Sunteți pe pagina 1din 427

OSCE

PEDIATRIA

|| NEONATOLOGIA

Contenido
RECIEN NACIDO TÉRMINO Y PRETERMINO .......... 9
RNBP y PEG ........................................................ 13
Percentiles Peso para EG Sochipe 2010 .......... 13
HIJO DE MADRE DIABETICA ............................ 17
TERMORREGULACIÓN................................ 17
APNEA DEL RECIEN NACIDO ........................... 18
SINDROME DE DISTRESS RESPIRATORIO......... 19
ENFERMEDAD MEMBRANA HIALINA
(EMH) ........................................................ 19
TAQUIPNEA TRANSITORIA .......................... 20
SINDROME ASPIRATIVO MECONIAL ........... 20
HIPERTENSIÓN PULMONAR PERSISTENTE.. 20 7. FARINGOAMIGDALITIS AGUDA
SEPSIS NEONATAL .......................................... 26 BACTERIANA .................................................. 36

>37 semanas asintomático con FR madre Definición ....................................................... 36


Corioamnionitis ......................................... 27 Diagnóstico .................................................... 36
>37 semanas asintomático con FR madre: Clínica ............................................................ 36
RPM o Profilaxis inadecuada...................... 27 Tratamiento ................................................... 36
ICTERICIA NEONATAL ..................................... 27 8. OTITIS MEDIA AGUDA ............................ 36
1. GIARDASIS.............................................. 32 Definición ....................................................... 36
Definición....................................................... 32 Clínica ............................................................ 36
Diagnóstico .................................................... 32 Tratamiento ................................................... 36
Clínica ............................................................ 32 9. LARINGITIS OBSTRUCTIVA ...................... 37
Tratamiento ................................................... 32 Definición ....................................................... 37
2. ENTEROBIASIS ........................................ 32 Clínica ............................................................ 37
Definición....................................................... 32 Tratamiento ................................................... 37
Diagnóstico .................................................... 32 10. BRONQUITIS AGUDA OBSTRUCTIVA ... 37
Clínica ............................................................ 32 Definición ....................................................... 37
Tratamiento ................................................... 32 Clínica ............................................................ 37
3. PEDICULOSIS .......................................... 33 Tratamiento ................................................... 37
Definición....................................................... 33 11. NEUMONIA ........................................ 37
Diagnóstico .................................................... 33 Definición ....................................................... 37
Clínica ............................................................ 33 Clínica ............................................................ 37
Tratamiento ................................................... 33 Tratamiento ................................................... 37
Definición....................................................... 34 12. DERMATITIS SEBORREICA ................... 38
Diagnóstico .................................................... 34 Definición ....................................................... 38
Clínica ............................................................ 34 Diagnóstico .................................................... 38
Tratamiento ................................................... 34 Clínica ............................................................ 38
5. RGE ........................................................ 34 Tratamiento ................................................... 38
Definición....................................................... 34 13. DERMATITIS ATOPICA........................ 39
Diagnóstico .................................................... 34 Definición ....................................................... 39
Clínica ............................................................ 34 Clínica ............................................................ 39
Tratamiento ................................................... 34 Tratamiento ................................................... 39
6. RESFRIO COMUN ................................... 36 14. DERMATITIS DEL PAÑAL ..................... 40
Definición....................................................... 36 Definición ....................................................... 40
Diagnóstico .................................................... 36 Diagnóstico .................................................... 40
Clínica ............................................................ 36 Clínica ............................................................ 40
Tratamiento ................................................... 36
Tratamiento ................................................... 40 Tratamiento ................................................... 44
15. GRANULOMA UMBILICAL ................... 41 25. URACO PERSISTENTE .......................... 44
Definición....................................................... 41 Definición ....................................................... 44
Tratamiento ................................................... 41 Diagnóstico .................................................... 44
16. ONFALITIS .......................................... 41 Tratamiento ................................................... 44
Definición....................................................... 41 26. HIDROCELE......................................... 44
Tratamiento ................................................... 41 Definición ....................................................... 44
17. IMPETIGO .......................................... 41 Diagnóstico .................................................... 44
Definición....................................................... 41 Clínica ............................................................ 44
Clínica ............................................................ 41 Tratamiento ................................................... 44
Tratamiento ................................................... 41 27. DISPLASIA DE CADERA ........................ 45
18. PULICULOSIS ...................................... 42 Definición ....................................................... 45
Definición....................................................... 42 Diagnóstico .................................................... 45
Diagnóstico .................................................... 42 Clínica ............................................................ 45
Profilaxis ........................................................ 42 Tratamiento ................................................... 45
19. PIE PLANO .......................................... 42 28. ESCOLIOSIS......................................... 46
Definición....................................................... 42 Definición ....................................................... 46
Examenes....................................................... 42 Diagnóstico .................................................... 46
Clínica ............................................................ 42 Tratamiento ................................................... 46
20. TESTICULO EN ASCENSOR .................. 42 29. PERTHES............................................. 46
21. FIMOSIS ............................................. 43 Definición ....................................................... 46
Definición....................................................... 43 Diagnóstico .................................................... 46
Complicación ................................................. 43 Clínica ............................................................ 46
Clínica ............................................................ 43 Tratamiento ................................................... 46
Derivar ........................................................... 43 ADMINISTRACION SIMULTANEA DE VACUNAS:
22. CRIPTORQUIDIA ................................. 43 En general se han obtenido niveles de
protección similares a la administración de
Definición....................................................... 43 vacunas separadas. ....................................... 59
Clínica ............................................................ 43 VACUNACIONES INTERRUMPIDAS: No se
23. SINEQUIA VULVAR ............................. 43 requiere reiniciar toda la serie, pudiendo
continuarse el esquema con la siguiente
Definición....................................................... 43
dosis .............................................................. 59
Clínica ............................................................ 43
VACUNACIONES NO INICIADAS: .................... 59
Tratamiento ................................................... 43
• Niños menores de 1 año debe
24. HERNIA UMBILICAL ............................ 44 comenzar con la serie a los mismos
Definición....................................................... 44 intervalos de tiempo establecidos. ........... 60
Clínica ............................................................ 44
• Niños mayores de 1 año deben recibir PAUTA DE COTEJO.......................................... 78
MMR, DPT y Polio y luego 2 dosis mas a ESTACION 2 HOJA DE INSTRUCCIONES ........... 81
intervalos de 2 o 4 meses ......................... 60
UD. DEBE CONTESTAR LAS PREGUNTAS EN LA
• Después de los 4 años no se usa DPT HOJA DE RESPUESTAS .................................... 82
sino DT ...................................................... 60
ESTACIÓN 2 HOJA DE RESPUESTAS ................ 83
Respetar el intervalo mínimo
recomendado para cada vacuna .................... 60 ESTACIÓN 2 .................................................... 83

Al aumentar el intervalo entre las dosis no INSTRUCCIONES ............................................. 84


disminuye la efectividad................................ 60 ESTACIÓN 10.............................................. 84
No hay contraindicaciones para ESTACIÓN 10.............................................. 85
administración simultánea de vacunas
ESTACIÓN 10 PAUTA DE CORRECCIÓN ...... 85
(administrar en sitios distintos del cuerpo),
excepto para cólera y fiebre amarilla. ........... 60 RESPUESTAS ................................................... 87

Si diferentes vacunas no se administran ESTACIÓN 15.............................................. 87


simultáneamente, se recomienda: ................ 60 Aspecto: turbio Color amarillo¨ Ph : 5
Intervalo entre 2 vacunas vivas Nitritos: positivos Leucocitos > 20-30 por
diferentes: 4 semanas intervalo. .............. 60 campo........................................................ 87

ESTACIÓN 1 PAUTA PARA LA ACTRIZ .................. 62 ESTACIÓN 15 HOJA DE RESPUESTA ............ 87

ALIMENTACIÓN Y NUTRICIÓN ........................ 62 ESTACIÓN 15 PAUTA DE CORRECCIÓN ...... 87

ESTACIÓN 1 – ALMUERZO PAUTA DE COTEJO63 UD ES MÉDICO DEL SAPU Y RECIBE A UNA ..... 88

ESTACIÓN 14 PAUTA PARA LA ACTRIZ ................ 65 ESTACIÓN 12 HOJA DE RESPUESTA ............ 89

ALIMENTACIÓN Y NUTRICIÓN ........................ 65 ESTACIÓN 12.............................................. 90

ESTACIÓN 14 PREPARACIÓN DE MAMADERAS UD DEBE VER LAS DISTINTAS SITUACIONES


PAUTA DE COTEJO ............................................. 67 CLÍNICAS EN EL ARCHIVO ADJUNTO Y HACER EL
DIAGNOSTICO .................................................... 90
EN LA HOJA DE RESPUESTA................................ 67
HOJA DE RESPUESTA ...................................... 91
ESTACIÓN 3.................................................... 67
ESTACIÓN 13 PAUTA DE COTEJO .................... 93
ESTACIÓN 3.................................................... 68
INFORMARSE DE LOS ASPECTOS NECESARIOS
ESTACIÓN 3 ............................................... 69
RELACIONADOS CON ESTE CONTROL DE SALUD 94
ESTACIÓN 4 ....................................................... 71
ESTACIÓN 11 CONTROL SANO ........................ 94
HOJA DE RESPUESTAS .................................... 72
ESTACIÓN 11 - CONTROL SANO PAUTA DE
ESTACIÓN 5.................................................... 73 COTEJO .......................................................... 96
ESTACION 5 PAUTA PARA LA ACTRIZ ............ 73 ESTACIÓN Nº: 2 PAUTA
ESTACIÓN 5 PAUTA DE COTEJO ...................... 74 DEL ALUMNO..................................................... 97

ESTACIÓN 6.................................................... 75 ESTACIÓN Nº: 2 PAUTA DE


CORRECCIÓN ..................................................... 97
ESTACIÓN 6.................................................... 76
ESTACIÓN Nº: 2 PAUTA DEL
UD ES MEDICO DEL CESFAM Y DEBE REALIZAR LA
EVALUADOR....................................................... 98
EVALUACIÓN NUTRICIONAL DE SOFÍA, LACTANTE
DE 12 MESES CON EL ANTECEDENTES DE RNPreT ESTACIÓN Nº: 2 PAUTA DE
30 SEMANAS ..................................................... 78 PACIENTE SIMULADO......................................... 98
INDIQUE EL DIAGNÓSTICO ................................. 99 ESTACIÓN Nº: 8 PAUTA
ESTACIÓN Nº: 2 PAUTA DEL PACIENTE DEL ALUMNO ........................................... 114
Y EVALUADOR ................................................ 99 EN LA HOJA DE RESPUESTAS ....................... 114
CON TOS DE PERRO .............................. 100 ESTACIÓN Nº: 8 PAUTA
NO RESPIRABA RÁPIDO ........................ 100 DE CORRECCIÓN ...................................... 114

AL TERMINAR SI EL ALUMNO NO LE HA ESTACIÓN Nº: 8 HOJA DE


INFORMADO, CONSÚLTELE SI DEBE RESPUESTAS ............................................ 115
LLEVARLA A URGENCIA AL TERMINAR SÓLO ESTACIÓN Nº: 6 PAUTA
DELE LAS GRACIAS ESTACIÓN Nº: 2 DE CORRECCIÓN ...................................... 119
PAUTA DE CORRECCIÓN .......................... 100 ESTACIÓN Nº: 11
ESTACIÓN Nº: 3 PAUTA DEL ALUMNO ............................... 120
PAUTA DEL ALUMNO ................................... 101
RESPONDA LAS PREGUNTAS DE LA MADRE...... 120
ESTACIÓN Nº: 3 PAUTA DE ESTACIÓN Nº: 11 PAUTA DE
CORRECCIÓN ............................................... 101 CORRECCIÓN................................................ 120
ESTACIÓN Nº: 3 PAUTA
ESTACIÓN Nº: 11 PAUTA DE PACIENTE
PACIENTE SIMULADO................................... 102 SIMULADO ................................................... 121
NOMBRE DEL Y LO LLEVA A CONTROL SANO POR PRIMERA
ALUMNO:______________________________
VEZ .......................................................... 121
______ NÚMERO DEL ALUMNO:________ .. 104
ESTÁ TOMANDO SOLO PECHO, NO LE HA
NO OLVIDE COLOCAR SU NOMBRE Y DADO OTROS ALIMENTOS ....................... 122
NÚMERO INTRODUZCA SU HOJA DE
RESPUESTAS EN EL BUZÓN ESTACIÓN Nº: 4 DE LA MADRE................................................... 125
PAUTA DE CORRECCIÓN .......................... 104 ESTACIÓN Nº: 13 PAUTA DEL PACIENTE
RESPONDA SUS PREGUNTAS............................ 106 SIMULADO ................................................... 127

ESTACIÓN Nº: 5 PAUTA DE CALCULE QUE NO SE LE ACABE EL TIEMPO


CORRECCIÓN ............................................... 106 Y .............................................................. 127

5 PAUTA DEL 3.- QUÉ HACER PORQUE A SU HIJO LE


EXAMINADOR .......................................... 107 GUSTA.......................................................... 128

ESTACIÓN Nº: 5 PAUTA ESTACIÓN Nº: 14


PACIENTE SIMULADO................................... 107 PAUTA DEL ALUMNO ............................... 128

ACOMPAÑADO POR SU MADRE ............... 108 NOMBRE DEL


ALUMNO:____________________________
CONSIDERE SUS MANOS YA LAVADAS...... 108 ____ NÚMERO DEL
UN CONTROL SANO ................................. 108 ALUMNO:____________________________
Y LE DARÁ EL RESULTADO ........................ 108 ____ ........................................................ 129

SI NO LE DIJO QUE SE PUSIERA LOS ESTACIÓN Nº: 15


ZAPATOS,..................................................... 109 PAUTA DEL ALUMNO ............................... 130

ESTACIÓN Nº: 6 RESPONDA A SUS INQUIETUDES.................... 131


PAUTA DE CORRECCIÓN .......................... 109 CUANDO ERA CHICO Y TOMABA PECHO NO
ESTACIÓN N°: 13 PAUTA TENÍA PROBLEMAS HACÍA CAQUITA VARIAS
DE CORRECCIÓN ...................................... 113 VECES AL DÍA, DESPUES AL SUSPENDER EL
PECHO SE PUSO MAS ESTÍTICO, PERO LA AYER USTED EXAMINÒ A UNA NIÑA DE UN
MADRE LO SOLUCIONABA CON JUGOS ........ 132 AÑO ............................................................. 144
NO LE GUSTA IR AL BAÑO EN SU COLEGIO YA PRUEBAS HEPÁTICAS Y UN HEMOGRAMA
QUE LA PUERTA NO CIERRA BIEN Y LOS QUE ............................................................. 144
COMPAÑEROS MOLESTAN Y HACEN BURLAS, LA HOJA DE RESPUESTAS ......................... 144
ADEMAS ESTAN MUY SUCIOS ...................... 132
H E M O G R A M A ................................... 146
PEDRO TOMA 2 LECHES CON CHOCOLATE
DIARIAS, COME LA COMIDA NORMAL DE LA (Detalle todos los pasos que siguió para su
CASA, NO LE GUSTAN LAS VERDURAS, DE evaluación) .................................................. 147
FRUTA COME SOLO PLÁTANO (UNO TODOS LOS ESTACIÓN Nº: 4
DÍAS) Y LAS BEBIDAS LE ENCANTAN ............ 132 PAUTA DE CORRECCIÓN .......................... 147
VA A HACER MAL, SI NO SE VA A USTED ESTÁ DE TURNO EN UN SAPU: ATIENDA
ACOSTUMBRAR ........................................... 132 EL TELÉFONO y RESUELVA LA SITUACIÓN E
ESTACIÓN Nº: 16 PAUTA INDIQUE EL DIAGNÓSTICO ........................... 148
DE CORRECCIÓN .......................................... 132 ESTACIÓN Nº: 17 PAUTA
ESTACIÓN Nº: 17 PAUTA DE CORRECCIÓN ...................................... 148
DEL ALUMNO ............................................... 135 OBSERVE LAS 5 FOTOS Y ANOTE LOS
ESTACIÓN Nº: 17 PAUTA DE DIAGNÓSTICOS EN LA HOJA DE
CORRECCIÓN ............................................... 136 RESPUESTAS................................................. 149

FOTO DE LA RADIOGRAFÍA PARA LOS ESTACIÓN Nº: 5 PAUTA


CÁLCULOS .................................................... 138 DE CORRECCIÓN ...................................... 149

ESTACIÓN Nº: 17 PAUTA FOTO 3..................................................... 152


PACIENTE SIMULADO................................... 138 ESTACIÓN Nº: 16
LUEGO LE INDICARÁ EL DIAGNÓSTICO ..... 139 PAUTA DEL ALUMNO ............................... 152

ESTACIÓN Nº: PAUTA ESTACIÓN Nº: 16


DEL ALUMNO ............................................... 139 PAUTA DE CORRECCIÓN .......................... 153

AL EXAMEN FISICO ....................................... 140 ESTACIÓN Nº: 8


PAUTA DEL ALUMNO ............................... 154
ESTACIÓN Nº: PAUTA DE
CORRECCIÓN ............................................... 140 ESTACIÓN Nº: 8 PAUTA
DE CORRECCIÓN ...................................... 154
ESTACIÓN Nº: PAUTA DE
PACIENTE SIMULADO................................... 141 ESTACIÓN Nº: 8 PAUTA
PACIENTE SIMULADO............................... 155
ESTACIÓN Nº: 2
PAUTA DEL ALUMNO ................................... 142 ESTACIÓN Nº: 12
PAUTA DEL ALUMNO ............................... 156
AL CONTROL SANO .......................................... 142
EL TRATAMIENTO SI LO REQUIRIERA................ 156
ESTACIÓN Nº: 2 PAUTA DE
CORRECCIÓN ............................................... 142 ESTACIÓN Nº: 12 PAUTA
DE CORRECCIÓN .......................................... 157
ESTACIÓN Nº: 2 PAUTA
PACIENTE SIMULADO................................... 143 ESTACIÓN Nº: 15 PAUTA
DEL ALUMNO ............................................... 158
EL ALUMNO LE HARÁ PREGUNTAS........... 143
NOMBRE DEL
ALUMNO:______________________________
______ NÚMERO DEL Neumonía neonatal...................................... 220
ALUMNO:______________ ......................... 159 Síndrome de aspiración meconial ................ 222
USTED ATIENDE A RAÚL UN NIÑO DE 10 AÑOS, Síndrome de escape aéreo ........................... 223
CUYA MADRE ESTÁ MUY PREOCUPADA POR QUE
EL NIÑO MUESTRA UNA OBESIDAD IMPORTANTE Hipertensión pulmonar persistente.............. 223
(IMC > P95) CON TALLA NORMAL .................... 160 Malformaciones pulmonares del RN ............ 224
ESTACIÓN Nº: 10 PAUTA Hiperbilirrubinemia .......................................... 224
PACIENTE SIMULADO................................... 160
Hijo de madre diabética ................................... 233
º:
ESTACIÓN N 9
Trastornos de la glucosa .................................. 236
PAUTA DE CORRECCIÓN............................... 162
Hipoglicemia ................................................ 236
ESTACIÓN Nº: PAUTA
DEL ALUMNO ............................................... 164 Hiperglicemia ............................................... 238

ESTACIÓN Nº: 2 PAUTA DE Trastornos del calcio, fosforo y magnesio ........ 240
CORRECCIÓN ............................................... 165 Hipocalcemia ............................................... 240
º:
ESTACIÓN N PAUTA DE Hipercalcemia .............................................. 241
PACIENTE SIMULADO................................... 166
Hipofosfemia................................................ 242
ESTACIÓN Nº: PAUTA
Hiperfosfemia .............................................. 242
DEL ALUMNO ............................................... 167
Hipomagnesemia ......................................... 242
ESTACIÓN Nº: 2 PAUTA DE
CORRECCIÓN ............................................... 167 Hipermagnesemia ........................................ 243

ESTACIÓN Nº: PAUTA DE Sepsis neonatal y meningitis ............................ 244


PACIENTE SIMULADO................................... 169 Infecciones congénitas. Síndrome de TORCH ... 247
ESTACIÓN Nº: PAUTA Toxoplasmosis .............................................. 247
DEL INTERNO ............................................... 170
Rubéola ........................................................ 248
ESTACIÓN Nº: 10 PAUTA DE
Citomegalovirus ........................................... 249
CORRECCIÓN 171
VHS .............................................................. 250
ESTACIÓN Nº: PAUTA DE
PACIENTE SIMULADO................................... 173 Sífilis congénita ............................................ 250

NEONATOLOGÍA .............................................. 193 Enfermedad de Chagas ................................ 251

Examen físico y fenómenos parafisiológicos del Hepatitis B.................................................... 251


RN ................................................................... 194 Parvovirus B 19 ............................................ 251
Prematurez ...................................................... 203 VIH ............................................................... 253
Atención inmediata neonatal ........................... 208 Síndrome de Down .......................................... 254
Reanimación neonatal ..................................... 208 Consultas del RN en urgencias ......................... 255
Asfixia perinatal y EHI ...................................... 212 PEDIATRÍA AMBULATORIA ............................... 256
Convulsiones neonatales ................................. 215 Control de salud infantil (niño sano) II ............. 256
Síndrome de Distrés Respiratorio .................... 218 Controles de salud infantil por edad............. 256
Enfermedad de membrana hialina ............... 218 Desarrollo psicomotor ..................................... 260
Taquipnea transitoria de lRN........................ 220 Alimentación infantil ........................................ 261
Alimentación del menor de 2 años............... 261 Cardiopatía congénita cianótica ................... 300
Alimentación del mayor de 2 años ............... 265 Tetralogía de Fallot .................................. 300
Evaluación nutricional...................................... 265 Transposición de grandes arterias............ 300
Lesiones dermatológicas frecuentes del Atresia tricúspide ..................................... 301
lactante ........................................................... 268
Tronco arterial ......................................... 301
Dermatitis .................................................... 268
Shock y síndrome séptico................................. 301
PATOLOGÍA RESPIRATORIA .............................. 271
Enfermedad de Kawasaki ................................. 304
Infecciones respiratorias altas ......................... 271
Reanimación cardiopulmonar .......................... 306
Rinofaringitis aguda ..................................... 271
PATOLOGÍA DIGESTIVA .................................... 307
Faringoamigdalitis aguda ............................. 272
Síndrome diarreico agudo ................................ 307
Sinusitis........................................................ 273
Síndrome emético............................................ 311
Otitis media ................................................. 273
Constipación y encopresis................................ 313
Otitis externa ............................................... 274
PATOLOGÍA INFECCIOSA .................................. 313
Generalidades de IRAs bajas: Resumen guía
Exantemas ....................................................... 313
MINSAL............................................................ 274
Síndrome febril agudo sin foco ........................ 318
Laringitis aguda obstructiva ......................... 274
Meningitis y encefalitis .................................... 320
Bronquitis aguda catarral (no obstructiva) ... 276
Meningitis .................................................... 320
Bronquiolitis................................................. 277
Encefalitis..................................................... 323
Bronquitis obstructiva aguda ....................... 278
Convulsión febril .............................................. 324
Coqueluche .................................................. 279
Vacunas ........................................................... 327
Infección respiratoria por Influenza ............. 280
Hepatitis viral ................................................... 332
Neumonía ........................................................ 282
Infección urinaria ............................................. 333
Fibrosis quística ............................................... 287
Infecciones osteoarticulares ............................ 338
Asma................................................................ 289
PATOLOGÍA ENDOCRINA .................................. 340
Bronquitis obstructiva aguda ....................... 294
Talla baja.......................................................... 340
PATOLOGÍA CARDIOVASCULAR ........................ 297
Obesidad.......................................................... 345
Cardiopatías congénitas................................... 297
Cetoacidosis diabética ..................................... 348
Cardiopatías congénitas acianóticas............. 297
PATOLOGÍA NEFROUROLÓGICA ....................... 350
Comunicación interventricular................. 297
Síndrome hemolítico urémico .......................... 350
Comunicación interauricular .................... 297
Alteraciones del equilibrio acido base .............. 354
Conducto arterioso persistente ............... 298
Síndrome edematoso....................................... 363
Defectos del canal AV .............................. 298
PATOLOGÍA HEMATOLÓGICA ........................... 365
Estenosis pulmonar ................................. 299
Síndrome purpúrico ......................................... 365
Estenosis aórtica ...................................... 299
Púrpura de Schonlein Henoch ...................... 367
Coartación de aorta ................................. 299
Trombocitopenia inmune primaria (PTI)....... 367
PATOLOGÍA NEUROLÓGICA ............................. 368 Cuerpo extraño ................................................ 415
TEC .................................................................. 368 Cuerpo extraño en la vía aérea..................... 415
Compromiso de conciencia .............................. 370 Cuerpo extraño intranasal ............................ 416
Intoxicaciones .................................................. 371 Ingestión de cuerpo extraño ........................ 417
Síndrome convulsivo (Eventos paroxísticos de Hemorragia digestiva ....................................... 418
etiología epiléptica) ......................................... 379 Neutropenia, leucemia y linfoma ..................... 420
Enuresis ........................................................... 381 Neutropenia ................................................. 420
CIRUGÍA INFANTIL y ONCOLOGÍA..................... 382 Cáncer en pediatría ...................................... 420
Patología quirúrgica del RN.............................. 382 PATOLOGÍA INMUNOLÓGICA ........................... 423
Atresia de esófago ....................................... 382 Reacciones alérgicas y anafilaxia ...................... 423
Obstrucción intestinal neonatal ................... 384
Atresia y estenosis intestinal .................... 385
Malrotación intestinal .............................. 386
Íleo meconial ........................................... 387
Enfermedad de Hirschprung neonatal ..... 388 RECIEN NACIDO TÉRMINO Y PRETERMINO
Expulsión completa del producto de la concepción,
Malformación ano – rectal ........................... 389 independiente de la duración del embarazo, que después de
dicha separación respire o dé señal de vida: (Latido cardiaco,
Hernia diafragmática congénita ................... 392 pulso del cordón umbilical, mov. de mm voluntarios y/o
movimiento respiratorio), tanto si ha cortado o no el cordón
Gastrosquisis y onfalocele ............................ 393 umbilical y esté o no desprendida la placenta.
El límite de viabilidad:
Enterocolitis necrotizante ............................ 395
• >500 grs;
Reflujo gastroesofágico.................................... 399 • > 22 semanas.
Clasificación
Estenosis hipertrófica del píloro ...................... 401 Peso al Nacer Edad Gestacional
Abdomen agudo .............................................. 402 BPN <2500 grs PreTérmino <37 semanas
MBPN <1500 grs Término 37-41 sem
Apendicitis aguda......................................... 405 EBPN <1000 grs Post-Término ≥42 sem

Malrotación intestinal y vólvulo ................... 405 RECIEN NACIDO NORMAL


 Atención Inmediata: Mínimo 2 personas
Adenitis mesentérica ................................... 406 1. Evaluación Rápida del RN y manejo de
emergencias
Constipación ................................................ 406 2. Termorregulación: Ambiente a 28°, secar con
sábanas tibias, apego precoz, gorro, cuna
Dolor perimenstrual ovulatorio .................... 406
radiante.
Divertículo de meckel .................................. 406 3. Apgar: Al minuto y a los 5 minutos.
Escroto agudo.................................................. 407
Torsión testicular ......................................... 407
Torsión de hidátide de Morgagni ................. 408
Fimosis ............................................................ 408
Testículo no descendido .................................. 410
Sinequia vulvar ................................................ 412
4. Ligadura de cordón umbilical
Hernia inguinal e hidrocele testicular............... 414 5. Examen físico completo
6. Identificación
7. Antropometría: Peso, talla y CC porque todos los niños tragan líquido amniótico.
Además la sonda se pasa para ver si tiene estenosis o
8. Edad Gestacional
atresia de esófago, aunque si tiene estenosis o algún stock
debe tener síntomas inmediatamente ( sialorrea, distress
a. TEST USHER:
respiratorio) por lo tanto no es necesario realizarlo. Lo
i. Cabello:
único que se debiese hacer es limpiar la boca.
1. Escaso, fino, aglutinado, De realizarlo, cuidar Depresión por reflejo vagal (bradicardia,
difícil de separar (hasta las menor respiración, cianosis.)
37 semanas)
2. Abundante, grueso,
individualizable (>38
semanas) ii. Pabellón
auricular:
1. Escaso cartílago, no
vuelve a su posición
después de plegarlo: < de
36 semanas
2. Menos deformable,
cartílago regular, tarda en
volver a su posición: 37 o
38 semanas
3. Poco deformable,
cartílago grueso y rígido,
vuelve rápido a su
posición: más de 39
semanas
iii. Pezón:
1. Menos de 0.5 cm o
ausente: < de 36 semanas
2. 0.5-1 cm diámetro:
37 o 38 semanas
3. 1-4 cm diámetro: más
de 39 semanas iv. Genitales
1. Masculinos: escroto liso,
micropene y testículos no
descendidos (<36
semanas); escroto con
plieuges, testículos
descendidos (más de 38
semanas)
2. Femeninos: labios
menores sobresalen
sobre los mayores (<36
semanas); labios mayores
cubren a los menores
(más de 38 semanas)
v. Pliegues plantares: ( es el más importante )
1. 1 o más en 1/3 anteriores
del pie: < de 36 semanas
2. Pliegues en 2/3 anteriores
del pie: 37 o 38 semanas
3. Pliegues en toda la planta
del pie: más de 39
semanas
b. TEST DE BALLARD
9. Profilaxis ocular: Colirio CAF
10. Profilaxis hemorrágica: vit. K. 1 mg IM, RNPT
0,5mg IM
11. Muestra de Cordón: Grupo, Rh, Coombs
Por norma no es obligación pasar la sonda nasogátrica para
aspirar secreciones
Romina Rojas
Guardia
 Transición: 2. Higiene
1. Apego materno 3. Parámetros antropométricos: peso, talla CC.
2. Lactancia materna precoz (antes de 4 hrs) 3. Signos vitales: a. Recuperación peso hasta peso RN y sobrepasarlo
o FC= 120-160 o FR= 40-60 o T° o Diuresis: 4. Verificar el brote de BCG: 15-30 días tras la vacuna en el sitio de inyección.
92% primer día. 2do día el 100%. Indicar cuidados respectivos.
o Deposiciones: 94% primer día Puerperio: 5. Siguiente control al mes.
1. Alimentación: LME o si requiere fórmula
2. Examen físico completo RECIEN NACIDO PRETERMINO
3. Cambios de piel: ictericia fisiológica, cianosis, eritema tóxico, lesiones de la piel, <37 semanas de EG. Factores de Riesgo:
equimosis, descamación. Maternos Fetales Uterino
4. Postura. Madre soltera, bajo peso y Anomalías congénitas, Sobredistensión por PHA,
5. Deposiciones, diuresis. talla, tabaquismo, <20 o >40 muerte fetal, embarazo malformaciones, infecciones,
6. Cambios de peso: baja un 10% de su peso RN los primeros 5-7 días. años. Enfermedad sistémica múltiple. DPPNI, PP, tu cuerpo extraño, mioma,
7. Cambios ombligo: Momificación. Cuidar onfalitis. grave, metrorragia <20sem, cordón umbilical incompetencia cervical.
antec. Parto prematuro,
8. Screening PKU-TSH, tiene que tener mínimo 40 hrs de vida si es de término
ARO, ITU
9. Vacunación BCG: se puede hacer en puerperio o bien el día del alta debe irse
con su vacuna puesta.
 Alta: Puede ser en 2 días si el parto es vaginal, 3 si es cesárea, procurar binomio.
1. Alimentación: Fomentar LME libre demanda (no >4 hrs entre tomas). Morbilidad Asociada
2. Peso: pérdida adecuada, evaluar técnica LM.
Inmadurez de sistemas Alt. termorregulación: hipotermia
3. Temperatura: Sobre abrigo o enfriamiento.
EMH, Depresión respiratoria, Distress. Trast. Nutricional, baja peso 12-15%, (Sat O2: 88-92 %)
4. Examen físico completo. inmadurez S.Digestivo por lo que se
• Cardiopulmonar: Buscar soplos Mayor predisposición a infecciónes retrasa alimentación.
• Piel y Cuidados de la piel Ictericia precoz y patológica HIC
• Cordón umbilical, ya va a estar más momificado. Cardiopatías (CIA, CIV, DAP) Enf. Hemorragípara
5. Apego y actitud materna: ver si hay o no apégo. Hipoglicemia, HipoCa, Alt ELP, poliglob.
6. Toma exámenes TSH y PKU: verificar si estan tomados
7. Inmunización BCG: verificar que este vacunado A largo plazo BP y Talla baja, retinopatía y ceguera, hipoacusia, DBP (con 1 m de
oxígeno continuado o VM
Consideraciones: LME/LD <4hrs, no bañar (sólo algodón y agua), limpiar cordón con alcohol 70°
en cada muda. Seguimiento GES: todo RN <1500 GR O <32 semanas o que haya requerido cuidado intenstivo
para pesquisar y tratar oportunamente problemas secundarios a patología neonatal. Niños con
Situaciones normales esperables: Cambio de deposición (color y consistencia), Estornudos fctes. DBP tienen otro seguimiento, reciben vacuna VRS.
Y secresión nasal, Caída del cordón 7-10 días, Se puede bañar 2-3 días desde que se cae el
cordón, pañal con orina naranja e ictericia.

Control con pediatra a los 15 días:


1. Alimentación: LME, Fórmula, mixta
RNBP y PEG
Fact. Maternos Constitución, Emb. Gemelar, privación nutricional,
En Chile alrededor del 6% de los niños tienen bajo peso al nacer y un 0,4% Extremo Bajo Peso. Esta población
está expuesta a un alto riesgo de morbimortalidad. efermedad crónica, drogas, N. S/E bajo, SHE, DMG, y DMPG,
tabaquismo y OHC
Se
Clasificación P/EG Clasificación Peso
Fact. Placentarios Emb. Gemelar, TORCH, necrosis, abruptia, inserc. Anormal,
PEG <p10 Macrosómico >4000 g BPN <2500 grs thrombosis vasos fetales
AEG p10-p90 Normal 3999-3000 grs MBPN <1500 grs
GEG >p90 Insuficiente 2999-2500 g EBPN <1000 grs Fact. Fetales Cromosómicas, metabólicas, infecciones.

debe diferenciar de RCIU, ya que estos tienen una falla en alcanzar el potencial biológico de su
crecimiento, mientras que los PEG son constitucionalmente pequeños.

Percentiles Peso para EG Sochipe 2010


PEG: Principalmente asociado a insuficiencia placentaria e hypoxia crónica,
poliglobulia e hipoglicemia. Tienen mayor susceptibilidad al parto y distress.
Complicaciones Asociadas: Mortinato, Anomalía congénita, Depresión perinatal,
hemorragia pulmonar, HTPulm, Hipotermia, HipoGlicemia,
HipoCalcemia, HipoNatremia, Policitemia, Trombocitopenia, Leucopenia y
alt. Inmunológicas, ECN.
.
Estas complicaciones también pueden presentarse en RNT pero con menor
frecuencia e incluso con menor severidad

HIPOGLICEMIA

<40mg/dL en Sangre y <45 en plasma.


Clínica: Alt. Conciencia, apatía, hipotonía, temblores, pobre succión y alimentación,
vómitos, respiración irregular, taquipnea, apneas, cianosis, convulsiones, coma
Etiología:
Aumento uso periférico: Hijo de madre diabética, Eritroblastosis, Drogas maternas, Hiperplasia o hiperfunción de Cel
Beta , Producción insulina por TU, iatrogenia.
Inadecuado Aporte: RNPT, RCIU, Ayuno prolongado, Stress perinatal, Policitemia, Exanguinotransfusión, Insuf.
Adrenal, Deficiencia hipotalámica y/o hipopituitaria, déficit de glucagón, alt, en metabl. Aa.
Grupos de Riesgo
PEG
Prematuros
Hijos de madre diabética
SD. Beckwhit-Wiedemann: Niños macrosómicos, con hernia umbilical y macroglosia. Tienen problemas a nivel de
páncreas, por lo que debe ser derivado.
Resumen Tratamiento
1) ABC+TERMORREGULACIÓN
2) TTO DE LOS SINTOMAS
3) TTO DE LA PATOLOGÍA
4) EX COMPLEMENTARIOS PARA DIAG Y MANEJO.
5) SCREENING BÁSICO EN RNPEG: HEMOGRAMA, GLICEMIA
CALCEMIA, P DE COAGULACIÓN, (PCR),RX TX (PATOL ASOC)
HIPOCALCEMIA POLICITEMIA
Niveles séricos de Calcio <7 mg/dL sin corregir RCIU, DMG, SHE, STFF, STMF, Tirotoxicosis neonatal,
Calcio Iónico <3,5 mg/dL Tabaquismo, Cardiopatía Pinzamiento tardío del Hipotiroidismo,
Clínica: Temblores, Hipertonía, Irritabilidad, Convulsiones, Laringoespasmo, Arritmias. cianótica materna, RN cordón , Posición del RN bajo Hiperplasia Srr Congénita,
HipoCa Precoz HipoCa Tardía postTermino el nivel de la madre. T13, T18,T21,
Altitud.
RNPT, generalmente asintomática, Rpta RNPT o RNT.
hormonal adecuada, pero los receptores
hormonales no permiten la elevación de la Clínica: Mayoría asintomáticos, otros: plétora mas visible en mucosas, plantas y palmas.,
calcemia. rubicundez, taquipnea, distress, cianosis, temblores, letargia, irritabilidad, problemas de
alimentación.
F.R.: DM materna, SHE, HipoMg, Uso de Etiol.: Bajo aporte, HipoMG,
Furosemida, Alcalosisrespiratoria o metab. Hipoparatiroidismo, Tto.
Complicaciones asociadas: Hipoglicemia, Hipocalcemia, Hiperbili, IC, Alt.
Asfixia Neonatal, Anticonvulsivante, Sd. Di George,
Coagulación e infartos o hemorragias cerebrales, ECN.
Exsanguineotransfusión Exsangineotranfsusión, Malabs. Intest.
Manejo:
Tratamiento: Se recomienda Hto venoso central 2-4 horas de vida a RN de riesgo o sintomáticos.
- Asintomático RN sano, no requiere. - Con 2 Hto >65% pero <70% sólo se tratarán si son sintomáticos.
- Sintomático o críticamente enfermo, especialmente si hay alteración CV (ej. - Con 2Hto >70% se tratarán aunque sean asintomáticos.
Arritmias): 45mg/Kg/d EV hasta normalizar niveles séricos. Eritrofereses por VVP o catéter umbilical venoso si ésta no fuera posible. Uso de 2 VVP para
- Convulsiones o tetania:10-18 mg/Kg EV a pasar LENTO (riesgo arritmias, bradicardia, infundir SSF por una y extraer sangre por la otra (mismos volúmenes):
PCR).
- 1 cc Gluconato de Calcio 10% equivale a 9 mg de Ca elemental.
- * Extravasación: necrosis y calcificaciones periféricas.
Para una corrección rápida por alteraciones cardiovasculares por ejemplo el aporte es de
Donde el Vol sanguíneo es de 80-100 ml/Kg, y el objetivo es llevar el Hto a 55-60% También se
95mg/kg/día, eso se divide en 3 o 4 dosis. Una formula rápida son 3cc/kg/día, y eso se divide entre
puede usar: 15-20ml/Kg de vol a extraer sin fórmula.
3 a 4 dosis.
En RN asintomático reiniciar Lactancia tras procedimieno, en RN sintomático o asoc a asfixia,
Cuando el calcio sigue bajo aun con bolos de calcio, se toma calcemia y este sigue bajo, pensar
hipoglicemia, infección u otro agravante debe estar 24-48 en Reg. 0 post eritroféresis con Hto
en una hipocalcemia refractaria, la que puede deberse a una Hipomagnesemia no tratada.
<65%, ya que si es mayor hay riesgo de ECN por hipoperfusión.
Controlar Hto a las 6-8 hrs post procedimiento.

RNGEG

Definición Incidencia
Hto. Venoso central >65%. Valor Normal es de 50-60%
Dada por la hipoxemia. La poliglobulia determina un estado de mayor viscosidad, por lo que el Aquel que presenta un peso por encima del Alta tasa de muerte fetal y neonatal.
flujo disminuye, se produce hipoperfusión y aumenta el riesgo de trombosis. p90 Tienden a tener EG de entre 38-42 semanas y
Etiología un peso mayor a 4000g. Complicaciones
secundarias, se dan especialmente en hijos de
Hipoxia Crón. Intrauterina Transf. Placento-fetal Otras madres diabéticas y son policitemia e
hipoglucemia principalmente.
Causas Conducta - GSA y Rx Tórax
Hijos de madres diabéticas Debe valorarse la causa probable. - Bilirrubinemia ante aparición de ictericia Tratamiento:
Neonatos post Termino Debe buscarse signos de T obstétrico y/o Lo más importante es pesquisar y prevenir la hipoglicemia.
Transposición de grandes vasos depresión perinatal. - DMG: HGT precoz a las 1-2 hrs y luego a las 6-8 hrs. Luego controles posteriores según
Eritroblastosis fetal Monitorizar glicemia y evaluar posibilidad de clínica. Si se pesquisa la hipoglicemia manejar.
Niños constitucionalmente grandes policitemia. - DM IR: Parto distócico, Monitorear 1-2-4-6 y luego según clínica. Manejar
Sd. Wiedemann-Beckwith hipoglicemia, si se descarta iniciar aporte oral precoz, fraccionado y según tolerancia.
Sd. Parabiótico (gemelos) - Observar durante 48 hrs la aparición de síntomas de morbilidad como temblores,
irritabilidad, apnea, dificultad respiratoria, ictericia, palidez; y descartar MFC

HIJO DE MADRE DIABETICA TERMORREGULACIÓN


Todo aquel niño nacido de una madre con diabetes mellitus, los que están expuestos a un La estabilidad de la temperatura corporal es expresión de un equilibrio entre la producción de
ambiente con trastornos metabólicos dependientes del tipo de diabetesde la madre, pero calor y la pérdida de calor.
principalmente hiperglicemia lo que lo obliga a una secresión importante de Insulina Fetal, lo El recién nacido tiene mayor pérdida de calor y además menor capacidad para aumentar la
que altera el crecimiento y desarrollo. producción de calor dado:
- Alta relación superficie/volumen: mientras más pequeño mayor es la superficie total
Trastornos específicos Observados: expuesta al ambiente.
Macrosomía, Hipoglicemia, Hipocalcemia, MFC cardiácas y SNC, EMH, - Menor Aislamiento Cutáneo: Piel y tej. sc delgados, más aun en PT.
Hiperbilirrubinemia, Policitemia, Asfixia perinatal y Tx del parto, Miocardiopatía Hipertrófica.
- Control Vasomotor: Regulación de la perdida, por vasoconstricción cutánea,
Embarazo: durante la gestación se puede producir en la madre hipoglicemia, cetoacidosis,
respuesta que se logra en RNT aprox. a los 15 días.
polihidroamnios, preeclampsia, y ello derivar en: abortos y mortinatos, parto prematuro, RCIU
(compromiso placentario), macrosomía (por el hiperinsulinismo fetal) y malformaciones. - Postura corporal: Posición en flexión les ayuda a guardar calor, RNPT tienen su
Parto: Son alteraciones secundarias a macrosomía, distocia y Tx, SFA. Periodo Neonatal: miembros extendidos.
Termogénesis No Termorreguladora: metabolismo basal, alimentos, postura. Termogénesis
1. Hipoglicemia. Termorreguladora: Grasa Parda alrededor de los vasos, mm del cuello, axila, mediastino y
2. El periodo de mayor riesgo 48 horas de vida y 1ª 6 horas son las más críticas. alrededor de los riñones, disminuye las perdidas produciendo calor por reacciones exotérmicas.
3. Hipocalcemia: Por la disminución de la respuesta a la hormona paratiroidea y/o En el Hipotiroidismo congénito, asfixia, BetaBloqueo, diazepam y anestésicos disminuyen la rpta.
hipomagnesemia asociada. Si ésta no se corrige con el aporte e.v., debe administrarse Metabólica al frío.
simultáneamente magnesio.
4. Hipomagnesemia: Debe evaluarse en hipocalcemias refractarias También tiene mayor susceptibilidad para absorber calor, sobre abrigo y espacios con altar
5. EMH: El índice de lecitina/ esfingomielina esmás confiable usar como valor predictivo mayor temperaturas hacen que el RN reaccione con sudoración, la cual es ltda. los primeros días y con
a 3 o medir fosfatidilglicerol, para predecir madurez pulmonar. HMD tienen retardo en la mayor razón mientras más PT.
producción de surfactante (a las 36 sem aprox)
6. Poliglobulia: Aumento de la EPO circulante en HMD, secundario a la hipoxia Manejo del Ambiente Térmico
7. Hiperbilirrubinemia - Conducción: pierde calor a través del contacto. Ropa, colchón, sábanas, etc.
8. Miocardiopatía hipertrófica. - Radiación: entre cuerpos a distancia por ondas electromagnéticas (sol, radiadores,
vidrios, etc.) Pierde calor hacia cualquier objeto más frío que él y ganará calor de
9. Malformaciones: SNC: anencefalia, encefalocele, síndrome de regresión caudal, aquellos objetos calientes a los que se exponga.
mielomeningocele, espina bífida, holoprosencefalia. Cardiovasculares: CIA, CIV, DAP.
Renales: agenesia, doble uréter, trombosis de vena - Convección: hacia el aire que lo rodea o que respira.
renal. Gastrointestinales: atresia anorrectal, colón izquierdo pequeño, raro, no - Evaporación:
quirúrgico. Ambiente térmico neutral: Rango de Temperatura ambiental en el cual el gasto metabólico se
mantiene al mínimo y la regulación de la temperatura se efectúa por mecanismo físicos no
Laboratorio: evaporativos, manteniéndose la temperatura corporal profunda en rangos normales. – 27 a 28
- Glicemia seriada °C
Considerar: EG - PESO- EDAD POSTNATAL-VESTIMENTA-SALUD RN.
- Calcemia Y del ambiente: T - SUPERFICIES RADIANES CERCA – CORRIENTES Y HUMEDAD.
- Hto y Hb
En el caso del prematuro, hay tablas de referencia que dan la t° a la que se debe colocar - Secarlo y cubrirlo con sabanas limpias y tibias.
inicialmente la incubadora para que aproximarse al ATN considerando el peso, la edad - Si el niño está en BCG se puede colocar piel con piel con su madre en una pieza a 26-
gestacional y la edad postnatal. 28 °C, cubierto, controlado su temperatura verificando que se estabilice

Temperatura <36.5°C
Se debe evaluar la causa: ambiental, antecedentes de asfixia o la ingestión materna de diazepam
o anestésicos. Se debe medir la T rectal y si está bajo 37° significa q el niño se enfrió y está en
riesgo de hipoglicemia y acidosis.
De acuerdo a la causa se tomaran las medidas correspondientes: Corregir la T° ambiental , abrigar
al niño, evaluar glicemia, GSA, Pesquisa de infección.
Inestabilidad térmica es signo precoz de ECN e Infección.

Temperatura >37°C
Evaluar condiciones ambientales y grado de abrigo. Se debe medir la temperatura rectal, si la
gradiente Ax/R disminuye a menos de 0,5 o se invierte es un signo de alza térmica importante o
un ambiente térmico demasiado cálido. Monitorizar cada 30 minutos hasta que se estabilice la
temperatura, con las medidas tomadas.

APNEA DEL RECIEN NACIDO


Ausencia de flujo aéreo en la vía respiratoria por al menos 20 segundos, o menos si se acompaña
de bradicardia o cianosis.
La respiración normal en el RN es periódica: patrón irregular en que se alternan pausas cortas
con ventilación normal y no produce hipoxemia ni bradicardia.

Mucho mas frecuente en RNPT


Clasificación
Etiología Presentación
Primaria o Ideopática – Inmadurez Es la más Central – sin flujo aéreo ni movimientos
fcte. Se pta. Al 2°-3° día en un prematuro sin respiratorios.
hallazgos patológicos

Secundaria: En PT y T, Alt. Metabólicos, HIC; Obstructiva – con movimientos


asfixia, convulsiones, infecciones, DBP, DAP respiratorios, pero sin flujo aéreo
hipotermia, etc.
Mixta – Apnea central prolongada que al final
se le agregan esfuerzos respiratorios sin flujo.

Monitorizar a todo RN <34 semanas, dado el alto riesgo de apnea.


La forma práctica de evaluar si el niño se encuentra en una ambiente térmico neutral es tomando El monitor respiratorio no detecta apneas obstructivas y ésta debe sospecharse en caso de
su temperatura axilar y verificando que esta se encuentra entre 36.5 y 37°C. bradicardia o cianosis sin etiología aparente.
Contribuye al reisgo de enfriamiento que los niños nacen desnudos y mojados; el feto en la vida
intrauterina está a una temperatura 0,5°C mas alta que la de la madre, por lo que el naciemiento
es la primera experiencia de frío y la primera vez que la termorregulación empieza a funcionar.

Medidas Inciales al nacer:


Tratamiento Diagnóstico
Ideopática Secundaria Antecedentes Baja EG, EMH en niño anterior, Metrorragia, Asfixia perinatal,
HMD, , eritroblastosis fetal, sexo masculino
Monitor Cardiorrespiratorio y SatO2 Tratar la causa
permanente. Clínica Signos de dificultad respiratoria precoz (antes 6 horas) y
Mantener cuello en posición neutra y aspirar progresiva: aleteo nasal, quejido polipnea y req. O2 en ascenso.
secreciones.
Mantener la temperatura corporal lo más
estable posible. Corregir la Hipoxemia Ex. Físico Disminución diámetro AP torax, MP↓ total leve o en las bases,
Teofilina: estimular centro respiratorio y crépitos
mejora contractilidad del diafragma.
Rx Tórax Aumento densidad homogéneo y difuso (vidrio esmerilado y
Tiende a desaparecer cuando el prematuro
broncograma aéreo)
alcanza las 35 – 37 semanas de EG
pH y GSA Hipoxemia e hipercapnia, Acidosis respiratoria o metaból.
El DgD debe plantearse con otras casuas de Distress, principalmente Bronconeumonia por Str.
Grupo B. Si no se logra diferenciar se trata para ambas enfermedades: surfactante +ATB.
Cuando no responde con medidas generales de Oxigenoterapia y estimulación se debe usar CPAP
(presión positiva continua) y si ésta fracasa, VM.
SINDROME DE DISTRESS RESPIRATORIO
Patología Frecueunte del RN que responde a múltiples etiologías.
Clínica:
 Taquipnea (>60)
 Retracciones torácicas intercostales subcostales o supraesternales
 Quejido respiratorio audible
 Cianosis
 Aleteo nasal Tratamiento
Pulmonares Extrapulmonares Prenatal Inducción de maduración pulmonar con corticoterapia.cuyo uso
Sd. Aspirativo Meconial DAP también disminuye ECN, HIC, DAP, DBP. Estudio de maduración el L.A.
Taquipnea transitoria Enf. CV, SNC
Enfermedad Membrana Hialina Sd. Hiperviscosidad (policitemia) Neonatal Grales: Hospitalizar en UCI, control temperatura, hidratación,
Sd. Hipertensión Pulmonar Persistente Hipotermia monitorización y equilibrio ácido-base.
Infecciones Metabólicas (hipoglicemia)
MFC (hipoplasias, tumores, etc) Surfactante: Instilación endotraqueal de surfactante exógeno natural o
artificial. Son 2 dosis (la segunda si está conectado a VM o tiene FiO2
<30% a las 8 – 12 hrs, si ya no está en respirador y tiene una FiO2>30%
ya no la necesita).
ENFERMEDAD MEMBRANA HIALINA (EMH) Se da la primera dosis en las primeras 2 horas de vida como profilaxis.
Principal causa en el prematuro. Propio del RNPT, también se conoce con SDRI. La condición
fundamental es que el niño NO tenga desarrollado los alveolos y ni su capacidad para producir
surfactante. Más frecuente a menor EG. Se debe a un déficit o ausencia de surfactante pulmonar Oxigenoterapia: Hood, CPAP, VM
en prematuros por lo cual se produce atelectasia y colapso alveolar por lo que no permite el
intercambio gaseoso Manejo de Patología Asociada: Hipoglicemia, hipocalcemia,
- Disminuye la distensibilidad pulmonar. hiponatremia, infección DAP, HIV, Ruptura alveolar.
- Disminuye la capacidad funcional residual. Pronóstico
- Alteración V/Q (un pulmón atelectásico fluctuante) • Letalidad global: 20-30%
• Tercera causa de mortalidad infantil o Aspiración entotraqual si el niño nace deprimido
• 10-20% con membrana hialina y VM displasia broncopulmonar (apgar <3) Hospitalización en UCI Neo Soporte
TAQUIPNEA TRANSITORIA ventilatorio:
Curso benigno, alteración transitoria en la adaptación respiratoria neonatal. Más frecuente en o Depende del grado de insuficiencia respiratoria:
RNT y por nacidos por cesárea. Se debe a líquido intra-alveolar el cual normalmente debiera • Oxigenoterapia en Hood si respira
absorberse rápidamente para ser reemplazado por aire. Dura alrededor de 2-3 días. Presenta solo
Taquipnea y retracciones. • Ventilación mecánica si necesita
ayuda
Diagnóstico: • Excepcionalmente se usa CPAP o
- Clínica de SDR: predomina la taquipnea (80-90-100) Antibioterapia: Ampicilina con
- Ex pulmonar normal o con leve ↓MP Gentamicina
- Rx tórax normal o líquido en fisuras (cisuritis)
- Requerimiento de O2 < 0,40
HIPERTENSIÓN PULMONAR PERSISTENTE
Tratamiento: Medidas generales, régimen cero (si la función respiratoria es muy alta o con
requerimientos de oxígeno cercanos al 40%), Oxigenoterapia en Hood según GSA, realimentar al Cuadro muy grave no tan frecuente. Persistencia de la
bajar los requerimientos de oxígeno. circulación fetal x mala adaptación; Se mantiene una alta
resistencia vascular y presión pulmonar lo que lleva a
SINDROME ASPIRATIVO MECONIAL
hipoperfusión pulmonar y cortocircuito de derecha a
Cuadro grave, que se da principalmente en RNT y post Término (>42 s, LA con meconio, apgar
izquierda a través del ductus y foramen oval
bajo). Es complicación de asfixia perinatal.
Se ve en niños con asfixia prolongada (SAM, hernia
o Asfixia intrauterina mantenida aumenta la motilidad gastrointestinal y relaja el esfínter diafragmática). Es muy raro que sea HTP persistente primario
anal paso de meconio al líquido amniótico (no ocurre bajo 34 sem).
(arterias con musculaturas engrosadas).
o La hipoxia induce esfuerzos respiratorios profundos aspiración de meconio en vías
altas, mientras que principalmente al nacer se produce la aspiración a nivel de vías bajas. Factores Hipoxia crónica y aguda, Acidosis e hipercapnia
10-15% de los partos tienen meconio en el LA, 50% hace SAM. Mortalidad 40%. Predisponentes Hipoplasia pulmonar (hernia diafragmática)
Uso prenatal de inhibidores de prostaglandinas
Antecedentes Embarazo prolongado, SFA
Clínica SDR inicio precoz, presencia de meconio en LA , en la piel del niño, Clínica SDR con CIANOSIS que no cede con aumento de la FiO2 (dx dif de
SFA, alteración LCF. CC cianótica)

Ex. Físico Enfisematoso, hiperinsuflado, con ↑ diámetro AP, MP↓ con Ex. Físico Enfisematoso, hiperinsuflado, con ↑ diámetro AP, MP↓ con
estertores húmedos. estertores húmedos.

Rx Tórax Opacidades irregulares de aspecto nodular o cordonal, Rx Tórax Descarta SAM, neumonía o hernia diafragmática
hiperinsuflación, puede haber neumotórax Ecocardio doppler Descarta CC, muestra cortocircuitos por ductus y foramen oval.
Permite medir la severidad de la HTP.
pH y GSA Hipoxemia e hipercapnia.
Considerar otras complicaciones de la asfixia: encefalopatía hipóxico-isquémica (convulsione sy
alt. Conciencia), Insuf. Renal aguda (pre o renal), Shock cardiogénico.

Tratamiento
Prevención: Buen control del embarazo
 Medidas generales: Monitoreo – Corrección de acidosis – Hipoglicemia –
Hipocalcemia – Alt. Coagulación,
 Atención inmediata: Aspiración de vía aérea superior antes de la primera respiración
(boca y luego nariz) luego ventilar y reanimar.
Tratamiento: DISPLASIA BRONCOPULMONAR
Prevención:
 Prenatal: evitar factores predisponentes Alteración crónica de función pulmonar en niños que
(solucionar asfixia) han requerido VM, y Necesidad de O2 por periodos
 Postnatal: largos.
o Buena reanimación Incidencia: de 10 a 20% de RNPT que requieren VM,
cardiopulmonar aspiración nariz boca de Mayor frecuencia a menor Peso Nacimiento y edad
inmediato. Tb Tráquea. gestacional
o Hospitalizar en UCI y medidas
generales
 Manejo de factores que aumenta la RVP
(pasa que se iguales las presiones y los shunts se cierren)
 Corregir hipoxemia y acidosis
Diagnóstico:
 Oxigenoterapia
• Fi02, para mantener pa02 > 50, ojala cercana a 100mmHg
• VM si es necesario con sedación para evitar lucha contra el respirador
• Evitar o corregir hipotensión sistémica Antecedentes RNPT, VM y O2 x más de 24 hrs
• Volumen o drogas vasoactivas (dopamina, dobutamina, a veces Criterios de Displasia Requerimiento de 02 por más de 28 días
adrenalina) 02 desde el nacimiento hasta más de 36 semanas de edad
• Se busca lograr vasodilatación pulmonar Vasodilatadores: gestacional corregida (significa un mes más con 02)
o La mayoría de los vasodilatadores son sistémicos y pulmonares, por lo que el desbalance entre
la presión pulmonar y sistémica sigue. Rx Tórax Panel de abeja
• Oxido nítrico inhalado: o Único vasodilatador pulmonar selectivo. Imágenes radiolucidas (hiperaireadas) similares a fibrosis
o Efectividad del 60% en casos que no responden a terapia habitual pulmonar
• ECMO (oxigenación extracorpórea de membrana) o Sistema de pH y GSA Hipoxemia e hipercapnia.
circulación estracorporea. Se salta el corazón y los pulmones o
Ultima alternativa.
Tratamiento: o Usar cada vez menos la VM por traumas debidos a la presión y volumen o Si
respira solo usar oxigeno en ambu o CPAP o Si no respira solo VM y O2 a la menor
concentración y tiempo posible o Diureticos si hay edema pulmonar o Broncodilatadores
(por signología obstructiva) o Corticoides (solo para sacar al niño del respirador): Dosis bajas
y tiempo corto o Kinesioterapia respiratoria: Son niños hipersecretores o Saturometria de
hemoglobina pre alta continua por 12-24 hrs: indic. O2 domic.
o Control broncopulmonar post alta o Vacunación:
prevenar, neumo 23, antiinfluenza El RN contrae la
infección tras su paso por el canal de parto.
- Etiologías: Str. Grupo B, E. Coli, Listeria, Chlamydia, Mycoplasma. Virales
(VHS,CMV,Ruebola, Influenza, etc)
- Clínica: SDR precoz (polipnea, quejido, cianosis; crépitos y disminución del MP.
- Antecedentes: Infección Materna, RPM, Cultivo vaginal, meconio, LA mal olor.
- RxTorax: Infiltrados, condensaciones, broncograma aéreo, atelectasias, derrames
pleurales
- Se pueden tomar cultivos de aspiración traqueal , Hemocultivos. Pedir Hemograma
(leucocitosis, leucopenia y desviación izquierda) - Tratamiento:
- Medidas generales (CSV y estabilidad del medio interno, soporte ventilatorio y
hemodinámico) La hipoxia produce una sucesión de eventos:
- Ingreso a UCI 1) Período inicial de respiraciones profundas (boqueo)
Cese de los movimientos respiratorios: Apnea primaria, hay cianosis pero el tono muscular está
- Tto. ATB: Ampicilina + AMG, y reevaluar según antibiograma.
conservado. En este momento la respiración puede reiniciarse en la mayoría de los casos con
- Soporte ventilatorio necesario. estímulos táctiles y administración de O2.
Si la asfixia continúa se produce:
2) Período de respiraciones profundas y jadeantes
Apnea secundaria que se manifiesta como cianosis y palidez, hipotensión y ausencia de tono y
reflejos. En este periodo en RN responde a estímulos y puede fallecer si no se inicia
oportunamente ventilación asistida con oxígeno.
3) Diving Reflex: Hay disminución y redistribución del débito cardíaco privilegiándose el flujo

NEUMONIA CONNATAL ASFIXIA NEONATAL


Síndrome caracterizado por Depresión Cardiorrespiratoria, Cianosis, Palidez, secundaria a hacia cerebro, corazón , suprarrenales y placenta (feto), en detrimento del flujo hacia los
hipoxemia y/o isquemia tisulas pulmones, riñones, intestino y músculo esquelético.
Causa importante de secuelas motoras y cognitivas. La asfixia fetal produce FMO y su sintomatología depende del grado de adaptación individual del
Se caracteriza por hipoxemias e hipercapnia, además de acidosis. órgano afectado.
Clínica de Sarnat
Criterios Diagnósticos (AAP y ACOG): mín. 2 Grado I Grado II Grado III
- Apgar <3 a lso 5 minutos
Conciencia Hiperactivo Letargia Estupor coma
- pH en sangre de cordón <7
Tono muscular Normal hipotonía flaccidez
- Manifestaciones sistémicas de asfixia incluyendo Encefalopatía Hipóxica
Isquémica Reflejos Aumentados disminuidos ausentes
- Antecedentes SFA Reflejo de moro Aumentado Débil-incompleto ausente
Etiología:
Reflejo de succión Débil Débil-incompleto ausente
- 90% intrauterino: 70% intraparto y 20% preparto - 10% neonatal.
Alteraciones del intercambio gaseoso a nivel Alteraciones del flujo placentario Convulsiones Raras frecuentes infrecuentes
placentario - hipertensión arterial con toxemia EEG Normal anormal anormal
Alteraciones siempre agudas gravídica: lo mismo que la eclampsia
Duración 24 horas 2-14 días Horas o semanas
- desprendimiento de placenta - hipotensión materna:
- placenta previa sangrante accidente, crisis convulsiva, eclampsia. - Grado I: buen pronostico, sin secuelas va a durar 24 hrs y estar
- prolapso del cordón - alteraciones de la contractilidad - Grado II: 20 a 30% secuelas
- circulares irreductibles uterina. - Grado III: 50% de mortalidad y 99% de secuelas.
- posmadurez: mujer adulta, se altera la
circulación y por lo tanto la entrega de SECUELAS:
oxígeno. SNC: Parálisis Central y RDSM variable
Cardiovascular: Isquemia miocárdica, insuficiencia cardiaca, cardiomegalia,
congestión pulmonar.
Otras causas Respiratorio: Hipoxemia, hipercapnia y acidosis. HTPulmonar. Riñón y vías
- distocias de presentación: parto en podálica, presentaciones transversas urinarias: Necrosis Tubular Aguda, Insuf. Renal Digestivo: Disminuye el
- asfixia materna transito intest, ECN, úlceras.
- infecciones intrauterinas – diabetes graves – eritroblastosis fetal Hemato: Leucopenia, leucocitosis, trombocitopenia, CID.
Apoyo Diagnóstico
- ecografía cerebral
- TAC Manejo
DCR LEVE - Observación por unas horas
- EEG
- examen neurológico precoz y al alta - CSV por 4-6 hrs
- isoenzimas cerebrales y cardiacas - Si está asintomático, regresa con su madre
- pruebas de coagulación, ELP, calcemia, nitrógeno ureico, GSA, hemograma. DCR MODERADA - Observación por 24 – 48 hrs
Clasificación DCR (depresión cardiorrespiratoria) - Si hay compromiso neurológico hospitalizar y EEG urgente
DCR Leve moderada severa
- Postergar alimentación hasta estabilizar la parte
SFA si si si cardiovascular, se re-establezca el reflejo de succión y se
APGAR (1´y a los 5´) <3/>7 3-5 a los 5´ <3 a las 5´ ausculten ruidos intestinales.

pH >7,1 <7,1 <7,0 DCR SEVERA - Hospitalizar en UCI


- Conectar a VM
síntomas no no Si (EHI)
- CSV continuo
- Restricción de líquidos por riesgo de edema
- Controlar glicemia, calcemia, anemia, acidosis metabólica.

- Uso de vasoactivos o anticonvulsivos


- Hipotermia terapéutica: 34°C por 72 hrs máx, debe
iniciarse antes de las 6 horas de vida (disminuye el
metabolismo).
- Removedores de Radicales libres (alopurinol)
- Bloqueadores de calcio
Mal pronóstico
- Grado y duración EHI, Convulsiones precoces y prolongadas de difícil manejo, EEG y eco cerebral anormales, Encefalopatía grado III
- Insuficiencia cardiorrespiratoria
- Examen neurológico anormal al alta. - Apgar <3 en los 10 minutos.
ALGORITMO DE REANIMACION NEONATAL
SEPSIS NEONATAL Str. Grupo B Nosocomial, Comunit. o Materno
Síndrome clínico con manifestación de infección sistémica durante las primeras 4 semanas de E. coli St. Epidermidis
vida que debe ser confirmado con al menos un hemocultivo que se encuentre positivo (el cultivo Lysteria monocytogenes E. coli
puede ser de sangre o LCR). Otros G- Klebsiella
*Infección Connatal: Síndrome clínico con manifestación de infección sistémica durante las St. Aureus, Cándida, E. coli, Klebsiella.
primeras 4 semanas de vida que no necesita cultivos. *Sepsis Clínica: SIRF asociado a infección Clínica : Inespecífica y Variada
sospechada o conocida Apneas
SIRF: Rechazo alimentario y regurgitación
Temperatura: Inestabilidad térmica; >37,9° O <36° Distensión abdominal
Glóbulos blancos: Debe tener 1 de los siguientes; Dificultad respiratoria
- Leucocitosis: G.B. > 34.000 (minsal : >25.000 >20.000 PT) Hipotonía y Letargia
Inestabilidad térmica
- Leucopenia: G.B. <4.000 (minsal : <6.000)
Sepsis neonatal precoz: Abrupta, con falla multisistémica, distress respiratorio severo, cianosis
- >10% Neutrofilos inmaduros y apneas. Antecedentes de RPM y Coirioamnionitis Sepsis neonatal tardía: Subagudo,
Otros: insidioso, alteraciones hemodinámicas, desaceleración frecuencia cardiaca, aumento
• Taquicardia persistente >2 D.E. o > percentil 90 (>180´) parámetros ventilatorios. -Las nosocomiales (Fcte stafilo epidermidis) suelen ser en pacientes
• Bradicardia < percentil 10 (<100´) prematuros, deteriorados, conectados a ventilador mecanico que se descompensan. -
• Taquipnea > 2 D.E. (>60´) + (quejido o retracciones o desaturación) Comunitarios, descontando resfríos y otras infecciones virales, la manifestación más temida es
• PCR > 10mg/dl (1mg/L). GOLD ESTÁNDAR Es muy caro la meningitis (manifestación tardía de la listeria y del streptoco grupo B). Dg es difícil, ya que
Sepsis Precoz: aquella infección con Cultivo + en sangre o LCR las primeras 72 hrs (0-6 días habitualmente no se obtienen cultivos +, orientan al Dg PCR alta y leucocitosis.
MINSAL) DgD.
Sepsis Tardía: Aquella que se produce entre las 72 hrs y los 18 días (7-28 días - Convulsiones c/síntomas autonómicos
MINSAL) - Sobredistensión pulmonar
FACTORES DE RIESGO: - Hipoglicemia o hipocalcemia severas
- Deficiencia Inmunológica (fagocito, Igs, LT, NK) - Hipotermia o sobrecalentamiento
- Prematurez - Dolor Agudo
- Corioamnioitis - Efectos de drogas que se le haya puesto
- RPM – RPO - Poliglobulia
- Inmadurez Vasomotora del Prematuro - Ductus Sintomático.
Etiología
Precoz Tardía
Estafilococo aureus Vancomicina o Clindamicina
Otros usados eb MO multiresitente Ciprofloxacino (+ estudiado).
Hemograma Leucitosis o Leucopenia Linezolid
RAN <1500 (-1000 es leve, -500 mod <500 severa) Daptomisina Colistin
Puede darse trombocitopenia

PCR Poca utilidad de detección, pick tardío. Pedir entre 6-12 hrs
Sirve como marcador de tto. Exitoso
Alta si >10mg/dL
Procalcitonina Sin utilidad en sepsis precoz por valores maternos. Utilidad en sepsis LABORATORIO
tardía. Peak a los 12 hrs.
Duración Tto
IL-6 e IL-8 Elev. precoz, se eleva los primeros 5 minutos y dura 1 hr. Sepsis clínica comprobada con cultivos (+) en sangre o LCR + mín 10 días
Hemoculivo Gold Standard. Positivo a partir de las 48 hrs. Baja sensibilidad. Sepsis clínica con hemocultivo (-) (más común) 7 días
HC (-) en 48 hrs y sin clínica Suspender tratamiento
Reacción Verdadero Gold Standard Prevención
Polimerasa en Muy caro
- Lavado de Manos
Cadena Obtiene patógeno en menos de 8 hrs
- Nutrición Precoz
Busqueda de Cultivo Urinario(tardía), no pedir Aspiración traqueal, pobre valor cultivo <37 semanas asintomático c/ FR: madre: Corioamionitis clínica, RPM >18 hrs o
foco aspirado gástrico. PL en pacientes con sospecha de sepsis tardía o Sospecha de meningitis Ampicilina + Cefotaxima Profilaxis ATB inadecuada
manifestación neurológica. Sospecha de SMR Agregar Vancomicina por lo tanto: Vancomicina + AMG - HC(-) + niño bien + Lab
normal suspender ATB y alta (suponiendo que Sospecha infección por catéter Gentamicina +
Vancomicina cultivos llegan en 48hrs).
-HC (-) + niño bien + Lab alt. (PCR alta,etc) continuar ATB hasta 5 días si madre recibió profilaxis ATB.
-HC (+) continuar tto ATB. Si es sepsis tardía necesitará P.L.
>37 semanas asintomático con FR madre Corioamnionitis
- HC (-) + niño bien + Lab normales suspender ATB y alta en 48 hrs.
-HC (-) + niño bien + Lab alterados (PCR alta,etc) continuar ATB hasta 5 días si madre recibió profilaxis ATB.

- Uso de catéteres centrales


- Profilaxis antimicrobiana para SGB: o PNC G 5.000.000 ev luego
A quien tratar: 2.500.000 c/4 hrs hasta el parto.
o Ampicilina 2 g luego 1g c/4 hrs o
Por lo tanto en sepsis precoz tomar: hemograma, glicemia, PCR <24hrs y en tardía considerar urocultivo Cefazolina 2g ev luego 1 g c/8 o
y PL Clindamicina 900 mg c/8hrs en alérgicos.
Tratamiento
Primera Línea: Ampicilina + Gentamicina
Segunda Línea: Cloxacilina + Amikacuna (buena llegada a BHE)
Tercera Línea: Vancomicina + Carbapenémico
GERMEN ANTIBIOTICOS -HC (+) continuar tto ATB. Si es sepsis tardía necesitará P.L.
En la práctica si se sale anormal el laboratorio, se toman cultivo y se inicia ATB. Si
Estreptococo grupo B PNC o Ampicilina salieran después negativos los cultivos, se suspenden los ATB.
E.coli Ampicilina + Gentamicina
E- coli BLEE (+); son resistentes a aminoglicosidos. Ampicilina + Carbapenemico
ICTERICIA NEONATAL
Coloración amarilla de piel y mucosas, generalmente visible con bilirrubina total o mayor a
Bacilos gram (-); klebsiella, salmonella, proteus, Ampicilina o Cefotaxima 5 -6 mg/dL
shigella Etiología
Bacilos g – serratia, pseudomona o enterobacter Carbapenemicos • ICTERICIA FISIOLOGICA: Tras 48 hrs de vida, dura 1 semana, predominio indirecto
o Aumento de la producción gran masa GR con VM mas corta o Transporte supera
capacidad de unión a albúmina o Captación, Conjugación y excreción hepática aumento
Listeria monocytogenes Ampicilina + Gentamicina BNC o Circulación enterohepática aumento de la Reabs
>37 semanas asintomático con FR madre: RPM o Profilaxis inadecuada • ICTERICIA POR INCOMPATIBILIDAD GRUPOS ABO RH
-Si laboratorio esta normal + niño se ve bien alta en 48 hrs -Si • ICTERICIA POR LECHE MATERNA
laboratorio sale anormal tomar hemocultivo: • ICTERICIA SECUNDARIA POLIGLOBULIA O REABS DE HEMATOMAS O
-Si salen (-) + niño se ve bien alta en 48 hrs. HEMORRAGIAS
• ICTERICIA POR ANEMIA HEMOLITICAS
• ICTERICIA PATOLOGICA: clínicamente evidente en primeras 24 hrs, persistente, >1
s en RNT y >2 s RNPT
Examenes:
• Grupo sanguíneo y Rh de madre e hijo
• Test de coombs directo de Rn
• Bilirrubinemia
• VDRL de madre y Rn
• Hematocrito con frotis
• Recuento de reticulocitos

Área del cuerpo Rango de bilirrubina sérica


total ( mg/dl)

Cabeza y cuello 4-9


Tronco superior 5-12
Tronco inferior y muslos 8-14
Brazos y piernas 11-15
Piernas y plantas 15-20
<1000
Tto: 1000-1490
 Fototerapia continua salvo para amamantar, voltear c/ 4-6hrs. según
factores de riesgo. 10 mg% 12 mg% 14 mg% o Considerar: Aumento de perdidas sensibles, hipo o
hipertermia, protección ocular, aumentar aporte lácteo.
o Suspensión Exanguineotransfusión en RNPT
 Controlar bili total cada 8-12 o 24 horas(según patología)
 2 valores en descenso Peso E
 Al menos uno de ellos < o = a 13 mg % Grs. < 24 hrs
 Bajo curva de alto riesgo < 1000 En
 Si falla, fototerapia o con bili >20-25 mg/dL pasar a exanguineotransfusión <= 1 mg%/Kg
1000 - 1490 <= 1 mg%/Kg En
a) Factores de riesgo alto: Causa hemolítica, coombs +, RNPT 35-36 semanas, Poliglobulia, Cefalohematomas
extensos, equimosis múltiples extensas,sepsis y asfixia. 1500 - 1990
14
b) Factores de riesgo moderado: Hijo anterior recibió fototerapia, <37 semanas, lactancia no exitosa, baja de peso
2000 - 2800 16
importante, cefalohematomas pequeños, incompatibilidad O/A, O/B, coombs (-), caso social (ruralidad). c) Factores de
riesgo bajo: RNT sano y s/ otros factores
Esquema de tratamiento en ictericia neonatal de RN de término según edad y nivel de bilirrubina total.

Edad solicitud fototerapia fototerapia Exsanguineo de estudio intensiva transfusión

Esquema de tratamiento con fototerapia en RNPT

Peso (grs) <24hrs 25-48hrs >48 hrs

todos >=10 mg% >=14 mg% >=18 mg%


25-48 >=10 mg% 13-15 mg% >=15 mg% >=20 mg%
49-72 >=13 mg% >=16 mg% >=18 mg% >=25 mg%
>72 >=15 mg% >=18 mg% >=20 mg% >=25 mg%

CAUSAS ICTERICIA PATOLÓGICA APARICIÓN PRECOZ (<24 hrs)

HEMOLÍTICA POR ISOINMUNIZACIÓN INFECCIÓN INTRAUTERINA

Antecedente Incompatibilidad Rh ABO Infección Materna


Hermanos afectados Sufrimiento fetal
Clínica Hidrops fetal Específica
Hepatoesplenomegalia Hepatoesplenomegalia
Coluria Hábito séptico
Hipercolia
Ictericia nuclear

Bilirrubina +15 mg/dL Variable


Indirecta, libre Mixta

Exámenes Específicos Coombs + Serologís


Prueba de eluido de GR IgM
Anemia Radiología
Eritroblastosis Fondo de ojo
Reticulocitosis LCR
Microesferocitosis Cultivos
Hemoglobinuria

Observaciones ABO: Paso transplacentario de IgG anti A o anti B en una madre O a un hijo A o B. Habitualmente no manifiesta
compromiso fetal, de lo contrario puede haber anemia y hepatoesplenomegalia. Coombs directo + 40-50%
Rh: Rh(D). Paso transplacentario de igG anti Rh de madres Rh (-) sensibilizadasa hijos Rh (+).
Compromiso fetal. Coombs directo (+) confirma el Dg.

CAUSAS ICTERICIA PATOLÓGICA APARICIÓN PRECOZ (2° - 7° día)

INFECCIOSA ↑CIRCULACIÓN HEMOLÍTICA NO OTRAS


ENTEROHEPÁTICA ISOINMUNE

Antecedente Infección posible o probable Obstrucciones intestinales Familiares Diabetes, toxemia, Sd.
Sepsis Íleo paralítico por drogas Anom. Morfológicas Lucey-Driscoll, anoxia
Infección localizada Enzimopenias neonatal, SDRA,
Hemoglobinopatías Poliglobulia
Déficit vit. E

Clínica Hábito séptico Específica Esplenomegalia Específica


Específica

Bilirrubina Variable Variable +12,9 mg/dL Variable


Mixta indirecta Indirecta Indirecta

Exámenes Específicos Cultivos Radiología Anemia Según etiología


Anemia Morfología GR
Leucopenia Hemoglobinas
Desviación izq. Vitamina E
Aumento PCR
LCR, orina

Ictericia Normal:
- Muy común: fisiológica
- Frecuentes: E. Hemolítica (Rh, ABO, grupos menores), Poliglobulia, Reabs. Hematomas y cefalohematomas,
prematuridad, sepsis - Menos frecuente: obstrucción intestinal, Enfermedad metabólica del metabolismo de la bilirrubina: Gilbert,
hipotiroidismo Ictericia Tardía:
- Frecuentes: Ictericia por LM, por hipoalimentación
- Menos frecuente: alteraciones hepatobiliares, atresia de vías biliares, hepatitis, ictericia por alimentación
parenteral, TORCH
CAUSAS ICTERICIA PATOLÓGICA APARICIÓN TARDIA (2° - 7° día)

OBSTRUCTIVAS PSEUDOOBSTRUCTIVAS ENDOCRINO - HEPATOPATÍAS


(atresia vías biliares; agenesia METABÓLICAS CONNATALES
intrahepática vías biliares)

Antecedente Infección materna Familiares Familiares Familiares


Infección prenatal (TORCH) Enf. Hemolítica Galactosemia Enfermedad Gilbert
Colestasis recurrente familiar Madre HbsAg+ Hipotiroidismo Enfermedad Dubin.
Frco (clopromazina) Johnson
Nutrición parenteral
Fibrosis quística

Clínica Hepatomegalia Hepatoesplenomegalia Específica Específica


Coluria Coluria
Acolia Hipocolia
Ictericia verdínica
Esplenomegalia

Bilirrubina Aumento continuado Intermitencia Variable Según tipo


Directa Directa Indirecta, Mixta Indirecta, mixta

Exámenes Ecografía Alfa 1fetoproteina Cuerpos reductores Biopsia hepática


Específicos Lipoproteina X Rosa vengala Orina LPC
Rosa bengala HBsAg T4, TSH Colecistografía oral
Exploración Qx Alfa 1 antritripsina Punción medular Bilirrubina mono y
Test meconio Sangre periférica diconjugada
Tripsina heces Radiología
Tripsina inmuno
reactiva

Tratamiento Intervención de Kasai Ecografía Específico Fenobarbital


Profilaxis colangitis Vitaminas liposolubles Vitaminas liposolubles
Trasplante hepático Específico Fototerapia
Estaño – proto -
porfirina
1. GIARDASIS
Definición Diagnóstico Clínica
Enfermedad Parasitaria  Examen deposiciones:  Síntomas: Anorexia, dolor abdominal leve e
Agente: Giardia Lamblia - Coproparasitológico inconstante, falla en crecimiento pondoestatural.
Reservorio: humano y otros - ELISA Ag en deposiciones  Signos: Fetidez fecal, diarrea (lientérica restos de
animales - PCR guardia lamblia (alto costo) alimentos, esteatorrea grasa en deposiciones).
Mecanismo de transmisión:
Contaminación Fecal
Vía de contaminación: oral Control Tratamiento
Epidemiología:  Medidas preventivas: Educación,  Metronidazol: 15 mg/kg/día en tres tomas, 7 días; 2
Prevalencia: preescolares, filtración de aguas, hervir agua si no series con intervalo de 7 días.
lactantes, está filtrada, eliminación de heces en  Tinidazol: Desde los 6 años: 200 mg/día =3 días;
inmunodeprimidos. forma sanitaria. menores de 6 años 100 mg/día.
Factores de riesgo: Mal  No está indicado la terapia familiar  Albendazol
saneamiento ambiental, como medida primaria.  Secnidazol
hábitos higiénicos
inadecuados.
2. ENTEROBIASIS
Definición Diagnóstico Clínica
Agente infectante: Test de Graham: Tocaciones con cinta adhesiva  Signos y síntomas: prurito anal y nasal, irritabilidad.
Enterobius vermicularis en márgenes perianales lectura posterior en  Complicaciones: apendicitis, vulvovaginitis,
Presentación en grupos de microscopio. salpingitis.
personas: familia, personas
que viven en asilos, Control: Tratamiento
internados, etc. Educación a pobl. Eliminación de fuentes de  Albendazol: 400 mg dosis única.
Mecanismo de transmisión: infección.  Mebendazol: 100 mg dosis única.
Contaminación ambiental. Baño diario, cambio frecuente de ropa interior,  Pamoato depirantel.
- Directa: ciclo ano-mano- ropa para dormir y sábanas.  Tratar a paciente y familia
boca. Limpieza diaria de la casa con aspiradora
- Indirecto: inhalación. durante varios días después del tratamiento de
Reservorio: Humano. los casos. Control del paciente, de los contactos y del ambiente
Reducción del hacinamiento inmediato:
Mantener limpias las instalaciones sanitarias.  Desinfección concurrente: cambio de ropa de cama
e interior de la persona infectada.
Ropa: exponerla a temperatura de 55°C durante
unos segundos, hervirla o lavarla con agua caliente.
3. PEDICULOSIS
Definición Diagnóstico Clínica
Agente infectante: Hallazgo del parásito,  Infestación comienza en región retroauricular y luego se extiende a toda la
Pediculus humanus frecuentemente la liendre. cabeza.
Capitis (cuero  Aparecen pápulas eritematosas; en casos severos se desarrollan lesiones
cabelludo) y Corporis múltiples por grataje, sobreinfección secundaria y linfoadenopatías, en
(cuerpo). cadena cervical posterior y auricular.
Epidemiología: Más Control: Tratamiento
frecuente en niños y Revisión periódica del niño. A Familia y contactos
de sexo femenino. Tratamiento de contactos
Transmisión: Por
contacto.

*Uso de cotrimoxazol 80/400 en infestación masiva (no mata liendres)


*Acidos (acético, vinagre) como coadyuvantes para despegar liendres.
4. ESCABIOSIS
Definición Diagnóstico Clínica
Invasión de la capa  Prurito de predominio Periodo de latencia de 15-50 días.
córnea de la piel. nocturno. Prurito Noct. O cuando hace calor. Región interdigital, codos, axila, muñeca, zona
Agente: Sarcoptes  Morfología y distribución genital, región glútea, cara interna de muslos rodillas y tobillos.
Scabiei var hominis. de las lesiones. En el niño es más generalizada, y tiende a comprometer también cuero cabelludo,
Epidemiología: A  Antecedentes cara palmas y plantas.
cualquier edad y epidemiológicos, tanto de 1. Surco arcarino: lesión lineal, representa el túnel de la hembra.
nivel familiares como 2. Vesicula perlada o perla escabiótica: pequeñas vesículas en la epidermis.
socioeconómico. contactos. 3. Nódulos escabióticos: lesiones nodulares induradas. Reacción de
 Ante las dudas: hipersensibilidad retardada
- Acaro test: buscar la
presencia de ácaro
bajo el microscopio
en las células
descamadas de la
piel.
- Pintar con tinta china
el surco acarino.
Tratamiento
Puede persistir el prurito por varias Todos los contactos del paciente y simultaneo
semanas después del tto. Se La aplicación del escabicida en todo el cuerpo, del cuello hacia abajo, en las uñas.
pueden usar antihistamínicos orales En niños en el cuero cabelludo también. Repetir aplicación a los 7 días.
o lubricación de la piel.
Ropa de cama y del paciente deben
ser lavadas con agua caliente y
planchadas. El lavado en seco
también mata al parasito. Parasito
muere en 4 días si no está en
contacto con el huésped, por lo
tanto las cosas que no se puedan
lavar deben ser guardadas en una
bolsa por 7 a 10 días.
Avisar al colegio u hospital para
evitar brotes.

Complicaciones Permetrina 12-24 hrs y repetir en una semana


Vaselina Azufrada: por 3 noches lavandose antes de cada aplicación y repetir en
 Prurigo escabiótico: una semana
lesiones eritemato
escamosas.
 Nodulos escabioticos.
 Infección bacteriana
secundaria por strepto
beta hemolítico

5. RGE
Definición Diagnóstico Clínica
Ascenso del contenido  Estudio radiológico: Baritado de  Vomitos y  Asma bronquial
gástrico hacia el esófago esófago, estómago y duodeno. regurgitación  Tos crónica
torácico. Para comprobar indemnidad del  Desnutrición  Laringitis
En la mayoría de los TDS.  Irritabilidad y rechazo  Neumonías por
lactantes es clara la  Esofagoscopia: visión directa de de la alimentación aspiración
presencia de mucosa esofágica. Tomar  Pirosis  Crisis de apnea y
regurgitaciones biopsias.  Dolor restroesternal cianosis
postprandiales no es  Biopsia de mucosa esofágica:  Disfagia  Síndrome de muerte
necesario hacer ningún para ver si hay esofagitis, ver la  Esofagitis súbita abortada
examen. hiperplasia de la capa basal del  Hemorragia digestiva
epitelio escamoso.  Estenosis esofágica
 Medición prolongada de pH
esofágico
Tratamiento
Los síntomas en el lactante tienden a resolverse espontáneamente en la mayoría.
En niños con desnutrición a causa del RGE debe verificarse periódicamente que el problema nutricional se esté
resolviendo, si no es así hay que plantear cirugía.
Posición en decúbito prono porque ayuda al vaciamiento gástrico y reduce el gasto energético y los episodios de
llanto.
Alimentos poco voluminosos. Se puede espesar la FL para darle una mayor densidad que disminuye el número de
episodios de regurgitación y vómitos.
No fumar en presencia de los niños.
Evitar utilizar ropa ajustada.
Medicamentos:
- Procineticos: metoclopramida 0,3 mg/kg administrados 15-20 min antes de cada alimentación 3
veces al día.
- Domperidona: 0,3 mg/kg tres veces al día.
- Betanecol
6. RESFRIO COMUN
Definición Diagnóstico Clínica
Enfermedad viral, leve. Se Clínico. Inicio repentino, CEG, obstrucción nasal, romadizo, estornudos, tos
caracteriza por el seca poco frecuente y a veces fiebre hasta 38.5°C
compromiso de vías aéreas En <1 año puede haber trastornos de alimentación y el sueño.
superiores y dura aprox. 2-5
días. Complicaciones Tratamiento
Sinusitis, otitis media, adenoiditis, NO ATB.
bronquitis obstructiva, neumonía Descongestionantes solo en >3m.
Reposo, líquido y antipirético (paracetamol 5-10 mg(kg (gtas) o 10-15
mg/kg (comp) SOS. Si la fiebre persiste o dificultad respiratoria acudir
a centro médico
7. FARINGOAMIGDALITIS AGUDA BACTERIANA
Definición Diagnóstico Clínica
Inflamación de faringe y/o Clínica Inicio brusco con decaimiento, dolor de cabeza, dolor de garganta
amígdalas provocada por Frotis faringeo y fiebre alta (>38.5°C). Se puede observar enrojecimiento y
Str. Betahemolítico grupo A,. aumento de las amígdalas y en ocasiones acompañarse de
manchas blancas y dolor en ganglios submaxilares.
Complicación Tratamiento
Abceso periamigdalino y retrofaringeo Reposo
GNF Post Str Antipiréticos
Endocarditis ATB: PNC Benzatina >25 kg: 1.200.000 UI <25 kg: 600.000 UI o
Amoxicilina 50-80 mg/kg/día c/12 hrs por 7 días.
8. OTITIS MEDIA AGUDA
Definición Clínica Tratamiento
Inflamación aguda del oído Intensos dolores de oídos de comienza Es necesario acudir a un centro médico y, en otitis con supuración
medio y trompa de brusco, fiebre y, en lactantes, irritabilidad y resistentes al tratamiento habitual, se recomienda realizar un
Eustaquio, que puede afectar como manifestación del dolor. Se puede cultivo de secreción en laboratorio.
a uno o los dos oídos. Esta presentar supuración del oído y dificultad Aplicar calor local alivia el dolor y no se debe taponear el conducto
inflamación puede ser para escuchar. auditivo externo.
causada por virus Puede llegar a necesitar una intervención Si hay fiebre  en reposo
respiratorios o bacterias quirúrgica para drenar el fluido desde el oído 3 o más episodios en un año, secreción persistente del oído o
Corresponde a una medio. hipoacusia por más de 3 semanas  consultar un médico otorrino.
complicación del resfrío S/ fiebre y BCG  otitis ya no es contagiosa.
común, ataca principalmente Paracetamol SOS y Amoxicilina.
a niños menores de 3 años.
9. LARINGITIS OBSTRUCTIVA
Definición Clínica
Inflamación aguda de la Evoluciona rápidamente, causando disfonía o afonía, tos disfónica (tos de perro), estridor inspiratorio, diferentes
laringe, generalmente de grados de dificultad respiratoria y fiebre moderada.
origen viral, que provoca Esta infección puede ir de moderada a intensa, llegando al agotamiento del paciente y disminución de síntomas
diversos grados de respiratorios
obstrucción. Tratamiento
Si los síntomas son leves se pueden manejar en el hogar, pero es necesario acudir a un centro de salud para recibir
las instrucciones adecuadas. El personal de salud decidirá si el paciente requiere hospitalización si el paciente
empeora.

10. BRONQUITIS AGUDA OBSTRUCTIVA


Definición Clínica
Enfermedad generalmente Tos de intensidad variable, fiebre moderada, respiración agitada, sibilancia audible (ruido en el pecho), dificultad
producida por virus, respiratoria y para alimentarse.
caracterizada por la Esta enfermedad se presenta mayoritariamente en niños menores de dos años. En niños de tres meses o menos
obstrucción de bronquios y puede haber episodios de apnea.
bronquiolos. Tratamiento
Se debe acudir a un centro médico, donde se definirá la gravedad del caso de acuerdo a una escala de puntajes
predeterminados que definirán el tratamiento a seguir y se instruirá a la persona responsable del cuidado del
niño, a fin de evitar complicaciones posteriores, como por ejemplo, neumonía.

11. NEUMONIA
Definición Complicación Clínica
Enfermedad de origen viral Por sus múltiples complicaciones y riesgo Sus síntomas más comunes son tos, fiebre y dificultad
o bacteriano, provoca una de apnea y paro cardiorespiratorio, es respiratoria, pudiendo presentarse dolor abdominal, puntada en
inflamación aguda del necesario acudir rápidamente a un centro el costado, vómitos, calofríos y expectoración. En el lactante se
parenquima pulmonar médico, realizar una radiografía de tórax y aprecia compromiso del estado general, rechazo al alimento,
Hemograma con VHS y, en niños menores de quejido, hundimiento debajo de las costillas y aleteo nasal
3 años, hospitalizar. Una secuela importante Tratamiento
de la neumonía son las bronquiectasias, es Dado que es difícil establecer la etiología de la infección, la
decir, zonas de los bronquios que quedan neumonía se trata con antibióticos, que deben ser prescritos por
dañadas, aumentando la posibilidad de un profesional médico.
infecciones a repetición.
12. DERMATITIS SEBORREICA
Definición Diagnóstico Clínica
Trastorno autolimitado Importante: Placas eritematosas de un tinte rosado, con morfología
benigno, de causa  Edad del paciente: < 3 meses redondeada o incluso circinada, con bordes bien definidos,
desconocida, que afecta  Localización de las lesiones: Cuero aisladas o confluentes, que se cubren de una descamación
áreas seborreicas. Proceso cabelludo, frente, párpados, surco amarillenta de aspecto grasiento. No se acompaña de síntomas
inflamatorio que se nasogeniano. generales ni prurito.
presenta con pequeñas  Ausencia de síntomas y Suele afectar cuero cabelludo, frente, pabellones auriculares,
pápulas cubiertas por descamación untuosa. cejas, pliegues retroauriculares y nasogenianos, párpados y
costras, localizadas en cuello. Se puede extender a tronco, con predominio de zona
regiones seborreicas. periumbilical, axilas, ingles, tórax anterior y espalda. Afectación
Dura entre 2-10 semanas de zona del pañal es muy frecuente, especialmente en zonas
inguinales, pliegue interglúteo y genitales externos.
Tratamiento
 Evolución benigna y tendencia a curación espontánea.
 Corticoides tópicos: Tto de elección, no más de 3 o 4 días.
 Corticoides orales: 0,5 mg/kg/ día de prednisona, asociado a corticoterapia tópica durante pocos días.
 Queratoliticos: En las lesiones de cuero cabelludo, se precisa la disolución del componente costroso o
queratósico. Aplicación tópica de ácido salicílico en vaselina al 3 o 5% o bien de agentes oleosos como
aceite de oliva. Tras la aplicación durante 1 o 2 hrs se procede a retirar las escamas reblandecidas.
Luego se puede aplicar una crema de corticoide de mediana potencia.
 Chamous cn azufre
 Antifungicos: ketoconazol 2% aplicado 2 veces al día durante 10 a 14 días.
13. DERMATITIS ATOPICA
Definición Clínica
Es un conjunto de DA Lactante:
anomalías y lesiones  Empieza hacia los 5 meses
cutáneas que aparecen en  Algunos desarrollan eczema seborreico, que de aspecto gradual va adquiriendo el aspecto de la dermatitis.
los individuos atópicos  Localización más en cara, respetando alrededor de los ojos, nariz y boca.
 Son pápulas o placas eritematosas y edematosas, muchas con erosiones, exudación y costra.
DA infantil:
 Inicia a los dos años.
 Lesiones características sobre todo en las flexuras, en especial en codos y rodillas.
 Lesiones eczematosas con vesículas pero el intenso prurito hace enseguida que se transformen en
erosiones, con exudación y formación de costras.

Diagnóstico Tratamiento
Se basa en la clínica. Muchos tienen  No es una alergia
eosinofilia periférica y elevada de la IgE.  Deben evitarse todas las circunstancias y elementos que
causan prurito
 Duchas cortas con agua no excesivamente caliente y con un
gel de pH acido.
 Luego crema emoliente
 La base del tto son los corticoides
 En niños > 2 años los inhibidores tópicos de la calcineuria
suponen una alternativa a la terapeútica con esteroides.
14. DERMATITIS DEL PAÑAL
Definición Diagnóstico Clínica
Proceso cutáneo irritativo e Clínico y en caso de sobreinfección  Puede ser dermatitis irritativa de las zonas convexas,
inflamatorio debido a candidiasica o bacteriana debe tomarse irritativa lateral, irritativa por químicos, irritativa
especiales condiciones de cultivo de las lesiones sospechosas perianal.
humedad, maceración,  Tipo más frecuente de dermatitis, en el 70-80% de los
fricción y contacto con casos es de contacto irritativa al amoniaco (urea) que
orina, heces y otras están entre la piel y el pañal. En este caso respeta los
sustancias. pliegues.
 Entre las causas infecciosas la más frecuente es la
candidiasis. Se puede encontrar el impétigo buloso. Con
poca frecuencia la sífilis congénita podría producir
lesiones semejantes. No suele respetar pliegues y suele
tener pustulas.
 Dermatosis inflamatorias: también puede tener
expresión en la zona del pañal; entre ellas están la
dermatitis seborreica, la dermatitis atópica y la psoriasis
del lactante.
Complicación Tratamiento
 Puede haber sobrinfección por cándida  Modificación del pH de la piel a base de sediluvios en medio
albicans que se manifiesta como ácido: 5 litros de agua tibia con 4 cucharaditas (20 ml) de
eritema intenso de tinte rojo violáceo acido acético (vinagre blanco) y a continuación la aplicación de
con formación de pápulas y pápulo – tintura de eosina.
pústulas d extensión perif´rica.  En dermatitis moderadas o severas dar corticoids atópicos de
 Granuloma glúteo complicación de DP baja o moderada potencia.
asociada a utilización de corticoides  Sobreinfeccion por cándida: antifúngico tópico.
tópicos de alta potencia.  Sobreinfección bacteriana: ATB tópico
15. GRANULOMA UMBILICAL
Definición Tratamiento
Pequeño nódulo de tejido
friable que puede medir  Los pequeños granulomas pueden ser tratados con aplicaciones tópicas de nitrato de plata, granulomas
hasta 1 cm, que aparece en de mayor tamaño o los que no mejoran con tto requieren de resección quirúrgica.
el lecho del ombligo con una  Cuando no ceden con 2-3 aplicaciones de nitrato de plata o tienen un color rojo cereza, debe hacernos
superficie lisa o irregular y pensar en un pólipo de mucosa intestinal por vestigio del conducto onfalomesentérico.
frecuentemente
pedunculado de color
rosado.

16. ONFALITIS
Definición Tratamiento
Se presenta con eritema Pueden ser extremadamente graves, provocando sepsis, debido a la permeabilidad de los vasos umbilicales que
umbilical, edema y secreción persisten hasta aproximadamente los 20 días de vida, por lo que su tratamiento debe ser tomar muestra para
maloliente. La edad identificar el germen y antibiograma e indicar tratamiento antibiótico.
promedio de presentación
es de 3° o 4° día de vida.

17. IMPETIGO
Definición Clínica
Infección cutánea superficial No ampolloso: más frecuente, se inicia en zonas de traumatismo mínimos. Pápulas eritematosas que
causada por Streptococcus rápidamente evoluciona a una vesícula que se rompe y forma un exudado que se seca formando costras
Pyogenes, St. Aureus o amarillentas merecerías.
ambos. Altamente
contagioso, se propaga Ampolloso: predomina en niños pequeños. Causado por S.aureus. Aparecen ampollas superficiales de paredes
fácilmente por contacto lisas contenido transparente. Se rompe con facilidad deejando una superficie erosiva eritematosa con apariencia
directo. exudativa.
Diagnóstico Tratamiento
Es clínico y la confirmación se hace  Buena higiene general
mediante gram y cultivo del contenido  Eliminación de costra y la utilización de antisépticos
líquido o de la superficie de la lesión. tópicos.
 Tratamiento antimicrobiano tópico: acido fusico y
mupirocina.
 Antibióticos sistémicos: impétigos con multiples lesiones,
localizaciones difíciles de tratar, si afecta a varios
miembros de la familia.
18. PULICULOSIS
Definición Diagnóstico Profilaxis
Es la infestación de pulgas, es Parasitosis propia de condiciones higiénicas  Tratamiento de los animales y del medio
un ataque hematofágico, de desfavorables. ambiente.
parásitos externos  Diferenciar picadura de pulgas de las de  Mantener buena higiene personal y de la
permanentes y corrientes de otros insectos. familia.
perros y gatos, cuyas formas  Lesión papular con una mancha roja al  Asear rigurosamente.
inmaduras viven en el medio centro.
ambiente.
La saliva contiene sustancias
que provocan hipersensibilidad
(dermatitis alérgica).
19. PIE PLANO
Definición Examenes Clínica
Disminución o ausencia del  Radiografía de tórax:  Preescolar o escolar con caminata torpe, caídas
arco longitudinal o disminución  Diagnóstico de pie plano frecuentes.
de la bóveda plantar.  Rx de ambos pies con proyección  Costumbres sedentarias, desgaste de zapatos por el
 Normal hasta 4-5 frontal y lateral con apoyo bipodal lado medial
años (rx con carga)  Genu valgo
 Abundante tejido  Mediciones utilizadas  Ausencia de arcos longitudinales, huella plantar sin
adiposo plantar.  Angulo astrágalo calcáneo arco, antepie abducido
 Laxitud articular y aumentado (N:20- 40°) y  Tres maléolos, pie flexible e indoloro.
ligamentosa astrágalo medializado. Tiene 3 grados
exagerada.  Angulo astrágalo- 1er MTT
 Debilidad muscular negativo (N:0-15°)
relativa  Angulo costa bartani Cuando derivar: Todo pie plano sintomático o doloroso, severo
 Pie plano laxo infantil aumentado (N:120-130°) (grado III), rigido, >4-6 años.
(98% de los casos)
Alteración musculo ligamentosa

20. TESTICULO EN ASCENSOR


Definición: Permanece en el conducto inguinal y desciende al escroto espontáneamente o con maniobras de vasalva. Es por un reflejo
cremastérico exagerado. No es patológico pero requiere segumiento.
21. FIMOSIS
Definición Complicación Clínica
Es la estrechez del prepucio que  Balanitis: infección del espacio La fimosis fisiológica el 90% se resuelve los primeros
impide su retracción y con esto balanoprepucial, higiene deficiente de la 3 años de vida.
la exposición del glande. zona. La fimosis secundaria a la cicatrización del prepucio,
Presente en casi todos los RN  Parafimosis: inflamación aguda del tracción brusca, infecciones balanoprepuciales a
cumpliendo una función prepucio, reducción forzada de una repetición, balanitis xerótica.
principalmente protectora. estrechez anular.
 Itu
Tratamiento Derivar
 Absolutas: balanitis xerotica obliterante,  Si persiste sobre los 2 años sin tendencia a
balanitis recurrente. mejorar.
 Relativas: ITU en < 1 año, ITU recurrente,
uropatias obstructivas de las vías urinarias.
 Circuncisión y corticoides tópicos
(betametasona 1% en crema por 4-6
semanas)
22. CRIPTORQUIDIA
Definición Clínica
Es la falta de descenso Causa multifactorial: persistencia dl canal  A adecuadas condiciones de tranquilidad y temperatura
testicular permanente peritonovaginal, mala implantación del  Testículo no palpableultrasonido inguinal descarta
desde el retroperitoneo al aubernaculum testis, anomalías epidídimo presencia de tejido testicular.
escroto en su trayecto de testiculares. Testículos no palpables bilaterales examen cromosómico y
descenso normal. endocrinológico

 Al diagnóstico cirugía, edad de indicación es entre los 12-


18 meses
23. SINEQUIA VULVAR
Definición Clínica
Unión del borde del  Asintomática
introito de los labios  Irritación local
menores por su borde  Infección vaginales o urinarias
libre, lo que origina una  Retención urinaria
cicatriz de grosor variable.  Antecedentes: dermatitis del pañal, vulvitis, goteo postmiccional, ITU.
Condición adquirida, por Tratamiento
estado hipoestrogénico  80% resuelve espontáneamente
fisiológico de edad  Seguimiento clínico cada 3-6 meses hasta 8-9 años
prepuberal e irritación  Vaselina tópica: masaje digital suave, en niñas con pañales lubricar el introito en cada muda después del
crónica de la región vulvar. aseo.
 Estrógenos tópicos: estradiol o estriol en el sitio 3 veces al día hasta que se separe, máximo 15 días.
Derivar: cuando sea persistente en pubertad
24. HERNIA UMBILICAL
Definición Clínica
Defecto en el cierre del  Pequeña tumoración blanda, del tamaño de una cereza que se reduce fácilmente.
anillo facial del ombligo  Aumenta con maniobras de valsalva.
que permite la herniación  Indoloras
de vísceras o epiplón.  Estrangulación
Frecuente en la lactancia y
niñez. Tratamiento
La mayoría se resuelve si persiste sobre los 5 años de edad
espontáneamente

25. URACO PERSISTENTE


Definición Diagnóstico
El uraco representa a los restos obliterados de la alantoides y cuando  Se sospecha por la salida de orina por el ombligo
se presenta una falla en la obliteración se habla de un  Se hace evidente una vez que cae el cordón umbilical
uraco peristente.
Luego de la semana 12 de gestación, el cordón se alarga y los restos
de la alantoides se obliteran constituyendo el uraco. Tratamiento
Cuando existe paso de orina se trata de una fístula del uraco.  Quirúrgico
Entre el ombligo y la vejiga por encima del peritoneo y por debajo de  Extirpación completa del uraco con resección y cierre de la
la unión de los músculos abdominales (los rectos) en la línea media porción vesical
(línea alba).  Sin tratamiento quirúrgico, la piel periumbilical se inflamará y
presentará los signos clínicos y físicos de onfalitis por orina.

26. HIDROCELE
Definición Diagnóstico Clínica
acúmulo de líquido Signos como consistencia renitente, Aumento de volumen fluctuante a nivel escrotal, siendo de mayor
peritoneal a nivel de la transiluminación positiva, color azuloso, tamaño durante el transcurso del día.
vaginal testicular en el ausencia de dolor y de signos inflamatorios El hidrocele del cordón o quiste del cordón en hombres, o quiste de
escroto, por una fina locales y hallazgos típicos al ultrasonido Nuck en mujeres, corresponde a una persistencia del canal
persistencia del canal permite hacer el diagnóstico diferencial. peritoneo-vaginal a nivel del cordón con cierre proximal y distal. En
peritoneovaginal general es asintomático y también se trata en forma expectante,
operándose sólo en niños mayores de un año
Tratamiento
El tratamiento es conservador, con cierre espontáneo del proceso vaginal, especialmente durante el primer año de
vida (95%). Cuando ocurre el cierre, el hidrocele deja de ser fluctuante y se transforma en hidrocele no
comunicante, signo de buen pronóstico de resolución espontanea. Si persiste el hidrocele o aparece después del
año, tiene indicación operatoria, que consiste en el cierre del canal peritoneo vaginal persistente.
27. DISPLASIA DE CADERA
Definición Diagnóstico Clínica
Patología que se caracteriza Radiografia Examen físico
por la anormal relación • Ángulo acetabular >40°  Signo Ortolani-Barlow (+)  subluxación y luxación
entre la cabeza femoral y el • Ausencia de ceja cotiloidea  Abducción limitada <45°
acetábulo, además de la Ecografía  Otras deformidades congénitas (pie bot, tortícolis
sobrecarga acumulativa Techo óseo cubra < 50% de la cabeza congénita)
que daña las estructuras femoral.
articulares y que puede Necesidad de imágenes se adelanta y
causar artrosis. debiera realizarse una ecografía entre la 2°
Incluye la luxación, y
subluxación y la 6° semanas de vida, manteniéndose la
inestabilidad o luxabilidad necesidad de una radiografía de control a
en que la cabeza puede los 3 meses
entrar y salir de la cavidad
cotiloídea, además de una
serie de signos radiológicos
que indican una
inadecuada formación del
acetábulo.

Factores de Riesgo Tratamiento


 Sexo femenino (mayor laxitud  Arnés de Pavlik
ligamentosa por altos niveles  Hasta los 6 meses
estrogénicos)  Durante todo el día
 Presentación podálica  3 meses Rx  Ok (se continua usando durante 1 mes
 Oligohidroamnios (12hrs al día))
 Tortícolis congénita Se realizan controles mensuales con rx pelvis hasta el alta
28. ESCOLIOSIS
Definición Diagnóstico
Desviación lateral de la columna vertebral. La  Dolor de espalda o lumbago.
desviación lateral debe tener una magnitud  Sensación de cansancio en la columna después de pararse o sentarse por mucho
mínima de 5°. tiempo.
Generalmente de causa idiopática.  Hombros y cadera que aparecen desiguales (un hombro puede estar más alto que el
Escoliosis infantil es < o = 3 años, escoliosis otro).
juvenil de 4 a 10 años, escoliosis adolescente de
 Curvaturas de la columna más hacia un lado.
11 a 18 años.
Afecta más a las niñas, curva empeora en  Un hombro que es más alto que el otro.
periodos de crecimiento rápido.  La pelvis está inclinada
Hay una escoliosis congénita en que las costillas Tratamiento
o vertebras no se forman correctamente. - Curvas leves (<20°): Observar, si aumenta más de 5° en 6 meses tratar.
Escoliosis neuromuscular asociada a una - Curvas moderadas de 20 a 40°: Corset de 23 hrs al día (TLSO, milwoukee), ejercicios
enfermedad del SNC como una paralisis. kinésicos, sólo detiene la progresión.
- Curvas graves (>40°): quirúrgico (corrección, instrumentalización, y artrodesis
posterior).

29. PERTHES
Definición Diagnóstico Clínica
Osteocondritis, que se da RX sin embargo éste puede resultar normal en una Más frecuente en varones (5/1) y se presenta
en niños entre 4 y 8 años, etapa precoz de la enfermedad y se debe completar habitualmente con cojera indolora. Sí existe dolor, este
se localiza en la cabeza del el estudio con cintigrafía ósea. aumenta con la actividad y mejora con el reposo. También
femoral. Necrosis avascular Gammagrafía ósea: la gammagrafia ósea con se puede manifestar por dolor en el muslo, en la región
idiopática de la cabeza tecnecio 99 es un método diagnóstico que muestra inguinal o en la rodilla. Puede verse contracturas
femoral, provocando una la perfusión e integridad de la vascularización del musculares y limitación de la rotación.
falta de riego sanguíneo a hueso, identificando áreas en donde no hay
la cabeza del fémur que captación del radiotrazador en un estadio temprano
provoca una necrosis Tratamiento
parcial de éste. Etapa Aguda: Eliminar el dolor y recuperar la Etapa crónica: Actualmente existe una nueva posibilidad
movilidad en forma completa, mediante: tracción terapéutica que consiste en el uso curativo y profiláctico del
de las partes blandas, reposo, AINEs, kinesioterapia. oxígeno bajo presión por encima de la atmosférica y se basa
Paciente menor de 6 años, lesión tipo I y II se hace en utilizar la capacidad de transporte del oxígeno disuelto
kinesioterapia y control expectante de su evolución. en el plasma sanguíneo incrementando más de 20 veces
Mayores de 6 años se busca una “cobertura dicho volumen transportado con lo cual se enriquece la
cefálica” por medio: ortopédico: Férula de Atlanta oxigenación de los tejidos comprometidos
(produce abducción de los muslos y centra la
cabeza femoral), quirúrgico: Osteotomía acetabular.

DESARROLLO PSICOMOTORRETRASO Y REGRESIÓN DEL DSM


RETARDO MENTAL

DESARROLLO PSICOMOTOR

El desarrollo psicomotor, o la progresiva adquisición de habilidades en el niño, es la manifestación externa de la maduración del sistema
nervioso central (SNC).

La proliferación de las dendritas y la mielinización de los axones son los responsables fisiológicos de los progresos observados en el niño.

La maduración del SNC tiene un orden preestablecido y por esto el desarrollo tiene una secuencia clara y predecible: el progreso es en sentido
céfalocaudal y de proximal a distal.

Resulta impensable el examen neurológico del niño que no incluya la valoración de su desarrollo psicomotor.
Se entiende por DSM normal el que permite al niño alcanzar las adquisiciones adecuadas para cumplir las funciones que corresponden a su
edad.

Cuanto más lejos del promedio es menos probable que sea normal.

Aplicación rutinaria a edades clave de algunas tablas de desarrollo o test de screening, para averiguar si esta en riesga o en RDSM.

Antes de hablar de los test de screening, es importante saber cuales son los reflejos arcaicos.
Reflejos arcaicos

• Sufren una evolución significativa en el primer año de vida.


• Su persistencia,asimetria,intensidad anormales o reaparición son signos de disfunción del SNC.
• Varios patrones evolutivos han sido identificados.

Tónico Nucal
• Alcanza su máximo a los 2 meses y desaparece entre los 4 y 6 meses.
• Alteraciones motoras como la paralisis cerebral producen mentencion del reflejo despues de los 6 meses
• Hacia el lado de la rotacion se produce una extencion de las extremidades ipsilaterales y flexion de las contralaterales.

Reflejo de Moro
• Abducción seguida de aducción y flexión de las extremidades superiores, tras hacer que la cabeza caiga.
• Aparece desde el nacimiento, a los 2 meses es incompleto y puede verse hasta los 4 - 6 meses.

Marcha Automática
• Presente desde el nacimiento y desaparece hacia los 2 – 3 meses.

Prehensión Palmar
• Respuesta involuntaria de flexión y agarre del objeto. Desde el nacimiento y desaparece hacia los 3 meses.

Prehensión Plantar
• Presente desde el nacimiento y desaparece hacia los 9 – 10 meses

Búsqueda
• Presente desde el nacimiento y desaparece hacia los 3 meses.

Incurvación de Tronco (Galant)


• Flexión lateral del tronco hacia el mismo lado ante un estímulo paravertebral.
• Presente desde recién nacido hasta los 3 - 9 meses.

Evolución del tono muscular

• Progresa desde el predominio flexor en el recién nacido a un equilibrio balanceado en el tono flexor-extensor de las extremidades.
• Ocurre en primera instancia con las extremidades superiores luego con las inferiores.
• La extensibilidad de las articulaciones va acrecentándose lo que permitirá su mayor funcionalidad.

Evolución de la motricidad gruesa

• Progresiva diferenciación de actos amplios e indiferenciados a precisos y refinados.


• En proyección céfalo-caudal.
• De axial a distal.

• Esta transformación es posible por:


– Evolución del tono muscular: disminución de la hipertonía flexora de extremidades, aumento del tono del tronco, aumento de la
extensibilidad articular, todo esto, que permite la marcha.
– Desaparición de los reflejos arcaicos.
– Desarrollo de reacciones de equilibrio, para el balanceo y evitar la caída.
– Los niños por ej., primero tienen que liberar el pulgar (disminuye tono flexor a un eq. Flexoextensor) parapoder tomar un objeto.
Por eso es importante que primero desaparezca la prension palmar pqra
– poder desarrolar la pinza…esta todo coordinado.

Reacciones de Maduración

• Paracaídas Lateral :
– Aparición a los 6 meses y persiste durante toda la vida.

• Paracaídas Horizontal :
– Aparición a los 9 – 10 meses y persiste.

• Landau :
– Suspensión horizontal en prono, produce flexión de piernas y extensión cervical y de tronco.

Evolución de la motricidad fina


• Funciones de la mano. Antes de alcanzar su función como órgano motor se debe:
– Liberar de la acción sinérgica tónicoflexora
– Acción tónico nucal.(no explico en que consistia)
– Superar la hipertonía de extremidades superiores.
– Coordinación con el sentido de la vista.

Evolución de la motricidad fina

4 – 5 meses coge objetos con toda la mano.(debe tener liberado el pulgar)


7 meses coge con el pulgar y los 2 dedos próximos.
9 – 11 meses logra realizar la pinza

Evolución sensorial
• Estudios han mostrado que el niño al nacer ya está dotado de una serie de capacidades sensoriales.
• RN y lactantes menores se comportan como miopes y a los 4 meses adquieren una agudeza visual importante.
• Capacidad de seguimiento de objetos.

HITOS DEL DESARROLLO PSICOMOTOR

Tono muscular y postura flexora.


Recién nacido Reflejos arcaicos presentes y simétricos.
Levanta la mejilla en posición prona.

Fija la mirada en el examinador y sigue a 90 grados.


1 mes
Alerta al sonido, se sobresalta, vocaliza.
En suspensión ventral y mantiene cabeza a nivel del tronco.

Sonríe en respuesta al examinador, vocaliza.


2 meses Levanta cabeza por varios segundos en posición prona.
Mantiene manos empuñadas gran parte del tiempo.
Se sobresalta con sonidos fuertes.

Abre sus manos y las observa con atención.


Mantiene levantada la cabeza en prono.
3 meses
Sigue objetos en plano vertical y horizontal, observa los rostros.
Afirma por segundos un objeto puesto en su mano, incorpora sonido de consonante "G' (agú).

Se ríe fuerte, a carcajadas.


Mantiene la cabeza firme al mantenerlo sentado.
4 meses Alcanza un objeto, lo coge con la palma y lo lleva a la boca.
Gira la cabeza en busca del sonido.
Se sonríe espontáneamente.

En supino levanta la cabeza e intenta sentarse en prono, levanta cabeza y tronco y se gira a supino.
Manotea objetos, los agarra y transfiere de una mano a la otra.
5-6 meses
Balbucea y localiza el origen del sonido.
Se sienta con apoyo y mantiene el tronco firme.

Se mantiene sentado solo. Apoyando sus manos adelante (trípode) apoya su peso en los pies y flecta sus piernas.
Con energía golpea fuertemente objetos contra la mesa, los lleva a la boca.
7-8 meses Se gira de supino a prono e intenta gatear.
Dice disílabos (da-da, ba-ba).
Estira los brazos para ser tomado.
Se sienta solo por largo rato, sin ningún apoyo. Se pone de pie afirmado de muebles.
9-10 meses Hace adiós con la mano, aplaude, come con los dedos.
Desconoce a extraños, dice "papa" o “mama", busca el objeto caído (permanencia del objeto)

Camina con poco apoyo. Usa pinza fina índice pulgar. Dice 3-4 palabras con significado. Gatea bien. Ayuda al vestirse.
11-12 meses
Comprende órdenes simples.
Camina sin apoyo, se agacha en cuclillas.
Dice varias palabras (mucha jerigonza).
13-15 meses
Hace rayas con un Iápiz.
Apunta con el índice para pedir lo que necesita
Sube las escaleras gateando con ayuda, se sube a una silla.
Ayuda a desvestirse.
18 meses Comienza a comer solo.
Hace torre de 4 cubos, tira una pelota, apunta a 3 partes de su cuerpo, pide cosas por el nombre, dice varias palabras en
forma incorrecta.
Sube y baja escaleras de pie solo (ambos pies en un peldaño), corre e intenta saltar con los dos pies juntos.
24 meses: Se comunica diciendo dos o tres palabras formando frases simples.
Hace torre de 6 cubos, patea una pelota, dice su nombre.

RETRASO Y REGRESIÓN DEL DSM

• Se denomina RDSM a la lenta adquisición de los hitos del desarrollo.


• Se denomina regresión del DSM a la pérdida de los hitos del desarrollo previamente adquiridos.

Esta diferenciacion es importante para el dg


• RDSM : por una encefalopatia por alguna noxa al nacer, pero sin noxas posteriores
• Regresion DSM : puede indicar una enf Progresiva …puede ser un niño con un DSM normal, pero tiene una noxa como una meningitis

El RDSM es uno de los problemas más comunes evaluados por el Neurólogo Infantil. 2 importantes preguntas deben ser resueltas:
– ¿Es el retraso específico para un área del desarrollo (lenguaje o motor) o es global?.
– ¿Corresponde solo a retraso o existe regresión?...por la importancia etiologica.

El diagnóstico diferencial del RDSM corresponde a la regresión del DSM.


El RDSM puede ser causado por una encefalopatía estática o progresiva, en cambio la regresión del DSM puede indicar una enfermedad progresiva
del sistema nervioso.

• ESTA TABLA LA VA A PRGUNTAR


No muestra alerta especial a la madre.
No fija la mirada.
2 meses No tiene sonrisa social.
No sujeta la cabeza en supino.
No levanta la cabeza en prono.
No ayuda a sentarse.
No gira.
4-5 meses Persiste reflejo de prehensión palmar.
No es capaz de sujetar un cascabel.
No gorjea.
No se sienta sin apoyo.
No mantiene un objeto en cada mano.
6-8 meses
No busca objeto caído.
No se ríe.
No se para afirmado.
9 –10 meses No tiene pinza.
No dice pa-pa; ma-ma.
No busca objeto escondido.
12 meses No camina con apoyo de ambas manos.
No gatea.
No camina solo.
15 meses No pone o saca objetos de un recipiente.
No se interesa en fenómeno de causa efecto.
No dice palabras sueltas.
18 meses
No entiende orden verbal gestual.
No sube o baja escaleras.
No dibuja (garabateos).
2 años
No hace o repite frases de 2 palabras.
No entiende orden verbal.
No salta en ambos pies.
No da vuelta página de un libro.
2 años 6 meses No usa al menos un pronombre personal.
No categoriza similaridades.
No establece contacto visual ni juega con otros niños.
Incapaz de pararse en un pie por segundos.
No dibuja círculo.
3 años
No hace torres de 8 cubos.
No conoce su nombre completo.
No salta en un pie.
No se lava y seca las manos.
4 años No va solo al baño.
No construye oraciones, no entiende preposiciones.
No se le entiende lo que habla.

CAUSAS DE RDSM GLOBAL O PARCIAL (NO REGRESIÓN)

Retraso del desarrollo global


 Malformaciones cerebrales.  Desórdenes perinatales.
 Alteraciones cromosómicas.  Encefalopatías progresivas.
 Infecciones intrauterinas.

Retraso predominante del lenguaje


 Esclerosis hipocampal bilateral.  Déficit auditivo.
 Síndrome perisilviano bilateral congénito.  Autismo infantil.

Retraso predominante motor


 Ataxia.
 Hemiplejía.
 Hipotonía. Desórdenes neuromusculares.
 Paraplejia.
ENCEFALOPATÍAS PROGRESIVAS…CAUSAS DE REGRESION (INICIO ANTES DE LOS 2 AÑOS)
Encefalopatía por VIH
EIM
Desórdenes del metabolismo de los aminoácidos (homocistinuria – Enfermedad de la orina olor a jarabe de arce –
Fenilquetonuria).
Desórdenes de las enzimas lisosomales (Gangliosidosis – Gaucher – Krabbe – Mucopolisacaridosis – Niemann Pick – Deficiencia en
sulfatasas).
Desórdenes de la glicosilación de proteínas.
Hipotiroidismo.
Enfermedades mitocondriales (Alexander, MELAS, Leigh).

Síndromes neurocutáneos
Neurofibromatosis - Esclerosis tuberosa.

Otros desórdenes de la sustancia gris


Lipofuscinosis – Distrofia neuroaxonal infantil – Lesch-nyhan – Síndrome de Rett.

Otros desórdenes de la sustancia blanca


Canavan – Galactosemia – Adrenoleucodistrofia – Pelizaeus merzbacher.

ENCEFALOPATÍAS PROGRESIVAS (INICIO DESPUÉS DE LOS 2 AÑOS)


EIM
Desórdenes de las enzimas lisosomales (Gaucher – Krabbe – Gangliosidosis – Leucodistrofia metacromática - Mucopolisacaridosis –
Niemann Pick).

Enfermedades infecciosas
Encefalopatía por VIH - Sífilis congénita – PEES.

Otros desórdenes de la sustancia gris


Lipofuscinosis – Enfermedades mitocondriales.

Otros desórdenes de la sustancia blanca


Adrenoleucodistrofia – Alexander.

RETARDO MENTAL
Definición
• Según la AAMD:
– “Un funcionamiento intelectual muy por debajo del promedio, que se origina durante el período del
desarrollo y que se acompaña de alteración de uno o más de los siguientes aspectos: maduración,
aprendizaje y adaptación social”.
• Según el DSM-IV:
– “Un funcionamiento intelectual general significativamente por debajo de la media que se acompaña
de limitaciones significativas en el funcionamiento adaptativo, con un inicio antes de los 18 años”.(
Mayor a 18 años es demencia y menor a 5 años es RDSM)
• Según el CIE-10:
– “Un nivel reducido del funcionamiento intelectual que resulta en una menor capacidad para
adaptarse a las demandas diarias del entorno social normal. Se afectan las capacidades lingüísticas,
motoras, sociales y cognoscitivas”.

El retardo mental

• Es un trastorno del desarrollo de tipo general que se inicia antes de los 18 años y se caracteriza por una
capacidad intelectual y adaptativa disminuída. Por funcionamiento intelectual bajo el promedio se entiende un
rendimiento que está más allá de 2 DS bajo la media de una población a una edad determinada.
• Se define como un coeficiente intelectual < a 70.
• El bajo funcionamiento intelectual debe traducirse en un menoscabo de la conducta adaptativa.
¿Por qué es importante el estudio del RM?.

• Afecta entre un 3 a 4% de la población.


• En un 25 a 40% es de origen biológico.
• El RM leve es 15 veces más frecuente en los estratos socioeconómicos más pobres.( la leve es la mas fcte)
• La prevalencia de otros trastornos mentales en los pacientes con RM es aproximadamente 3 a 4 veces mayor.
• En personas mayores la prevalencia es menor debido a las tasas de mortalidad más altas de aquellos con RM
grave o profundo.

CLASIFICACIÓN
DSM-IV CIE-10
RM leve. 50-55 a 70 50 a 69
RM moderado. 35-40 a 50-55 35 a 49
RM grave. 20-25 a 35-40 20 a 34
RM profundo. <20-25 <20

RM BIOLÓGICO RM SOCIOCULTURAL
ETIOLOGÍA Noxas (infecciosas, metabólicas, Ambiente pobre en cantidad y
traumáticas, vasculares, hipoxias, calidad de estímulos en los
etc) que afectan al SNC. primeros años de vida
(deprivación sociocultural).
MORBIMORTALIDAD Mayor que la población general. Igual que la población general.

ESTIGMAS FÍSICOS Más frecuentes. No existen.

GRADO DE RETRASO Generalmente moderado, severo Generalmente leve o limítrofe.


o profundo.
RECONOCIMIENTO Período de lactancia a preescolar. Generalmente en edad
preescolar.
DISTRIBUCIÓN POR Igual en los distintos estratos. 15 veces mayor la prevalencia en
CLASE SOCIAL los estratos más bajos.
EPIDEMIOLOGÍA (esto como que lo repite)

• La prevalencia de RM (OMS) se estima entre el 1 a 3% de la población.


• En Chile afecta entre un 3 a 4% de la población.
• La relación hombre/mujer es 1,5:1.Es mas fcte en hombres por el sd cromosoma x fragil ( ya que las mujeres
desctivan un x… pero el hombre no)
• La gran mayoría de los pacientes RM se clasifican dentro de RM leve (85%).

• Entre los 0 y 4 años la prevalencia de RM es de 0,5%, a los 5 años es de 2,2%, a los 6 años de 4%, entre los 10 y
14 de 8% y entre los 16 y 17 de 2,8% (Goodman y Gruenberg).

ETIOLOGÍA

• Los factores etiológicos pueden ser biológicos, sociales o una combinación de ambos.Se ha podido establecer
un origen biológico en aproximadamente un 25% de los casos de RM. Por lo tanto existe un 75% en que no se
encuentra causa identificable.

• A mayor severidad del RM mayor es la correlación con un origen biológico identificable. A menor severidad del
RM más es su correlación con factores socioculturales.
FACTORES BIOLÓGICOS. Factores prenatales.
Factores genéticos. Síndrome TORCH.
Síndrome de Down. Síndrome alcohólico fetal.
Síndrome del cromosoma X frágil. Exposición prenatal a sustancias.
Síndrome de Prader-Willi. Complicaciones del embarazo.
Síndrome del grito del gato (“Cri-du-chat”).
Fenilcetonuria. Factores perinatales.
Síndrome de Rett.
Neurofibromatosis. Trastornos adquiridos en la infancia.
Esclerosis tuberosa. Infecciones.
Síndrome de Lesh-Nyhan. Trauma encefálico.
Enfermedad de la orina olor a jarabe de arce. Otros.
Otros trastornos por deficiencias enzimáticas.
FACTORES AMBIENTALES Y SOCIOCULTURALES.

1) Síndrome del cromosoma X frágil.

• La causa más frecuente de RM familiar.


• 1:4000 hombres, 1:8000 mujeres.
• Dismorfias: Frente prominente - Cara alargada - orejas grandes.
• Un 20% presenta conductas autistas.
• En la pubertad es fcte que consulten por macroorquidismo.
• Corresponde a una mutación por amplificación de secuencias repetidas:

Existe una mutación en el gen denominado FMR-1. Ocurre un aumento de las repeticiones del trinucleótido CGG.

Existen 2 estados de mutación:

1) Premutación (60 a 200). (Mujeres portadoras de una premutación son capaces de transmitirles a sus hijos varones el
cuadro.)
2) Mutación completa (200 a 1000).
Cuando ocurre la mutación completa se produce una hipermetilación que impide la síntesis de la proteína
FMR-1 y se manifiesta el cuadro.
2) Exposición prenatal a sustancias.
Síndrome alcohólico fetal.

• 1,9 por 1000 RNV.


• Hipertelorismo - filtrum largo – microoftalmia – microcefalia- RDSM.
• Se diagnostica en el contexto de una historia (+) para consumo materno de alcohol más las dismorfias.
• Evolucionan con trastornos del aprendizaje y ADHD.
• Su dg es por:
1. Consumo materno de OH ( efecto acumulativo)
2. Dismorfias

Evaluación psicológica. ( la hace un sicologo)


• El funcionamiento intelectual general se determina mediante escalas de inteligencia estandarizadas.
• Se deben realizar distintos test psicológicos, entre ellos destacan:
– Escala de inteligencia de Stanford-Binet.
– Escala de inteligencia Weschler para niños (WISC-R).

• El funcionamiento adaptativo puede ser medido utilizando una escala estandarizada, como la escala de
Vineland de comportamiento adaptativo.
• En esta escala se puntúan destrezas de comunicación de la vida diaria, de socialización y motoras y se obtiene
una composición de la conducta adaptativa que se compara con las capacidades esperadas a una edad
determinada.
• Pueden salir con CI bajo pero con capacidad adaptativa normal (vineland normal) y no van a tener retardo
mental…………el Vineland debe hacerse anualmente y tambien los test sicologicos ( ya que los niños se las
aprenden

EXÁMENES DIAGNÓSTICOS

• Debe realizarse de acuerdo a los hallazgos en la historia clínica y examen físico.


– Análisis de sangre y orina.
– Amniocentésis - Biopsia de vellosidades coriónicas.
– R(x) de cráneo, TAC, RM encefálica y el EEG.
– Estudio genético - Cariograma - FISH (Síndrome de Angelman/Prader-Willi - Síndrome de Smith-
Magenis – CATCH 22 – Síndrome de Williams – Síndrome de Wolf-Hirschhorn – Síndrome de cri du
chat – Síndrome de Langer-Giedion -Síndrome de Miller-Dieker).

– Además es necesario realizar, entre otros: Aminoacidemia y aminoaciduria - Ácidos orgánicos en


orina - Mucopolisacaridos en orina - Pruebas tiroídeas - Ácidos grasos de cadena muy larga.
– Medición de actividad enzimática.

DIAGNÓSTICO DIFERENCIAL
Debe realizarse con:

• Trastornos específicos de la lectoescritura y del lenguaje.


• Demencias.
• Trastornos generalizados del desarrollo.
• Handicaps sensoriales.

• Otros:
– Rendimiento intelectual interferido por trastorno del ánimo, estado postictal.

PRONÓSTICO
• RM leve y moderado no tiene una disminución en la expectativa de vida.
• La presencia de RM severo y comorbilidad afecta el pronóstico.
• Un bajo nivel intelectual determina una serie de limitantes vinculadas a las posibilidades de ocupación en la
vida adulta y de logro de una vida más o menos independiente.
MAVR

ASMA BRONQUIAL EN PEDIATRIA

Definición: proceso inflamatorio crónico de la vía aérea, que se caracteriza por la presencia de obstrucción bronquial difusa
reversible e hiperreactividad bronquial. Con factores genéticos y ambientales (tabaco, contaminación, infecciones, etc)
involucrados. La intensidad y frecuencia de los síntomas está determinada por el grado de inflamación y obstrucción
bronquial existente. Es la enfermedad crónica más común en los niños.

Diagnóstico diferencial:

 Fibrosis quística
 DBP
 Bronquiolitis obliterante
 Disquinesia ciliar
 Neumonitis intersticial

Diagnóstico:
 Clínica (lo más importante): episodios recurrentes de sibilancias, tos y dificultad respiratoria. 
Síntomas
nocturnos, con ejercicio u otros gatillantes. Historia familiar de asma, antecedentes de rinitis alérgica, dermatitis
atópica. No siempre hay correlación entre el grado de obstrucción y la severidad de los síntomas.

 Estudios complementarios:
o Espirometría: patrón obstructivo que revierte con broncodilatador (mejora VEF1: 12%). Gold estándar
para confirmar dg en atención primaria (AP). En periodo asintomático puede estar normal, lo cual no
descarta asma (Alta E, baja S). No aplicable en < 6 años.

 Pruebas de provocación bronquial:



- Test de ejercicio (+): disminución ≥ 10% VEF1.

- Test de metacolina (disminución > 20 % VEF1 con [ ] < 8mg/ml )

 Rx tórax: no se usa de rutina para evaluación de asma. Si en sx atípicos, complicaciones, etc.


 Prick Test: estudio alérgico, atopía.
o Medición de Oxido Nítrico exhalado (FENO) identifica inflamación
eosinófila, >20ppb: inflamación. En niños > 7-8 años.

¿Qué podemos utilizar para diagnosticar?


RN -> Hemograma
4 años -> hemograma + prick test
6-7 años -> Hemograma + prick test + espirometría

Tratamiento de mantención
Objetivos: Control síntomas, mejorar calidad de vida, prevenir exacerbaciones, mantener función 
pulmonar, actividad
física, prevenir efectos adversos medicamentos.

Medidas no farmacológicas: Educación, evitar gatillantes (ácaros, polen, tabaco, etc).

Medicamentos:
o De rescate: B2 agonistas vida media corta (SBT), anticolinérgicos, corticoides sistémicos.

o Control: Corticoides inhalados, B2 larga duración (salmeterol), anti-leucotrienos.


 Broncodilatadores:
- β 2 agonistas: Salbutamol (t1/2: 6 hrs), Salmeterol (c/ 12 hrs)
- Anticolinérgicos: Bromuro de Ipatropio
 Antiinflamatorios:
- Corticoides Inhalatorios: Beclometasona (100-300 ug/d), Fluticasona (100-200 ug/d), Budesonida (100-
200 ug/d)
- Antileucotrienos: Montelukast

CEFALEAS

- Importante causa de consulta en pediatría


- Consultantes entre 5-15 años
- Mujeres > hombres, pero hombres consultan a menor edad (promedio 7 años v/s 10 años)

Diagnóstico

- Hacer la diferencia entre cefalea primaria y secundaria (Tabla 1)


- Anamnesis detallada:
 Antecedentes familiares.
 Caracterización del dolor: tipo, ubicación, inicio, duración, frecuencia, horario de presentación, evolución,
intensidad, síntomas asociados, si existen desencadenantes, si sabe cuándo va a venir (presencia o no de aura),
agravantes y atenuantes.
- Examen físico:
 Presión arterial, temperatura.
 Examen de piel cuidadoso: Búsqueda de signos de enfermedades neurocutáneas (manchas café con leche,
hipocromías)
 Fondo de ojo
 Perímetro cefálico
 Signos neurológicos focales: Signos meníngeos
 Examen neurológico completo.
- Tipo de cefalea más común en niños: MIGRAÑA SIN AURA (Diagnóstico según tabla 2). Luego cefalea tensional
(asociada a eventos académicos, problemas familiares, etc.)
Exámenes:

- Diagnóstico clínico.
- Sólo neuroimágenes en casos indicados (tabla 3)
- PL en caso de sospecha de meningitis (previa toma de TAC)
- EEG no es de rutina.
Manejo:

- Explicar y tranquilizar al niño y a la familia


- Medidas no farmacológicas: Identificar gatillantes.
 Mejorar patrones de sueño (horario y duración fija)
 Alimentación: No saltarse comidas ni ayunar. Evitar queso, cafeína, aditivos y colorantes, glutamato monosódico
(comida china-embutidos-conservas), chocolate, alcohol.
 Otros: Menstruación, ejercicio intenso, estrés, cambios de clima.
- Medidas farmacológicas:
 Episodio agudo:
o Paracetamol: 10-20 mg/kg vo
o Ibuprofeno: 5-10 mg/kg vo
o Naproxeno: 2,5-5 mg/kg vo
o Asociar antieméticos en caso de náuseas-vómitos
o Sumatripán: 25-50-100 mg/dosis (spray nasal). Tratamiento específico y sólo se ha estudiado su
efectividad en adolescentes.
 Profilaxis:
o Antiepilépticos (divalproato de sodio-topiramato)
o Antidepresivos tricíclicos (amitriptilina-nortriptilina)
o Betabloqueadores (propanolol)
o Bloqueadores canales de calcio (flunarizina)
o Otros: Riboflavina, ciproheptadina, coenzima Q10, extracto de petasita.
Tablas:
Estación 1:

Cabro chico con vómitos y diarrea de 3-5 días, llega al SU: dan las características de su examen físico (ojos hundidos,
lagrimas, fontanela, boca, piel, signo de pliegue, etc).

a) Diagnóstico: Síndrome diarreico agudo (recordar que no existe el Dg de GECA en el cabro chico, según Dra
Shellman los vómitos generalmente acompañan al Sd diarreico agudo), y deshidratación leve. ** ahora si este
semestre dijo otra cosa háganle caso a ella obviamente xD.
b) Etiología: Norovirus y Rotavirus
c) Tratamiento del cuadro agudo: calcular el volumen de rehidratación en el agudo según el tipo de
deshidratación
d) Que indicaciones le mandaría para la casa: Dejar la indicación a la madre de que en cada episodio diarreico y
emético rehidratar con sales de rehidratación oral con el mismo calculo y además calcular la hidratación de
mantenimiento para el caso.

Guagua que se pone amarilla antes de las 24 horas (por tanto patológica).

Parte 1:

a) ¿Qué preguntas le haría a la madre?: lactancia, algún traumatismo en el parto, cefalohematoma, fiebre (en
búsqueda de infección).
** al examen físico (que está en la ficha clínica) tenía un cefalohematoma simple, no extenso.

Que examen le solicitaría: creo que solo era bilirribuna total, pues la diferencial solo se pide después de la
persistencia del cuadro.

Parte 2:

a) Clasificar el riesgo: creo que era moderado, y en los exámenes (entregado por la Dra tenia una bilirrubina
superior a la esperada para su riesgo, por tanto
b) Tratamiento: fototerapia, con todas sus indicaciones, es decir, Fototerapia, antifaz ocular, equipo de
fototerapia a 35 cms (no me acuerdo bien de la distancia XD), hidratación diaria ( recordar que son 60ml/kg/dia
si es de termino y 70ml/kg/día si es de pre término a esa cantidad se le suma un 10-20% mas por la mayor
perdida de liquido por la foto , esa cantidad se va aumentando paulatinamente día a día hasta llegar a los 150
ml/kg/dia de la formula), continuar con la lactancia, rotar en el día.
Estación 3:

Cabro chico de 8? Años consulta por disnea al esfuerzo y silbidos.

a) Diagnóstico: Asma moderada (apréndanse los criterios GINA y del consenso chileno)
b) 3 preguntas que haría para confirmar su diagnóstico: le silba el pecho en la noche, en el esfuerzo, cuando se
resfría el resfrió se le va al pecho, despierta en la noche con síntomas, despierta en la mañana con síntomas,
etc.
c) Tratamiento: Creo que era Corticoides inhalados + B2 adrenérgico de larga o corta duración (ya no me acuerdo
cuál de los 2 era). ¿Lo deriva? O ¿tto en APS?
d) Es patología GES: Si
Estación 4:

Cabro chico con vómitos, diarrea consulta en SU: detallan el examen físico las fontanelas, ojos, lagrimas, boca, etc.
Además están los resultados de exámenes: solo recuerda que tenía un BUN elevadísimo, una hiperkalemia y creo que era
una deshidratación eunatrimica.

a) Diagnóstico:
- SDA

-Deshidratación SEVERA (todos pusimos moderada por las características al examen físico, pero nadie se percató del
BUN, que indica IRA por tanto es severa).

- Acidosis metabólica hiperkalemica

b) indique tratamiento: hacer el cálculo de mantenimiento con un suero estándar para deshidratación eunatremica
(indiquen cual es la composición hidroelectrolítica del suero, es decir, cuanto NaHCO3, KCl, y otros).

c) Indique 3 medidas para la hiperkalemia: Salbutamol, Insulina, Corrección de la acidosis metabólica, kayexalate,
etc.

Estación 5:

Niño con fiebre y petequias en el cuerpo (no recuerdo si tenía signos de irritación), compromiso de conciencia:

a) Diagnóstico:
Purpura Febril: Observación de Meningococcemia y Meningitis meningococica.

No me acuerdo bien de esta estación, pero tienen que hacer el estudio diagnóstico y el tratamiento

Estación 6:

Galletitas, y jugo (si están de los últimos, no habrá nada :D).

Estación 7 (actuación):

Tienen que hacer pasar al papa del niño(a), tiene que:

a) Construir la alimentación de un niño de 6 meses:


7,5% de LPC + 2,5% de azúcar+ 3% cereal  explicar claramente cómo hacerla

Dieta no láctea: explicar de cómo hacer la papilla: cocer en agua un papa chica, una hoja verde, una zanahoria o zapalla,
huevo aun no!, carne del porte de una caja de fosforo (todo en la misma olla) y todo eso tirarlo a la licuadora, pasarlo por
cedazo y dárselo las veces que corresponde con la formula láctea.

b) Ver que vacuna le corresponde según PAI: obviamente le corresponde la polio oral, neumo y pentavalente,
pero tienen que preguntarle si tiene todas las vacunas al día en caso de que sea necesario reconstruir. La niña
tenía todo al día, por tanto indicar cuál es la que corresponde y darle las indicaciones y advertencias
pertinentes, como que tenga fiebre le duela el brazo, le salga una pelota y que si tiene fiebre sobre 38,5 recién
darle PCT 2 gotas por Kg (SOS), temperaturas menor a eso solo con compresas húmedas en guata y piernas (no
en la cabeza), además de los paños en el brazo para el dolor.
c) Indicarle al padre/madre como está el estado nutricional del niño (apréndanse los gráficos por número y para
qué sirve cada uno y para qué edad, se les hará más rápido y no les consumirá tanto tiempo buscando el
gráfico). La cosa es indicar P/E y T/E (si mal no recuerdo P/T es solo para mayores de 6 meses).
d) Y por último pregunten por los hábitos, a todos nos bajaron puntos por eso.

ADMINISTRACION SIMULTANEA DE VACUNAS: En general se han obtenido niveles de protección similares a la


administración de vacunas separadas.
VACUNACIONES INTERRUMPIDAS: No se requiere reiniciar toda la serie, pudiendo continuarse el esquema con la siguiente
dosis
VACUNACIONES NO INICIADAS:
• Niños menores de 1 año debe comenzar con la serie a los mismos intervalos de tiempo establecidos.

• Niños mayores de 1 año deben recibir MMR, DPT y Polio y luego 2 dosis mas a intervalos de 2 o 4 meses
• Después de los 4 años no se usa DPT sino DT
Respetar el intervalo mínimo recomendado para cada vacuna
Al aumentar el intervalo entre las dosis no disminuye la efectividad
No hay contraindicaciones para administración simultánea de vacunas (administrar en sitios distintos del cuerpo),
excepto para cólera y fiebre amarilla.
Si diferentes vacunas no se administran simultáneamente, se recomienda:
Intervalo entre 2 vacunas vivas diferentes: 4 semanas intervalo.

EDAD (MESES) PAI NO PAI


RN BCG
1 mes RTV (rotarix): 6-14sem. (1ª). Intervalo min. 4 sem.

DPT + Hib + POLIO


2 MESES (O.P.V).+Hep B PREVENAR (1ª): Intervalo.: 0-2-4 meses
ROTAVIRUS (rotarix): 14-24sem. (2ª)

DPT + Hib + POLIO


4 MESES (O.P.V).+Hep B PREVENAR (2ª)

DPT + Hib + POLIO PREVENAR (3ª)/


6 MESES (O.P.V).+Hep B INFLUENZA
VARICELA/
Twinrix (1º )(Hep. A y B)/ Intervalo: 0-1-6 meses
12 MESES TRES-VIRICA(MMR) Havrix (1º) Hep. A Intervalo:0 y 6-12 meses

13 meses Twinrix (2º )(Hep. A y B)

18 MESES DPT, OPV Havrix (2º) Hep. A

19 meses Twinrix (3º )(Hep. A y B)

2 AÑOS PNEUMO 23

4 AÑOS DPT

1º BASICO TRES-VIRICA(MMR)
TOXOIDE DT
2º BÀSICO

VACUNAS:

CONTRAINDICACIONES

1. Enfermedad aguda moderada o severa: contraindicación temporal


2. Reacción severa, incluyendo compromiso del CNS, a una dosis previa de la vacuna: contraindicar esa vacuna,
hasta que otra causa de tal reacción severa sea confirmada.
3. Reacción alérgica severa o anafiláctica a:
- Dosis previa de la vacuna: contraindicar esa vacuna
- Cualquier componente de la vacuna: contraindicar esa vacuna.
- Antibióticos usados en esa vacuna: Polio Inactivada (IPV), Tres Vírica (MMR), Varicela.
- Gelatina : MMR, Varicela.
4. Hipersensibilidad anafiláctica conocida al huevo: Contraindicadas Vacuna influenza, Fiebre Amarilla.
5. Inmunodeficiencia:
- En el receptor: contraindicadas Polio oral (OPV), BCG, MMR, Varicela ( inmunodeficiencia humoral pura
no es contraindicación para vacuna varicela)
- En contacto domiciliario: OPV
6. Infección VIH:
- Sintomática : contraindicadas OPV, BCG, MMR, Varicela.
- Asintomática: contraindicada BCG en países con bajo riesgo de TBC y Varicela.
7. Embarazo: contraindicadas OPV, BCG, MMR, Varicela.
8. Enfermedad neurológica en evolución o inestable: contraindicada DPT

NO SON CONTRAINDICACIONES
• Enfermedad aguda benigna (virosis resp, diarrea) en un niño sano.
• Niño tomando ABT.
• Madre del niño embarazada.(excepto vacuna varicela)
• Prematuridad
• Contacto reciente con patología infecciosa.
• Lactancia materna.
• Historia de alergia inespecífica, personal o familiar.
• Historia de alergia a los ABT contenidos en la vacuna, salvo reacción anafiláctica.
• Historia de alergia al pollo o plumas de aves, no anafiláctica.
• Historia familiar de SMSL en vacunación DPT.
• Administración concomitante de trat. de desensibilización.
• Historia de convulsiones febriles (usar antipiréticos)
• Condición neurológica estable tal como Parálisis Cerebral o Síndrome de Down
• Enfermedad crónica
• Cirugía reciente o inminente

PRÁCTICA ECOE
PEDIATRÍA
ESTACIÓN 1
ALIMENTACIÓN Y NUTRICIÓN

INSTRUCCIONES

UD. ATIENDE COMO MÉDICO GENERAL QUE TRABAJA EN EL CESFAM A LA MADRE DE EMILIO.
LACTANTE DE 6 MESES.
ELLA DESEA PREGUNTARLE A UD.
ACERCA DE DUDAS QUE TIENE DE CÓMO
PREPARAR LA ALIMENTACIÓN NO LÁCTEA
PARA SU BEBE
ELLA DEJÓ A SU HIJO CON LA ABUELA

SÓLO PREGUNTE A LA MADRE ACERCA


DE LA ALIMENTACIÓN DE SU HIJ0
ELLA LE MANIFESTARÁ SUS DUDAS.

ESTACIÓN 1 PAUTA PARA LA ACTRIZ


ALIMENTACIÓN Y NUTRICIÓN

CONTEXTO: Ud. es la madre primeriza de Emilio lactante de 6 meses se llama Antonia. Él es un niño
sano, de buen peso y estatura. Ud. lo amamantó hasta este momento y ahora debe volver al trabajo.
Ahora el niño toma pecho 3 veces: a las 07, 19 y 23 hrs., una mamadera de 180 cc de leche materna
o leche Purita a las 15 hrs. Ud quiere iniciar el almuerzo a las 11 horas

Ud. se controló el embarazo en el CESFAM y tuvo un parto normal en el hospital. De Talca. Emilio
pesó al nacer 2.950 grs y su talla fue de 50 cm. Fue un recién nacido normal; Ud. le dio solo lactancia
materna hasta ahora, tiene todas sus vacunas al día según el plan nacional de inmunizaciones (PNI).
Usted tiene varias dudas sobre alimentación, que quiere resolver con el médico.

Entrevista clínica: Preguntas que le van a hacer a la madre:

Qué dudas tiene Sra. Antonia.


Yo entro a trabajar pronto, y tengo una prima que cuidará de mi hijo necesito saber y aprender cómo
prepararle el almuerzo a Emilio y así enseñarle a mi prima.
¿Cómo lo está alimentando?
Sólo con leche materna y desde hace 3 a 4 días estoy intentando que reciba postres de manzana o
pera molida, le gusto y recibe todo lo que le doy, como ½ taza.
¿El niño recibe vitaminas y hierro?
Sí: le damos hierro y vitaminas.
Qué desea saber señora Pamela:
Primero, ¿qué tipo de verduras y vegetales debo usar para preparar la sopa puré y en qué cantidad?
Debe decir zapallo o zanahoria del porte de 1 caja de fósforos; papas: 1 papa chica; y verduras
verdes: algunas hojas de acelga o espinaca inicialmente y después otras verduras, como porotos
verdes, brócoli, etc.
Qué más desea saber
Debo agregar pollo, pavo o carne?
Si una cucharada de carne molida cocida e incluso le puede dar pescado
Si no señala el pescado le pregunta
Puedo darle pescado? Debe responder que sí
Puedo agregar legumbres
No
Qué más desea saber
¿Debo usar sal o aceite?
aceite vegetal sí 3 cc al momento de servir la pailla.
Qué más
Qué cantidad de almuerzo debo darle
Comience con media taza y debe llegar a tres cuartos de taza a una taza. Y ½ taza de postre
Alguna otra pregunta
Qué postre le doy y qué cantidad
Puede darle fruta molida: pera, manzana, plátano, etc. en volumen de media taza o de un
plato de postre o en un plato de compota, sin azúcar) Piña Y frutilla no
Está bien media taza? Si
¿Puedo darle miel? No
Ud le dice que encuentra que la miel están sana y ¿Por qué?
Debe contestarle que la miel puede contener un germén peligroso para los menores de 1 año
Algo más
¿Puedo darle bebida?
Debe contestarle
que no ¿Por qué?
Debe contestarle que las bebidas gaseosas contienen mucho azúcar y se va a acostumbrar al gusto,
además contribuyen a la obesidad ¿Todo está claro Sra.?
Sí, entendí bien. Muchas gracias
Si le preguntan si tiene otras dudas o preguntas debe decir: No,
está todo claro, Muchas gracias.

ESTACIÓN 1 – ALMUERZO PAUTA DE COTEJO

NOMBRE ---------------------------------------------

A- COMUNICACIÓN Y PROFESIONALISMO PUNTAJE PTOS PUNTAJE


OBTENIDOS
SI NO

Hizo contacto visual y saludó a la madre del paciente 0,5 2


Permitió a la madre del paciente hablar sin interrumpir 0,5
Usó terminología coloquial y evitó terminología médica 0,5
Se preocupa de verificar si la madre entendió las instrucciones 0,5
B- DESEMPEÑO CLÍNICO

Respuestas sobre sopa puré: 4,5


A. Zapallo o zanahoria: porte 1 caja fósforos o algo semejante 0,5
0,5
B. Una papa chica (o media o ¾ de papa grande) 0,5
C. Algunas hojas de verdura 0,5
D. Debe mencionar al menos 1 acelga, espinaca 0,5
0,5
E. Consistencia: puré o sopa-puré o papilla 0,5
F. Volumen: ¾ de taza o una taza entera 0,5
G. Aceite vegetal 3cc al momento de servir: sí 0,5
H. Sal: NO
I. Pescado , carne, pollo o pavo

Respuestas sobre postre: 3,5


A. Postre debe ser de fruta molida o rallada 0,5
0,25
B. Menciona al menos 2 frutas: pera, manzana, plátano, etc. 0,5
C. Piña y frutillas NO 0.75
D. Azúcar agregada: NO 0,5
0,5
E. No dar miel 0,25
F. Justificar por qué 0,25
G. No dar bebidas
H. Justificar por qué

10 PTOS. = 100% TOTAL


OBTEN
IDO =

FIRMA DEL DOCENTE…………………………………………………..

ESTACIÓN 14
HOJA DE INSTRUCCIONES

Ana madre de Carolina, niña de 4 meses, acude a la consulta porque por razones económicas
debe volver a su trabajo, solo podrá darle leche
materna a las 7- 19 – 23- 3 y necesita saber qué hacer y cómo preparar las mamaderas en caso necesario.
La niña pesa 6ooo grs , fue une RNTAEG 38
semanas con crecimiento pondoestatural normal

UD. DEBE DARLE LAS INSTRUCCIONES


CLARAS Y PRECISAS DE CÓMO
PREPARAR LAS MAMADERAS , ESCRIBASELAS EN UN PAPEL

ESTACIÓN 14 PAUTA PARA LA ACTRIZ


ALIMENTACIÓN Y NUTRICIÓN

CONTEXTO: Ud. es una madre primeriza se llama Ana, es el sustento de su hogar y debe volver a
trabajar, lo que no le permitirá darle de mamar a su a las 11 y 15 horas. Carolina es su primera hija
actualmente tiene 4 meses. Ella es una niña sana, de buen peso y estatura. Ud. la está
amamantando hasta ahora con leche materna exclusiva

Ud. se controló el embarazo en el CESFAM y tuvo un parto normal en en hospital de La Florida. pesó
al nacer 2980 grs y su talla fue de 49 cm. Fue una recién nacida normal. Ella es sana y tiene todas sus
vacunas según PNI. Usted tiene varias dudas sobre cómo preparar las mamaderas de su hija, o qué
hacer para no perjudicar la nutrición de su bebe, que quiere resolver con el médico.

Entrevista clínica: Preguntas que le van a hacer a la madre: Sra. Ana cuáles
son sus dudas?
• ¿Cómo puedo alimentar a mi hija en los horarios que no podré amamantarla? Ella se
alimenta solo con pecho materno y a las 11 horas y 15 horas no podré amamantarla, por
lo tanto quiero que Ud. me indique cómo hacerlo?
• Puedo guardar la leche materna?
• Y si no me alcanza que le doy?
• El médico debe explicarle que la mejor opción es que Ud. se extraiga la lecha materna y
la guarde le debe decir cuánto tiempo la puede guardar en al menos 1 opción de la que
se señala abajo
• Si no le dice le pregunta cuánto le dura refrigerada
Debe contestar 72 horas
Si no puedo juntar suficiente leche, qué hago?
El médico debe explicarle paso a paso cómo preparar la mamadera, con un lenguaje claro y escribirle
en un papel

PASO ACCIÓN

PASO 1 Debe preparar mamaderas de 150 cc de leche purita

Lo primero es hervir el agua


Agregar a la mamadera una mínima cantidad de agua

PASO 2 Agregar:
- 2 1/4 medidas de leche purita
- 1 medida maltosa dextrina o ½ medida de azúcar
- 3 ml de aceite de canola u omega 3- 6

PASO 3 Batir los ingredientes

PASO 4 Agregar agua hasta completar los 150 cc y nuevamente batir

PASO 5 Esperar que se entibie y administrar a la bebe

PASO 6 Dar a las 11 AM, 15 PM

¿Todo está claro Sra.?


Sí, entendí bien. Muchas gracias
Dr. le podría dar NAN 1? Debe responder que sí y explicarle

PASO ACCIÓN MAMADERA DE NAN 1

PASO 1 Debe preparar mamaderas de 150 cc


Lo primero es hervir el agua
Agregar a la mamadera una mínima cantidad de agua

PASO 2 Agregar:
- 5 MEDIDAS DE nan 1

PASO 3 Batir los ingredientes

PASO 4 Agregar agua hasta completar los 150 cc y nuevamente batir

PASO 5 Esperar que se entibie y administrar a la bebe

PASO 6 Dar a las 11 AM, 15 PM


Si resume y solo dice que debe seguir el mismo procedimiento anterior con la diferencia de que para
los 150cc de agua debe agregar solo 5 medidas de NAN 1 es correcto

Le quedó claro ? Si muchas gracias Dr


ESTACIÓN 14 PREPARACIÓN DE MAMADERAS PAUTA DE COTEJO
NOMBRE ………………………………………………..

ESTACIÓN 3
CONTROL DE SALUD

FRANCISCA, PACIENTE DE 8 AÑOS SIN


ANTECEDENTES MÓRBIDOS PREVIOS, CONSULTA POR UN CUADRO DE 4 DÍAS DE EVOLUCIÓN
CARACTERIZADO POR FIEBRE HASTA 38°C, CEFALEA Y
CORIZA. Al SEGUNDO DÍA DE EVOLUCIÓN NOTA
ENROJECIMIENTO EN LAS MEJILLAS Y
POSTERIORMENTE APARECEN LESIONES
ERITEMATOSAS EN TRONCO Y EXTERMIDADES
ASOCIADO A DOLOR EN LA ARTICULACIÓN DE LAS MUÑECAS IZQUIERDA Y DERECHA.
LA TRAE SU MADRE, EMBARAZADA DE 7 MESES
QUIÉN SE ENCUENTRA MUY PREOCUPADA POR LA
SITUACIÓN DE SU HIJA
MIRE EL SETS DE FOTOS ADJUNTAS Y CONTESTE
EN LA HOJA DE RESPUESTA

ESTACIÓN 3
ESTACIÓN 3
ESTACIÓN 3
HOJA DE RESPUESTAS
NOMBRE…………………………………………………………………………………………………………

ESTACIÓN 3
PAUTA DE CORRECCIÓN

PUNTAJE
1- ¿Cuál es el diagnóstico más probable? ERITEMA INFECCIOSO 1

2- ¿Cuál es la etiologías más probable? Parvovirus B19 1

3- ¿Cuál es la terapia más apropiada? 0,75


1- Medidas generales
0,75
2- Hidratación 0,75
0,75
3- Manejo de la fiebre con paracetamol 8
gotas cada 8 horas
4- Control médico en 72 horas o en caso
de reaparecer la fiebre

4--Tiene indicación de No O, 5
Terapia específica

5- Menciones 2 complicaciones 1
1- Anemia hemolítica
1
2- Hepatitis
3- Miocarditis
5- Representa algún riesgo para la madre Si porque es causa de hidrops fetal 1
embarazada?

6-Mencione otra manifestación cutánea que Sindrome purpúrico en guante y calcetín 1


se asocian a este agente etiológico

6- Tiene indicación de inmunización la madre? No 1


no hay vacuna

TOTAL 10

ESTACIÓN 4

FERNANDO, PACIENTE DE 4 AÑOS , PESA 15 KG,


SIN ANTECEDENTES PREVIOS,CONSULTA POR UN CUADRO DE 6 DÍAS DE EVOLUCIÓN
CARACTERIZADO POR TOS IRRITATIVA, A RATOS
EMETIZANTE, CON EXPECTORACION MUCOSA Y
COMPROMISO DEL ESTADO GENERAL, A LO QUE
SE AGREGA EN LAS ÚLTIMAS 24 HORAS FIEBRE
HASTA 37,8°C
AL EXAMEN SE APRECIA EN REGULARES
CONDICIONES GENERALES, AFEBRIL, DECAIDO,
CON DISCRETA DIFICULTAD RESPIRATORIA,
SATURACIÓN DE O2 DE 91% AL AMBIENTE
AL EXAMEN DESTACA MURMULLO VESICULAR
GLOBALMENTE DISMINUIDO , CON SIBILANCIAS
BILATERALES Y ESCASOS ESTERTORES FINOS EN
LA BASE DERECHA

UD DEBE VER LAS RX Y EXAMENES ADJUNTOS Y CONTESTAR EN LA HOJA DE RESPUESTAS

ESTACIÓN 4
PROTEÍNA C REACTIVA 4 MG/LT
VHS 2
HEMOGRAMA HCTO 37% HB 12,8 gr/dl LEUCOCITOS 12.500 x 10 3 /ul CON BAC 0 SEG 50 LINFO 35
MONO 15 PLAQUETOPENIA 467 x10 3 /ml ESTACIÓN 4
HOJA DE RESPUESTAS
NOMBRE………………………………………………………………………………………

ESTACIÓN 4 RESPIRATORIO
PAUTA DE CORRECCIÓN

Opción 1 y 2

6. ¿Cuál es el diagnóstico más probable? • NEUMONIA ADQUIRIDA EN LA COMUNIDAD 1

• SBO

7. ¿Cuál es la etiología más probable? • Mycoplasma pneumoniae 1

8. Señale claramente las indicaciones que debería dejar a 1- Hospitalización 0, 55


este paciente 0,5
2- Oxígeno
0,5
3- Control de temperatura y en caso de fiebre dar
Paracetamol 150 mg cada 1
8 horas oral
4- Salbutamol 2 puff cada 4 horas o NBZ de salbutamol 1
1 cc en 3 cc SF cada 4 horas
1
5- Claritromicina 15mg/kg/día por 10 a
14 días ( 0,5 y 0,5 ) 1
6- Claritromicina 250mg/5cc dar 2,25 cc cada 12horas
por 10 días ( o,33- 1
0,33-0,33)
7- Hidrocortisona dosis de carga de 10 mg/kg/por dosis 0, 5
es decir 150 mg por 1 vez
8- Hidrocortisona de mantención 5 mg/kg/dosis
75mg/cada 6 horas
9- PCR en aspirado nasofaríngeo para Mycoplasma
pneumoniae

9. Señale 2 complicaciones de este cuadro clínico • Insuficiencia respiratoria 0,25


0,25
• SDRA
• Pleuroneumonia
10. Mencione 2 manifestaciones extrapulmonares que • Eritema multiforma 0,25
pueden acompañar a este cuadro clínico 0,25
• Miringitis bulosa
• Eritema nodoso
• Serositis
• Hepatitis
• Meningoencefalitis

a. TOTAL 10

ESTACIÓN 5

UD. ESTÁ DE TURNO Y RECIBE A LA MADRE DE


ANDREA, LACTANTE DE 8 MESES QUE SE
ENCUENTRA CON UN CUADRO DE DIARREA
UG HAGA LAS CONSULTAS NECESARIAS PARA
ORIENTARSE Y DEFINIR EL ESTADO DE LA PACIENTE Y
LE DA LAS INDICACIONES

ESTACION 5 PAUTA PARA LA ACTRIZ

Ud. es la madre de Andrea, su bebé tiene 8 meses y nació por parto normal, peso 3500 grs, ha
sido sana, tiene su calendario de vacunas al día, solo ha recibido las vacunas del PNI. Ud la
controla regularmente en el CESFAM y siempre ha sido sana. Desde ayer se encuentra con fiebre
hasta 38°C que ha bajado con Paracetamol, su hija pesa 8 kg por lo cual le dio 16 gotas de
paracetamol cada 8 horas, y con esto manejo adecuadamente el cuadro febril. A esto se asocian
deposiciones liquidas más o menos 5 al día, sin mucus ni sangre, de color verde. La niña no ha
vomitado y le recibe bien el pecho y agua.
A pesar de lo anterior se encuentra activa, juega, se sonríe, orina bien, y Ud la nota como
siempre salvo cuando le sube la temperatura y presenta fiebre pero Ud. se encuentra
preocupada y desea saber que debe hacer.
La alimenta al pecho y 2 comidas sopas puré con verduras , carne , pollo , pavo o pescado,
postres de frutas y hace poco agregó legumbres

El médico le va a preguntar.
Por la alimentación La alimenta con pecho y 2 comidas sopas puré con verduras, carne , pollo ,
pavo o pescado, postres de frutas y hace poco agregó legumbres
Por las vacunas solo PNI , si le pregunta por la vacuna del rotaviris ,
Ud, contesta que no la recibió
Si asiste a sala cuna Si hace 1 mes
Si hay alguien más enfermo en la familia No
Por su estado nutricional, Ud. le contesta que ha sido sana y sube bien de peso
Si está activa Si como siempre salvo cuando está con fiebre Orina normalmente Si, y
como ha tomado mucha agua , hace bastante orina
Toma agua y pecho Si, sin problemas

Después el Dr. debe darle las indicaciones y le explica


1- Que el cuadro probablemente es viral y pasará solo
2- Tiene que darle 100 ml de agua o mezcla hidratante después de cada deposición o lo que
quiera recibir, lo importante es que tome agua
3- El niño debe seguir tomando pecho y que se reincorpore la alimentación normal lo antes
posible, si le dice que inicialmente le saque las verduras verdes y que luego las reincorpore
está correcto 4- NO debe tomar bebidas ni jugos mientras esté con diarrea. .
5- Si tiene fiebre es preferible paracetamol. A la dosis que le está dando
. NO DEBE darla diclofenaco o ibuprofeno
6- Lo manda a control en 24 horas al CESFAM
7- La educa y señala que en cualquiera de estas condiciones debe consultar antes:
deposiciones con sangre, disminución de la diuresis vómitos frecuente, mucho decaimiento
o sueño, fiebre alta .

8- Le indica probioticos ( bioflora, perenteryl, Biolactus) Ud. pregunta no necesita


antibióticos?
El médico responde que no porque es una infección viral que no se cura con antibióticos
El medico pregunta si entendió?
Ud. contesta que si y le da las gracias

ESTACIÓN 5 PAUTA DE COTEJO


-NOMBRE---------------------------------------------------------------------------------------
A- COMUNICACIÓN Y PROFESIONALISMO
Saluda , se despide y se identifica O,5
Fue cordial y se mostró interesado en solucionar dudas de la madre 0,5
Fue claro y asertivo en sus explicaciones 0,5
Permitió al paciente hablar sin interrumpir 0,5
Evitó terminología médica al hacer las preguntas y dar indicaciones 0,5
B -DESEMPEÑO CLÍNICO
Antecedentes del niño: A. Alimentación 0,5
B. Vacunas o antecedentes mórbidos

C. Asistencia a Sala-Cuna / Jardín infantil 0,5


D. Estado nutricional o controles periódicos 0,5

E. Si hay otro miembro de la familia con diarrea

0,5

0,5
Diagnóstico y Tratamiento
A. Explica que es diarrea por virus y es autolimitada 0,5
B. Deben darle al niño 100 de agua (o líquidos fisiológicos o MH) por 1 cada
deposición líquida que presente.
C. Debe volver a la alimentación normal cuanto antes o bien le da un régimen sin residuos
y después vuelve a la alimentación normal en
forma gradual 0,5
D. N debe tomar bebidas ni jugos mientras esté con diarrea. .
E. Le indica probioticos
F. No debe darle AINES 0,5
G. Si tiene fiebre es preferible paracetamol.
H. Menciona que debe traerlo a control en 24 a 48 hrs o antes si se 0,5 agrega: sangre
fecal, oliguria, fiebre alta, aumento de la diarrea,
vómitos frecuentes, decaimiento excesivo, etc. 0,5
I. NO debe dar antibióticos.
0,5
0,5

0,5
MAXIMO DE 10 PUNTOS

FIRMA DEL DOCENTE…………………………………………………..

ESTACIÓN 6

UD. SE ENCUENTRA EN EL SUI , ELENA LACTANTE DE


5 MESES CON ANTECEDENTE DE UN CUADRO
OBSTRUCTIVO HACE 2 MESES, CONSULTA POR UN
CUADRO DE 3 DÍAS DE EVOLUCIÓN CARACTERIZADO
POR CORIZA , TOS,SECRECIÓN Y ERITEMA
CONJUNTIVAL BILATERAL, DEPOSICIONES
DISGREGADAS EN 2 OPORTUNIDADES, SIN SANGRE
NI MUCOSIDADES, A LO QUE SE ASOCIA, EN LAS
ÚLTIMAS HORAS, DIFICULTAD RESPIRATORIA
PROGRESIVA
AL EXAMEN SE APRECIA DECAÍDA FEBRIL, PRESENTA ESCASA SECRECIÓN CONJUNTIVAL, FARINGE
CONGESTIVA Y ESTERTORES DE MEDIANA BURBUJA Y
SIBILANCIAS INSPIRATORIAS EN AMBOS CAMPOS
PULMONARES Y CRÉPITOS BIBASALES
CON DIFICULTAD RESPIRATORIA EVIDENTE, SATURANDO 92% AL AMBIENTE

UD DEBE VER LA RX ADJUNTA Y CONTESTAR EN LA HOJA DE RESPUESTAS

ESTACIÓN 6

ESTACION 6
HOJA DE RESPUESTA
NOMBRE…………………………………………………………………………….

ESTACIÓN 6 PAUTA DE CORRECCIÓN

1. ¿Cuál es el diagnóstico más probable? SINDROME BRONQUIAL OBSTRUCTIVO MODERADO 1


BRONCONEUMONIA

2. ¿Cuál es la etiología más probable? ADV 1

3. ¿Señale la conducta más adecuada con este 1. 2. Hospitalizar 0,5


lactante? Administración de O2 húmedo por naricera hasta lograr
3. saturación de 95% Realizar terapia acortada con 1
Salbutamol 2 puff cada 10 minutos por 5 veces y luego 1
cada 4 horas
Corticoides ev hidrocortisona dosis de carga de 10
4. mg/kg/día y luego 5
mg/kg/día cada 6 horas 1
Tomar exámenes Hemograma, PCR, VHS
Estudio viral por PCR o IFI en aspirado nasofaríngeo 0,5
5. Aseo ocular
Mantener hidratado, aporte de 100cc de agua o MH 0,5
después de cada deposición alterada
0,5
6.
7.

1. Describa los hallazgos que se ven en la Rx de Hiperinsuflación pulmonar 0,5


tórax al menos 2 Diafragmas descendidos 0,5
Infiltrado alveolar e intersticial en el 1/3 medio pulmón derecho
Condensación en 1/3 medio derecho

2. Señale 2 complicaciones de este cuadro clínico • Insuficiencia respiratoria grave 0,5


• Atelectasias 0,5
• BN con sobreinfección bacteriana

3. Mencione 2 factores de riesgo • Ambiente con contaminación atmosférica 0,5


• Uso de calefacción a gas , carbón 0,5
• Baja temperatura ambiental
• Asistencia a sala cuna
ESTACION 9 INSTRUCCIONES

UD ES MEDICO DEL CESFAM Y DEBE REALIZAR LA EVALUACIÓN NUTRICIONAL DE SOFÍA, LACTANTE DE 12 MESES CON EL
ANTECEDENTES DE RNPreT 30 SEMANAS

- UD DEBE PESAR Y MEDIR A LA


LACTANTE RELATAR PASO POR
PASO EN VOZ ALTA Y REALIZAR EL DIAGNOSTICO NUTRICIONAL

ESTACION 9
EVALUACIÓN NUTRICIONAL

PAUTA DE COTEJO

Nombre del alumno: _____________________________________________

FIRMA DEL EXAMINADOR………………………………………………………………………………………………………………….


TABLAS DE CRECIMIENTO:
ESTACION 2 HOJA DE INSTRUCCIONES

ALFONSO LACTANTE DE 10 MESES PREVIAMENTE

SANO, PRESENTA EN LAS ÚLTIMAS 24 HORAS, CRISIS

DE LLANTO, IRRITABILIDAD, RECHAZO DE LA

ALIMENTACIÓN, PALIDEZ Y DEPOSICIONES CON

SANGRE ROJA Y FRESCA. AL EXAMEN FÍSICO SE

APRECIA EN REGULARES CONDICIONES, LETARGICO,


Y LLAMA LA ATENCIÓN LA PRESENCIA DE

DISTENSIÓN ABDOMINAL CON DOLOR A LA PALPACIÓN DEL HEMIABDOMEN DERECHO.

LA MADRE LE TRAE UNA FOTO DE LAS DEPOSICIONES

UD. DEBE CONTESTAR LAS PREGUNTAS EN LA HOJA DE RESPUESTAS

ESTACIÓN 2
ESTACIÓN 2 HOJA DE RESPUESTAS
NOMBRE……………………………………………………………………………………….

ESTACIÓN 2
PAUTA DE CORRECCIÓN

1. Cuál es el diagnóstico más 1. OBS ABDOMEN AGUDO 0,5


probable? 2. OBS INVAGINACIÓN INTESTINAL 0,5

2. Señale que estudio • RX DE ABDOMEN SIMPLE 1


imagenológico realizaría en este paciente y PUEDE MOSTRAR
los hallazgos que espera encontrar y que IMÁGENES DE
apoyen su diagnóstico? OBSTRUCCIÓN
INTESTINAL
• ECOGRAFÍA PUEDE
MOSTRAR LA IMAGEN EN 1
DIANA O DONNA
3. Señale 2 diagnósticos SINDROME DISENTÉRICO 0,5
diferenciales DIVERTICULO DE MECKEL 0,5
4. Señale 2 elementos clínicos que 1. Fosa iliaca vacía al examen : signo de 0,5
contribuyen al diagnóstico de esta Dance
patología 2. Palpación de masa en HD y región del 0,5
colon transverso : Signo de
Morcilla
3. Tacto rectal + Signo de la Tenca
5. Señale 4 factores predisponentes 1. 2. Hiperplasia de placas de Peyer 0,5
para presentar esta patología Infecciones por Rotavirus, 0,5
3. 4. 5. 6. Shiguelosis 0,5
7. Alergias alimentarias 0,5
Divertículos
8. Pólipos
9. Hematomas subserosos
Defectos en la pared del lúmen
intestinal
Duplicaciones intestinales linfomas
10. Cuál es el manejo más adecuado en 1. DESINVAGINAIÓN neumática o 1
este paciente? hidrostática si tiene menos de 24
horas de evolución y no presenta
signos de compromiso sistémico o
de perforación intestinal
2. Quirúrgica si tiene mas de 24 horas
de evolución o presenta signos de 1
perforación o compromiso sistémico

11. Mencione 2 complicaciones de este Shock séptico O,5


cuadro clínico Peritonitis por perforación intestinal 0,5
Sepsis grave
Necrosis intestinal por isquemia mesentérica

10 puntos máximo

ESTACIÓN 10
INSTRUCCIONES

ESTER, RN DE 20 DÍAS DE VIDA CONSULTA POR QUE LA


MADRE NOTA QUE RECHAZA LA ALIMENTACIÓN Y ESTÁ “
CALENTITA”, LE TOMA LA TEMPERATURA RECTAL Y MARCA 39°C, POR LO CUAL DECIDE CONSULTAR EN EL
CESFAM DESDE DONDE LA DERIVAN AL SUI, CON UN PERFIL
HEMATOLÓGICO Y SEDIMENTO DE ORINA TOMADO POR RECOLECTOR
ESTER, RNTAEG DE 38 SEMANAS, SEXO FEMENINO, NACE
POR PARTO NORMAL . PN 3200 GRS ALIMENTADA AL
PECHO MATERNO EXCLUSIVO, CON BUEN INCREMENTO PONDERAL
UD. REALIZA EL EXAMEN FÍSICO Y OBSERVA A UNA RN
FEBRIL HASTA 39°C, FR 60 POR MINUTO, ROSADA PÁLIDA,
CON LLENE CAPILAR NORMAL, FONTANELA A NIVEL, Y
RESTO DEL EXAMEN SIN HALLAZGOS PATOLÓGICOS

UD DEBE VER LOS EXAMENES ANEXOS y CCONTESTAR LAS PREGUNTAS EN


LA HOJA DE RESPUESTA

ESTACIÓN 10

EXAMEN DE ORINA (TOMADO POR RECOLECTOR)


ASPECTO: turbio
COLOR: amarillo
Ph 5,5
Nitritos : negativos
G, rojos . 0- 5 x campo
Leucocitos: 0-5 por campo
Células epiteliales ++
Bacterias ++

ESTACIÓN 10
HOJA DE RESPUESTAS NOMBRE…………………………………………………………………………………….

ESTACIÓN 10 PAUTA DE CORRECCIÓN

1. ¿Cuál es el diagnóstico más probable? SINDROME FEBRIL SIN FOCO 1

2. ¿Qué cuadros clínicos debe prioritariamente O,5


descartar en esta RN?
1. SEPSIS
0,5
2. ITU 0,5
3. MAB
4. ¿Qué exámenes debe solicitar en esta RN? 0,5
1. Sedimento de orina y Urocultivo POR
0,5
SONDEO
0,5
2. Hemograma 0,5
0,5
3. PCR
4. Hemocultivos 1-2
5. Citoquímico y cultivo LCR

6. ¿Cuál es la conducta más adecuada con esta RN? 1


1. Hospitalizar
1
2. Iniciar terapia ATM 1 /1
3. Indicar Ampicilina
200mg/kg/día + cefotaxima
200mg/kg/día
0,5
4. Paracetamol para la fiebre
0,5
5. Reevaluar con los exámenes solicitados y si
el LCR se encuentra alterado y compatible
con MAB ajustar dosis de ATM

7. 10 PUNTOS

ESTACIÓN 15

UD. ES MÉDICO DEL SUI Y ATIENDE A ANA LACTANTE


DE 1 AÑO QUE ES TRAÍDA POR SU MADRE, DADO
QUE PRESENTA FIEBRE HASTA 39°C DESDE HACE 2
DÍAS ASOCIADO A VÓMITOS EN 3 OPORTUNIDADES
Y RECHAZO ALIMENTARIO, RECIBE LÍQUIDOS POR
BOCA Y LECHE MATERNA EXAMEN FÍSICO PESO: 10 KG.
BUEN ESTADO GENERAL, HIDRATADA. CARDIOPULMONAR: NORMAL
ABDOMEN: BLANDO, INDOLORO, SIN
VISCEROMEGALIAS, RUIDOS INTESTINALES
NORMALES
BIEN PERFUNDIDA
RESTO EXAMEN, NORMAL
UD LE SOLICITA EXÁMENES QUE SE ADJUNTAN
UD. DEBE:
EVALUAR EL CASO, REVISAR LOS EXAMENES Y
CONTESTAR LAS PREGUNTAS EN LA HOJA DE
RESPUESTAS

ESTACIÓN 15

EXÁMENES:

SEDIMENTO DE ORINA (TOMADO POR SONDERO VESICAL)


Aspecto: turbio Color amarillo¨ Ph : 5 Nitritos: positivos Leucocitos > 20-30 por campo
Glóbulos rojos 5 por campo Bacterias +
Células epitaliales negativas
HEMOGRAMA
Hcto. 31 % Hb 10,2 g/dl Leucocitos 16.500 /mm3
Bas 1, EOs 1, Mielo o, Juv 2, Bac 2, Sg 64; Momo 25
VHS 72 mm/h PCR 70 mg/lt

ESTACIÓN 15 HOJA DE RESPUESTA


NOMBRE……………………………………………………………….

ESTACIÓN 15 PAUTA DE CORRECCIÓN

1. ¿ Cuál es el diagnóstico más probable? Pielonefritis o ITU 1

2. ¿Cuál es el agente etiológico más E.coli 1


probable?
3. ¿Cuál es la conducta más adecuada y Indica
qué indicaciones le da a la madre? 1- Líquidos abundantes 0,5
2- Control de temperatura y en caso de fiebre de
0,5
38,5°C o más dar paracetamol
16 gotas cada 6-8 horas
1
3- Cefadroxilo 250mg/5cc 30- 50mg
/kg/día dividido cada 12 horas
4- Cefadroxilo 3 cc cada 12 horas oral por 10 días 1
5- Control en 24- 48 horas en su consultorio con 1
médico
6- En caso de vómitos, aumento del compromiso del O,5
estado general o fiebre alta mantenida debe
consultar

4. Mencione 2 complicaciones 1- Absceso renal 1


2- Sepsis grave 1
3- Daño renal crónico

5. Mencione que estudio debería realizar 1- Sedimento de orina y Urocultivo de control 0,75
al termino satisfactorio del tratamiento en 0,75
2- Ecografía renal y vías urinarias diferida
esta paciente 0,75
3- Uretrocistografía
4- DMSA opcional ( bono 0,5 ptos)

10 puntos

ESTACIÓN 12

UD ES MÉDICO DEL SAPU Y RECIBE A UNA


EMILIO RN DE 24 DÍAS DE VIDA QUE ES TRAÍDO
POR SU MADRE PORQUE NOTA QUE DESDE
HACE 1 SEMANA PRESENTA VÓMITOS DE
ASPECTO CLARO O CON LECHE, DESPUÉS DE
CADA ALIMENTACIÓN. ELLA ES UNA MADRE
PREOCUPADA, LO ALIMENTA CON LACTANCIA
MATERNA EXCLUSIVA, Y NOTA QUE SU BEBE NO
SE VE “GORDITO” Y LLORA CONSTANTEMENTE,
SEÑALA QUE “CADA DÍA VOMITA MÁS”
HA CONSULTADO VARIAS VECES
PESO AL NACER 3560 GRS Y ACTUALMENTE
PESA 3 600 GRS, SE APRECIA ACTIVO, LLORA
CONSTANTEMENTE, SIN ALTERACIONES AL
EXAMEN FÍSICO
UD. VERIFICA SI LA MADRE TIENE LECHE Y LA TÉCNICA ALIMENTARIA ES CORRECTA.

UD DEBE CONTESTAR LAS PREGUNTAS EN LA HOJA DE RESPUESTAS

ESTACIÓN 12 HOJA DE RESPUESTA


NOMBRE……………………………………………………………………………………………..
ESTACIÓN 12
PAUTA DE COTEJO

5- Señale los 1- Sindrome emético 0,75


diagnósticos sindromático 2- Sindrome de mal incremento 0,75
ponderal
6- Cuál es el diagnóstico más probable? Estenosis hipertrófica del píloro 1

7- Señale 3 1- Enfermedad por RGE 1


diagnósticos diferenciales 2- Errores congénitos del 1
metabolismo del ciclo de 1
la urea
3- Acidemias orgánicas
4- Obstrucción intestinal duodeno,
íleon

8- Qué manifestaciones 3- Alcalosis metabólica 0,5


clínicas o de laboratorio se pueden ver en hipoclorémica 0,5
este cuadro clínico 0,5
4- Visualización de ondas
peristálticas gástrica
5- Palpación de la oliva pilórica

6- A qué otros 1- Gastroenteritis eosinofílica 0,5


diagnósticos clínicos se asocia este cuadro 2- Hernia hiatal 0,5
clínico
Señale 2 3- Ulcera péptica en síndrome
nefrótico congénito
4- Cardiopatías congénitas
5- Hipotiroidismo congénito
7- Cual es el examen de elección para 1- Ecografía abdominal 1
confirmar el diagnóstico y qué espera 2- Aumento en el grosor, longitud
encontrar y diámetro del 1
piloro

10 puntos máximo

ESTACIÓN 8

UD DEBE VER LAS DISTINTAS SITUACIONES CLÍNICAS EN EL ARCHIVO ADJUNTO Y HACER EL DIAGNOSTICO
ESTACIÓN 8

HOJA DE RESPUESTA

SITUACION 1 ENFERMEDAD MANO PIE BOCA 1

SITUACIÓN 2 ESCARLATINA 1

SITUACIÓN 3 ERITEMA INFECCIOSO 1

SITUACION 4 OBS ATRESIA DUODENAL O PANCREAS ANULAR 1

SITUACION 5 ONFALOCELE 1

SITUACIÓN 6 HERNIA DIAFRAGMATICA 1

SITUACIÓN 7 HERNIA INGUIMAL 1

SITUACIÓN 8 MIELOMENINGOCELE 1

SITUACIÓN 9 CEFALOHEMATOMA 1

SITUACIÓN 10 ANO IMPERFORADO 1

TOTAL 10 PUNTOS
ESTACIÓN 13
INTRUCCIONES

UD. ESTÁ EN LA PLAYA Y SACAN DEL MAR AUN NIÑO


DE 8 AÑOS INCONCIENTE
UD. DBE PRESTAR LOS PRIMEROS AUXILIOS
ESTACIÓN 13 PAUTA DE COTEJO

NOMBRE: ---------------------------------------------

ACCIÓN PUNTAJE

Evalúa respuesta 1 pto

Pide ayuda 1 pto

Compruebe que el niño no respira 1 pto

Comprueba si hay pulso carotideo entre 5 y 10 segundos 1 pto

Posiciona correctamente al paciente 1 pto

Inicia compresiones torácicos con el talón de la mano en 1 pto


la mitad inferior del esternón

Realiza masaje en profundidad correcta (5 cm) 1 pto

Realiza 30 compresiones efectivas en 15 a 18 segundos 1 pto

Al terminar compresiones, realiza dos ventilaciones con 1 pto


mascarilla ( 30 X 2)

Lograr elevar el tórax con al menos una de las 1 pto


ventilaciones

TOTAL 10 puntos

ESTACIÓN 11 LACTANTES
HOJA DE INSTRUCCIONES PARA EL ALUMNO
UD SE DESEMPEÑA COMO MÉDICO EN EL CESFAM PUDAHUEL Y ATIENDE A MARISOL QUE TRAE A SU HIJA
MARINA DE 6 MESES A CONTROL SANO
REALICE LA ENTREVISTA QUE LE PERMITA
INFORMARSE DE LOS ASPECTOS NECESARIOS RELACIONADOS CON ESTE CONTROL DE SALUD

ESTACIÓN 11 CONTROL SANO


PAUTA PARA LA ACTRIZ:
CONTEXTO: Ud. es una madre , que lleva a su primer hijo, Pedro, de 6 meses a control sano. Su nivel
educacional es de cuarto medio y trabaja como como dependiente en una tienda de ropa. Ud. es una
mujer preocupada. Se controló el embarazo en su consultorio y tuvo un parto normal en el Hospital
San Juan de Dios. El peso de nacimiento de Andrés fue 3150 grs y la talla fue de 50 cm. Fue un RN
normal; Ud. le da pecho desde el nacimiento y no ha tenido problemas. Ud. y Pedro se fueron de alta
a los 3 días del parto. El niño fue llevado a supervisión de salud a los 15 días y el médico verificó que
la lactancia materna iba muy bien. La vacuna BCG brotó normalmente al mes y Ud. es una madre
soltera que no cuenta con la colaboración del padre para la crianza del niño , pero pese a esto la
apoya su familia y está muy contenta con el niño

Entrevista clínica a la mamá de Pedro:

Preguntas que le van a hacer a la madre:


- Si el niño ha estado bien o ha tenido
enfermedades

No, no ha tenido enfermedades

-
ALIMEN
TACIÓN
:

- Con qué se alimenta el niño: Lo alimento sólo con pecho y ahora que cumplio 6 meses inicie
sopas verduras con pollo o carne
- Cuáles son sus intervalos de amamantamiento o mamaderas: Le doy aproximadamente cada
4 horas
- Si recibe alimentos o agua extra: A las 11 horas le doy la pailla y ocasionalmente le doy agua
- Si recibe vitaminas: Sí; se llama vitaminas ACD y le doy 10 gotas al día

- HÁBITOS DEL NIÑO:


- Horas de sueño: Duerme toda la noche, de 21 hrs. hasta las 6 de la mañana, y no se despierta
en la noche. En el día duerme como 3 horas - Posición al dormir: Duerme de espaldas
- Deposiciones: Tiene la caca amarilla formada
- Llanto excesivo del niño o si es difícil de calmar: Llora cuando tiene hambre; se calma
fácilmente
- Aseo: baños. Lo baño día por medio

- OTROS:

- Eritema glúteogenital (coceduras). le pongo bastante crema al mudarlo.


- Regurgitaciones (“vómitos”, “botar leche”): Sí, bota algo de leche después de mamar desde
pequeño ; eso me preocupa. ¿ SERÁ REFLUJO?
- Congestión nasal, rinorrea (romadizo, mocos). No
-
- DESARROLLO PSICOMOTOR:
Lo estimula?: Le converso, le canto, me río con él, le pongo música
Se sienta con apoyo?si
Se cambia los objetos de mano? Si
Dice dada, gaga? Si
Se retira pañal si se lo ponen en la
cara ¿ si Se rie a carcajadas? si

- CÓMO HA ESTADO LA MADRE (ÁNIMO, SI TIENE PROBLEMAS CON LA CRIANZA DEL NIÑO):
Estoy bien y cuento con apoyo de mi madre
- Si la madre piensa volver a trabajar No por el momento no, quiero estar con mi hi
ESTACIÓN 11 - CONTROL SANO PAUTA DE COTEJO

NOMBRE…………………………………………………………………………………………
PUNTAJE PUNTAJE
OBTENIDO
El alumno debe saludar y dar su nombre la a la madre 0,5
El alumno debe despedirse de la madre 0,5
Si el niño ha estado bien o ha tenido enfermedades 0,5
Alimentación:
A. Con qué se alimenta el niño: 1
B. Cuáles son sus intervalos de amamantamiento o mamaderas: 1
C. Si recibe alimentos o agua extra 0,5
0,5
D. Si recibe vitaminas
3- Hábitos del niño:

F. Horas de sueño 0,5


G. Posición al dormir 0,5
0,5
H. Deposiciones
0,5
I. Llanto excesivo o si es fácil de calmar
0,5
J. Aseo: baños y aplicación de crema o aceite en región glútea
4- Otros: al menos 2 ítems de los enumerados a continuación

A. Eritema glúteo genital (coceduras). 0,5


B. Regurgitaciones (“vómitos”, “botar leche”) 0,5
C. Congestión nasal, rinorrea (romadizo, mocos)

5-Desarrollo Psicomotor: al menos 2 ítems de los enumerados a continuación


A. Se sienta con apoyo?
B. Se cambia los objetos de mano? 0,5
0,5
C. Dice dada, gaga?
D. Se retira pañal si se lo ponen en la cara
E. Se rie a carcajadas? si

6-Cómo ha estado la madre (ánimo, si tiene problemas con la crianza del niño): 1

10 PUNTOS = TOTAL
100% OBTENIDO =

FIRMA DEL DOCENTE…………………………………………………..


ESTACIÓN Nº: 2 PAUTA DEL ALUMNO

Usted está en un consultorio.

Acaba de ingresar a su consulta la Sra. María Eugenia para el control sano de su hijo Kevin de 4 años, quien fue a comprar
un dulce con su abuela.

Interrogue a su madre y realice la anamnesis correspondiente al control sano, mientras Kevin llega.

ESTACIÓN Nº: 2 PAUTA DE CORRECCIÓN

SI NO

1. Saluda y se presenta. O,5 0

2. Pregunta por desarrollo del lenguaje. 1 0

3. Pregunta por control de esfínteres. 1 0

4. Pregunta por vacunas. 1 0

5. Informa de la vacuna correspondiente a los 4 años. 1 0

6. Pregunta por alimentación. 1 0


7. Pregunta por hábitos de sueño. 1 0

8. Pregunta por aseo dental. 1 0

9. Responde adecuadamente a la pregunta de la mamadera (debe 1 0


dejarla cuanto antes porque favorece las caries y la mal oclusión
dental).

10. Responde adecuadamente a la pregunta de la madre sobre 1 0


control de esfínter urinario nocturno (es normal hasta los 6 años y
no es motivo de castigo o exámenes).

11. Usa un lenguaje verbal y no verbal adecuado. 0,5 0

PUNTAJE 10 0

ESTACIÓN Nº: 2 PAUTA DEL EVALUADOR

Ubíquese detrás del alumno y mantenga silencio.


El alumno está realizando el control sano de un niño de 4 años.
Realizará la anamnesis correspondiente:
Observe las conductas, sugerencias y preguntas del alumno.
Evalúe como responde a las preguntas de la madre

según pauta de evaluación.

ESTACIÓN Nº: 2 PAUTA DE PACIENTE SIMULADO

Usted es María Eugenia, madre de Kevin de 4 años a quien lleva a control sano. (No está con usted porque fue a comprar
un dulce con su abuela).

El alumno deberá preguntar sobre lo señalado con números:


1.- Lenguaje:
Responda: Habla de todo pero le cuesta pronunciar la rr.
2.- Control de esfínteres (si usa pañales, si aún se hace, etc,):
Responda: Le pone pañal en la noche.
3.- Vacunas:
Responda: Tiene todas las vacunas hasta ahora.
4.- Hábitos de sueño (como duerme, cuanto duerme):
Responda: Ya no duerme siesta y a veces se despierta en la noche.
5.- Alimentación:
Responda: Toma dos mamaderas y come dos comidas. No come ensaladas.
6.- Aseo de los dientes:
Responda: Se lava los dientes todos los días.
Si le hace otras preguntas responda de acuerdo a su experiencia o use su creatividad pero responda a todos los alumnos
lo mismo si le hacen preguntas
semejantes.

Pregúntele:

a) Si ya debe dejar la mamadera.


b) Si es normal que todavía se moje en la noche.
ESTACIÓN Nº: 2

CONTENIDO : Laringitis obstructiva aguda.

COMPETENCIA : Diagnosticar laringitis obstructiva aguda y su severidad

DESEMPEÑO : Diagnóstico de laringitis aguda obstructiva y derivación


para consultar en Servicio de Urgencia

ESTACIÓN Nº: 2 PAUTA DEL ALUMNO

USTED ESTÁ DE TURNO EN UN SAPU

CUANDO SUENE, ATIENDA EL TELÉFONO,

RESUELVA LA SITUACIÓN

INDIQUE EL DIAGNÓSTICO

ESTACIÓN Nº: 2 PAUTA DEL PACIENTE Y EVALUADOR


USTED ES LA MADRE DE UNA NIÑA QUE TIENE
DIFICULTAD AGUDA PARA RESPIRAR
SÓLO SI LE PREGUNTAN DEBE INFORMAR

• QUE TIENE 13 MESES DE VIDA


• SE DURMIÓ SIN PROBLEMAS Y HACE 15 MIN. EMPEZÓ A LLORAR MUY RONCO,
CASI SIN VOZ
CON TOS DE PERRO
• CON RUIDO EN EL PECHO, SOLO CUANDO LLORABA
• NO SE LE HUNDIAN LAS COSTILLAS
NO RESPIRABA RÁPIDO
• CONTESTE SÓLO LO QUE EL DOCTOR LE PREGUNTE
• NO SE HA PUESTO MORADA
• ESTA DESPIERTA Y LE PIDE BRAZOS

AL TERMINAR SI EL ALUMNO NO LE HA INFORMADO, CONSÚLTELE SI DEBE LLEVARLA A URGENCIA AL TERMINAR SÓLO


DELE LAS GRACIAS ESTACIÓN Nº: 2 PAUTA DE CORRECCIÓN

LOGRADO NO LOGRADO

Pregunta la edad de la paciente 0,5 0

Pregunta si tiene tos de perro 0,5 0

Pregunta si tiene taquipnea 1 0

Pregunta si tiene disfonía 1 0

Pregunta si tiene estridor inspiratorio (en reposo, con el llanto) 1 0

Pregunta si tiene retracción de partes blandas 1 0

Pregunta si tiene cianosis 1 0

Pregunta si está consciente y conectado al medio 1 0

Diagnostica Laringitis obstructiva grado I 1 0

Le indica que la hidrate bien, la haga dormir con el cuello en extensión 0.5 0
y solo darle Paracetamol si tuviera fiebre
Le indica que concurra a primera hora al 0.5 0
consultorio para evaluación

Le señala signos de alerta frente a los cuales debe concurrir de 1 0


inmediato al Servicio de Urgencia (aumento de dificultad respiratoria,
cianosis,
compromiso de consciencia)

PUNTAJE TOTAL: 10 0

ESTACIÓN Nº: 3 PAUTA DEL ALUMNO

USTED SE ENCUENTRA CONVERSANDO CON LA MADRE DE


JORGE, UN ADOLESCENTE DE 12 AÑOS,
QUE ESTÁ PREOCUPADA PORQUE
USTED LE INFORMÓ QUE ESTÁ CON
SOBREPESO
Y
DESEA TOMAR MEDIDAS PARA
MODIFICAR
LOS HÁBITOS DE ALIMENTACIÓN
Y DE ACTIVIDAD FÍSICA
DE SU HIJO

USTED DEBE
DARLE SUGERENCIAS SOBRE AMBOS
ASPECTOS EN UN LENGUAJE
ACCESIBLE

ESTACIÓN Nº: 3 PAUTA DE CORRECCIÓN

INDICACIONES LOGRADO NO LOGRADO

1.- Se presenta 0.5 0


2.- Debe comer 4 veces al día más 1-2 colaciones saludables 1 0

3- Debe recibir 3 o 4 porciones de lácteos al día 1 0


(0.5ptos.) y bajos en grasas (0.5 ptos.)

4.- Debe comer 2 platos de verduras al día (crudas o cocidas) 0,5 0

5.- Debe comer 3 frutas al día 0,5 0

6.- Debe comer pescado mínimo 2 veces por semana 0,5 0

7.- Debe comer legumbres mínimo 2 veces por semana, en 0,5 0


reemplazo de carnes

8.- Debe comer carnes de ave o vacuno sin grasa 1 vez por 0,5 0
semana

9.- Debe comer huevo 2 a 3 veces por semana 0,5 0

10.- Comer cereales, pastas o papas cocidas 1 a 2 tazas por día 0,5 0

11.- Debe comer pan 2 unidades por día 0,5 0

12.- Debe recibir aceites 6 cucharaditas por día 0,5 0

13.- Debe ingerir azúcar 6 cucharaditas por día 0,5 0

14.- Debe tomar 6 a 8 vasos de agua diario 0.5 0

15.- Debe realizar actividad física mínimo 3 veces por semana 1 0

16.- Consulta por antecedentes familiares de sobrepeso, 0.5 0


obesidad y Diabetes

17.- Le informa que estos antecedentes del padre lo convierten 0.5 0


en un niño de riesgo y que se debe poner énfasis en cumplir las
indicaciones

PUNTAJE 10 0

ESTACIÓN Nº: 3 PAUTA PACIENTE SIMULADO

USTED ES LA PREOCUPADA MADRE DE JORGE,


ADOLESCENTE DE 12 AÑOS
AL QUE EL DOCTOR ENCONTRÓ CON SOBREPESO
Y
QUE DESEA LE INDIQUE LAS MEDIDAS A TOMAR PARA QUE SU
HIJO COMA SANO Y HAGA ACTIVIDAD FISICA

EL ALUMNO YA LA ENTREVISTÓ Y EXAMINÓ A JORGE


AHORA DEBE DARLE SUGERENCIAS VERBALES
SOBRE ESTILO DE VIDA Y ALIMENTACIÓN
SALUDABLE

SOLO SI EL ALUMNO LE PREGUNTA


POR ANTECEDENTES DE JORGE,
DEBERÁ INFORMARLE
QUE NACIÓ PREMATURO
Y
QUE EL PADRE ESTÁ OBESO
NO HAY MÁS ANTECEDENTES, PUES JORGE ES MUY SANO
4 HOJA DE RESPUESTA
NOMBRE DEL ALUMNO:____________________________________ NÚMERO DEL ALUMNO:________

1.- Indique qué vacunas le corresponde colocar a una lactante sana de 2 meses de vida

2.- ¿Existe alguna otra vacuna que le puede sugerir a la madre de la lactante mencionada? (mencione 2)

3.- ¿Qué vacunas están contraindicadas en un paciente con leucemia Linfoblástica Aguda en tratamiento actual con
quimioterapia? (mencione 2)

1.-

2.-

4.- ¿Qué vacunas no presentarían mayor riesgo en su administración en este mismo caso? (mencione 4)

1.-

2.-

3.-

4.-

NO OLVIDE COLOCAR SU NOMBRE Y NÚMERO INTRODUZCA SU HOJA DE RESPUESTAS EN EL BUZÓN ESTACIÓN Nº: 4
PAUTA DE CORRECCIÓN

Si
Vacunas indicadas PAI
Triple (DPT) 1
Sabin (Polio) 1
Hib 1
Hepatitis B 1
Vacunas No PAI
Rotavirus 1.5
Neumococo (Prevenar) 1.5
Vacunas Containdicadas
BCG 0.5
Tres vírica 0.5
Varicela 0.5
Sabin (Polio) 0.5
Vacunas Permitidas
Triple (DPT) 0.5
Hib 0.5
Hepatitis A y B 0.5
Neumococcica 0.5
Influenza 0.5
Polio Inactivada 0.5
PUNTAJE 10

5 PAUTA DEL ALUMNO

FRENTE A USTED ESTÁ LA SEÑORA MARÍA

MADRE DE ELENA DE 12 AÑOS, DEPORTISTA,

SELECCIONADA DE VOLLEYBALL, QUIEN HACE UNA

SEMANA SUFRIÓ UN SÍNCOPE AL REALIZAR

DEPORTES, POR LO QUE USTED LE SOLICITÓ UN

ECG QUE AHORA ELLA LE TRAE


ANALICE EL ECG Y COMUNÍQUELE A LA MADRE SUS HALLAZGOS EN CUANTO A:

- FRECUENCIA CARDIACA - EJE QRS - APRECIACIÓN DE LOS COMPLEJOS QRS

DELE INDICACIONES A LA MADRE

RESPECTO A LA CONTINUACIÓN DE

EDUCACIÓN FÍSICA DE SU HIJA ELENA Y

RESPONDA SUS PREGUNTAS

ESTACIÓN Nº: 5 PAUTA DE CORRECCIÓN

RESPUESTA CORRECTA LOGRADO NO


LOGRADO

Calcula adecuadamente la frecuencia cardíaca 1.5 0


90 x´ +- 10 rango : 80 - 100

Refiere que tiene taquicardia 1 0

Calcula adecuadamente el eje QRS - 10º 1.5 0


(rango : -20º a + 10º)

Dice que los complejos QRS están normales 1 0

Le recomienda que por ahora no realice educación física 1 0

La deriva a cardiología infantil 1 0

Argumenta adecuadamente la derivación basada en síncope en ejercicio 1.5 0

Pregunta por antecedente familiar o refiere que ese antecedente es 0.5 0


importante
Es claro y empático, no asusta a la madre pero la deriva con convicción 1 0

PUNTAJE TOTAL 10 0

5 PAUTA DEL EXAMINADOR

UBÍQUESE FRENTE A LA PACIENTE Y


OBSERVE LA FORMA EN QUE EL ALUMNO:

- SE DIRIGE A LA MADRE
- MIDE EL EJE QRS CON EL TRANSPORTADOR
- LE EXPLICA A LA MADRE LOS HALLAZGOS DEL ECG QUE ES NORMAL
- RESPONDE ADECUADAMENTE A LAS PREGUNTAS DE LA
MADRE

PUEDE ANOTAR EN LA HOJA DE EVALUACIÓN LAS RESPUESTAS


DE CADA ALUMNO PARA QUE QUEDE MÁS CLARO EL
FUNDAMENTO DE SU CALIFICACIÓN

ESTACIÓN Nº: 5 PAUTA PACIENTE SIMULADO

Usted es María la madre de Elena de 12 años, deportista, seleccionada de voleibol en su colegio, ella hace una semana se
desmayó en el gimnasio, llevaba 30 minutos jugando y se sintió mal, se desmayó, perdió el conocimiento durante unos
segundos, se pegó en el hombro al caer. La llevaron a enfermería, estaba pálida y sudorosa
El doctor la examinó y le pidió un electrocardiograma que ahora le viene a mostrar

Espere que él haga unas mediciones y le informe si el electrocardiograma está normal o no Pídale al alumno que le
explique cómo se sabe si el ECG está normal

- Si no le dice que tiene taquicardia, pregúntele si tiene el corazón acelerado

Pregúntele si

- ¿Es peligroso lo que le pasó?


- Como María es deportista, ¿debe reducirle o suprimirle el deporte?
- Espere a ver si le dice que tiene que ir a un cardiólogo, en caso de que no lo haga pregúntele si valdría la pena
evaluarla por uno
- Si le pregunta si alguien de la familia tuvo algo al corazón, dígale que solo sabe que un hermano de su marido
tuvo un problema al corazón y murió repentinamente a los 30 años
- - Si no le pregunta de la familia, pregúntele usted si esto puede ser hereditario porque un hermano de su
marido tuvo un problema al corazón y murió repentinamente a los 30 años
6 PAUTA DEL ALUMNO

A SU CONSULTA ACUDE PARA HACERSE UN CONTROL SANO,

ANDRÉS DE 15 AÑOS

ACOMPAÑADO POR SU MADRE

PROCEDA A PESAR Y MEDIR AL ADOLESCENTE,

SACAR EL INDICE DE MASA CORPORAL

INFORME LOS RESULTADOS A ANDRÉS

Y SU MADRE

CONSIDERE SUS MANOS YA LAVADAS

6 PAUTA PACIENTE SIMULADO

USTED ES ANDRÉS, UN ADOLESCENTE DE 15 AÑOS,

QUE ACUDE PARA REALIZARSE

UN CONTROL SANO

EL MÉDICO LO PESARÁ Y MEDIRÁ,

LE CALCULARÁ EL ÍNDICE DE MASA CORPORAL

Y LE DARÁ EL RESULTADO
HAGA SOLO LO QUE EL DOCTOR LE SOLICITE Y NO PREGUNTE NADA
SI NO LE DIJO QUE SE PUSIERA LOS ZAPATOS,
HÁGALO AL TERMINAR
Y VUELVA A TOMAR EL BANANO LUEGO SOLO DE LAS GRACIAS

ESTACIÓN Nº: 6 PAUTA DE CORRECCIÓN

LOGRADO NO LOGRADO

1.- Saluda a la madre y a Andrés 0.5 0

2.- Se presenta 0.5 0

3.- Le explica a Andrés lo que le 1 0


va a realizar

4.- Le solicita a Andrés que se saque los zapatos y deje 1 0


a un lado el banano

5.- Pesa a Andrés 2 0

6.- Mide a Andrés 2 0

7.- Le solicita que se ponga los zapatos 0,5 0

8.- Calcula el IMC (peso/talla al 1 0


cuadrado)

8.- Les informa los resultados 1 0

9.- Se despide 0.5 0


PUNTAJE 10 0
ESTACIÓN N° 7 PAUTA DEL ALUMNO

UD. SE ENCUENTRA DE TURNO EN EL SERVICIO DE URGENCIA DEL HOSPITAL DE PEÑAFLOR Y RECIBE A MARÍA JOSÉ
GONZÁLEZ DE 4 AÑOS DE EDAD, QUIÉN FUÉ EXAMINADA POR EL MÉDICO DE TURNO ANTERIOR
POR PRESENTAR DESDE HACE 2 DÍAS FIEBRE ALTA, TOS CON
DESGARRO NO OBSERVADO, COMPROMISO DEL ESTADO GENERAL Y DOLOR AL INSPIRAR

TRAE UNA RADIOGRAFÍA DE TÓRAX QUE MUESTRA UNA


OPACIDAD NO HOMOGÉNEA CON BRONCOGRAMA AÉREO QUE COMPROMETE EL LÓBULO INFERIOR DERECHO.

LA PACIENTE PESA 20 KG

EN EL FACSÍMIL QUE SE ENCUENTRA A CONTINUACIÓN,

ANOTE EL DIAGNÓSTICO DE LA PACIENTE

EXTIENDA LA RECETA

TAL COMO SE LA ENTREGARÍA A LA MADRE

ESTACIÓN Nº: 7 PAUTA DE CORRECCIÓN

PJE.

1.- Anota correctamente el diagnóstico de la paciente: 1


Neumonia bacteriana
(Neumococcica o por Mycoplasma)

2.- Escribe correctamente el nombre y los datos de la paciente en la receta 1

3.- Escribe correctamente el nombre del medicamento 1


(Amoxicilina o
Claritromicina)
4.- Escribe correctamente la presentación del medicamento 1
a) Si indicó Amoxicilina: Suspensión de 500 mg.
b) Si indicó Claritromicina: Suspensión de 250 mg.

5.- La dosis, fraccionamiento y vía de administración son correctos 1.5


(0.5 ptos. c/u)
a) Si indicó Amoxicilina: 5 ml cada 8 hrs. vía oral
b) Si indicó Claritromicina: 3 ml cada 12 hrs. vía oral

6.- Los días totales de tratamiento están correctos 1


a) Si indicó Amoxicilina:7 a 10 días
b) Si indicó Claritromicina por plantear Neumococo: 7 a
10 días
c) Si indicó Claritromicina por plantear Mycoplasma: 10 a 14 días

7.- Indica antipirético 1


a) Paracetamol gotas: 40 gotas cada 8 hrs. en caso de fiebre
b) Ibuprofeno susp. de 200 mg: 5 ml cada 8 hrs. en caso de fiebre

8.- Firma con su nombre o coloca su nombre bajo la firma 1

9.- Coloca la fecha 0.5

10.- Escribe con letra clara 1

PUNTAJE TOTAL 10

ESTACIÓN N°: 13 PAUTA DEL ALUMNO

USTED ACABA DE ATENDER A PABLO CÁRDENAS DE 1 AÑO DE EDAD, QUIEN PRESENTA FIEBRE, OTALGIA, RINORREA.

DESPUES DEL EXÁMEN FISICO UD HACE EL DIAGNÓSTICO DE UNA OTITIS MEDIA AGUDA.

EL PACIENTE PESA 10 KG.

INDIQUE EL ANTIBIÓTICO DE PRIMERA LÍNEA, PRESENTACIÓN, DOSIS PONDERAL, CANTIDAD TOTAL A


ADMINISTRAR, FRECUENCIA Y DURACIÓN DEL TRATAMIENTO
(LIMÍTESE SOLO AL TRATAMIENTO ANTIBIÓTICO)

EN EL FACSÍMIL QUE SE ENCUENTRA A CONTINUACIÓN EXTIENDA LA RECETA CORRESPONDIENTE Y ENTRÉGUESELA A LA


MADRE

ESTACIÓN N°: 13 PAUTA DE CORRECCIÓN

PJE.

1. Escribe el nombre de la paciente en la receta. 1.5

2. Escribe correctamente el nombre del medicamento: 1.5


(Amoxicilina)

3. Escribe correctamente la presentación del medicamento: 0.5


(Jarabe o suspensión)

4. Señala la concentración del jarabe: 1


Jarabe o suspensión de 250 o 500 mg/5ml

5. La dosis indicada por kilo de peso es correcta: 1.5


(75-100 mg/k/día)

6. Señala la cantidad de cc a administrar: 1.5


250 mg/5 ml =5-6.5 cc c/8hrs o 6.5-10 cc c/12 hrs.
500 mg/5 ml = 2.5-3.3 cc c/8 hrs. o 3.3- 5 cc c/12 hrs.

7. El fraccionamiento es correcto: (c/8 o 12 hrs.) 0.5


0.5
8. Indica días totales de tratamiento:
( 7-10 días)

9. Firma y/o se identifica con letra legible: (0.5 p) y coloca la


fecha: (0.5 p) 1.5

TOTAL 10

ESTACIÓN Nº: 8 PAUTA DEL ALUMNO

UNA MADRE CONSULTA PORQUE SU HIJO DE 12 MESES


PRESENTA
UN AUMENTO DE VOLUMEN INGUINAL
DESDE EL DIA ANTERIOR,
QUE SE HACE MÁS EVIDENTE CON EL LLANTO

OBSERVE LA FOTO

CONTESTE LAS PREGUNTAS

EN LA HOJA DE RESPUESTAS

NO OLVIDE COLOCAR SU NOMBRE Y NÚMERO DE ALUMNO Y


DEPOSITE LA HOJA EN EL BUZÓN

ESTACIÓN Nº: 8 PAUTA DE CORRECCIÓN


PREGUNTA RESPUESTA PUNTAJE

1.- Indique el diagnóstico Hernia inguinal 2

2.- Anote tres diagnósticos - Quiste de cordón 2


diferenciales
- Adenopatías inguinales
- Testículo no descendido

3.- Escriba detalladamente todo lo - Menciona el diagnóstico 3


que le explicaría a la madre
- Que tiene indicación quirúrgica a la
brevedad
- Menciona posibles complicaciones:
(atascamiento, estrangulación)
- Que lo derivará a cirujano pediatra

4.- Menciona en qué situación - Si no se puede reducir con compresión 3


debe acudir a un Servicio de manualmente (atascamiento) acudir a
Urgencia servicio de urgencia
- Por peligro de estrangulación

TOTAL 10

ESTACIÓN Nº: 8 HOJA DE RESPUESTAS

1.- INDIQUE EL DIAGNÓSTICO

2.- ANOTE TRES DIAGNOSTICOS DIFERENCIALES

3.- ESCRIBA DETALLADAMENTE TODO LO QUE LE EXPLICARIA A LA MADRE

4.- EN QUE SITUACION DEBE ACUDIR A UN SERVICIO DE URGENCIA


9 PAUTA DEL ALUMNO

Usted está realizando una educación sobre cómo preparar una mamadera de 90 cc
de leche sucedánea de la materna (maternizada), a una madre cuyo hijo de 2 meses de vida queda con hambre y no
está subiendo bien de peso, también la madre entra a trabajar
y no logra extraerse mucha leche,
por lo tanto el niño debe recibir un relleno además del pecho

De las instrucciones desde el principio, infórmele también qué leche va a usar y a qué dilución la va a preparar,
explicando todo en voz alta, mientras realiza el procedimiento

Para la manipulación
considere la mamadera y sus manos ya limpias

9 PAUTA DE PACIENTE SIMULADO

Usted es una madre a quien el doctor le ha indicado para su hijo de 2 meses,


dar una mamadera de leche maternizada ya que está quedando con hambre y no sube bien de peso,
no logra sacarse mucha leche
y además ya va a tener que volver a trabajar
No sabe nada respecto de cómo prepararla

Espera que el doctor sea muy claro en cuanto a las indicaciones

9 PAUTA DE CORRECCIÓN
SI NO
1. Comunicación
0,5 0
Saluda a la madre/evaluadora
0,5 0
Se presenta

2. Indica que primero se debe

Lavar las manos 1 0

Lavar la mamadera 1 0

3. Preparar el agua

Hervir el agua y dejar enfriar hasta que esté tibia (0.5 ptos. c/u) 1 0

4. Preparar el biberón

Colocar la mitad del agua necesaria por biberón (aprox.50cc) 1 0

Agregar 3 medidas rasas de sucedáneo de leche materna o maternizada (1 medida por 30cc) 1 0

No agregar azúcar 1 0

Colocar el disco protector o el chupete con su tapa y agitarla 1 0

Agregar el resto de agua (aproximadamente 40 cc) 1 0

Se despide 1 0

PUNTAJE TOTAL 10 0

6 PAUTA DEL ALUMNO

Usted está realizando una educación sobre cómo preparar una mamadera de 100 cc de leche,
a la madre primeriza de Daniela, una lactante
Eutrófica y sana de 20 meses de vida.
Ella recibe 2 comidas y toma poco pecho por lo que está quedando con hambre.

Por lo tanto debe recibir un relleno.

De las instrucciones
Completas y desde el principio, infórmele también qué leche y a qué
dilución la va a preparar explicando todo en voz alta,
mientras realiza el procedimiento

Para la manipulación
Considere la mamadera y sus manos ya limpias
6 PAUTA PACIENTE SIMULADO

Usted es una madre primeriza cuya hija


Daniela de 20 meses de vida está bien de peso y sana, pero queda con hambre porque ya le está saliendo poca leche.

El doctor le ha indicado dar a Daniela una mamadera de fórmula.

Usted no sabe nada respecto de cómo prepararla y tampoco con que leche.

Espera que el doctor sea muy claro en cuanto a las indicaciones


ESTACIÓN Nº: 6 PAUTA DE CORRECCIÓN

PJE.
1. Comunicación
0,5
Saluda a la madre/evaluadora

0,5
Se presenta

2. Indica que:

Primero se debe lavar las manos 1

Luego lavar la mamadera 1

Va a usar leche Purita Cereal 1

3. Preparación del agua


1
Hervir el agua y dejar enfriar hasta que esté tibia

4. Preparación del biberón

Colocar la mitad del agua necesaria por biberón (aprox. 50cc) 1

Agregar 2 medidas rasas de leche Purita Cereal 1

Esta leche no necesita agregado de azúcar pues es dulce 1

Tampoco necesita agregar Nestum pues ya viene en la leche 1

Colocar el disco protector o el chupete con su tapa y agitarla 0.5

Agregar el resto de agua (aproximadamente 50 CC) 0.5

TOTAL 10
ESTACIÓN Nº: 11 PAUTA DEL ALUMNO

USTED ESTÁ EN EL CONSULTORIO LO HERMIDA

Y HACE EL CONTROL SANO DE KEVIN,

RECIEN NACIDO DE 7 DIAS DE VIDA

INTERROGUE A SU MADRE PARA:

REALIZAR LA ANAMNESIS

CORRESPONDIENTE AL

INGRESO DEL RECIEN NACIDO

RESPONDA LAS PREGUNTAS DE LA MADRE

ESTACIÓN Nº: 11 PAUTA DE CORRECCIÓN

LOGRADO NO LOGRADO

Pregunta por antecedentes del embarazo y parto 1 0


Pregunta por antecedentes familiares 1 0

Pregunta por edad gestacional 1 0

Pregunta por peso y talla al nacer 1 0

Pregunta por antecedentes del período perinatal inmediato 1 0

Pregunta por alimentación 1 0


Pregunta por problemas o dudas 0.5 0

A la pregunta sobre darle agua o relleno responde que no los necesita 0.5 0

A la pregunta sobre los cólicos responde adecuadamente (prevenirlos 1 0


con una buena técnica de alimentación, que son frecuentes alrededor
de los 2 o 3 meses, que generalmente no significan patología y que
habitualmente pasan solos) (2 razones)

A la pregunta sobre las ventajas de la lactancia materna responde 1 0


adecuadamente (mejor desarrollo intelectual, nutricional, emocional,
menos infecciones, mayor apego, mayor rapidez para recuperar peso
a la madre, ayuda a prevenir embarazo, menor costo, menor riesgo de
contaminación, menor riesgo de maltrato) (5 razones)

Saluda y se despide 1 0

PUNTAJE 10 0

ESTACIÓN Nº: 11 PAUTA DE PACIENTE SIMULADO

USTED ES LA MADRE DE UN RECIEN NACIDO

Y LO LLEVA A CONTROL SANO POR PRIMERA VEZ

EL ALUMNO LE HARÁ PREGUNTAS

RESPONDA DE ACUERDO A LAS SIGUIENTES CARACTERÍSTICAS:

NO HAY ENFERMEDADES IMPORTANTES EN LA FAMILIA,

EMBARAZO NORMAL, PARTO NORMAL,

38 SEMANAS DE EDAD GESTACIONAL,

PESO NACIMIENTO 3.200 GRS, TALLA NACIMIENTO50 CM,


NO TUVO COMPLICACIONES DESPUES DEL PARTO,

ESTÁ TOMANDO SOLO PECHO, NO LE HA DADO OTROS ALIMENTOS

SI LE PREGUNTA POR DUDAS DÍGALE:

¿TENGO QUE DARLE AGUA O RELLENO?

¿QUE PUEDO HACER CON LOS CÓLICOS?


FOTO Nº 1

FOTO Nº 2

FOTO Nº 3

Opción 1 Etapa del botón mamario.


Opción II Crecimiento de la areola y del pezón, apareciendo tres contornos. Opción III Mayor desarrollo de mama y areola
pero sin separación de sus contornos. Se inicia el crecimiento del pezón (sin menarquia)
12 PAUTA DE CORRECCIÓN

COLOQUE EL NÚMERO DE OPCIÓN QUE CORRESPONDE A CADA FOTO


INDICANDO ADEMAS LA EDAD BIOLÓGICA

OPCION EDAD BIOLOGICA

FOTO 1 II 11 años 1.5 PUNTO


1.5 PUNTO

FOTO 2 1.5 I 1.5 PUNTO 10 años y PUNTO


6 meses

FOTO 3 III 12 1 años 1.5 PUNTO


1.5 PUNTO

TOTAL = 9 PUNTOS + 1 PUNTO DE BASE

13 PAUTA DEL ALUMNO

USTED ESTÁ EN UN CONSULTORIO HACIENDO EL CONTROL SANO


DE UN NIÑO DE 6 MESES DE EDAD

YA REALIZÓ LA ANAMNESIS Y EL EXAMEN FÍSICO

CONSIDERANDO LA EDAD DEL NIÑO,

MENCIONE A LA MADRE LAS PRINCIPALES

INDICACIONES O SUGERENCIAS PARA

PREVENIR ACCIDENTES

RESPONDA A LAS PREGUNTAS

DE LA MADRE

13 PAUTA DE CORRECCIÓN

LOGRADO NO LOGRADO

Menciona la prevención de caídas 1 0

Menciona prevención de quemaduras 1 0

Menciona que debe viajar en un automóvil con silla fija en el asiento 1 0


trasero y con cinturón de tres puntas

Menciona cuidado con el sol, uso de bloqueador 1 0

Responde adecuadamente a la recomendación de juguetes o juegos 1 0


(propicie el juego libre y al aire libre, evite juguetes peligrosos)

Responde adecuadamente a la pregunta sobre quemaduras (lavar 1 0


con abundante
agua fría de la llave)
Responde adecuadamente a la pregunta sobre TEC (observar 1 0
conciencia, conducta y mantener la calma)

Menciona prevención de aspiración de cuerpo extraño porque se 1 0


lleva todo a la boca, le sugiere juguetes plásticos especialmente
diseñados para succión

Menciona la supervisión constante del niño y advierte que las 1 0


piscinas son muy peligrosas

Saluda y se despide 1 0

PUNTAJE 10 0
PJE.
Menciona prevención de asfixia por inmersión. 1

Menciona prevención de quemaduras. 1

Menciona que debe viajar en un automóvil en el asiento trasero, con silla adecuada. 1

Menciona prevención de intoxicaciones con productos de aseo o medicamentos. 1

Menciona prevención de aspiración de cuerpo extraño. 1

Menciona la prevención de caídas. 1

Menciona la supervisión constante del niño y advierte que es un periodo de exploración. 1

Responde adecuadamente a la recomendación de juguetes o juegos (es normal que le 1


guste jugar solo, propicie el juego libre y al aire libre, evite juguetes peligrosos).

Responde adecuadamente a la pregunta sobre quemaduras (lavar con abundante agua fría 1
de la llave).

Responde adecuadamente a la pregunta sobre TEC (observar conciencia, conducta y 1


mantener la calma).

TOTAL 10

ESTACIÓN Nº: 13 PAUTA DEL PACIENTE SIMULADO

USTED ES LA MADRE DE UN NIÑO DE 6 MESES

QUE LO LLEVA A CONTROL SANO

EL ALUMNO YA CONVERSÓ CON USTED Y EXAMINÓ AL NIÑO AHORA DEBE DARLE SUGERENCIAS VERBALES PARA

PREVENIR ACCIDENTES

CALCULE QUE NO SE LE ACABE EL TIEMPO Y


PREGÚNTELE:

1.- QUÉ HACER EN CASO DE QUEMADURA

2.- QUÉ HACER SI SE CAE Y SE PEGA EN LA CABEZA

3.- QUÉ HACER PORQUE A SU HIJO LE GUSTA


LLEVARSE TODO A LA BOCA

4.- AHORA QUE HACE CALOR, SI LO PUEDE


BAÑAR EN UNA PISCINITA PLÁSTICA EN EL
JARDÍN

5.- SI NO LE DICE CÓMO LLEVARLO EN AUTO, PREGÚNTELE SI YA PUEDE VIAJAR EN EL


ASIENTO DELANTERO AMARRADO CON EL
CINTURÓN DE SEGURIDAD

ESTACIÓN Nº: 14 PAUTA DEL ALUMNO

USTED ESTÁ DE TURNO EN UN HOSPITAL PEDIÁTRICO

LE TRAEN UNA LACTANTE DE 9 MESES SEXO


FEMENINO SIN ANTECEDENTES MÓRBIDOS,
ASISTE A SALA CUNA,
NUNCA RECIBIÓ LACTANCIA MATERNA,
VACUNAS PAI AL DÍA,
MAL PROGRESO PONDERAL
DESDE AYER TIENE FIEBRE DE 39,7°
PRESENTÓ DOS CONVULSIONES Y VÓMITOS, EN DOS
OCASIONES EXPLOSIVOS,
SE ENCUENTRA IRRITABLE, CON QUEJIDO, NO DEMANDA NI ACEPTA ALIMENTACIÓN.
NO SE TRANQUILIZA EN BRAZOS DE SU MADRE
AL EXAMEN Tº 39.5 AX, FRECUENCIA CARDÍACA 165 x¨,
FRECUENCIA RESPIRATORIA 60 x¨,
ES DIFÍCIL EVALUAR LA RIGIDEZ DE NUCA POR LA
IRRITABILIDAD,
LA FONTANELA ANTERIOR NO ESTÁ ABOMBADA
EXAMEN PULMONAR SIN RUIDOS AGREGADOS
RESTO DEL EXAMEN NORMAL

RESPONDA LAS PREGUNTAS DE LA


HOJA DE RESPUESTAS
NO OLVIDE COLOCAR SU NOMBRE Y NÚMERO
INTRODUZCA SU HOJA DE RESPUESTAS EN EL BUZÓN
ESTACIÓN Nº: 14 HOJA DE RESPUESTAS

NOMBRE DEL ALUMNO:________________________________ NÚMERO DEL


ALUMNO:________________________________

1.- DIAGNÓSTICO SINDROMÁTICO


_______________________________________________________________
_______________________________________________________________

2.- DIAGNÓSTICO PRESUNTIVO


_______________________________________________________________
_______________________________________________________________

3.- INDIQUE LOS 4 EXÁMENES MÁS IMPORTANTES QUE


SOLICITARÍA Y QUÉ ESPERARÍA ENCONTRAR EN ELLOS
PARA EVALUAR AL PACIENTE Y ACLARAR EL DIAGNÓSTICO

a)
_______________________________________________________________

b)

c)

d)
_______________________________________________________________
_______________________________________________________________
_______________________________________________________________ ESTACIÓN Nº: 14 PAUTA
DE CORRECCIÓN
LOGRADO NO LOGRADO

1.- Diagnóstico sindromático:


Síndrome febril complicado o Sepsis o
S. meníngeo 1.5 0

2.- Diagnóstico presuntivo:


Meningitis bacteriana 1.5 0

3.- Estudios complementarios:


a) Punción lumbar 1 0
Refiere purulento, abundantes glóbulos blancos predominio
PMN, glucosa disminuida) 1 0
Gram 0.5 0
Cultivo 0,5 0
Latex de LCR 0,5 0

b) Hemograma 0,5 0
Leucocitosis con desviación a izquierda VHS elevada 0,5 0
0.5 0

c) PCR 0,5 0
Sobre 60 0,5 0

d) Hemocultivos 1 0

e) Si menciona ELP y no lo ha hecho con alguno de los exámenes 0.5 0


anteriores (si mencionó los 4 previos no se considera)

PUNTAJE TOTAL 10 0

ESTACIÓN Nº: 15 PAUTA DEL ALUMNO

USTED ESTÁ ATENDIENDO EN UN CONSULTORIO

ACUDE LA MADRE DE PEDRO,


UN NIÑO DE 6 AÑOS

INTERROGUE A SU MADRE POR EL MOTIVO

DE CONSULTA

RESPONDA A SUS INQUIETUDES

ESTACIÓN Nº: 15 PAUTA DE CORRECCIÓN

PREGUNTA POR LOGRADO NO


LOGRADO

Motivo de consulta 1 0

Tiempo de evolución de la constipación (desde cuando es estítico), si la 1 0


asocia con algo (relación con el ingreso al colegio, dieta, etc.) (0.5 ptos.
c/u)

Características de las deposiciones (consistencia, frecuencia, 1 0


asociación con rectorragia, encopresis o dolor , si la constipación es
fluctuante) Menciona 5 (0.20 ptos. c/u)

Alimentación recibida (frutas, verduras, líquidos, comida chatarra) 1 0

Uso de medicamentos que pueden influir en la 1 0


estitiquez (Ej. Uso de fierro)

Antecedentes mórbidos personales o familiares 1 0

RECOMIENDA
Alimentación rica en fibras 1 0
Líquidos naturales y agua 1 0

Hábitos de ejercicio, sueño y llevarlo al baño 3 veces al día (refiere al 1 0


menos 2), idealmente permitirle defecar tranquilo

Si no resulta la dieta, indica Lactulosa como medicamento o similar 0.5 0


(maltin, etc), eventualmente fleet enema evacuante

Lo cita a control en un plazo razonable para ver evolución 0.5 0

PUNTAJE 10 0

El evaluador puede subir o bajar hasta dos puntos anotando la razón, por ejemplo preguntar por hitos del desarrollo que no
corresponden a la edad ESTACIÓN Nº: 15 PAUTA DE PACIENTE SIMULADO

USTED ES LA MADRE DE UN NIÑO DE 6 AÑOS, PEDRO EL ALUMNO LE HARÁ PREGUNTAS, RESPONDA:


LO LLEVA A CONTROL PORQUE ES ESTÍTICO Y LE CUESTA
MUCHO HACER CAQUITA
CUANDO ERA CHICO Y TOMABA PECHO NO TENÍA PROBLEMAS HACÍA CAQUITA VARIAS VECES AL DÍA, DESPUES AL
SUSPENDER EL PECHO SE PUSO MAS ESTÍTICO, PERO LA MADRE LO SOLUCIONABA CON JUGOS
O LE COLOCABA SUPOSITORIOS DE GLICERINA O ENEMAS
CUANDO INGRESÓ AL COLEGIO COMENZÓ A AGUANTARSE
NO LE GUSTA IR AL BAÑO EN SU COLEGIO YA QUE LA PUERTA NO CIERRA BIEN Y LOS COMPAÑEROS MOLESTAN Y HACEN
BURLAS, ADEMAS ESTAN MUY SUCIOS
DOS VECES HA LLEGADO CON LOS CALZONCILLOS SUCIOS
Y A USTED LE DA TEMOR QUE LOS NIÑOS SE EMPIECEN A
BURLAR DE ÉL, LOS NIÑOS SON CRUELES A VECES
PEDRO TOMA 2 LECHES CON CHOCOLATE DIARIAS, COME LA COMIDA NORMAL DE LA CASA, NO LE GUSTAN LAS
VERDURAS, DE FRUTA COME SOLO PLÁTANO (UNO TODOS LOS DÍAS) Y LAS BEBIDAS LE ENCANTAN
PEDRO NO TOMA REMEDIOS Y ES SANO,
EN LA FAMILIA NADIE TIENE ENFERMEDADES
Y TAMPOCO NADIE ES ESTÍTICO
CUANDO LE INDIQUE REMEDIOS PREGÚNTELE SI ESO NO LE
VA A HACER MAL, SI NO SE VA A ACOSTUMBRAR

SI LE REALIZAN OTRAS PREGUNTAS RESPONDA DE ACUERDO A


SU EXPERIENCIA O USE SU CREATIVIDAD

ESTACIÓN Nº: 16 PAUTA DE CORRECCIÓN

FOTO 1
1 Un niño de 6 años, previamente sano presenta desde hace dos días aumento de volumen cervical, de 3x3 cms,
móvil, no adherido a planos profundos ni superficiales, levemente sensible, sin otra sintomatología

PUNTAJE
A Señale 3 etiologías posibles (0.5 ptos. c/u) 1. Linfadenitis 1.5
2. Enf rasguño de gato
3.Tbc
4. Linfoma
5. Toxoplasmosis
6. S. MONI
B Señale dos exámenes de utilidad para el 1.Hemograma 1
diagnóstico diferencial (0.5 ptos. c/u) 2. IgM Bartonella
3. IgM Virus Epstein Barr
4. Ig M Toxoplasmosis
5. Punción

FOTO 2

2 Observe esta foto de la región perianal de un niño de 4 años con prurito EN la misma zona

PUNTAJE
A ¿A qué parásito corresponde? Oxiuros 1.5

B ¿Qué examen de control solicita después del Test de Graham 1


tratamiento?

FOTOS 3 y 4

3 Una lactante presenta estas lesiones y su hermana de 7 años presentan prurito y estas lesiones en las muñecas

PUNTAJE
A ¿Qué infección presentan? Sarna 1
B ¿El agente causal es? Sarcoptes scabiei 1

C ¿En base a qué se realiza el diagnóstico? La clínica 1

D En caso de duda se puede solicitar el examen Acarotest 1


llamado

E Señale dos complicaciones (0.5 ptos. c/u) Infección bacteriana, impétigo 1

FOTO 1 FOTO 2
FOTO 3 FOTO 4
ESTACIÓN Nº: 17 PAUTA DEL ALUMNO

EN LA PANTALLA DEL COMPUTADOR OBSERVE LA

RADIOGRAFÍA DE CADERAS DE

EMILIA UNA LACTANTE DE 3 MESES DE EDAD

QUE VIENE A CONTROL SANO

CON LA RADIOGRAFÍA DE SCREENING SOLICITADA EN

EL CONTROL ANTERIOR

UTILICE UNA HOJA CON LA FOTOGRAFÍA PARA

HACER LOS CÁLCULOS, EXPLIQUE EN VOZ ALTA

LUEGO EMITA EL DIAGNÓSTICO

RESPONDA LAS PREGUNTAS DE LA MADRE


ESTACIÓN Nº: 17 PAUTA DE CORRECCIÓN

PREGUNTAS RESPUESTAS PUNTAJE

1.- ¿Podría decir que la Rx es adecuada para Si es adecuada para emitir un diagnóstico porque en la Rx 2
emitir un diagnóstico? ¿En qué elementos se aprecian las alas simétricas, los agujeros obturadores
anatómicos se basa para su decisión? simétricos y el extremo proximal del fémur
es también simétrico

2.- ¿Qué método alternativo se pudo usar Ecografía de caderas 1


para el diagnóstico?

3.- Mida los ángulos acetabulares Dibuja los ángulos y los mide resultando ser mayores a 30º 2

4.- Emita el diagnóstico basado en la Displasia bilateral de caderas o enfermedad del desarrollo 2
conclusión radiológica de la cadera bilateral, con subluxación bilateral

5.- ¿En qué basa su diagnóstico? Ángulos > 30º 1


Cejas cotiloideas ausentes
Proyección de las cabeza femoral en subluxación

6.- ¿Qué indicaciones recomienda? Derivación inmediata a traumatólogo para inicio 1


tratamiento con correas de Pavlik

7.- ¿Qué pudo haber producido esto? Herencia, posición intrauterina, hiperlaxitud ligamentosa 1

PUNTAJE TOTAL 10
PREGUNTAS RESPUESTAS PJE.
1.¿El ángulo acetabular es normal o patológico? Es patológico porque mide más de 30°. 1.5
¿Cuál? ¿Por qué? Ambos son patológicos.

2. ¿Las cabezas femorales se proyectan en buena No, porque se proyectan fuera 1.5
posición? del cuadrante inferointerno.
¿Cuál? ¿Por qué? Ambas.

3. Las cejas cotiloídeas, No, porque no están bien desarrolladas. 1


¿Se observan o no? ¿Por qué?

4. Según sus observaciones la conclusión Displasia bilateral de caderas. 1


radiológica es:

5. Su conclusión se basa en: 1. Angulo mayor de 30°. 1


(indique 2 razones) (0.5 p c/u)
2. Falta de desarrollo de cejas cotiloideas.
3. Cabeza femoral proyectada fuera del cuadrante inf
int.

6. ¿Qué recomienda a la madre de este niño? Llevarlo a traumatólogo infantil.


1.5

7. En qué situación solicitaría usted una - Antecedente familiar de displasia.


ecografía de caderas antes del mes de edad 1
(mencione 2) (0.5 p c/u)
- Ante la duda de displasia.
- RN con ex físico alterado.
- Presentación podálica.
- Oligohidroamnios.

8. Señale 3 factores de riesgo para presentar - Sexo femenino - Madre primípara.


displasia de caderas. (0.5 p c/u) 1.5
- Presentación podálica.
- Antecedente familiar de displasia. - Otras
displasias esqueléticas.
- Oligohidroamnios - Útero bicorne.

TOTAL 10

ESTACIÓN Nº: 17
FOTO DE LA RADIOGRAFÍA PARA LOS CÁLCULOS

ESTACIÓN Nº: 17 PAUTA PACIENTE SIMULADO

USTED ES LA MADRE DE EMILIA, UNA NIÑA DE 3 MESES, QUE VIENE A SU CONTROL SANO
Y
TRAE LA RX DE CADERAS QUE LE SOLICITARON EN EL CONTROL ANTERIOR EL ALUMNO REVISARÁ LA FOTO Y HARÁ LOS
CÁLCULOS
Y
LUEGO LE INDICARÁ EL DIAGNÓSTICO

USTED LE HARÁ LAS SIGUIENTES PREGUNTAS:

1.- ¿ESTÁ BIEN TOMADA LA RX?


2.- ¿CÓMO LO SABE USTED?
3.- ¿EXISTE OTRA FORMA DE EVALUAR A MI HIJA QUE NO SEA CON LA RX?
4.- ¿CÓMO SE HACEN LAS MEDICIONES PARA EVALUARLA?
5.- ¿QUÉ DIAGNÓSTICO TIENE EMILIA?
6.- ¿QUÉ COSAS VIÓ EN LA RX QUE LE HACEN DAR ESE DIAGNÓSTICO?
7.- ¿QUÉ ME RECOMIENDA HACER AHORA?
8.- ¿QUÉ PUDO CAUSARLE ESTO?

EMILIA ES UNA NIÑA SANA, SU PRIMERA HIJA, PRODUCTO DE UN


EMBARAZO Y PARTO NORMAL
USTED TIENE EL ANTECEDENTE DE HABER TENIDO QUE
HACERSE TRATAMIENTO CUANDO CHICA POR ALGO PARECIDO

SI LE REALIZAN OTRAS PREGUNTAS RESPONDA DE ACUERDO A


SU EXPERIENCIA O USE SU CREATIVIDAD

ESTACIÓN Nº: PAUTA DEL ALUMNO

Usted está en un consultorio donde llega Margarita, madre de

Antonella, lactante menor de 3 meses quien viene con una Rx de caderas que solicito otro médico.
Interrogue a su madre y realice

la anamnesis correspondiente, analice la Rx, realice el diagnóstico y manejo.


AL EXAMEN FISICO

Ortolani Barlow (-)


Pliegues simétricos
Sin limitación a la abducción
ESTACIÓN Nº: PAUTA DE CORRECCIÓN

SI NO

12. Saluda y se presenta. O,5 0

13. Pregunta por factores de riesgo displasia de caderas(o nombra)(3 3 0


minimo)
• Presentación nalgas
• Antecedentes familiares
• Sexo femenino
• Primigesta
• Embarazo múltiple. Oligohidramnios Primogénito.
• Malformaciones asociadas
• Producto macrosómico
14. Pregunta si se ha realizado estudios anteriores 0,5 0

15. Describe la técnica de la Rx y relata que es adecuada. 1 0


Esta centrada por:
-Simetría de alas iliacas
-Simetria de agujeros obturadores
-Simetría de porción proximal del femur

16. Traza líneas: 1 0


-Hilgenreiner
-Perkins
-Arco de Shenton

17. Traza correctamente ángulo acetabular 1 0

18. Diagnostica la Rx como normal ya que: -Tiene un A acetabular 2 0


<30º
-Nucleo de osificación de cabeza del femur se encuestra en
cuadrante inferomedial
-Arco de Shenton es continuo
-Linea de perkins divide porción proximal del femur en relación 1/3-
2/3

19. Explica a la madre que la Radiografía esta normal, y que al no 1


tener mayores factores de riesgo se da el alta ortopédico.

PUNTAJE 10 0

ESTACIÓN Nº: PAUTA DE PACIENTE SIMULADO

Usted es Margarita madre de Antonella, ella es su segunda hija y nacio de


“cabecita” parto natural a las 38 semanas. No ha tenido ninguna enfermedad desde su nacimiento, y nadie en su familia ha
presentado alguna enfermedad a la cadera.

Ud trae consigo una radiografía tomada a Antonella hace una semana, indicada por otro Dr, no sabe porqué y se lo
pregunta al Médico.

Ud está preocupada ya que una amiga del trabajo tiene una hija con displasia de caderas a la cual tuvieron que operar y no
quiere que esto le pase a su hija.

Pregunte al doctor si esta seguro del diagnóstico ya que ud vee que antonella mueve poco las piernas y aun no gatea.
ESTACIÓN Nº: 2 PAUTA DEL ALUMNO

USTED ESTÁ EN SU CONSULTA


HACIENDO EL CONTROL SANO DE LOS
6 MESES A JUAN ,A QUIEN CONOCE DESDE RECIÉN
NACIDO, POR LO QUE YA TIENE SUS
ANTECEDENTES PERINATALES (TODO NORMAL),
MÓRBIDOS Y FAMILIARES (NADA SIGNIFICATIVO)

INTERROGUE A SU MADRE Y REALICE

LA

ANAMNESIS CORRESPONDIENTE

AL CONTROL SANO

No debe hacer examen físico, ni dar indicaciones,


Solamente preguntar los demás aspectos correspondientes a un niño de 6 meses para aclarar si todo va bien y ha
alcanzado lo que corresponde a su edad.

ESTACIÓN Nº: 2 PAUTA DE CORRECCIÓN

PJE.
1. Saluda a la madre. 1

2. Pregunta por: XXX


- Alimentación recibida 1
- Suplementos Vitamínicos y de fierro recibidos (fierro o sulfato ferroso). 1

- Vacunas 1
- Desarrollo psicomotor grueso: Sentarse con apoyo. 0.5
- Desarrollo motor fino: Coger objetos y transferirlos de mano. 0.5

- Desarrollo social y sensorial: Sonríe, sigue objetos con la vista, se alegra con personas 0.5
conocidas.

- Desarrollo del Lenguaje: Balbucea – sílabas y disílabas. 0.5


3. Hábitos: Sueño-deposiciones. 1

4. Responde correctamente la pregunta de las vacunas: Triple 1.5


(Difteria, tétanos, tos convulsiva), Polio, Haemophilus Influenzae, Hepatitis B.
La Triple le puede producir fiebre.

5. Hace prevención de accidentes y recomienda el no uso de andador. 1.5

PUNTAJE TOTAL 10

Puede subir o bajar hasta dos puntos anotando la razón, por ejemplo preguntar por hitos del desarrollo que no
corresponden a la edad

ESTACIÓN Nº: 2 PAUTA PACIENTE SIMULADO

USTED ES LA MADRE DE UN NIÑO DE 6 MESES, KEVIN Y LO LLEVA A CONTROL SANO.


EL ALUMNO LE HARÁ PREGUNTAS

• RECIBE SOLO LECHE MATERNA, CADA VEZ QUE PIDE

• LE DA ADEMÁS VITAMINAS ACD 5 GOTAS AL DÍA

Y GOTITAS DE FIERRO 12 AL DÍA

• TIENE PUESTAS TODAS LAS VACUNAS Y LE ACABAN DE


ADMINISTRAR LA DE LOS 6 MESES

• SE SIENTA CON AYUDA DE ALMOHADONES.


• COGE OBJETOS CON LAS MANOS Y SE LOS PASA DE UNA MANO A OTRA.

• SONRÍE A CARCAJADAS, CONOCE A LA MAMÁ, PAPÁ Y HERMANOS.

• DICE BA BA Y TA TA.

• DUERME SIESTA EN LA MAÑANA Y EN LA TARDE Y DESPIERTA UNA VEZ EN LA NOCHE A TOMAR PECHO.

• HACE CAQUITA NORMAL DOS VECES AL DÍA Y PIPÍ ABUNDANTE, CLARO Y SIN MAL OLOR.

PREGÚNTELE PARA QUÉ SON LAS VACUNAS QUE LE PUSIERON


Y SI LE VAN A PRODUCIR FIEBRE
PREGÚNTELE SI LO PUEDE PONER EN ANDADOR

SI LE HACEN OTRAS PREGUNTAS RESPONDA DE ACUERDO A SU EXPERIENCIA O USE SU


CREATIVIDAD
ESTACIÓN Nº: 3 PAUTA DEL ALUMNO

AYER USTED EXAMINÒ A UNA NIÑA DE UN AÑO

QUE ES TRAÍDA POR SU MADRE

PORQUE LA ENCUENTRA “MARILLA”.

USTED LA EXAMINÓ Y CONFIRMÓ ESTE HECHO

Y LE SOLICITÓ

PRUEBAS HEPÁTICAS Y UN HEMOGRAMA QUE

LOS ENCUENTRA EN HOJAS PLASTIFICADAS

LEA LOS EXÁMENES Y ANALÍCELOS

LUEGO RESPONDA LAS PREGUNTAS EN LA HOJA DE RESPUESTAS

NO OLVIDE COLOCAR SU NOMBRE Y NÚMERO DE ALUMNO EN


LA HOJA DE RESPUESTAS
INTRODUZCA SU HOJA DE RESPUESTAS EN EL BUZÓN
3 PAUTA DE CORRECCIÓN

ITEM RESPUESTA PUNTAJE

1 El hemograma esta: Alterado 0.5


(Refiérase solo a si está normal o alterado)

2 Las pruebas hepáticas son: (Refiérase solo a Normales 0.5


si están
normales o alteradas)

3 El diagnóstico más probable es: Anemia hipocroma, microcítica, no regenerativa, 2


lo más probable es que sea ferropriva

4 Escriba tres razones en las que fundamenta 3


el diagnóstico :
Puede mencionar 3 de estos 4 (1 punto
c/u, pero si informa las razones correctas 1.- hemoglobina bajo el nivel normal para la
pero no da las cifras límites solo 0.5 edad 10,5 - 11 gr. 2.- hipocromía por CHCM baja
puntos) inferior a 30 33
3.- microcitosis por VCM bajo inferior a 70 80
4.- no regenerativa por falta de reticulocitosis o
inferior a 3

5 ¿Le indica algún tratamiento?: describa Sulfato ferroso 3 – 5 mg /kg/día hasta 1


normalizar la hemoglobina y por mínimo 3
meses para llenar los depósitos

6 Indique dos aspectos que le explica a la 2


madre respecto al Diagnóstico y
tratamiento: 1.- Es frecuente la falta de fierro en los niños
Puede mencionar 2 de estos 5 2.- Es importante que ingiera carne de vacuno
(1 punto c/u) o carnes rojas 3.- No debe ingerir te
4.- El tratamiento es largo
5.- Tendrá deposiciones negras por el fierro del
medicamento

7 ¿Lo cita a control? Si, con hemograma en un mes 1


¿Cuando? y ¿con qué?
(0.5 puntos si responde solo a una
pregunta)

TOTAL 10
ESTACIÓN Nº: 3 PAUTA DEL ALUMNO

HEMOGRAMA

HEMOGLOBINA 9 gr./Dl.

HEMATOCRITO 26 %

VCM 69 fentolt
CHCM 28 %
HCM 23 pg/cel
RETICULOCITOS 1 %
LEUCOCITOS 8.400 x mm3
PLAQ 550.000 x mm3
VHS 35 mm hora

BAS EOS JUV BAC SEG LINF MONO


0 3 0 2 58 31 6

CARACTERÍSTICAS AL FROTIS

GLOBULOS ROJOS microcitosis hipocromía moderada


LEUCOCITOS normales
PLAQUETAS normales

PRUEBAS HEPÁTICAS

TRANSAMINASAS GOA : 28-30 U/ml


BILIRRUBINA TOTAL : 0.6 mg/dl
% BILIRRUBINA DIRECTA: 20%
PROTROMBINEMIA : 90%

4 PAUTA DEL ALUMNO


Paula Escolar, 12 años, acude con su madre a su consulta para hacerse un control de salud.

Paula tuvo su menarquia a los 10 años.

Actualmente pesa 54 kilos y mide 1,54 m.

DIAGNOSTIQUE SU ESTADO NUTRICIONAL


(Detalle todos los pasos que siguió para su evaluación)

ESTACIÓN Nº: 4 PAUTA DE CORRECCIÓN

PJE.
1. IMC Peso / talla2 0.5

54 / 2.3716= 22.7 0.5

IMC = 22,7 0.5

2. Edad biológica E B x Menarquia = 12 años 8 meses 2


E. cronológica menarquia = 10 años
E. cronológica actual = 12 años
EB menarquia - EC menarquia = 2 años 8 meses 1.5
EC actual 12 + 2 años 8 meses = 14 años 8 meses
E B = 14 años y 8 meses

1.5

3. Diagnóstico nutricional Eutrófica 3.5

TOTAL 10
17 PAUTA DEL ALUMNO

USTED ESTÁ DE TURNO EN UN SAPU: ATIENDA EL TELÉFONO y RESUELVA LA SITUACIÓN E INDIQUE EL DIAGNÓSTICO

CERCA DE 30 SEGUNDOS DESPUÉS DEL TIMBRE


LLAME AL NÚMERO......

USTED ES LA MADRE DE UNA NIÑA QUE ACABA DE PRESENTAR


UNA CONVULSIÓN Y DICE….

“DOCTOR: LO LLAMO YA QUE MI NIÑA ACABA DE


PRESENTAR ALGO QUE CREO QUE PUEDE SER UNA
CONVULSIÓN…”

SÓLO SI LE PREGUNTAN DEBE INFORMAR

• QUE TIENE 13 MESES DE VIDA.


• ES LA PRIMERA VEZ EN SU VIDA QUE LE SUCEDE.
• QUE HOY BRUSCAMENTE EMPEZÓ CON FIEBRE.
• LA NIÑA ESTABA SANA ANTES DE LA CONVULSION.
• LA CONVULSIÓN DURÓ MAS O MENOS UN MINUTO Y MEDIO.
• SE LE FUERON LOS OJOS HACIA ATRÁS Y SE LE SACUDIÓ TODO EL CUERPO.
• PERDIÓ LA CONSCIENCIA.
• SI LE PREGUNTA POR RELAJACIÓN DE ESFÍNTERES (ORINARSE O DEFECARSE) RESPONDA QUE USA PAÑALES Y
NO SE HA DADO CUENTA.
• CONTESTE SÓLO LO QUE EL DOCTOR LE SOLICITE.
SI EL ALUMNO NO LE PREGUNTA, CONSÚLTELE SI DEBE LLEVARLA A URGENCIA

AL TERMINAR SÓLO DELE LAS GRACIAS Y CORTE.

ESTACIÓN Nº: 17 PAUTA DE CORRECCIÓN


PJE.

Pregunta la edad de la paciente. 1

Pregunta sobre el estado de la niña antes del episodio. 1

Pregunta si tuvo fiebre. 1

Pregunta si le ha pasado esto antes. 1

Pregunta tiempo de duración de la convulsión. 1

Pregunta características de la convulsión (única, localizada o generalizada, 2


pérdida de conciencia, relajación de esfínteres).

Diagnostica convulsión febril simple. 2

Le indica que debe consultar en urgencia. 1

TOTAL 10

5 PAUTA DEL ALUMNO

OBSERVE LAS 5 FOTOS Y ANOTE LOS DIAGNÓSTICOS EN LA HOJA DE RESPUESTAS

ESTACIÓN Nº: 5 PAUTA DE CORRECCIÓN

FOTO DIAGNÓSTICO PUNTAJE


1 HIDROCELE 2

2 HERNIA UMBILICAL 2

3 PARAFIMOSIS 2

4 BALANITIS 2

5 FIMOSIS 2

TOTAL 10
FOTO 2
FOTO 3

FOTO 4

FOTO 5

ESTACIÓN Nº: 16 PAUTA DEL ALUMNO


EN LA HOJA DE RESPUESTAS
DETALLE EN FORMA COMPLETA

QUE VACUNAS LE CORRESPONDEN A


UN NIÑO DE 4 MESES DE EDAD
SEGÚN EL CALENDARIO DE
VACUNACIÓN DEL
PROGRAMA ACTUALIZADO DE
VACUNAS DE CHILE

NOMBRE DOS
VACUNAS EXTRA PAI QUE
TAMBIEN LE RECOMENDARÍA
COLOCAR

ESTACIÓN Nº: 16 PAUTA DE CORRECCIÓN

4 MESES VACUNAS PROTEGE VIA PUNTOS


INDICADAS CONTRA

Vacunas DPT Difteria


PAI Tétanos IM 2
Coqueluche

POLIO O SABIN ORAL Poliomielitis Oral 2

HAEMOPHILUS Haemophilus Influenzae tipo B IM 2


INFLUENZAE B

Virus Hepatitis B
HEPATITIS B IM 2
Vacunas ROTARIX Rotavirus Oral 1
no PAI
PREVENAR Neumococo IM 1
(7 serotipos)

TOTAL 10

ESTACIÓN Nº: 8 PAUTA DEL ALUMNO

USTED EXAMINÓ A MATÍAS DE


6 MESES DE EDAD, DIAGNOSTICÓ
BRONQUITIS OBSTRUCTIVA.

DECIDE INDICARLE COMO TRATAMIENTO SALBUTAMOL 2 PUFF C/ 6 HRS

DEBE ENSEÑARLE A LA MADRE EL USO DEL INHALADOR DE


DOSIS MEDIDA (IDM)

DELE A LA MADRE LAS INDICACIONES DE LA TÉCNICA CORRECTA DE USO DEL INHALADOR

HAGA LA RECETA Y ENTRÉGUESELA A LA MADRE

ESTACIÓN Nº: 8 PAUTA DE CORRECCIÓN


PJE.

Colocar al niño sentado en la falda de la madre. 1

Conectar el inhalador a la cámara agitando previamente el inhalador. 1

Colocar bien la cámara incluyendo la boca y nariz del niño. 1

Se aplica 1er puff y se espera aproximadamente 10 seg. 1

Le dice que entre cada puff debe esperar 1 o 2 minutos y retirar la cámara de la cara. 1

Le explica que no importa o incluso es mejor que llore. 0,5

Le dice que no se puede aplicar sin cámara porque los efectos adversos son más intensos. 1

Le explica que los inhaladores no son dañinos ni se acostumbrará si lo usa solo cuando se lo 1
indican.

Le explica que los inhaladores se dan en el consultorio porque es patología cubierta por el 1
AUGE.

Llena en forma correcta la receta. 1

Utiliza un lenguaje comprensible para el paciente. 1

TOTAL 10

ESTACIÓN Nº: 8 PAUTA PACIENTE SIMULADO


USTED ES LA SEÑORA AMELIA Y VIENE CO SU HIJO MATÍAS DE 6 MESES.

LE DIAGNOSTICARON BRONQUITIS OBSTRUCTIVA Y LE INDICARON INHALADOR.

EL DOCTOR LE ENSEÑARÁ A USAR EL INHALADOR.

PREGÚNTELE:

SI IMPORTA QUE LLORE CUANDO LE PONE EL INHALADOR.

SI SE LO PUEDE COLOCAR DIRECTO A LA BOCA PARA NO TENER QUE COMPRAR LA CÁMARA.

SI NO LE VA A HACER MAL USAR INHALADORES PORQUE USTED HA ESCUCHADO QUE PRODUCE


ACOSTUMBRAMIENTO.

¿EN EL CONSULTORIO LE VAN A DAR EL INHALADOR O TIENE QUE COMPRARLO?

ESTACIÓN Nº: 12 PAUTA DEL ALUMNO

USTED ESTÁ REALIZANDO UN TURNO DE PEDIATRÍA EN UN HOSPITAL.


EL MÉDICO DEL TURNO ANTERIOR ATENDIÓ A UN PACIENTE DE
19 MESES. PRESENTABA DESDE HOY FIEBRE DE 38,3, BUEN
ESTADO GENERAL, FRECUENCIA CARDÍACA 100 Y FRECUENCIA RESPIRATORIA 25.
LE SOLICITÓ ESTUDIOS QUE LOS TIENE SOBRE LA MESA.
ANALICE LOS RESULTADOS.

INDIQUE EL DIAGNÓSTICO QUE SOSPECHA


Y
EL TRATAMIENTO SI LO REQUIRIERA

HOJA DE EXÁMENES

- Radiografía de tórax normal.


- Recuento de glóbulos blancos 13. 500
Neutrófilos: 70 %
Linfocitos: 20 %
Monocitos: 10%
- Proteína C reactiva: 40
- Orina completa (por sondaje)
Leucocitos: 18 por campo
Hematíes: 6 por campo
Glucosa: no contiene
Proteínas: no contiene
Nitritos: positivos
Urocultivo: pendiente

ESTACIÓN Nº: 12 PAUTA DE CORRECCIÓN

PJE.

Hemograma con: Leucocitosis y desviación izquierda (0.5 p c/u) 1

PCR: Aumentada. 1

Orina: Alterada. 1

Diagnóstico presuntivo: Infección urinaria. 2

Indicaría tratamiento: Si. 1

Tratamiento adecuado: Cefalosporina de 1ª (Cefadroxilo). 2

Dosis adecuada: 50 mg/kg/día cada 12 hrs. 1

Tiempo de tratamiento: 7 a 10 días. 1

TOTAL 10
ESTACIÓN Nº: 15 PAUTA DEL ALUMNO

USTED ESTÁ EN EL CONSULTORIO LO HERMIDA FRENTE A USTED ESTÁ FRANCISCO DE 8 AÑOS


ACOMPAÑADO POR SU MADRE.

ES TRAÍDO POR PRESENTAR TOS Y FIEBRE DE 39º DESDE HACE 3 DIAS

AL EXÁMEN FÍSICO SE ENCUENTRA FEBRIL (38,5 º AX.)

USTED SOLICITÓ UNA RADIOGRAFÍA DE TÓRAX QUE LA TRAEN EN ESTE MOMENTO


Y LA PUEDE OBSERVAR EN EL NEGATOSCOPIO

DESCRIBA

LOS HALLAZGOS PATOLÓGICOS DE LA


RADIOGRAFÍA

DIAGNÓSTICO

TRATAMIENTO
(MEDICAMENTOS, DOSIS, DURACIÓN Y
VÍA DE ADMINISTRACIÓN)

CONTROL RADIOGRÁFICO ¿CUANDO?


Y ¿EN QUÉ CASOS?
15 PAUTA DE CORRECCIÓN

NOMBRE DEL ALUMNO:____________________________________ NÚMERO DEL ALUMNO:______________

SI NO

1. Analiza placa: AP y L 0.5 0

2. Describa los campos pulmonares: Describe zona de 3 0


condensación no homogénea con broncograma aéreo
en
lóbulo inferior izquierdo

3. Describa los senos costofrénicos: 1 0


Libres

4. Diagnóstico radiológico: 2 0
Neumonía Lóbulo Inferior Izquierdo

5. Etiología posible:
Neumocócica

5.Tratamiento: 1 0
- Amoxicilina

- Dosis: 80-100 mgs por kg. 0.5 0

- Vía : oral 0.5 0

- Duración: 7 días 0.5 0

6. Control radiográfico:
- ¿Cuándo?: 4-8 semanas después 0.5 0

- ¿En qué casos?: Neumonía redonda, compromiso 0.5 0


pleural, bronquiectasias

PUNTAJE TOTAL 10

10 PAUTA DEL ALUMNO


USTED ATIENDE A RAÚL UN NIÑO DE 10 AÑOS, CUYA MADRE ESTÁ MUY PREOCUPADA POR QUE EL NIÑO MUESTRA UNA
OBESIDAD IMPORTANTE (IMC > P95) CON TALLA NORMAL

EVALÚE EN EL PACIENTE LOS FACTORES DE RIESGO FAMILIARES

COMUNÍQUELE A LA MADRE DEL NIÑO,


DE ACUERDO AL RIESGO ENCONTRADO,
LOS ESTUDIOS DE
LABORATORIO BÁSICO PERTINENTES
QUE DEBE REALIZARLE
(NO HACE FALTA ANOTARLOS)

ESTACIÓN Nº: 10 PAUTA PACIENTE SIMULADO

Usted es la madre de un niño con obesidad. Usted tiene diabetes y colesterol alto (mas de 300 miligramos).
La hermana del niño es obesa.
Su esposo tiene hipertensión arterial.
El abuelo paterno del niño falleció de un infarto al corazón a los 50 años.

10 PAUTA DE CORRECCIÓN

PJE.
1. Comunicación:

Saluda a la madre/evaluadora. 0,5

Se presenta. 0,5

2. Interroga sobre:

Obesidad en la familia. 1

Diabetes en la familia. 1

Hipertensión en la familia. 1

Dislipidemia en la familia. 1

Infarto al miocardio o accidente cerebro vascular. 1

3. Exámenes que solicita:

Glicemia. 2

Perfil lipídico. 2
TOTAL 10
ESTACIÓN Nº: 9 PAUTA DE CORRECCIÓN

DESARROLLO PSICOMOTOR DE UN NIÑO SANO

Señale los hitos presentes en las fotos Edad aproximada a la que debe estar presente PJE.

PÁGINA 1

Foto A Se sienta con apoyo 0.5 6 meses 0,5 1

Foto B Gatea 0.2 9 meses 0,5 0,7

Foto C Levanta la cabeza en prono 0.5 3 a 4 meses 0,5 1

PÁGINA 2

Foto A Camina sin apoyo 0.5 12 a 14 meses 0,5 1

Foto B Se para con apoyo 0.2 9 a 10 meses 0,5 0,7

Foto C Camina tomado de la mano 0.2 12 meses 0,5 0,7


(o con apoyo)

PÁGINA 3

Foto A Coge objetos 0.5 4 meses 0,5 1

Foto B Sonrisa social 0.5 2 meses 0,5 1

Foto C Gira sobre su eje 0.2 5 meses 0,5 0.7

Foto D Coge objetos y los lleva a la 5 meses 0,5 0,7


boca 0.2
PÁGINA 4
Escriba los reflejos del recién nacido, presentes en las fotos:

Foto A Prehensión plantar 0,5

Foto B Marcha 0,5

Foto C Succión 0,5

TOTAL 10

2
Temas clave de Pediatría y Cirugía Infantil

A B C

AB

1
6
Temas clave de Pediatría y Cirugía Infantil

C D

ESTACIÓN Nº: PAUTA DEL ALUMNO

Usted está en un SU.

Acaba de ingresar a su consulta la Sra. Kimberly Rojas

Quien trae a su hijo Briyatan, lactante de 6 meses quien hace 2 días presenta fiebre alta, hoy esta un poco mejor aunque lo vee con “manchas en la piel”.

Interrogue a su madre y realice la anamnesis correspondiente, diagnóstico, agente etiológico y manejo.

1
6
Temas clave de Pediatría y Cirugía Infantil

AL EXAMEN FISICO:

Lactante en BEG, afebríl(37ºC).


Máculas y pápulas rosa pálido, con forma de almendra, bien delimitadas que se tornan blancas a la presión, granulosas al tacto.

ESTACIÓN Nº: 2 PAUTA DE CORRECCIÓN

SI NO

1. Saluda y se presenta. O,5 0

2. Pregunta por fiebre y caract 1 0

3. Pregunta por CEG y síntomas acompañantes(vómitos diarrea,etc). 1 0

4. Pregunta por vacunas. 1 0

5. Pregunta si hay alguien más enfermo en la casa 1 0

6. Pregunta por alimentación, ropa, cambio de shampoo/jabón. 1 0

7. Hace el diagnóstico de exantema súbito. 1 0

8. Agente etiológico virus herpes 6/7. 1 0

1
6
Temas clave de Pediatría y Cirugía Infantil

9. Responde adecuadamente a la pregunta de si es contagioso(si, 1 0


hasta que comiencen a desaparecer las lesiones en piel, aprox 2
dias).

10. Calma a la madre y explica que es un episodio benigno y 1 0


autolimitado, explica probables complicaciones(convulsión
febríl y encefalitis/hepatitis raro)

11. Deja tratamiento sintomático con antipiréticos en caso de fiebre. 0,5 0

PUNTAJE 10 0

ESTACIÓN Nº: PAUTA DE PACIENTE SIMULADO

Usted es Kimberly, madre de Biyatan de 6 meses a a quien lleva al SU por presentar fiebre de 39ºC desde hace 3 días y hoy por la mañana despertó con
manchas en todo el cuerpo, primero en tronco y luego en cara, son ropjas y ud tiene mucho miedo a que su hijo se muera

Es su 1º hijo, y tiene todas las vacunas al dia(este mes corresponde ponerle vacunas nuevamente). Su hijo es sano hasta el momento sin antecedentes de
riesgo

No ha cambiado de ropa, shampoo,jabón, no le pican las ronchas a su hijo.

UD ESTA MUY PREOCUPADA CREE QUE SU HIJO SE VA A MORIR!!! QUIERE A TODA COSTA QUE EL DR LO DEJE HOSPITALIZADO!!!!

Una vez más tranquila pregunte al Dr si es contagioso ya que a veces Briyatan comparte con su primo de 1 año, y en tal caso desde cuando deja de ser
contagioso.

1
6
Temas clave de Pediatría y Cirugía Infantil

ESTACIÓN Nº: PAUTA DEL ALUMNO

Usted está en un SU.

Acaba de ingresar a su consulta la Sra. Dominga Infante, quien trae a su hija Donatella, escolar de 9 años quien hace 1 semana presenta fiebre alta,
vómitos, y decaimiento.
Interrogue a su madre y realice la anamnesis correspondiente, diagnóstico, agente etiológico y manejo.

AL EXAMEN FISICO:
Paciente decaída, conciente, orientada, pálida, febril, 38,5º ax,
FC 100x´, PA100/60, FR 20x´,
Piel seca Se observan 5 maculopápulas rojas en abdomen levemente pruriginosas de 5 mm
Lengua saburral Sin adenopatías
Pulmones libres
Abdomen difusa y levemente sensible, poco depresible,
“plástico”
Hígado a 3 cms y bazo a 2 cms del reborde costal

EXAMENES DE LABORATORIO
Hemograma
Hb 11,5 GB 8500 Neutr 63%
Bac 15 VHS 35
PCR 80 (N 0 – 10)
Orina N
Rx tx N
Hemocultivo pendiente
ESTACIÓN Nº: 2 PAUTA DE CORRECCIÓN

SI NO

1
6
Temas clave de Pediatría y Cirugía Infantil

1. Saluda y se presenta. O,5 0

2. Pregunta por fiebre y características, si ha disminuido con algo 1 0

3. Pregunta por CEG y síntomas acompañantes(,dolor de cabeza). 1 0

4. Pregunta por vómitos y caract, dolor abdominal, 1 0


diarrea/constipación y orina

5. Pregunta si hay alguien más enfermo en la casa 1 0

6. Pregunta si ha comido mariscos crudos e indaga en hábitos 1 0


higiénicos.

7. Solicita Exámenes 1 0
-Hemocultivo
-Hemograma
-VHS
-PCR
-OC y Urocultivo
-Rx Tx

8. Diagnostica Fiebre Tifoídea 1 0


Etiología Salmonella typhi (bacilo de Eberth)

9. Indica tratamiento: CLORANFENICOL 50 MG/KG/DÍA cada 1 0


8 hrs a las 48 hrs afebril bajar a 25 mg/kg/día. Mínimo por
14 días
10. Medidas no farmacológicas
-REPOSO mínimo 3 semanas desde el inicio
-REGIMEN liviano
-MEDIDAS HIGIÉNICAS
“aislamiento entérico”

1
6
Temas clave de Pediatría y Cirugía Infantil

11. Probables complicaciones: 0,5 0


-Peligro de perforación intestinal a la tercera semana -Recaída 7-
10%
-Colecistitis

PUNTAJE 10 0

ESTACIÓN Nº: PAUTA DE PACIENTE SIMULADO

Su hija Donatella presenta desde hace 8 días fiebre sostenida de 38-39.5º, decaimiento, inapetencia, sudoración nocturna, mareos, dolor de cabeza
intenso que la hace llorar, náuseas, vómitos de escasa cantidad, no explosivos, dolor abdominal generalizado, constipación, orina escasa, oscura.

La fiebre remite con paracetamol a 37.8 y al cabo de 4 hrs asciende nuevamente a 39

En un viaje a Puerto Monnt hace dos semanas comió mariscos crudos, pero
“cocidos con limón”.
Vive en una casa con buenas condiciones higiénicas y tiene todos los servicios básicos.

Nadie más en la familia se enfermó.

1
6
Temas clave de Pediatría y Cirugía Infantil

ESTACIÓN Nº: PAUTA DEL INTERNO

LEA ATENTAMENTE LAS INSTRUCCIONES QUE SE DETALLAN A


CONTINUACIÓN

USTED ESTÁ ATENDIENDO EN EL SERVICIO DE URGENCIA DEL HOSPITAL DE SAN EXPEDITO DONDE
CONSULTA LA SRA. ELEUTERIA CON SU HIJO
RUPERTINO DE 2 AÑOS 7 MESES QUIÉN PRESENTA TOS EMATISANTE,CEG y SECRECIÓN CONJUNTIVAL PURULENTA BILATERAL. HACE 5 DIAS PRESENTÓ
TOS SECA, CORIZA MUCOSA, CONSULTÓ EN SU
POLICLÍNICO DONDE SE LE DIAGNOSTICÓ FARINGITIS
AGUDA. HOY HA TENIDO FIEBRE Y ESTA MÁS DECAÍDO, POR LO CUAL CONSULTA.

LA MADRE TRAER LA RX QUE LE SOLICITO HACE


UNOS MOMENTOS EL MEDICO QUE LE ENTREGO A USTED EL TURNO.

REALICE LA ANAMNESIS CORRESPONDIENTE, PLANTEE UNA HIPÓTESIS DIAGNÓSTICA, Y DE


INDICACIONES

EX FISICO:

Tº: 39 °C
FC: 180 X'
FR: 60 X'
CEG
RETRACCIÓN COSTAL (+)
SIBILANCIAS AUDIBLES (+)
HEMODINÁMICAMENTE ESTABLE.

PULMONAR:
DISMINUCIÓN DIFUSA DEL MURMULLO PULMONAR
ESPIRACIÓN MUY PROLONGADA SIBILANCIAS EN AMBOS TIEMPOS CREPITES BILATERALES.

SAT02: 94%

LEUCOCITOS: 18 900/MM3
NEUTROFILIA DE 10 773/MM3
PCR 146 MG/L

1
7
Temas clave de Pediatría y Cirugía Infantil

ESTACIÓN Nº: 10 PAUTA DE CORRECCIÓN

PUNTOS
SALUDA Y SE PRESENTA A LA SRA MARIA 1 0

1
7
Temas clave de Pediatría y Cirugía Infantil

1.- Anamnesis: 3 0
Pregunta por antecedentes perinatales
Pregunta por patologias previas
Pregunte por medicacion recibida
Antecedentes sociales, va al jardin vive en casa con calefaccione ¿hay problemas de abrigo?
¿Alguien fuma en la casa?
¿Alguien mas de la familia esta enfermo?
Como se alimenta ¿Recibio lactancia materna? Si va a jardín
infantil?

- -

2.- Hipótesis diagnóstica: NEUMONIA VIRAL 2 0

3.- Diagnóstico Etiológico probable: 2 0


PROBABLEMENTE POR ADENOVIRUS

4 Indicaciones a la madre 1 0
Reposo en cama
Adecuada hidratación
Alimentación fraccionada según tolerancia sin residuos
Evitar exceso de abrigo
Control de Tº c/ 3 hrs (pregunta a dra Rodríguez)
Paracetamol 10-15 mg/Kg/Dosis máximo c/6-8 hrs si Tª>38ºc Axilar SBT 2puff c/4 hrs
Signos de alarma

5.- Control: 1 0
Control en 48 horas, anticipar el control si presenta signos de alarma

1
7
Temas clave de Pediatría y Cirugía Infantil

TOTAL PUNTOS 10 0

ESTACIÓN Nº: PAUTA DE PACIENTE SIMULADO

POR FAVOR LIMÍTESE SOLO A CONTESTAR LO QUE EL INTERNO LE PREGUNTA.

USTED ES LA SRA. ELEUTERIA, QUE TRAE AL SERVICIO DE URGENCIA DEL HOSPITAL DE SAN EXPEDITO A SU HIJO RUPERTINO, UN PRE-ESCOLAR DE 2
AÑOS 7 MESES, POR PRESENTAR CUADRO CLÍNICO QUE COMENZÓ 5 DÍAS ANTES DE SU HOSPITALIZACIÓN, CARACTERIZADO POR TOS SECA, CORIZA
MUCOSA, CONSULTÓ EN SU POLICLÍNICO DONDE SE LE DIAGNOSTICÓ FARINGITIS AGUDA, INDICÁNDOSELE AMOXICILINA (65 MG/KG/DÍA) E
IBUPROFENO. POSTERIORMENTE SE AGREGÓ FIEBRE, TOS EMETISANTE, COMPROMISO DEL ESTADO GENERAL, DEPOSICIONES LÍQUIDAS, SECRECIÓN
CONJUNTIVAL PURULENTA BILATERAL. SU HIJO RUPERTINO PRESENTÓ DIFICULTAD RESPIRATORIA PROGRESIVA POR LO QUE DECIDIÓ CONSULTAR EN
EL SERVICIO DE URGENCIA. USTED ESTA MUY PREOCUPADA PORQUE SU HIJO ESTA MUY DECAÍDO Y HA TENIDO FIEBRE, HASTA 39 GRADOS.

SI EL ALUMNO PREGUNTA POR OTROS SINTOMAS SOLO INDIQUE QUE HA PRESENTADO DEPOSICIONES LIQUIDAS EN ALGUNAS OPORTUNIDADES (NO
RECUERDA
CUANTAS) PERO QUE NO HAN SIDO ABUNDANTES

TERMINADO DE INTERROGARLA, PREGUNTE ¿Cuál es el diagnostico de su hijo? y si hay que hospitalizarlo.

PREGUNTELE SI LE VA A DAR ALGUN ANTIBIOTICO.

Pregunte cuando debe traerlo a control

FINALMENTE SE DESPIDE Y AGRADECE

ESTACION 1 (2007)
NOMBRE:

Martín es un lactante de 2 meses, alimentado con leche materna exclusiva. Es un RNT AEG, peso de nacimiento : 3260 gr Talla : 50 cm CC : 35 cm
Durante el control actual : Peso : 4.700 gr Talla 57 cm CC 37.5cm
Examen físico normal y desarrollo psicomotor normal

1.- Realice la evaluación nutricional del niño según tablas OMS


3 puntos

P/E : -1DE T/E : N P/T : N

2.- Si la madre no puede alimentarlo con leche materna.Nombre 2 alternativas de alimentación e indique forma de preparación

1
7
Temas clave de Pediatría y Cirugía Infantil

2 puntos

Leche maternizada o Nan 13%


Purita 7,5% + azúcar 3% + aceite 1%

3.- Indique SOLO cuatro tareas normales del desarrollo psicomotor que presenta el niño a los 2 meses 3 puntos

- Sigue la argolla en ángulo de 90º


- En posición prono levanta el tórax
- Reacciona al sonido de la campanilla
- Busca con la mirada la fuente del sonido
- Sonríe frente a la sonrisa del examinador
- Ríe a carcajadas
- Intenta sostener la cabeza al ser llevado a posición sentado

4.- Identifique las vacunas que debe tener el niño hasta los dos meses
2 puntos

BCG al nacer
1ª dosis de DPT-SABIN- Hib – Hepatitis B

ESTACION 3 (2008)
NOMBRE:

Paciente de 3 años 6 meses, eutrófico, que cursa cuadro tipo viral respiratorio alto desde hace 3 días. Recibe medidas generales y paracetamol en caso
de fiebre.
Desde anoche con más fiebre que se ha mantenido constante, tos con secreciones y dificultad respiratoria progresiva, rechazo alimentario y vómitos
frecuentes.
Al examen físico destaca : febril 39.5ºC , polipnea, taquicardico y sudorosos, con retracción subcostal, aleteo nasal, quejido y saturando 89% con O2
ambiental.

1.- Diagnostico más probable ( lo más completo posible) 2 puntos


Neumonía
Insuficiencia respiratoria aguda

2.- ¿Qué exámenes pediría? ( nombre 4 ) 2 puntos


Hemograma
PCR
Gases arteriales
Rx de tórax

1
7
Temas clave de Pediatría y Cirugía Infantil

2.- Describa brevemente en una línea la Rx de tórax que ve en la pantalla


1 punto
Foco de condensación en lóbulo superior izquierdo

3.- ¿Cuál sería su conducta? ( lo más importante, máximo 3) 2 puntos


Hospitalizar
Oxigenoterapia
Antibióticos EV

4.- ¿Cuál es el gérmen más probable?: 1 punto


Streptococo neumoniae

5.- ¿Qué antibiótico dejaría, en que dosis y como lo fracciona en el día?: 2 puntos

Penicilina sódica, 100.000 a 200.000 U/kg/día, cada 6 horas EV

ESTACION 10 (2008)
NOMBRE:

Ingresa a Urgencia , lactante de 1 año 2 meses, que la madre lo trae porque hace 2 días presenta algo de coriza y fiebre y desde ayer con tos que moviliza
secreciones y se agrega dificultad respiratoria, inapetencia, y vómitos con la tos.
Al examen físico está febril con 38ºC, con frecuencia respiratoria de 50 x min, cianosis perioral al llanto, aleteo nasal presente y con retracción subcostal
e intercostal.
A la auscultación pulmonar destaca estertores bilaterales y se auscultan sibilancias en inspiración y espiración.

1.- Asigne el puntaje del Score de obstrucción bronquial en lactante a cada uno de los signos que tiene el paciente y concluya el Score final
FRECUENCIA RESPIRATORIA : 2 puntos (1)
SIBILANCIAS : 2 puntos (1)
CIANOSIS : 1 punto (1)
RETRACCIÓN : 2 puntos (1)

SCORE TOTAL: 7 (2) 6 puntos

2.- Según score calculado, ¿cuál sería el manejo adecuado? 2 puntos

2 puff de salbutamol cada 10 min por 5 veces.

3.- Si a la hora de tratamiento sigue con el mismo Score ¿ qué indica?. Nombre los medicamentos y sus dosis 2 puntos
2 puff de salbutamol cada 10 min por 5 veces + Hidrocortisona 5 – 10 mg/kg /dosis EV o Prednisona 1-2 mg/kg en dosis única

1
7
Temas clave de Pediatría y Cirugía Infantil

ESTACION 8 (2008)

NOMBRE:

EXAMEN DE ORINA

Aspecto : turbio
Ph : 5.5
Densidad : 1015
Glucosa : negativo
Proteínas : 15 mg/dl
Sangre : 2-4
Leucocitos : 4-6
Piocitos : 50 -60
Cilindros : negativo

1.- Según el examen anterior, ¿Cuál sería el diagnóstico más probable? 2 punto
Infección urinaria

2.- Si el examen de orina anterior perteneciera a un lactante de 5 meses que presenta fiebre sobre 38ºC, decaimiento, vómitos, intolerancia alimentaria y
sin foco aparente al examen físico.¿cuál sería el diagnóstico ahora? 2 puntos

ITU alta o Pielonefritis aguda

3.- ¿Cuáles serían los 2 gérmenes más probables que encontraría en el Urocultivo? 1 punto
E. Coli
Proteus mirabilis

4.- ¿Cuál sería el tratamiento más adecuado para este paciente en este momento? Nombre medicamento, dosis y frecuencia. 3 puntos
Cefotaxima 100 mg/kg/día cada 6 horas EV

4.- Una vez terminado su tratamiento ¿qué se debe hacer con este paciente?( en forma general) 2 puntos

Estudiar con imágenes


Queda con quimioprofilaxis

1
7
Temas clave de Pediatría y Cirugía Infantil

ESTACION 2 (OSCE 2008)


NOMBRE:

Recién nacido de sexo masculino, PN 1750 gr.,EG 32 semanas, Apgar


8-8-9, producto de cesárea por Síndrome Hipertensivo del Embarazo, que a los 10 minutos de vida inicia quejido respiratorio y retracción torácica. A su
ingreso a UCI neonatal presenta Tº 36.5 ºC axilar, FR 76 x min y FC 140 x min, cianosis peribucal y en su examen pulmonar muestra disminución del
murmullo pulmonar con estertores finos basales

OBSERVE RADIOGRAFIA QUE SE PRESENTA

1.- ¿Cuál es el diagnóstico más probable de este RN?:


Membrana hialina

2.- ¿Cuáles serían las medidas que evitarían o minimizarían esta patología?
Prevenir el parto prematuro
Uso de corticoides maternos

3.- ¿Cuál es la fisiopatología que produce esta enfermedad?


Falta de surfactante

4.- ¿Cuál es el tratamiento indicado ( generalidades)


UCI neonatal
Oxigenoterapia con VM
Uso de surfactante exógeno

ESTACION 10 (2008)

NOMBRE:

Lactante varón de 6 meses de edad, con antecedentes de PN 2.900 gr y talla 49 cm. La madre refiere que el menor ha sido sano hasta ahora, con la
salvedad que a los 15 días de vida le detectaron un soplo cardíaco que no fue estudiado en esa oportunidad.

1
7
Temas clave de Pediatría y Cirugía Infantil

Ahora consulta por notar que el bebé cuando despierta en las mañanas, en algunas ocasiones, llora intensamente, se pone morado alrededor de la boca,
posterior a lo cual queda flácido, pálido y como que “se queda en el llanto”.
Al examen clínico actual: peso 6.500 gr, rosado, con leve cianosis peribucal y de lechos ungueales, gran eritema de conjuntivas palpebrales. FC 120 x min,
Pr. Arterial 80/40 mmHg, pulsos palpables en las 4 extremidades, frémito en región precordial (+).
Pulmones normales. Corazón RR en 2 tiempos, con soplo holosistólico 3-4 / 6 en borde paraesternal, 2º ruido único y reforzado, diástole libre.
Abdomen normal con hígado en reborde costal.

1.- ¿Qué diagnóstico plantearía?

2.- ¿Qué exámenes pediría y describa brevemente lo que encontraría


( Nombre 3 exámenes)

3.- Conducta a seguir y cuando lo realizaría

ESTACION 7 (2008)

NOMBRE:

Usted recibe en el Servicio de Urgencia a un paciente de 4 años con cuadro de inicio brusco caracterizado por fiebre de 39,5ºC y gran compromiso del
estado general. En las últimas horas se agregan vómitos explosivos y compromiso sensorial caracterizado por tendencia al sueño e hipoactividad

1.- Señale 2 hipótesis diagnósticas más probables 2 puntos


Meningitis bacteriana ( 1 ) DEBE ESTAR
Septicemia
Cetoacidosis diabética
Intoxicación

2.- Al examen físico usted constata la presencia de signos meníngeos y lesiones en ambos pies que Ud. ve en la pantalla, por lo que decide hacer una
punción lumbar.
Según el gram que Ud ve en la pantalla, ¿qué gérmen es el causal del cuadro infeccioso?
Meningococo o neisseria meningitidis 2 puntos

1
7
Temas clave de Pediatría y Cirugía Infantil

3.- Señale 3 cambios CUALITATIVOS ( no numéricos) más característicos que presenta el LCR en una meningitis bacteriana
1.5 puntos ( 0.5 puntos cada uno)
Líquido turbio o pisco sour, Aumento de proteínas, Aumento de células con predominio de PMN, Disminución de la glucosa

4.- Indique el tratamiento antibiótico de elección en una meningitis bacteriana 2.5 puntos
ANTIBIOTICO _______ceftriaxona__(1)______
DOSIS ____100 (1)_MG/KG/DIA CADA ____24 (0.5)_HORAS

5.- Mencione 2 complicaciones agudas de una meningitis bacteriana


2 puntos
SSIADH Shock séptico
Convulsiones Edema cerebral
Colecciones subdurales Compr. Neurológico focal
CID Hidrocefalia aguda
Septicemia Miocarditis-Pericarditis

ESTACION 7 (2008)
NOMBRE

Lactante de 10 meses, consulta por palidez. Hemograma muestra los siguientes valores:
Hb: 9.0 g/dl
Hto: 27%
VCM: 65 f.
Reticulocitos: 2%
Frotis sanguíneo: Anisocitosis: + a ++
Microcitosis: ++
Hipocromía: ++

Al respecto responda:
1- Calcule el índice reticulocitario según la edad.
¿Es arregenerativa o regenerativa? 2 puntos
- Indice reticulocitario : 0.9 (1)
- Anemia arregenerativa (1)

2- El diagnóstico presuntivo completo es: 1 punto


Anemia microcítica hiprómica
Cuya causa más frecuente es: 1 punto
Anemia ferropriva

3- El tratamiento de elección es ( medicamento, dosis y tiempo ):


3 puntos
Sulfato ferroso 2 -4 mg/kg por 3 meses
Luego 1 – 2 mg/kg por 3 meses más

1
7
Temas clave de Pediatría y Cirugía Infantil

4- ¿Qué examen confirmaría el diagnóstico presuntivo?1 punto


Ferritina sérica

¿Qué parámetro hematológico ayudaría a ver la respuesta al tratamiento a los 15 días? 1 punto
Trombocitosis como respuesta medular

5.- ¿A qué edad se inicia tratamiento profiláctico de un niño de término?:


1 punto
A los 4 meses

ESTACION 9 (2008)
NOMBRE

Lactante de 8 meses, sin antecedentes mórbidos de importancia, que hace 4 días inicia cuadro respiratorio con coriza y tos. Desde ayer con fiebre
cuantificada en 38,5ºC, alrededor de 4 vómitos en el día y hoy con diarrea líquida abundante aproximadamente 7 veces en el día.
Al examen algo irritable, febril, FC 120 x min, llene capilar conservado, con mucosa bucal seca, llanto sin lágrimas, ojos levemente hundidos, signo del
pliegue (++), con orina en el pañal y deposiciones líquidas verdosas en el pañal.

1.- Diagnóstico completo del caso anterior


Diarrea aguda con deshidratación moderada ( 3 puntos)

2.- ¿Qué plan utilizaría para el manejo de este paciente y en que consiste dicho plan ( en forma breve y general)? ( 4 puntos)
Plan B
Utilizar sales de rehidratación aproximadamente 400-600 cc o
50 a 100 cc/kg en 4 horas y luego evaluar

3.- ¿Cuál sería la causa etiológica más frecuente?


Rotavirus ( 1 punto)

4.- ¿Qué hidratación le indicaría a un paciente con shock hipovolémico?


Suero fisiológico 20-30 cc/kg en bolo ( 2 puntos )

1
8
Temas clave de Pediatría y Cirugía Infantil

ESTACION 5 (2008)
NOMBRE:

Ingresa a Urgencia un lactante de 18 meses, peso 10.800 gr, que la madre refiere que estando dormido siente que se queja y al observarlo se pone
“tieso” y comienza con movimientos repetidos de todo el cuerpo incluido las cuatro extremidades, desviación de la mirada que dura aproximadamente 5
minutos, quedando con sueño posterior.
Al examen físico está somnoliento, pero despierta y llora al examinarlo, Tª axilar de 39ªC, FC 100 x min y se encuentra una faringe algo congestiva.
En ese momento vuelve a presentar movimientos clónicos de la hemicara derecha y extremidad superior derecha.

1.- ¿Cuál sería su hipótesis diagnóstica? 2 puntos


Convulsión febril compleja

2.- ¿Cuál sería el manejo más adecuado en Urgencia? (generalidades)


3 puntos
ABC
Oxígeno
Vía venosa o vía rectal para uso de Anticonvulsivante
3.- ¿Qué droga utilizaría y en que dosis? Nombre 2 alternativas
2 puntos
Diazepam 0.5 mg/kg
Lorazepam 0.1 mg/kg
4.- ¿Qué patología es muy importante de descartar? 1 punto
Meningitis o Encefalitis

5.- Nombre 4 indicaciones de usar tratamiento profiláctico con anticonvulsivante permanente 2 puntos
- Convulsiones febriles complejas
- Presencia de trastornos neuromadurativos
- Historia de epilepsia familiar
- Primer episodio en menor de 1 año
- Ansiedad de los padres

ESTACION 6 (2007)

RNT AEG 41 sem, sexo masculino, PN 3450 gr., Apgar 2-4-5, producto de parto vaginal, expulsivo prolongado, líquido amniótico con meconio, que
evoluciona desde los primeros minutos de vida con quejido.

1
8
Temas clave de Pediatría y Cirugía Infantil

Frecuencia respiratoria 74 x min, retracciones torácicas y cianosis peribucal. El examen cardíaco muestra una FC de 150 x min y sin soplos.
El examen pulmonar tiene MP presente, estertores húmedos bilaterales.

La Radiografía de tórax, es la que se muestra ( pantalla)

El examen de gases arteriales es:


pH 7.35
pO2 45
pCO2 30
HCO3 21.9
EB - 5.7
Sat Hb 72%

1.- Indique el diagnóstico lo más completo posible

RNT 41 sem AEG


Síndrome aspirativo meconial ( 3 puntos)
Insuficiencia respiratoria aguda
Depresión respiratoria o asfixia neonatal

2.- Usaría oxigenoterapia? SI (1 punto)

3.- Usaría antibióticos? SI ( 1 punto)

Si su respuesta es positiva, ¿cuáles?


AMPICILINA + AMINOGLUCÓSIDOS ( 2 punto)

4.- Nombre 2 complicaciones posibles de esta patología

RUPTURAS ALVEOLARES ( 3 puntos)


HIPERTENSION PULMONAR PERSISTENTE

ESTACION 9 (OSCE 2008)

Preescolar de 2 años 6 meses que cursa con infección respiratoria alta. Durante las primeras 48 horas de evolución presenta alza febril ( sobre
los 39ºC) asociada a un único episodio de convulsión tónico-clónica generalizada de más o monos 3 minutos de duración.
Al examen físico, en el servicio de Urgencia, tiene un examen neurológico normal.

1.- ¿Cuál es su diagnóstico completo?


CONVULSION FEBRIL SIMPLE ( 2 puntos)

1
8
Temas clave de Pediatría y Cirugía Infantil

2.- Justifique su respuesta, señalando 3 características del tipo de crisis que apoyan su diagnóstico
Generalizada
Duración < 15 min ( 3 puntos)
Sin recurrencia en las 24 horas
Ex neurológico normal

3.- ¿Qué características de cualquier paciente que convulsiona por fiebre se asocia a aumento del riesgo de epilepsia? Señale dos:
Examen neurológico o DSM anormal antes del primer episodio
Historia familiar de crisis afebriles o epilepsia ( 3 puntos)
Convulsión febril compleja

4.- ¿Cómo maneja en Urgencia una convulsión febril? Nombre solamente

ABC
Oxígeno ( 2 puntos)
Diazepam o Lorazepam
Antipiréticos

ESTACION 1 (OSCE 2008)

Ignacio de 8 meses es un lactante aparentemente sano, su madre refiere que se alimenta con leche materna, almuerzo y un postre.
Peso de nacimiento : 3.020 gr Talla 52 cm. RNT AEG
Examen físico normal y desarrollo psicomotor : dice da-da, coge pastilla con movimiento de rastrillo y se sienta apoyado en almohada.
Antropometría:
Peso control de 6 meses 7.800 gr Talla 68 cm
Peso actual 8.650 gr Talla 71 cm

De acuerdo al caso:

1.- Realice Evaluación Nutricional del niño en el momento del control, según Curvas NCHS
P/E N P/T N T/E N ( 2 puntos)

2.- Establezca el diagnóstico nutricional para el niño según evaluación e incrementos


Lactante eutrófico incremento adecuado ( 2 puntos)

1
8
Temas clave de Pediatría y Cirugía Infantil

3.- Indique todas las vacunas que debe tener según norma y las edades que deben ser colocadas:

BCG al nacer ( 2 puntos)


DPT-HibTiter-Hepatitis B- Polio 2-4-6 meses

4.- Dentro del examen del desarrollo psicomotor ¿qué área encuentra deficitaria y porque?

Motora, porque debería sentarse sin apoyo ( 2 puntos)

5.- ¿Qué dientes debería tener al momento del examen de los 8 meses?

2 incisivos centrales inferiores y 2 incisivos centrales superiores

ESTACION 6 (2008)

NOMBRE

Prepare una mamadera con leche dada en el Consultorio para un niño sexo masculino de 4 meses, cuya madre no puede dar lactancia materna
Peso actual : 7 kg
Talla actual : 64 cm
Evaluación nutricional : P/E Normal
T/E Normal
( escriba la preparación ) 2 puntos

PURITA 7.5% + AZÚCAR AL 2.5 % + ACEITE 1.5%


170-180 cc cada mamadera por 6 veces
( 2 puntos)

Escriba las medidas o cucharaditas que corresponda: 3 puntos

PURITA 2 ½ medida o 3 cucharaditas de té

AZUCAR ½ medida o 1 cucharadita de té

ACEITE ½ cucharadita de té

Ahora proceda a realizarla ( 5 puntos)

1
8
Temas clave de Pediatría y Cirugía Infantil

- Colocar la mitad del agua primero


- Colocar correctamente las medidas
- Completar el volumen de 250 cc
- Al final el aceite

ESTACION 8 (OSCE 2008)

NOMBRE

Escolar de 5 años sin antecedentes previos, que presenta coriza, tos y fiebre cuantificada hasta 38ºC. Al cabo de 7 días consulta en Urgencia por
compromiso del estado general y orinas colúricas. Destaca al examen físico palidez intensa, ictericia de piel y mucosas, petequias en tórax y abdomen,
esplenomegalia leve y adenopatías cervicales palpables de hasta 1 cm. de diámetro.
Hb : 9.0 g/dl
Hto : 27%
VCM : 80 f
Reticulocitos : 10%
Frotis sanguíneo : Anisocitosis ++
Poiquilocitosis +++
Target cells ++
Punteado basófilo +
1.- ¿Qué diagnósticos diferenciales se plantean en este caso? Nombre a lo menos 3 3 puntos
Anemia hemolítica aguda Obs Anemia hemolítica inmune
Obs anemia hemolit. No inmune
Obs hemólisis por drogas
Obs intoxicación plúmbica
Anemia hemolítica crónica reagudizada
Obs talasemia
2.- ¿Tiene anemia regenerativa o arregenerativa? Justifique su respuesta
Anemia regenerativa 3 puntos
IR 6.75
Normocítica normocrómica
Frotis sanguíneo concordante
Anemia por hemólisis
3.- Indique 4 exámenes que solicitaría para orientar el diagnóstico definitivo. 4 puntos
- Test de Coombs directo
- Bilirrubina total y directa
- Estudio infeccioso ( Micoplasma, CMV, HIV, Ebstein Barr)
- Plumbemia
- Test de fragilidad osmótica
- Electroforésis hemoglobina

1
8
Temas clave de Pediatría y Cirugía Infantil

ESTACION 7
NOMBRE

Ingresa por Urgencia un preescolar de 2 años 7 meses, con antecedente de cuadro respiratorio alto de hace 2 días, desde ayer febril, decaído, inapetente
y hoy con mayor compromiso del estado general.
Al examen físico decaído, con tendencia a la somnolencia, Tº axilar de 38.0ºC, Frecuencia cardíaca 160 x min, presión arterial 90/60, saturando 93%, mal
prefundido y llene capilar lento.
Al examen segmentario destaca faringe congestiva, pulmonar con algunos estertores aislados, sin signos meníngeos.

1.- ¿Cuál sería su diagnóstico en este paciente?

Sepsis ( 2 puntos)
Obs Neumonía

2.- ¿Qué exámenes tomaría? Nombre 6


Hemograma (3 puntos)
PCR
Hemocultivos ( 2)
Ex. De orina
Urocultivo
Rx de tórax
Punción lumbar

3.- ¿Qué gérmenes podrían ser los causales de este cuadro? Nombre 2 importantes

Neumococo ( 2 puntos)
Estafilococo
Meningococo

4.- ¿Qué antibióticos y sus dosis por kg/día dejaría como esquema empírico?
Cefotaxima 100-150 mg/kg/día + Cloxacilina 100-150 mg/kg/día
( 3 puntos)

ESTACION 5 (OSCE 2008)


NOMBRE

Usted recibe en el Servicio de Urgencia a un preescolar de 3 años, cuya madre refiere que presenta fiebre desde hace 2 días, cuantificada en 39ºC.

1
8
Temas clave de Pediatría y Cirugía Infantil

Se acompaña de calofríos y hoy en la mañana se agrega rechazo alimentario y decaimiento.


Dado que en las últimas horas su estado empeora, con extremo decaimiento e hipoactividad, la madre decide acudir a Urgencia.
Como antecedente la madre relata que hace 1 semana sufrió una herida en la planta del pie, pero que la curó en su casa.
Al examen físico destaca mal estado general, pálido, frecuencia cardíaca de 160 x min, presión arterial 80/45, llene capilar 4 seg., Glasgow 14 y herida
plantar con secreción purulenta

1.- Hipótesis diagnóstica de este paciente


Shock séptico ( debe estar) 3 puntos
Herida plantar infectada

2.- Indicaciones iniciales en el box de Urgencia ( nombre solamente)


ABC 3 puntos
Oxígeno
Suero fisiológico en bolo 20 cc/kg rápido

3.- ¿Qué exámenes de laboratorio solicita inmediatamente? Nombre a lo menos 5 2 puntos


Hemograma-PCR-Hemocultivos(2)-ELP-Gases arteriales-Creatininemia-Cultivo de herida plantar-Glicemia

4.- Tratamiento antibiótico más adecuado según el caso que dejaría, señalando nombre, dosis y frecuencia diaria ( 2 puntos)

Cefotaxima + cloxacilina 100-150 mg/kg/día cada 6 horas

ESTACION 4 (2008)
NOMBRE

Consulta en Urgencia paciente de 4 años cursando cuadro respiratorio viral hace 3 días. Desde hoy con fiebre alta, mayor compromiso del estado
general, tos productiva, dolor tipo puntada de costado, dolor abdominal y vómitos ocasionales. Satura 95% sin O2 adicional.

1.- ¿Cuáles son sus presuntos diagnósticos?


Virosis respiratoria 2 puntos
Neumonía

2.- Conducta o que haría con este paciente


Hospitalizar 1 punto

3.- ¿Qué exámenes pediría? Nombre a lo menos 3

1
8
Temas clave de Pediatría y Cirugía Infantil

Hemograma-PCR-IFI-Rx de tórax 2 puntos

4.- Frente a la más frecuente etiología en este paciente ¿usaría antibióticos?¿cuál? 2 puntos
Si. PNS 200.000 U/kg/día cada 6-8 horas EV

Después de 3 días el paciente se encuentra más grave, con fiebre mantenida, satura 89% y sus exámenes de laboratorio empeoran.
La Rx de tórax es la que se muestra en el negatoscopio

5.- ¿Qué cambiaría en su manejo y cuáles serían?


Oxigenoterapia 3 puntos
Cambio ATB a Cefotaxima + Cloxacilina
Traslado a UCI

ESTACION 6

NOMBRE

Niña de 4 años que acude a control de salud pediátrico. La menor al momento de la consulta está asintomática, haciendo una vida normal, asiste al jardín
infantil. Refiere la madre el antecedente de cuadro respiratorio alto 3 semanas antes de ésta consulta y 2 episodios de neuropatías en los últimos 2 años.
El médico que la atiende señala encontrar una menor en buenas condiciones generales, rosada, con peso de 14 kilos y una talla de 99 cm.
Presión arterial 90/55, eupneica. En la faringe amígdalas hipertróficas, sin exudado, pulmones con buen MP y a la auscultación cardíaca se describe soplo
sistólico 2/6 en 2º-3º espacio intercostal izquierdo, con 2º ruido desdoblado fijo, más suave escape diastólico en borde esternal izquierdo bajo.

1.- ¿Cuál es su sospecha diagnóstica?


Cardiopatía congénita acianótica
Bajo peso

La Rx de tórax muestra cardiomegalia moderada a expensas de VD con aumento de vascularización pulmonar


ECG con eje a derecha con rSR en V1

2.- Ahora cuál sería su conclusión?


CIA

3.- ¿Qué otros exámenes Ud. solicitaría?

1
8
Temas clave de Pediatría y Cirugía Infantil

4.- ¿Cuál sería la conducta y tratamiento?

ESTACION 1 ( 2008)

NOMBRE

Ud. está en el Consultorio y debe realizar el Control del Niño Sano de un lactante de 2 meses, sexo femenino.

Datos que aporta el Carne de Salud:


Peso RN: 3500 gr
Talla RN: 50 cm
Perímetro craneano RN : 35 cm

a) Considerando que la balanza está calibrada, obtenga las mediciones antropométricas de la lactante

b) Utilice las tablas OMS y determine la evaluación nutricional

P/E
P/T
T/E

DIAGNOSTICO NUTRICIONAL :

c) ¿Qué vacuna(s) indica en este control?

d) Nombre 3 reacciones adversas que le puede pasar al lactante con la colocación de la vacuna

1
8
Temas clave de Pediatría y Cirugía Infantil

ESTACION 2 ( 2008)

NOMBRE:

RN de sexo femenino que nace por cesárea a las 38 semanas por FUR confiable
Peso RN: 2.900 gr
Talla RN: 50 cm
Apgar: 9 – 9- 10

Primer hijo de madre 35 años, con talla de 150 cm

a) Si según la evaluación de Ballard corresponde a 38 semanas, realice el diagnóstico de edad gestacional definitivo con los factores de corrección
utilizando la tabla de crecimiento intrauterino que Ud, dispone.

b) ¿Qué aspectos semiológico en el examen físico o somáticos ( Usher) del RN debe analizar para determinar la edad gestacional? Nombre

c) Nombre a los menos 4 reflejos arcaicos que debe buscar en un RN

d) ¿Qué consideraciones generales debe tener el RN para ser dado de alta con su madre? Nombre 4

ESTACION 6 (2008)

NOMBRE

Prepare una mamadera para una niña de 4 meses de edad, ya que la madre no le da pecho
Peso actual : 7 kilos

1
9
Temas clave de Pediatría y Cirugía Infantil

Talla actual : 62 cm

Diagnóstico nutricional : EUTROFIA

a) Prepare la mamadera utilizando Purita Fortificada 26%


Anote la cantidad de mamaderas al día, y la preparación con medidas o cucharaditas de t

ESTACION 1 (2007)

NOMBRE:

Nicolás es un lactante de 4 meses. Es un RNT AEG, peso de nacimiento: 3. 650 grs., talla: 52 cms, CC: 35 cms Durante el control actual encontramos los
siguientes parámetros: Peso: 6. 850 grs. Talla: 65 cms. CC: 44, 5 cms

1. Realice Evaluación Nutricional del niño según tablas OMS (3 puntos)

P/E: N T/E: N P/T: N

2. Indique 4 características del desarrollo psicomotor que presenta un niño sano a los 4 meses. (2 puntos)

- Levanta la cabeza y hombros al ser llevado a posición sentada.


- En posición prona se levanta a sí mismo.

1
9
Temas clave de Pediatría y Cirugía Infantil

- Gira la cabeza al sonido de la campanilla.


- Ríe a carcajadas.

3. Si la madre no estuviera dando lactancia materna, indique la forma de preparación láctea más apropiada en porcentajes para este caso. ( considerar
leche entregada en consultorio)(1 puntos)

Leche: 7,5 % Azúcar: 2,5% Aceite 1%

4. Señale las vacunas que debe tener este niño hasta los 4 meses. (2 puntos)

- BCG al nacer
- DPT Antipolio Hib Hep B (2 Meses)
- DPT Antipolio Hib Hep B (4 Meses)

5.- Nombre 3 complicaciones o reacciones adversas de la DPT ( 2 puntos)

- Dolor en zona de inoculación


- Fiebre
- Llanto irritable
Convulsiones

1
9
Temas clave de Pediatría y Cirugía Infantil

NEONATOLOGÍA

1
9
Temas clave de Pediatría y Cirugía Infantil

Examen físico y fenómenos parafisiológicos del RN


• Introducción o Primer examen de inmediato después del parto ▪ Objetivos
• Verificar recuperación ante el estrés del parto y la adaptación extrauterina
• Identificar anomalías graves y malfor maciones mayores

Evaluar la edad gestacional
o Segundo examen Dentro del primer día o Tercer examen Al alta
• Examen inmediato o Cardiorespiratorio
▪ Evaluar adaptación extra uterina Transición CR
• Expansión de los pulmones
• Inicio de la respiración regular
• Caída de la resistencia vascular pulmonar
• Cierre de shunts
 Se basa en la evaluación de 3 parámetros
• Frecuencia cardíaca
 Normal: 120 – 160 rpm en reposo. Mayor a 100 en el parto
 Se ausculta o se palpa en el cordón o a nivel braquial o Fenómenos
patológicos
 Frecuencias < 100 lpm Apnea secundaria
• Requiere reanimación con ventilación a PP
• Esfuerzo ventilatorio
 Debe ser vigoroso (llanto) Permite la expansión
 pulmonar
 Luego movimientos regulares con FR 40 – 60 rpm o Fenómenos fisiológicos
 Frecuencias de hasta 70 en la primera hora de vida
 Crépitos y ruidos húmedos en las primeras respiraciones
 Quejido respiratorio: Cierre de la glotis en espiración para evitar colapso de
la VA intrapulmonar en RNPT durante la primera hora
 Retracción costal discreta: A nivel subdiafragmático y esternal en RNPT
 Respiración periódica: Pausas < 10 segundos o Fenómenos patológicos
 Asimetrías de ruidos agregados con signos de DR
 Ausencia o disminución de tonos cardíacos
• Color
 Indicador de la función CR o Fenómenos fisiológicos
 Acrocianosis en paciente con color rosado generalizado o
Fenómenos patológicos
 Cianosis de piel y mucosas Hipoxia
 Palidez y/o llene capilar lento Hemorragia aguda, hipoxia,
acidosis.

1
9
Temas clave de Pediatría y Cirugía Infantil

o Abdomen
• Blando y deprimido al nacer o Distención posterior por
llene de aire o Órganos palpables ▪ Fenómenos patológicos
• Persistencia de abdomen deprimido Hernia
diafragmática • Abdomen distendido Visceromegalia, ascitis,
obstrucción.
o Apariencia general
• RN activo y llorando
• Moviliza las 4 extremidades manteniéndolas
semiflectadas con manos empuñadas ▪
Fenómenos patológicos
• Asimetría en movimientos Lesiones del
plexo braquial o cervical
• Lesiones traumáticas GEG nacidos por PV
• Malformaciones y deformidades mayores.
o Apgar
 Evaluación al 1 y 5 minutos de 5 signos clínicos
 Buenas condiciones Apgar 8 – 10
 Persistencia de Apgar bajo después de 5 minutos se
correlaciona con riesgo de compromiso neurológico
 Buen predictor de pronóstico ante progresiva
recuperación del puntaje
o Antropometría
 Peso, talla y CC
 Ubicarlas en las curvas de crecimiento uterino
• Examen posterior o Consideraciones
 Niño recién alimentado puede regurgitar
 Niño llorando dificulta la auscultación cardíaca y el examen
abdominal
 Evitar mantenerlo desnudo por largos períodos de tiempo
por enfriamiento
o Postura y actividad
 Extremidades Flectadas e hipertónicas en reposo
 Puede adoptar posición de reflejo tónico nucal
 RNPT tiene postura en mayor extensión a menor EG o Piel
 Color y textura o Rosada y suave o con aspecto marmóreo o
Acrocianosis: Desaparece en algunos días o Tejido
subcutáneo lleno o Descamación discreta de la piel, marcada
a mayor EG o Ictericia BR < 4 – 5 mg/dL

1
9
Temas clave de Pediatría y Cirugía Infantil

o RNPT Delgada, transparente, roja, poco tejido subcutáneo


• Lesiones fisiológicas
o Unto o vérmix caseosa: Material graso, mayor en el RNPT, sobre dorso, cuero
cabelludo y pliegues.
o Lanugo: Pelo fino sobre hombros y dorso, más abundante en el RNPT o Mancha
atávica: Manchas azul pizarra grandes en dorso, nalgas o muslos o Eritema tóxico
 Rash maculopapular de base eritematosa con vesículas de centro
amarillo (eosinófilos) preferentemente sobre tronco y extremidades.
 Entre los 3 y 10 días.
o Petequias y equimosis
 En cabeza y cuello asociadas a circular de cordón
 Generalizadas Sospechas alteración de la coagulación

Vérmix caseosa Lanugo Mancha atávica Eritema Tóxico

o Cabeza
 Forma y tamaño o Grande en relación al cuerpo
• Deformación plástica: Cabalgamiento óseo por paso por el canal de parto
• Fontanelas o Anterior
 1 – 4 cms
 Blanda, pulsátil y depresible en reposo o Posterior
 < 1 cm
 Triangular
• Suturas: Afrontadas, móviles, con cierto grado de cabalgamiento
• Caput succedaneum: Edema subcutáneo del cuero cabelludo por presión durante el trabajo de parto
• Cefalohematoma: Hematoma por hemorragia subperióstica.

1
9
Temas clave de Pediatría y Cirugía Infantil

Caput succedaneum Cefalohematoma

o Cara
 Ojos
• Cerrados. Párpados edematosos
• Iris grisáceo
• Hemorragias subconjuntivales y esclerales normales
• Pupila reactiva
• Rojo pupilar Opacidades en córnea y
cristalino anormales.
 Nariz o Respirador nasal
• Fenómenos patológicos o Atresia de coanas
 Evaluar pasando SNG
• Fenómenos fisiológicos o Milium sebáceo Glándulas sebáceas
▪ Boca o Labios rojos o Encías con relieve dentario, sin dientes
o Paladar sin fisuras
• Fenómenos fisiológicos o Perlas de
Ebstein: Pápulas blanquecinas en la línea media
de aprox 1 mm ▪ Oídos
• Forma e implantación del pabellón
• Alteraciones se asocian a
malformaciones del tracto urinario
• Tímpanos de apariencia opaca
o Cuello
 Es corto y simétrico

1
9
Temas clave de Pediatría y Cirugía Infantil

 Debe explorarse la movilidad y los aumentos de volumen: bocio, quiste tirogloso, hematomas
del ECM o Tórax
• Evaluar forma y simetría
• Movimientos respiratorios 30 – 60 por minuto ▪
Clavículas
• Descartar fractura GEG
 Nódulo mamario Palpable en niños maduros
 Pulmones
• Respiración abdominal predominante
• Respiración periódica
• El murmullo pulmonar es adecuado ▪ Corazón
• Fenómenos patológicos o Frecuencias < 90 y > 195 o Soplos persistentes por más de 24 horas o
sintomáticos
• Fenómenos fisiológicos o Soplos sistólicos eyectivos transitorios
o Abdomen
 Forma: Ligeramente excavado en las primeras horas y luego distendido
• Fenómenos patológicos o Abdomen persistentemente excavado + DR Hernia diafragmática o Abdomen distendido Obstrucción o
íleo paralítico (en niños con infección o sepsis)
▪ Ombligo y cordón o Debe contener 3 vasos: 2 arterias y 1 vena o Se suelta al 4to día y se cae
entre el 7 y el 10
• Fenómenos patológicos o Arteria umbilical única Asociada a síndromes
malformativos
• Fenómenos fisiológicos o Ombligo cutáneo: Prolongación de la piel por la base
del cordón o Hernia umbilical: Revierte espontáneamente
o Ano y recto
 Revisar ubicación y permeabilidad Malformaciones anorrectales o Genitales
 Masculinos
• Escroto pendular, con arrugas y pigmentado.
• Testiculos descendidos
• Prepucio adherido al glande
• Meato pequeño
• RNPT Escroto menos pigmentado y testículos en canal inguinal
▪ Femeninos o Hacia el término, labios mayores recubren a los menores y al clítoris o Himen visible
• Fenómenos fisiológicos o Secreción blanquecina mucosa en los primeros días o Pseudomenstruación
▪ Caderas
• Deben abducir en forma simétrica o Si hay signo de Ortolani o Barlow, sospechar luxación congénita
▪ Extremidades o Brazos y piernas simétricos
• Fenómenos patológicos o Alteraciones mayores: Ausencia de huesos, pie Bot, polidactilia, sindactilia y deformaciones.
o Fracturas • Examen neurológico o Actitud general y tono muscular

1
9
Temas clave de Pediatría y Cirugía Infantil

 Evaluar simetría de movimientos, postura y tono. o Reflejos arcaicos


 Reflejo de Moro: Respuesta a estimulo brusco o deflexión brusca de la cabeza
 Prehensión palmar y plantar: Flexiona los dedos de la mano o del pie
 Búsqueda: Vuelve su cabeza hacia un estímulo en mejilla o peribucal, buscando el pezón
 Succión: Movimiento al colocar objeto en la boca
 Marcha automática: Da pasos al sostenerlo del tronco ligeramente inclinado hacia adelante.
• Evaluación de la edad gestacional
o Introducción
 Riesgo de morbimortalidad neonatal se relaciona con el peso y la EG
 Para evaluar la EG se puede usar información obstétrica y del examen
físico o Información obstétrica
 FUR
 Ecografía: Mayor valor si fue precoz (antes de las 12 semanas)
 Otros: Altura uterina, primeros latidos CF, primeros movimientos Muy
variables o Información del RN
 Método de Usher Evaluación de caracteres físicos externos que se
diferencian a lo largo del último mes de gestación. Otorga una seguridad de ± 2
semanas.
• Pliegues
plantares o < 36
semanas: 1/3
anterior o 36 – 38
semanas: 2/3
anteriores o > 39
semanas: Toda la
planta
• Pabellón
auricular o < 36
semanas:
Plegable, no
vuelve a la
posición o 36 –
38 semanas:
Demora en
volver a la
posición o > 39
semanas: Vuelve
con rapidez a la
posición
• Pelo o < 37
semanas: fino,
aglutinado o > 38

1
9
Temas clave de Pediatría y Cirugía Infantil

semanas: grueso,
individualizable
• Nódulo mamario
o < 34 semanas:
No se palpa o 34
– 36 semanas:
0,5 cms o 73 – 38
semanas: 0,5 – 1
cm o > 39
semanas: 1 cm
• Genitales
masculinos o < 36
semanas: Escroto
con escasas
arrugas.
Testículos en
canal inguinal o
36 – 38 semanas:
Escroto con
moderada
cantidad de
arrugas.
Testículos
descendidos
o > 38 semanas: Escroto pendular arrugado. Testículos en escroto.
• Genitales femeninos o < 36 semanas: Labios mayores separados, sobresalen los menores o 36 – 38 semanas: Labios mayores
cubren a los menores
o > 38 semanas: Labios mayores cubren por completo a los menores. Puede haber leucorrea o
pseudomenstruación.
▪ Método de Ballard 6 criterios físicos y 6 neurológicos

2
0
Temas clave de Pediatría y Cirugía Infantil

• Otros fenómenos parafisiológicos o Caída del cordón


• Se momifica una vez ligado cayéndose entre los días 10 – 14. ▪
Fenómenos fisiológicos
• Secreción hemática ▪ Fenómenos patológicos
• Signos inflamatorios de la piel circundante Onfalitis
• Tumoración húmeda del fondo umbilical Granuloma umbilical
o Baja de peso
 Baja de peso normal hasta el 3er o 4to
• Balance nutricional negativo por escasa ingesta y elevado gasto
energético
• Pérdida de energía en llanto, movimiento y evacuación
• Pérdida de líquidos por la piel, respiración, orina y deposiciones.
▪ Recupera el peso entre los días 10 – 14 ▪ NO más del 10 – 12% de pérdida. o
Deposiciones de transición (Falsa diarrea)
 Meconio: deposición verde petróleo oscura, ligosa en las primeras 24 – 48 horas
• Formada por mucoproteínas y células epiteliales intestinales
descamadas
 Luego de la LM cambia de consistenia semejando diarrea verdosa, frecuente, que dura
2 – 3 días ▪ Posteriormente, pastosa color amarillo oro. o Fiebre de sed
 Fiebre en RN de elevado peso al nacer, entre las 40 – 42 semanas por poca ingesta
alimentaria.
 Niño ansioso por succionar con baja de peso importante
 Debe ceder en las siguientes 24 horas al suplementar la LM o con medidas físicas. o
Crisis hormonal (Pubertad precoz NN)
• Respuesta a la circulación de hormonas sexuales transmitidas vía
placentaria al a madre
 Fenómenos fisiológicos

2
0
Temas clave de Pediatría y Cirugía Infantil

• Nódulo mamario
• Secreción láctea (“Leche de brujas)
o Desaparece a la 3ra semana
o No extraer por riesgo de mastitis.
• Pseudomenstruación o 4to o 5to día de vida o Dura lo que dura
una menstruación (2 a 4 días)
• Acné neonatal o Comedones, pápulas inflamatorias y pústulas
en la cara o 2 primeras semanas de vida
o Ictericia fisiológica
 Después de las 24 horas de vida con peak al 2do o 3er día
 Desaparece a los 10 – 12 días de vida
 Observable con valores desde 4 – 5 mg/dL con valores máximos de 12 mg/dL

2
0
Temas clave de Pediatría y Cirugía Infantil

Prematurez
• Definición o RNPT
 OMS Nacimiento entre las 22 y 37 semanas EG ç
 AAP Nacimiento entre las 38 semanas o Edad corregida ECr – (40 – EG)
 RNPT extremos, corregir hasta los 2 años
 RNPT moderados y tardíos, corregir hasta el año
• Clasificación o Edad gestacional
 RNPT Extremo < 32 semanas o peso < 1500 gr
 RNPT Moderado 32 – 34 +6 semanas y peso > 1500 gr
 RNPT Tardío 35 – 36 + 6 semanas y peso > 1500 gr o Peso
 RNBP < 2500 gr
 RNMBP < 1500 gr
 RNEBP < 1000 gr
• Epidemiología o Cada año nacen 15 millones de niños prematuros
o Las complicaciones asociadas son la principal causa de
mortalidad en < 5 años o Tasa de RNPT es entre el 5 – 18%
• Etiología o Factores de riesgo
 Madre: Desnutrición materna, tabaquismo, leiomiomatosis materna.
 Infecciones: ITU, TORCH, RPM (> 18 horas), corioamnionitis
 Perinatales: SHE, PHA, intervalo intergenésico corto, útero bicorne, partos PT anteriores, hemorragia del 3er
trimestre.
• Fisiología de los RNPT o Ventilación inadecuada: Dada por
inmadurez del sistema nervioso, debilidad de los músculos
respiratorios, pulmones inmaduros, déficit de surfactante, falta de
desarrollo de la vaculatura pulmonar.
o Neurológico: Escasa migración neuronal, baja mielinización de la sustancia blanca,
crecimiento de la sustancia gris, susceptibilidad a hipoxia y cambios de osmolaridad,
sangrado frecuente, permeabilidad de la BHE aumentada
o Cardiovascular: Incapacidad del SNA para mantener adecuado tono vascular
o Sistema inmune: Inmunidad ineficaz, disminución de la reacción inflamatoria,
fagocitosis y acción bactericida de neutrófilos incompleta, disminución de IgG y
ausencia de IgA e IgM, incapacidad de limitar la infección a un territorio orgánico
(sepsis)
o Sistema hematológico: Menor número de GR y mayor de eritroblastos o
Gastrointestinales: Maduración de la succión deglución recién a las 32-34 semanas,
motilidad intestinal pobre con retraso de la evacuación, déficit de absorción de grasas
y vitaminas
o Endocrino: Hiperfunción tiroídea (encubre hipotiroidismo subyacente) Cribaje a los 3
días de vida y repetirlo al mes.

2
0
Temas clave de Pediatría y Cirugía Infantil

o Hipotermia: Metabolismo basal bajo, piel delgada, gran superficie corporal,


disminución de grasa subcutánea, deficiente control vasomotor o Metabólico
 Hidrosalino: Inmadurez renal que impide reabsorción de sodio y agua
correctamente
 HdC: Escasos depósitos de glucógeno, interrupción de glucosa umbilical
(hipoglicemia), pobre capacidad de regulación de insulina (hiperglicemia)
• Patologías asociadas o Alteraciones de la glucosa
▪ Hipoglicemia
• Definición o Glicemia z 45 mg/dl en asintomático o < 40 mg/dL en
sintomático
• Causas o Limitación de aporte de sustratos endógenos o Enzimas
glicogénicas maduran antes que las glicogenolíticas o Frecuente en
hipotermia e hipoxia (aumento del consumo de glucosa)
• Manejo
o Comenzar lo antes posible con fleboclisis con carga de glucosa 4 – 6 mg/kg/min asociada a malnutrición parenteral precoz o Ajustar
con glicemia seriada ▪ Hiperglicemia
• Definicón o Glicemia < 125 mg/dl
• Causas o Bajos niveles de insulina: inadecuada secreción +
aumento de la resistencia periférica + inhibición de la producción
endógena con la infusión exógena
• Manejo
o Control seriado de glicemias o Glucosuria si la glicemia > 90 mg/dl cada 12 horas mientras esté con NP o cargas de glucosa > 6 mg/dl
• Controlar o Desde las 2 horas después del nacimiento o Continuar
hasta que se alimente correctamente o Mantener glicemias
normales
o Anomalías cardiovasculares ▪ Conducto arterioso
• Definición: Estructura vascular que conecta la
aorta descendente proximal con la arteria
pulmonar
• Cierre funcional: Durante las primeras 48 horas
por o Aumento de la PaO2 o Disminución de las
PG circulantes
o Inversión de relación entre resistencia vascular pulmonar y periférica o
Puede reabrirse si hay hipoxia o acidosis
• Cierre anatómico o Se completa 2 – 8 semanas
• Epidemiología o 30% de los RN BPN o 20% RNPT
> 32 semanas hasta 60% < 28 semanas
• Etiología
o Insensibilidad al aumento de la oxigenación
o Caída anticipada de la presión pulmonar estableciendo shunt izquierda
derecha precozmente

2
0
Temas clave de Pediatría y Cirugía Infantil

• Clínica o Soplo sistólico de eyección que irradia al dorso o Síntomas: Taquicardia, pulsos
saltones postductales (izquierda), taquipnea, hepatomegalia, repercusión HD (apnea, DR,
congestión pulmonar, IC, acidosis metabólica)
• Diagnóstico o Ecocardiografía magnitud del DAP, dirección del shunt, función cardíaca, etc.
• Tratamiento o Manejo conservador
 RNPT < 1000 gr sin VM ni apneas: Restricción líquida y diuréticos o
Manejo farmacológico
 DA moderado-severo
• Indometacina 0.1-0.2 mg/kg/dosis cada 12 horas EV
• Ibuprofeno 5 – 10 mg/kg/dosis cada 8 horas VO
o Manejo quirúrgico: Falla de tratamiento previo o contraindicación
o Patología neurológica
• Factores predisponentes o Inmadurez del SNC o La anatomía está caracterizada por la fragilidad de la estructura vascular a nivel de
la matriz germinal
o Escasa migración neuronal o Pobre mielinización de la sustancia blanca o Crecimiento
exponencial de la sustancia grus o Susceptibilidad a la hipoxia, cambios de osmolaridad y tensionales ▪
Hemorragia IC del RN
• Epidemiología o Incidencia 12 – 15% de HIV grave en RNBPN o En RN 500 – 700 gramos
incidencias de hasta 70%
• Cuadros clínicos o Hemorragia intracerebral (asociada generalmente a HIV) del
cerebelo y cerebro
o HIV del RNPT Hemorragia más frecuente
 Casi siempre dentro de las primeras 24 horas
 Excepcional a partir de los tres días de vida o Presentaciones clínicas
 Forma aguda catastrófica
• Síntomas en las primeras 12 -24 horas de vida o Afectación central grave con crisis
tónicas generalizadas, descerebración, coma, hidrocefalia obstructiva aguda ▪
Forma de progresión lenta
• Síntomas fluctuanes y saltatorios o Alteración del estado de concienca
o Disminución de la motilidad o
Hipotonía
o Mirada en sol naciente ▪
Forma asintomática o silenciosa
• Es la más frecuente (50%)
• Pequeños sangrados detectados por ecografía o solo por disminución del hematocrito
• Diagnóstico ECO transfontaneral
 RN < 1500 gr y/o EG < 32 semanas + NN con factores de riesgo de HIV y HSE
 Realizar a las 48 horas y repetirla la primera semana y al mes

2
0
Temas clave de Pediatría y Cirugía Infantil

• Tratamiento o Medidas preventivas (incluyen reanimación rápida) o Evitar inestabilidad


HD y alteración de la autorregulación cerebral
o Retinopatía del prematuro ▪ Introducción
• Definición: Trastorno en el desarrollo de la proliferación vascular en la retina o Vascularización
incompleta y crecimiento anormal de neo vasos o Aumento de radicales libres locales
• Aumenta con disminución de la EG y PN o Inferior a 32 semanas y peso < 1000 gramos o Chile la incidencia
de retinopatía del prematuro es del 23.1 %
• > riesgo de desenlace adverso ocular o Primera causa de ceguera infantil o Discapacidad visual a largo
plazo 20% o El riesgo de deficiencia visuales alto o Formas severas: pérdida de la agudeza visual o
Resuelve espontáneamente ▪ Clínica
• Zona III
• Recuperación completa
• Se asocia a alteración del a refracción
• Mayor gravedad o Inicio precoz (5 – 9 semanas) o Zona I
o Evoluciona a enfermedad de Plus ▪ Tratamiento
• Ablación de la retina avascular periférica con fotocoagulación con laser
• Seguimiento cada 1 – 2 meses por 1 – 2 meses
o Hipoacusia neuronal bilateral del prematuro
 Definición: Screening auditivo a todo prematuro < 1500 gr y/o 32 semanas previo al alta ▪
Tamizaje
• Antes de los 6 meses
• Potenciales evocados auditivos automatizados de tronco (PEAAT), es el examen indicado para
la pesquisa en 2 etapas ( a las 2 y 4 horas), si primero es anormal, se realiza el segundo.
 Tratamiento
• Bilateral o
Implementación de audífono
o Terapia auditivo fetal (intervelación I, II, III o Implante coclear (hipoacusias severas)
• Unilateral o
Seguimiento en ORL

2
0
Temas clave de Pediatría y Cirugía Infantil

2
0
Temas clave de Pediatría y Cirugía Infantil

Atención inmediata neonatal


• Objetivos o Vigilar la adaptación cardiorrespiratoria o Vigilar que RN logre
termorregulación o Fomentar el apego y el inicio de la lactancia o Pesquisa precoz de
malformaciones congénitas o Realizar antropometría e identificación o Efectuar
profilaxis de patologías neonatales
• Proceso o Al ingresar la embarazada deben valorarse los antecedentes en
busca de factores de riesgo.
o Preparar la sala de partos, asegurando calefacción o Si al nacer se aprecia saludable, realizar un
pinzamiento tardío del cordón ya que disminuiría la anemia en lactantes (pese al mayor riesgo de fototerapia)
o La aspiración rutinaria de niños sanos debe descartarse o Al nacer, debe darse a contacto piel a piel, para
favorecer la estabilidad fisiológica del RN, el apego y la lactancia.
o Secar y cubrir con paños precalentados y la cabeza con gorro de algodón.
o Para asegurar la termorregulación, el contacto piel a piel debe darse de forma ininterrumpida durante la
primera hora de vida, con la madre o, si no puede ésta, con el padre. o Debe evitarse la aspiración gástrica por
alterar el inicio de la alimentación.
o Si el niño está saludable, debe facilitarse alimentación durante la primera hora o Evaluación iniciar debe
registrarse en la ficha del examen físico, junto a la atropometría, apgar, EG y adecuación según curva de Alarcon-
Pittaluga.
o Test neonatal de Ballard solo cuando haya discordancia muy grande respecto a estimar la EG o Todos los
neonatos deben recibir vitamina k (1 mg en >2000 g; 0,5 mg en < 2000 grM 0.3 mg en >
1000 gr) a las 6 horas de vida o Se debe mantener vigilancia por las primeras dos horas por cualquier alteración que permita sospechar
patología del niño.
o No es necesaria ninguna sonda o Confirmar visualmente diagnóstico de ano imperforado (evaluar
presencia de ano, vigilar eliminación de meconio)
o La profilaxis ocular para oftalmia NN por gonococo se trata con nitrato de plata al 1%, eritromicina al 0,5%,
tetraciclina al 1% o povidona al 2,5%, son igualmente eficaces.
o NO hay evidencia como para recomendar profilaxis ocular en RN por cesárea con membranas íntegras.
o Llevar a observación (por no mas de 4 horas)
 Malas condiciones maternas
 No ha alcanzado termorregulación ▪ No normaliza signos
vitales ▪ En espera de exámenes.

.
Reanimación neonatal
• Introducción o Aproximadamente el 10% de los RN requieren algún tipo de asistencia
para iniciar la respiración, sin embargo, solo 1% requiere medidas complejas de reanimación
o Principios para una reanimación exitosa Depende de el reconocimiento precoz del niño que necesita ser reanimado ▪ Anticipación
• En embarazos de alto riesgo, la madre debe ser trasladada a un centro con reanimación
neonatal avanzada y UCI neonatal.
• Si no es posible trasladarla, el niño debe nacer donde haya personal capaz de realizar
todas las maniobras de reanimación y estabilización posterior
 Información

2
0
Temas clave de Pediatría y Cirugía Infantil

• Buscar factores de riesgo prenatales o intra-parto de depresión neonatal,


medicamentos dados a la madre y condición fetal ▪ Personal entrenado y coordinado
• Partos de bajo riesgo: Al menos una persona en reanimación inicial y localizable otra
en reanimación completa
• Partos de alto riesgo: Al menos una en reanimación completa. o Neonato
severamente deprimido: Al menos dos (uno ventila e intuba y el otro monitoriza, masajea y
administra drogas)
• Partos múltiples: Tantos equipos como niños por nacer
 Asegurar condiciones ambientales y equipamiento disponible
• Temperatura > 26 ºC Cuna radiante funcionando
 Evaluación Vigilar (color ya no es criterio pues demora en conseguirse)
• Esfuerzo respiratorio
• Frecuencia cardíaca
• Etapas de la reanimación
 A: Vía aérea Asegurar vía aérea despejada y estimular
 B: Respiración Ventilación
 C: Soporte circulatorio Compresión torácica
 D: Drogas DVA o expansores de volumen o Pasos iniciales
 Mantener la temperatura
• Secar inmediatamente al nacer, cubrir con gorro y paños calientes
• Cuna radiante
• RNPT < 28 semanas: Cubrir la cabeza y el cuerpo (liberando la cara) con envoltorio plástico, sin
secarlo, y después colocar en cuna radiante.
 Evaluar: FC y respiración
 Despejar vía aérea y ventilar los pulmones (muchos responden en este paso)
 Evaluar: FC y respiración Si no establece respiración o si FC < 100 se inicia reanimación o Reanimación
 A: Vía aérea • Posición: o Poner de espalda con cabeza en posición neutral, con almohadilla de 2 cm
bajo el hombro para mantener la posición.
o En niños hipotónicos Tracción de mandíbula o uso de cánula Mayo
• Succión: Solo si la VA está obstruida o Succión faríngea agresiva puede retrasar el inicio de
la respiración espontánea y causar espasmo laríngeo y bradicardia vagal
o La presencia de meconio espeso en hipotónico es el único indicio para considerar
aspiraración inmediata de la orogaringe.
▪ Cateter 12 – 14 a fuente de aspiración a < 100 mmHg o Aspiración, intubación y succión en RN teñidos en meconio no es
recomendable.
o Niños hipotónicos tienen mayor riesgo de síndrome de aspiración meconial, pero
la aspiración traqueal no se ha asociado a disminución de éste, pero no hay evidencia para
suspender la práctica de aspirar orofaringe y endotraqueal directa a hipotónicos teñidos en
meconio.

2
0
Temas clave de Pediatría y Cirugía Infantil

▪ Si hay bradicardia o la intubación demora, ventilar con mascarilla. ▪ B: Ventilación


• Generalidades o Primeras 5 ventilaciones, mantener presión inicial durante 2 – 3 segundos
(ayuda a la expansión pulmonar)
▪ La mayoría responderá a los 30 segundos o Si con esto aumenta la FC pero no respira adecuadamente, ventilar a ritmo
30 – 60 resp/min “Ventilo – dos – tres”
 Adecuada ventilación generará rápido aumento de la FC (>100 lpm)
 Si no responde es probable mal control de la VA o ventilación inadecuada
(observar movimientos pasivos del tórax)
 Considerar asegurar VA con intubación o Continuar soporte hasta
respiración normal
• Particularidades
o RNT
 Comenzar con aire y evaluar si es adecuada con FC o, si no mejora,
con el movimiento de la pared torácica.
 Reanimados con VPP, lo mejor es comenzar con aire ya que el uso
de VPP con FiO2 100% no confiere ventaja y resulta en aumento en el tiempo
para iniciar la primera respiración o llorar, aun cuando no haya aumento de
la FC o saturación (que debe medirse pre ductal)
• Saturaciones esperadas o 2 minutos: 70 en PV y 62 en Cesárea o 5 minutos: 92 PV y 85
Cesárea o 10 minutos: 96 PV y 94 Cesárea
• Si se requiere VPP, utilizar Neo-puff con presión de inflado inicial de 20 cmH2O o Si no se
puede medir, usar insuflación mínima para conseguir aumento de la FC
o Intentar proporcionar 30 – 60 rppm para mantener FC > 100 lpm
o Prematuros < 32 semanas
▪ No alcanzarán los valores objetivos con aire y con FiO2 100% presentarán hiperoxia
• Comenzar con 30 – 90% FiO2 y titular
• Si no se dispone de mezclador de oxígeno y aire, iniciar con aire
▪ Si requiere VPP, usar insuflación inicial de 20 – 25 cmH2O
• Si se requiere continuar, considerar uso de PEEP o CPAP (cánulas nasales)
• Intubación endotraqueal o Cuando se requiere succión para eliminar obstrucción VA o Si
ventilación bolsa-mascarilla es ineficaz o prolongada o Cuando se realizarán compresiones torácicas
o Circunstancias especiales (hernia diafragmática congénita, PN < 1000 g) o Adecuada intubación:
Aumento de la FC o signo del empañado o Longitud del TET
 23 semanas 5.5
 30 semanas 7.0
 35 semanas 8.0
 40 semanas 8.5 ▪ C: Circulación
• Iniciar compresiones cuando la FC sea inferior a 60 lpm a pesar de ventilación adecuada
• Técnica

2
1
Temas clave de Pediatría y Cirugía Infantil

o Pulgares al lado del otro sobre el tercio inferior del esternón debajo de línea
intermamilar
o Comprimir hasta 1/3 del diámetro AP permitiendo reexpandir o Utilizar relación de 3
compresiones por 1 ventilación (120 lpm o 90 compresiones y 30 ventilaciones) “y uno y dos
y tres, ventila”
o Si la causa es probablemente cardíaca, cambiar a 15 compresiones por 2 ventilaciones
• Comprobar frecuencia después de 30 segundos y cada 30 segundos después
• Dejar de comprimir cuando la FC > 60 lpm ▪ D: Drogas
• Rara vez se utilizan
• La bradicardia es causada por insuflación pulmonar inadecuada o hipoxia, pero si la FC <
60 lpm, a pesar de ventilación y compresión, se indica el uso de DVA Usar Cateter venoso
umbilical o osteoclisis temporales.
• Drogas o Adrenalina
▪ Dosis
• 0.01 – 0.03 mg/kg EV
• 0.05-0.1 mg/kg Endotraqueal (no se prefiere)
o Fluidos Sospecha de pérdida de sangre o shock (pálido, pulso débil). Es raro que
ocurra
 Sangre
 Cristaloide isotónico (no albúmina) en bolo 10 ml/kg. Repetir hasta mejoría
o Glucosa Mantener en rango normal
o Temperatura
▪ Hipertermia: RN de madres febriles tienen mayor incidencia de depresión respiratoria, convulsiones, PC y mortalidad Mantener normotermia
▪ Hipotermia
• Terapéutica: EHI moderada o severa antes de las 6 horas por
72 horas • Cuando detener la reanimación o Si el ritmo no es detectable y permanece así por 10 minutos, se considera apropiado
suspender.
o RN sin FC al nacer que logran partir a los 10 minutos tienen alta probabilidad de
morir o daño neurológico grave
o FC < 60 lpm y no mejora en 10 – 15 min Depende del clínico o No iniciar
reanimación en situaciones de alta mortalidad y mal pronóstico.
 Prematuridad extrema (EG < 23 semanas y/o peso al nacer < 400 g
 Anencefalia
 Trisomia 13 o 18 o Iniciar siempre
 RNPT > 25 semanas
 Malformaciones congénitas manejables
• Recomendaciones o En neonatos no deprimidos, retardar el clampeo del cordón por al menos 1 minuto o Comenzar la
reanimación con aire o Saturar mano derecha (preductal) o RN < 28 semanas deben ser cubiertos arriba de su cuello por bolsa
plástica, sin secar, y posteriormente ubicar bajo cuna radiante

2
1
Temas clave de Pediatría y Cirugía Infantil

o Aspiración de meconio de nariz y boca antes de nacer no se recomienda o Adrenalina EV Dosis 0.01 – 0.03
mg/kg o Capnografía y evaluación clínica confirma posición del TET o Hipotermia terapéutica se usa en RNT
con EHI moderada/severa

Asfixia perinatal y EHI


• Introducción o La asfixia es un
mecanismo de daño tisular debido a la
interrupción del flujo sanguíneo
placentario, con repercusión clínica en
múltiples órganos.
o En todo parto normal siempre hay un grado de hipoxia transitoria.

2
1
Temas clave de Pediatría y Cirugía Infantil

 Si el mecanismo que le da origen se prolonga, se desencadena el reflejo de inmersión


Redistribución del gasto cardíaco hacia territorios vitales (cerebro, corazón, suprarrenales).
 Si se prolonga, cae el gasto cardíaco y se deteriora la perfusión cerebral
 Cuando hay daño cerebral siempre hay daño previo de otros órganos (pulmón, riñón, médula
ósea)
• Definiciones o Depresión
neonatal:
 Condición por compromiso neurológico o cardiorrespiratorio que demanda iniciar reanimación.
 Condiciona Apgar 1’ < 6 y > 7 a los 5’ (No permite establecer pronóstico) o Encefalopatía hipóxico
isquémica
 Afectación neurológica con compromiso de conciencia, alteración del tono, reflejos, a veces,
convulsiones por hipoxia sostenida
o Parálisis cerebral
 Desorden motor permanente de los movimientos, tono y postura (pocas veces del intelecto),
percepción o conducta por trastorno en el cerebro fetal en desarrollo.
 Cuadriplejía espástica es el tipo más frecuente asociado a asfixia perinatal
 Para atribuir PC a un parto debe ser por EHI moderada o grave o Enfermedad asfíctica
 Define la enfermedad de un RN caracterizada por encefalopatía aguda con afectación hipóxico-
isquémica de otros órganos por un mecanismo asfíctico durante el parto
• Enfermedad Hipóxico isquémica
o Definición
 Afectación neurológica con compromiso de conciencia, alteración del tono, reflejos, a veces, convulsiones
por hipoxia sostenida
 Evoluciona en horas o días después de reanimación del RN. o Diagnóstico: debe cumplir 4/4 criterios
 Evento centinela en el trabajo de parto (ej. Desprendimiento de placenta, procidencia del cordón)
 Estado fetal inquietante (FC o doppler alterado)
 Necesidad de reanimación NN al menos por 10 minutos o Apgar <5 a los 5’
 Alteración del examen neurológico después de la reanimación
 Compromiso asfíctico de al menos otro órgano o Pronóstico: Tres estadios de Sarnat
Estadio I II III
Conciencia Hiperalerta Letárgico Coma
Tono Aumentado Disminuido Sin tono
Reflejos Aumentados Débiles Ausentes
Función autonómica Simpática: Midriasis Parasimpático: Miosis Medianas fijas
Convulsiones No Sí Raro
EEG Normal Alterado Muy alterado
Duración 1 – 2 días 2-14 días Horas – semanas
Evolución 100% normal 20% secuelas 50% muere, 50% secuela
grave

2
1
Temas clave de Pediatría y Cirugía Infantil

o Tratamiento
▪ Generalidades
• Lo habitual es que se presente en un RN que requirió reanimación
y que a los 20 minutos no recupera su condición neurológica
• Se recomienda hospitalizar para observación y evaluar su
pronóstico según Sarnat. ▪ Manejo general
• 1. Reanimación neonatal
• 2. Obtener y mantener o Temperatura adecuada o Perfusión
periférica o Estabilidad metabólica: Glucosa, calcio, equilibrio AB o Control
de las convulsiones
• 3. Traslado a cuidados intensivos neonatales ▪ Manejo
específico
• Oxígeno o Mantener niveles normales (PO2 60 – 90 mmHg; SatO2
94-97%) o Recordar que la hipoxemia disminuye el flujo cerebral y aumenta
los radicales libres en los parénquimas amagados
• CO2
o Mantener niveles normales (PCO2 35-45 mmHg) o Recordar que
hipercapnia provoca vasodilatación cerebral (robo) y que la hipocapnia disminuye
el flujo cerebral
• Perfusión o PAM de 45-50 mmHg en RNT y de 30-40 mmHg en RNPT o Evitar bolos bruscos de volumen. Procurar mantener un
buen volumen circulante efectivo solo con los aportes necesarios.
o No practicar restricción a menos que haya secreción inapropiada de ADH
o El primer día usar volúmenes de 40-60 ml/kg/día, ajustando diuresis
• Glucosa o Mantener niveles de 75-100 mg/dL o Evitar la hiperglicemia (aumenta el
lactato) y la hipoglicemia (aumenta los aminoácidos éxito-tóxicos)
• Calcio o Asegurar niveles normales /7-8 mg/dL) o Considerar riesgo de convulsiones
y compromiso cardíaco de no corregir cifras bajas
• Convulsiones o Propias del estado 2 de Sarnat o Expresión de trastornos
metabólicos cerebrales o Manejo farmacológico
▪ Fenobarbital 20 mg/kg, seguido de 3-5 mg/kg/día cada 12 horas.
• Edema cerebral
o Recordar que la perfusión cerebral (PPC) = PAM – PIC, por lo que se requiere
volemia adecuada evitando la sobrecarga
o Para ello medir balance estricto: Reponer diuresis + pérdidas insensibles.
o No está aprobado el uso de corticoides ni manitol. Lo más efectivo es mantener
una PAM adecuada.
• Compromiso de otros sistemas o Cardiovascular: Uso de drogas vasoactivas.
Dopamina si hay problemas de perfusión periférica
o Renal: Seguimiento de parámetros funcionales. Sonda Folley desde el ingreso

2
1
Temas clave de Pediatría y Cirugía Infantil

o Digestivo: Prevenir la ECN con régimen cero 2 o más días, considerando nutrición
parenteral
o Hematológico: Corregir trastornos de coagulación o Respiratorio: Oxigenoterapia
y ventilación mecánica
• Exámenes o Todos los exámenes inicialmente para evaluar funciones de distintos
órganos y sistemas
o Pasadas las 24 horas hacer una eco cerebral para evaluar edema. Si va a
hipotermia, adelantar la eco.
o RNM útil cuando ha pasado fase aguda porque confirma el mecanismo hipóxico
isquémico y tiene valor pronóstico.
• Hipotermia terapeútica o Uso en EHI moderada o severa (Sarnat II o III) o
Hipotermia selectiva de la cabeza (34 – 35 ºC de temperatura central) antes de las 6 horas
por 72 horas. • Evolución de la asfixia perinatal o Generalidades
 La tasa de mortalidad global es del 20% y la de secuelas 30%.
 El riesgo de PC es entre el 5 – 10% en sobrevivientes de episodio asfíctico.
 La evolución depende de la gravedad, ausencia de convulsiones, normalidad de EEG y hallazgos en
neuroimágenes.
o Pronóstico
 Sarnat I (EHI leve): 98 – 100% desarrollo neurológico normal (mortalidad 1%)
 Sarnat II (EHI moderada) 20 – 37% mueren o presentan trastornos del neurodesarrollo
• La presencia de convulsiones aumenta de 50 a 70 veces el riesgo de PC
• Mortalidad más elevada si se inician en las primeras 12 horas de vida o son frecuentes.
 Sarnat III (EHI grave): Lo más probable es que muera. Los sobrevivientes con seguridad tendrán alteración
del neurodesarrollo: PC, retraso intelectual, discapacidad visual o epilepsia
• Consideraciones durante el traslado de paciente en hipotermia pasiva o Se deben trasladar con hipotermia pasiva de 35ªC central o
Se debe mantener la incubadora apagada, ya que la mayoría alcanza los 34 ºC en estas condiciones.
o Controlar la temperatura cada 15 minutos y realizar gráfico (curva de temperatura) o La hipotermia severa ( >32 ºC) es el principal
riesgo. SI la temperatura rectal baja de 35 ºC es necesario encender la incubadora en temperatura mínima y abrigar.

Convulsiones neonatales
• Definición o Alteración paroxística y esterotipada de la función neurológica por una
despolarización excesiva y coordinada de un grupo de neuronas.
 RNT en los primeros 28 días
 RNPT Hasta las 44 semanas o Son la manifestación más común de disfunción
neurológica en el neonato
• Epidemiología Riesgo de crisis es mayor entre menor sea la EG o RNPT < 1500 g 58/1000 o
RNPT < 2500 g 14/1000 o RNT 3/1000
• Fisiopatología o Cerebro plástico en constante remodelación y formación de sinápsis requiere
un balance hacia las sinapsis excitatorias durante este período

2
1
Temas clave de Pediatría y Cirugía Infantil

o Umbral muy bajo para convulsionar dado por:


▪ Desbalance entre circuitos excitatorios e inhibitorios
• GABA-A (inhibitorio) tiene actividad excitatoria sobre el hipocampo

Receptores de glutamato sobrexpresados
▪ Aumento de receptores NMDA en hipocampo y neocorteza Mayor actividad excitatoria ▪ Retraso de maduración de circuitos
postsinápticos inhibitorios
• Densidad de las sinapsis en las espinas dendríticas es máxima
• Período fisiológico de intensa excitación sináptica
▪ Red proconvulsiva en la sustancia negra se desarrolla antes que la red anticovulsivante ▪ Mayor presencia de Gap
Junctions
• Amplifican pequeños desbalances
• Sincronizan la actividad neuronal
▪ Período refractario de hiperpolarización más corto
• Célula recupera rápidamente la excitabilidad.
• Clasificación Difiere respecto a la de niños mayores y adultos por inmadurez de las conexiones corticosubcorticales o Clasificación de
Volpe
Tipo de Convulsión Frecuencia Descripción

Clónica 20% Movimientos rítmicos y repetitivos. Pueden ser focales (en una parte del cuerpo) o multifocales (en 2
o más partes del cuerpo)
Mioclónica
5% Contracturas musculares repetitivas, rápidas, no rítmicas. Pueden ser focales o multifocales.
Tónica
Sutiles 10% Movimientos y posturas sostenidas (extensión o flexión)
65% De la boca: Movimientos de succión o deglución
Ojos: Desviación horizontal de los ojos, apertura ocular sostenida Brazos y piernas: Pedaleo,
nado, boxeo.

• Etiología
o Amplia gama de patologías tanto sistémicas como del SNC. Destacan la EHI (causa más frecuente) y las
alteraciones metabólicas (causa común y con tratamiento específico)
o Siempre considerar ▪ EHI
• 50% de las CN (causa más común)
• Ocurre dentro de las primeras 24 horas
• Riesgo a largo plazo de déficit neurológico y cognitivo
• Causas Asfixia perinatal, cardiopatías congénitas, shock, ACV, displasia broncopulmonar, HT
pulmonar, etc.
 Hemorragia intracraneal (15-25%)

2
1
Temas clave de Pediatría y Cirugía Infantil

 Infecciones (12%) Meningitis, encefalitis, sepsis, TORCH, endocarditis.


 Alteraciones metabólicas: Hipoglicemia, hipocalcemia, alteraciones electrolíticas.
 Trastornos del desarrollo cortical:
• Paquigiria: Circunvoluciones aplanadas y disminuidas en número por alteración en la migración
neuronal
• Lisencefalia: esquizencefalia.
• Evaluación y diagnóstico o Historia detallada: antecedentes perinatales y familaires o Examen
físico y neurológico del RN o Laboratorio inicial
 Glicemia, calcio, magnesio, ELP, GSA, lactato, hemograma
 LCR: citoquímico y cultivo
 En caso de sospecha de error innato del metabolismo: cuantificación de aminoácidos y ácido láctico
o Neuroimágenes:
 ECO encefálica EHI, leucomalaciaperiventricular, HIC, malformaciones
 TAC cerebral Calcificaciones, hemorragias
 EEG Origen epiléptico
• Tratamiento Iniciarse mientras se estudia
o Medidas generales
 Ventilación y perfusión adecuadas
 Establecer vía permeable
 HGT
 Muestra para laboratorio inicial o Considerar trastornos metabólicos más frecuentes ▪ Hipoglicemia
• Bolo SG 10% 2ml/kg (200 mg/kg) en 1
minuto puede repetirse en crisis
• Infusión SG 10% 5 – 8 mg/kg/min ▪
Hipocalcemia
• Gluconato de calcio 10% 2ml/kg EV lento ▪
Hipomagnasemia
• Sulfato de magnesio 25% 0.4 cc/kg IM o
Bolo EV de 0.1 cc/kg en 60 minutos
o Antiepilépticos
▪ Fenobarbital 1ra línea
• Mecanismo de acción: Antagonista GABA-A
• Dosis de carga 15 – 20 mg/kg EV (repetir en dosis de 10 mg/kg máximo 2 veces, si no cede
pasar a 2da línea Fenitoína)
• Dosis de mantención no antes de 12-24 horas (vida media prolongada) 3 – 5 mg/kg/día en
2 dosis ▪ Fenitoína 2da línea
• Mecanismo de acción: bloqueador canales de sodio
• Dosis de carga 20 mg/kg EV a 1mg/kg/min (repetir la carga máximo 2 veces, si no cede pasar
a 3ra línea lorazepam o midazolam)

2
1
Temas clave de Pediatría y Cirugía Infantil

• Dosis de mantención: 12 horas post carga 2 – 5 mg/kg/día en 2 dosis ▪ Lorazepam


3ra línea
• Dosis 0.05 – 0.1 mg/kg EV infusión lenta
 Midazolam Última opción
• Dosis de carga 0.02 – 0.2 mg/kg EV
• Dosis de mantención 0.1 – 0.3 mg/kg/hora
 Crisis refractarias a antiepilépticos Terapia empírica
• Piridoxina 100 mg EV
• Biotina 10 mg/día VO
• Ácido folínico 2.5 – 5 mg/dia en 2 dosis
• Pronóstico o Tiene directa relación con la etiología y el manejo adecuado y precoz o En el EEG
inter ictal
 Normal 8 – 10% secuelas neurológicas
 Severamente anormal 90% secuelas neurológicas graves o Mortalidad ha disminuido

Síndrome de Distrés Respiratorio


• Definición o Cuadro de dificultad respiratoria en el RN que se inicia en las primeras horas y se caracteriza por
taquipnea, cianosis, quejido, retracción subcostal y compromiso de la saturación y ventilación alveolar.
• Clasificación etiológica: Se clasifica en relación a su principal causa:
 Asfixia perinatal
 Prematurez (reabsorción del líquido pulmonar)
 Trastornos de la circulación pulmonar
 Infecciones
 Problemas respiratorios crónicos
 Malformaciones congénitas

Enfermedad de membrana hialina


• Epidemiología o Causa más frecuente de SDR en el RN o
Característica del RNPT 80% antes de las 28 semanas o Incidencia general
10% en RNPT o Segunda causa de mortalidad en RN
• Fisiopatología o 1. Déficit o ausencia de surfactante por
inmadurez o 2. Aumento de la tensión superficial alveolar secundaria o 3.
Colapso alveolar progresivo o 4. Atelectasia pulmonar Lleva a hipoxemia
e hipercapnia.
o 5. La atelectasia condiciona una disminución del volumen residual, creando una alteración V/Q y un aumento del
trabajo respiratorio debido a disminución de la distensibilidad pulmonar
o 6. Finalmente, fatiga muscular.
• Etiopatogenia o Factores de riesgo: El principal es la prematurez.
Otros son:

2
1
Temas clave de Pediatría y Cirugía Infantil

 Cesárea sin trabajo de parto (no se induce la maduración pulmonar) ▪ Asfixia perinatal
 HMD
 Eritroblastosis fetal
 Antecedentes de EMH en hermanos o Factores protectores
 Mayor edad gestacional
 Parto vaginal (estímulo para maduración pulmonar)
 Uso de corticoides (maduración pulmonar)
• Clínica o Insuficiencia respiratoria en las primeras 0 – 6 horas de
nacido que avanza rápida y progresivamente
▪ Quejido, retracción costal, taquipnea, aleteo nasal, cianosis progresiva (dificultad respiratoria)
o Empeora de 6 a 72 horas por agotamiento, acentuando los síntomas o Período de mantenimiento de 3 a 5 días (por
madurez pulmonar), donde se estabiliza la sintomatología o Posteriormente, período de recuperación con franca mejoría.
• Diagnóstico Ante sospecha clínica, apoyarse en radiografía. o
Imágenes: Radiografía de tórax Volumen pulmonar disminuido, patrón
retículo granular difuso y homogéneo, broncograma aéreo (condensación
por colapso alveolar), velamiento de todo el campo pulmonar (casos
graves)
o Laboratorio: Evidencia insuficiencia respiratoria
 GSA Hipoxemia, hipercapnia y acidosis.
• Tratamiento o Preventivo
 Evitar nacimientos de RNPT
 Evaluar madurez pulmonar fetal
 Corticoides en el prenatal a la madre
 Evitar hipoxia, acidosis e hipotermia durante y después del parto.
o Medidas generales en paciente grave
 Tratamiento de la insuficiencia respiratoria: Oxigenoterapia que puede requerir ventilación asistida.
 Surfactante o Complicaciones
 Síndromes de escape aéreo
 Infecciones
 DAP
 HIC
 Displasia broncopulmonar
 Neumonías por aspiración (en relación a la ventilación mecánica)
 Hemorragia pulmonar
 Retinopatía del prematuro
• Pronóstico Depende de la edad gestacional la magnitud de la
insuficiencia respiratoria y las complicaciones asociadas.

2
1
Temas clave de Pediatría y Cirugía Infantil

Taquipnea transitoria de lRN


• Definición o Desorden autolimitado del parénquima pulmonar que se caracteriza por edema pulmonar
secundario a reabsorción y clearence disminuido del líquido pulmonar fetal.
o Tiene un curso corto y benigno caracterizado por dificultad respiratoria manifiesta por taquipnea con
retracción leve.
o Afecta principalmente al RNT
• Fisiopatología o Demora en la reabsorción del líquido pulmonar presente antes del nacimiento que genera
edema pulmonar.
o Es más frecuente en partos por cesárea (falta de estímulo), macrosomía (mayor cantidad de líquido) y
grandes aportes de líquidos a la madre.
• Clínica o Dificultad respiratoria desde el nacimiento: Principal manifestación Taquipnea. ▪ Rara vez
requiere oxígeno FiO2 > 50%.
o Examen físico: MP disminuido o Mejoría entre las 24 y 48 horas (puede prolongarse hasta 5 – 7 días)
• Diagnóstico: Ante la sospecha, se apoya en la epidemiología y las imágenes.
o Antecedentes perinatales: Cesárea, RNT o Clínica: Dificultad respiratoria (taquipnea) con requerimientos
menores del 50% FiO2.
o Radiografía de tórax
▪ Patrón congestivo (edema pulmonar)
• Aumento de la trama broncovascular
• Imágenes hileofugales
▪ Volumen del campo pulmonar conservado
• Tratamiento o Iniciar régimen cero (para facilitar la mecánica ventilatoria) y administrar fleboclisis habitual para
requerimientos HE
o Oxígenos según requerimientos (monitorizar saturación) o Exámenes para descartar cuadro
infeccioso (hemograma, PCR)
• Pronóstico o Bueno. Recupera en un 100%

Neumonía neonatal
• Definición o Cuadro infeccioso por complicación de infecciones connatales o
intrahospitalarias o Puede ser de inicio precoz o de inicio tardío.
• Epidemiología o Importante causa de mortalidad o Incidencia mayor en RNPT (10% vs 1%) o
Tipos
 Connatales
 Intrahospitalarias o Vía de infección
 Ascendente
 Hematógena
• Fisiopatología o Neumonía de inicio precoz
 Se adquiere los 3 primeros días de vida desde la madre (2 vías posibles)
• Aspiración intrauterina, intra o post parto de líquido amniótico infectado
• Transmisión trasplacentaria de MO

2
2
Temas clave de Pediatría y Cirugía Infantil

o Neumonía de inicio tardío


▪ Ocurre durante la hospitalización o después del alta debido a colonización de MO intrahospitalarios que pueden invadoir el organismo por injuria
traqueal, bronquial o vía hematógena. Patogenia
• Patogenia o Factores predisponentes
▪ Maternos
• RPM
• ITU 15 días previos al parto
• Colonización vaginal patológica
• Corioamnionitis
▪ RN
• Características anatómicas de la vía aérea
• Inmunología inmadura • Trauma de la vía aérea
• Presencia de meconio en la vía aérea.
• Etiología
o Connatal: SGB, E Coli, Listeria
▪ Otros: VHS, CMV o Intrahospitalaria: Klebsiella, pseudomonas, enterococo
• Clínica o Dificultad respiratoria: Taquipnea, quejido, cianosis, uso de musculatura accesoria o
Síntomas infecciosos: Taquicardia, fiebre, mala perfusión, rechazo alimentario o Síntomas respiratorios:
Apnea, disminución del MP, dificultad respiratoria (taquipnea, quejido, cianosis, uso de musculatura
accesoria)
• Diagnóstico: Ante la sospecha, solicitar laboratorio para dirimir etiología infecciosa o Laboratorio
 Hemograma: Leucocitosis con predominio segmentados
 Cultivos: Secreción traqueal y hemocultivo
 GSA: Insuficiencia respiratoria o Imágenes Radiografía de tórax
 Condensación pulmonar
 Broncograma aéreo (condensación)
 Derrame paraneumónico
 Atelectasias
 Opacidades nodulares
• Tratamiento o Medidas generales
 ABC: Segurar ventilación, equilibrio HE y AB y hemodinamia adecuada o Antibióticos empíricos de amplio
espectro
 Ampicilina + Gentamicina (aminoglicósido) o Manejo respiratorio
 Oxigeno en halo
 Ventilación mecánica
• Pronóstico o La mayoría evoluciona bien pero el pronóstico depende de la severidad de la
enfermedad, la EG, y el
MO

2
2
Temas clave de Pediatría y Cirugía Infantil

Síndrome de aspiración meconial


• Definición o Trastorno respiratorio por aspiración de meconio del líquido amniótico.
o Puede ocurrir antes, durante o inmediatamente después del parto.
o Los casos más severos son secundarios a procesos patológicos intrauterinos como asfixia crónica e infección
• Fisiopatología o El mecanismo involucra la presencia de meconio en el líquido amniótico,
aspiración y enfermedad pulmonar.
o 25% de los RN con líquido teñido en meconio presentan depresión respiratoria al nacer
por procesos intrauterinos como asfixia crónica e infección que lleva a presencia de meconio en
el líquido y gasping en el RN
o Hay hipertensión pulmonar persistente en los casos severos, lo que contribuye a la
hipoxemia o Mecanismo
 1. Estimulación de la motilidad intestinal y relajación del esfínter anal por asfixia intrauterina
 2. Paso de meconio al líquido amniótico
 3. Esfuerzos respiratorios profundos por hipoxemia
 4. Aspiración de líquido amniótico + meconio en la VAS
 5. Primeras respiraciones extrauterinas
 6. Aspiración de líquido amniótico + meconio a bronquios, bronquiolos y alvéolos.
o Consecuencias
▪ Obstrucción de la VA
• Si la obstrucción es completa, se producen Atelectasias distales; Si la obstrucción es parcial,
habrá neumotórax. ▪ Irritación química
• Es posterior (24 a 48 horas) y provoca neumonitis exudativa, daño epitelial, colapso alveolar y
finalmente, necrosis celular. ▪ Infección
• El meconio es un excelente medio de cultivo (principalmente E. coli)
 Inactividad del surfactante ▪ Hipoxemia: dada por
• Disminución de la ventilación
• Hipertensión pulmonar persistente
• Shunt de derecha a izquierda por aumento de la resistencia vascular pulmonar
• Clínica o Inspección: Vérmix, cordón y uñas teñidas de meconio o Manifestaciones
respiratorias
 Dificultad respiratoria: Taquipnea, cianosis, retracciones sub e intercostal, respiración abdominal,
quejido y aleteo nasal
 Tórax en barril
 Auscultación: Crépitos y roncus (condensación y obstrucción)
• Diagnóstico: La sospecha clínica se debe confirmar con radiografía o
Radiografía de tórax
 Infiltrados difusos o en focos múltiples
 Áreas de hiperinsuflación
 Síndrome de escape aéreo

2
2
Temas clave de Pediatría y Cirugía Infantil

• Tratamiento o Manejo inicial En el contexto de la dificultad respiratoria se


aspira.
 Prevenir hipoxia intrauterina
 Aspiración de líquido amniótico antes de la primera respiración de boca, orofaringe y fosas nasales
 Aspiración endotraqueal o Manejo específico
 Oxigenoterapia
 Corregir la acidosis
 Restricción de líquido
 Apoyo vasoactivo
 Sedación
 Antibióticos
 Considerar catéter umbilical arterial
• Pronóstico: Bueno pronóstico, depende del grado de dificultad respiratoria
al nacimiento.

Síndrome de escape aéreo


• Definición o Síndrome producido por aire que se escapa desde el espacio intraalveolar hacia lugares donde normalmente no
está presente
o Condiciones más comunes son el neumotórax, el neumomediastino, el enfisema pulmonar intersticial y el
neumopericardio
o Ocasionalmente: neumoperitoneo y enfisema subcutáneo

Hipertensión pulmonar persistente


• Definición o Es una transición anormal de la circulación fetal a la neonatal o Ocurre cuando la presión y
resistencia vascular pulmonar permanece alta después del nacimiento generando persistencia del shunt de derecha a
izquierda a través de las vías fetales de circulación
o Ocurre hipoxemia que no responde al soporte respiratorio convencional o Afecta más a RNT
• Epidemiología o Incidencia de 1 en 5000 RNV o Mortalidad alcanza hasta el 50%
• Clínica o RNT con antecedentes de asfixia o aspiración de meconio o Cianosis que no mejora con O2 o Taquipnea,
quejido, retracción costal o Segundo ruido aumentado o desdoblado, soplo de insuficiencia tricuspídea ( congestión) o
Signos de insuficiencia cardíaca o Labilidad de la PaO2
• Diagnóstico: Sospecha clínica. Complementar con estudios o Laboratorio
 PaO2 < 100 – 150 mmHg con FiO2 100%
 PaO2 pre ductal < post ductal o Imágenes
 Radiografía de tórax
• Campos pulmonares libres
• Signos de enfermedad pulmonar asociada
• Congestión venosa por falla VI
▪ Ecocardiograma Doppler

2
2
Temas clave de Pediatría y Cirugía Infantil

• Shunts DI por foramen oval y DAP


• Tratamiento o Corrección de factores que aumentan la presión pulmonar ▪ Hipotermia
 Hipoxia
 Acidosis o Traslado a centro especializado
 Ventilación mecánica
 Parálisis muscular y sedación
 Corrección de hipotensión sistémica con DVA, ECMO

Malformaciones pulmonares del RN


• Hernia diafragmática o Definición
 Presencia de vísceras abdominales herniadas hacia la cavidad torácica, siendo el sitio más común el
hemitórax izquierdo por defecto del diafragma posterior (Hernia de Bochdalek) 70%
 Hernia derecha con defecto anterior o posterior (Hernia de morgagni) es menos común
 El 50% se asocia a otras malformaciones: Defectos del tubo neural, cardiopatía congénita,
malrotación intestinal.
 HD congénita se asocia a trisomia 13, 18 y síndromes como la secuencia de Pierre-Robin o
Epidemiología Incidencia 1 en 2000-5000 RNV
• Displasia broncopulmonar o Definición
 Enfermedad pulmonar crónica caracterizada por daño pulmonar secundario a oxigenoterapia o VM
 Factores de riesgo: BPN, Menor EG, hombres, gravedad del cuadro respiratorio inicial o
Epidemiología
 Incidencia 10 – 20% de los RNPT que requirieron VM (relativamente frecuente)
 Mortalidad: Relativamente frecuente (7 – 30%) o Diagnóstico (criterios)
 VM y oxigenoterapia > 24 horas
 O2 > a 28 días o después de 36 semanas de EG ▪ Cambios crónicos en la Rx de tórax
• 2 a 3 día: Similar a EMH
• 4 a 10 día: Opacificación casi completa de ambos campos
• 10 a 20 día: Imágenes radolúcidas pequeñas con áreas irregulares de > densidad
• > 30 días: Imágenes radiolúcidas de mauor tamaño con bandas de > intensidad
o Tratamiento
▪ Prevención del daño.

Hiperbilirrubinemia
• Introducción o Puede afectar al 60% de los RN sanos en la primera semana de vida y al 85% de los RNPT
o RNT durante la primera semana
 8% > Percentil 95 17 mg/dL
 1 – 2% > 20 mg/dL (1:50)
 0,15% > 25 mg/dL (1: 700)

2
2
Temas clave de Pediatría y Cirugía Infantil

• Fisiología o Producción de bilirrubina en los primeros días de vida (8 a 10 mg/kg/día)


 75% de la bilirrubina diaria viene del catabolismo del hemo, prinicipalmente de la Hb
de los GR
 25% restante de la eritropoyesis ineficaz GR que se destruyen antes de pasar a la
circulación
• Esta BR vuelve a la circulación mediante la reabsorción en el intestino (circuito enterohepático)
 Producción total: 8 a 10 mg/kg/día o Distribución de la BR
 Características: Es lipofílica y tiene baja solubilidad Alta distribución tisular (piel)
 Ictericia clínica > 4mg/dL o Transporte, captación hepática y conjugación
 En el plasma, la BI circula unida a albúmina de manera reversible
• Esta unión se altera ante algunas situaciones: inmadurez, acidosis, fármacos
 En el hígado, los hepatocitos separan la BI de la albúmina
 La BI es insoluble en agua, solo se puede excretar si se convierte en soluble BD o BC
• Se solubiliza uniéndose al ácido glucorónico (glucoronización)
o Absorción y transporte intestinal
 La bilirrubina se excreta por la bilis
 La absorción intestinal en el RN es mayor, especialmente en la primera semana, formando el circuito
enterohepático.
• Etiopatogenia de la ictericia en RNT con LM
• Tres mecanismos o Flora intestinal escasa genera menor reducción en uribilinógeno y estercobilina o
La BC es relativamente inestable, pudiendo ser hidrolizada en el intestino ante la alcalinidad del duodeno
y yeyuno o Composición especial de la LM rica en grasa o ciertos lípidos.
o Toxicidad de la bilirrubina
 En alteraciones de la BHE la BR puede penetrar en el
• Bilirrubina es toxica para el SNC
SNC aún unida a albúmina
• Alteraciones en el metabolismo
 La toxicidad no depende exclusivamente de su unión a
neuronal y la transmisión sináptica albúmina, sin embargo, su disociación es importante • Baja solubilidad en agua y pH ácido

• Ácidos grasos genera precipitación en forma de


• pH Acidosis respiratoria con hipercapnia cristales Kernicterus (favorece el depósito en el
SNC)
• Fármacos o Sulfas, salicilatos, ceftriaxona o Ampicilina, aminofilina, furosemida, ibuprofeno en RNPT
(aumentan la BR libre) ▪ El nível de BR libre es el predictor más adecuado para neurotoxicidad
• No hay evidencia sobre un determinado punto de corte, si no de factores asociados
o Menor madurez (35 y 37 semanas) o Mayor velocidad de aumento de BR (Hemólisis) o Mayor duración
de valores elevados ▪ Otros trastornos predisponen al a neurotoxicidad
• Alteración de la BHE
• Hemólisis

2
2
Temas clave de Pediatría y Cirugía Infantil

• Hipoglicemia
• Hipoalbuminemia
• Acidosis respiratoria
 Cifra de 20 mg/dL (Hsia et al, 1952) solo aplicable en RNT con enfermedad hemolítica luego de 48 horas de
vida
 Estudios indican que RNT sanos y con peso adecuado tienen un mínimo riesgo aún con niveles de 25 mg/dL
o más
 No hay diferencias en la evolución neurológica a dos años en niños con niveles entre 25 y 30 respecto a
grupo control (Newman, 2006)
• Cuadros clínicos o Neurotoxicidad por hiperbilirrubinemia ▪ Dos
fases clínicas
• Temprana y aguda reversible (EBA moderada)
• Tardía y lenta Irreversible (EBA grave/Kernicterus) ▪ Cuadro clínico
Encefalopatía bilirrubínica aguda
• Epidemiología o Incidencia de 3:100.000 RNV o EBA grave
 Se ha asociado a LM ineficaz, gran baja de peso al nacer y alteraciones
genéticas
 Mortalidad 10 %; Secuelas 70%.
• Clínica o EBA Moderada: Apatía, somnolencia, irritabilidad o EBA Grave:
 Histología: Cerebro impregnado en bilirrubina con pérdida neuronal,
desmielinización y degeneración de las regiones afectadas (GB, hipocampo, nucleo VIII
par, cerebelo) Kernicterus
 Signos iniciales
• Succión débil, letargo, rechazo alimentario
• Llanto agudo, irritabilidad.
• Fiebre, convulsiones, coma
• Hipotonía con crisis de hipertonía, opistótonos ▪ Secuelas alejadas
• Trastornos extrapiramidales
• Hipoacusia
• Displasia dental
• Problemas de conducta
▪ Prevención
• Estímulo de la LM adecuada
• LM en las primeras 2 horas varias veces al día
• RN ictéricos con factores de riesgo evitar alta antes de las 48 horas o 36 –
37 semanas o LME
o Pérdida de peso > 10%

2
2
Temas clave de Pediatría y Cirugía Infantil

o Ictericia fisiológica
 Definición:
• Hiperbilirrubinemia transitoria durante la primera semana de vida cuando o Posterior a las 24 horas de vida o BR no
aumenta más de 5 mg/día o BC < 2.5 mg/dL o Valores superiores en RNT no son mayores de
 16 mg/dL en LM
 13 mg/dl en Fórmula o Duración < 7-10 en RNT
• Se relaciona con la LM y un mayor porcentaje de descenso del peso
 Etiología
• Ocurre cuando el ritmo de producción supera al de eliminación por o Aumento de la producción de BR
 Mayor volumen eritrocitario con vida media de eritrocito más breve
 Aumento del circuito enterohepático
 Aumento del catabolismo del hemo o Disminución de la eliminación de la BR
 Alteración en la captación y transporte intracelular mecanismos más lentos y menos
maduros en los primeros días
 Alteración de la conjugación Disminución de la actividad de la glucoroniltrasferasa
los primeros 3 días de vida
 Alteraciones de la excreción Producción muy exagerada o trastorno hepático
o Ictericia por LM
 Definición
• Causa más frecuente en RNT con LM
• Frecuencia 3 veces mayor de BR > 12 mg/dL y 6 veces
mayor > 15 mg/dL en comparación a los alimentados con
fórmula
 Clasificación
• Temprana (primera semana) o Etiología
 Aumento del circuito enterohepático por menor aporte
calórico e hídrico en los primeros días (Mayor reabsorción de
BI)
• Coexistenciacon hipernatremia
Aumento de la osmolaridad plasmática

2
2
Temas clave de Pediatría y Cirugía Infantil

(CONTROLAR SIEMPRE!) o Alteración de la


BHE o Entrada de BR en el SNC
 Variación en los lípidos de la leche Elevada actividad de
lipoproteína lipasa Liberación de Ac. Grasos en el intestino
• Disminuyen por competencia la captación
de BR en el hígado
• Efecto inhibitorio de la glucoronización
• Mayor reabsorción intestinal de BR
 Polimorfismo genético Genes UGT1A1 y OATP-2
• Tardía (luego de 7 – 10 días) o Definición: Valores entre
10 – 15 mg/dL durante varias semanas o Etiología
 Acción inhibitoria sobre la conjugación de esteroide en la
leche
 Aumento de los ácidos grasos no saturados que disminuyen
la conjugación del a BR
o Tratamiento:
▪ Si persiste con cifras > 22 mg/dL Fototerapia ▪ Indicación: NUNCA DEJAR LM o Ictericias patológicas: Por aumento de
la oferta o disminución de la eliminación patológicos ▪ Aumento patológico de la oferta de BR
• Enfermedad hemolítica o Proceso hemolítico por incompatibilidad sanguínea materno-fetal
 Sistema AB0 (más frecuente)
 Sistema Rh
 Subgrupos o Esferocitosis familiar o Déficit de G6PDHA o Infecciones graves
y Shock
• Hematomas y hemorragias o Cefalohematoma
o Hematomas en partos dificultosos o Hemorragias internas
• Incremento de la reabsorción intestinal (Aumento del circuito enterohepático) o
Retraso en la LM en RNPT muy pequeños o enfermos o Obstrucción intestinal o LM insuficiente o
Policitemia Mayor volumen globular determina destrucción eritrocitaria ▪ Disminución
anormal de la eliminación
• Síndrome de Crigler-Najjar Déficit de glucoroniltrasnferasa
• Síndrome de Glibert Hiperbilirrubinemia moderada por mutación genética ▪ Ictericias
prolongadas > 10 días
• Causas más frecuentes o Nacimiento antes de las 38 semanas
o Incompatibilidad AB0 o Ictericia por LM tardía • Clínica o
Presentación clínica
▪ Ictericia temprana (< 72 horas)
• Riesgo de ser por hemólisis ▪ Ictericia tardía (> 72 horas)
• Principal causa Disminución de la excreción de la BR
• Factores de riesgo: LM inadecuada, Déficit de G6PDHA, sexo masculino (asociación genética con
Déficit de G6PDHA)

2
2
Temas clave de Pediatría y Cirugía Infantil

o Evaluación clínica
▪ Examen físico
• Ictericia sigue distribución cefalocaudal (Kramer)
• Determinar visceromegalias, hematomas, púrpura ▪
Laboratorio
• Grupo y Rh
• Coombs indirecto en la madre (embarazo)
• Grupo del RN en el cordón
• Recuento reticulocitos
• Estudio diagnóstico o Ictericia temprana
▪ Grupo y Rh de la madre
• Madre grupo 0 y RN A o B Incompatibilidad AB0 (la más frecuente)
 Coombs positiva y antecedente de sensibilización Eritroblastosis fetal
 Rh y Coombs directo al RN si madre es Rh negativo Confirma, no descarta ▪ Recuento de
reticulocitos
• Aumentado en todo proceso hemolítico (> 4 – 5%) o RNT normal: 0.5 – 2% o RNPT
normal: Hasta 6%
 Ictericia y Coombs positivo en ausencia de incompatibilidad AB0 y Rh Hemólisis por incompatibilidad de subgrupos
 Hiperbilirrubinemia importante en RN con antecedentes familiares y personales Déficit de
G6PDHA
o Ictericia tardía
 Detectar problemas de LM
 Evaluar pérdida de peso
 Evaluar factores de riesgo
• 35 – 37 semanas
• Cefalohematoma
• Hematoma subaponeurótico

2
2
Temas clave de Pediatría y Cirugía Infantil

• Manejo
o Manejo en RN > 35 semanas (APA)
 Promover la LM
 Medir la bilirrubina en todos los RN ictéricos en las primeras 24 hroas
 Controlar RN < 38 semanas. Con LM, tienen más riesgo de bilirrubina elevada ▪
Evaluación de riesgo en todo RN ictérico antes del alta: 2 opciones (APA)
• Medir BR sérica antes del alta
• Evaluar factores de riesgo
 Seguimineto después del alta según factores de riesgo (útiles en ausencia) o Ictericia <
24 horas (hemólisis) o BR en zona de alto riesgo antes del alta o Nacimiento entre 35 y 37
semanas o LM insuficiente con > 10-12% de pérdida de peso o Hermanos que requirieron
fototerapia o Cefalohematoma
• Control posterior al alta: Evaluar LM, peso y deposiciones
• Guía según nomograma de Buthani

2
3
Temas clave de Pediatría y Cirugía Infantil

o Tratamiento específico
• 4 mecanismos o Acelerar la excreción de la BR por vías alternativas Fototerapia o Eliminar la
BR plasmática Exanguinotransfusión o Acelerar la captación y conjugación hepática Fenobarbital
y ciofibrato o Inhibición de la formación de BR en el paso de hemo a biliverdina tinmesoporfirinas
• Uso de curvas

2
3
Temas clave de Pediatría y Cirugía Infantil

▪ Fototerapia
• El más utilizado
• Mecanismo: Luz excita fotoquímicamente a la BR generando fotoproductos menos tóxicos y de rápida
eliminación
• Indicaciones o RN > 35 semanas o Según nomograma de Bhuthani o RN enfermos o prematuros
Según clínica
• Efectos adversos

2
3
Temas clave de Pediatría y Cirugía Infantil

o Erupciones morbiliformes o Exantemas o Eritemas o Respiración irregular o Fiebre y conjuntivitis o


Deposiciones disgregadas y frecuentes ▪ Exanguinotransfusión
• Extracción de la BR del plasma mediante cambio de sangre de un dador (2 volemias)
• Efectos o Sustracción de la BR del IV o Remoción de anticuerpos o Corrección de la
anemia
• Indicaciones o Según curva o BT > 25 mg/dL o ascenso > 0,5 mg/hr las primeras 72 horas
o Incompatibilidad Rh o eritroblastosis fetal si
 BR cordón > 4,5 mg/dL
 Ascenso > 1 mg/dL a pesar de fototerapia
 Aumento > 0,5 mg/dL a pesar de fototerapia
 Anemia con Hto < 35
 BT > 20 en cualquier momento o Otros trastornos hemolíticos
graves o RN sanos con valores muy elevados o Signos neurológicos sospechosos de EBA
Inmediato
• Complicaciones o Mortalidad 1% en RN estables y 10 veces más en RN enfermos o RNPT
o Complicaciones más frecuentes
 Hipotermia
 Hipocalcemia
 Arritmias, apneas y PCR
 Infecciones
 ECN ▪ Fármacos
• Fenobarbital o Inductor enzimático que aumenta la conjugación y excreción de BR por el
hígado o Poco usado por fototerapia
• Clofibrato o Estimula la glucuronil transferasa o Induce mejor y más rápido que
fenobarbital
• Tinmesoporfirina o Compite con la hem oxigenasa en la unión al hemo, impidiendo la
formación de biliverdina
• Ig EV o Disminuye uso de exanguinotransfusión en RN con enfermedad hemolítica por
Rh y AB0.

Hijo de madre diabética


• Introducción o La asociación entre diabetes y embarazo tiene una frecuencia del 1 al 5% y se da
de dos formas ▪ Diabetes pre-gestacional
• 5 – 10% de las embarazadas diabéticas
• Asocia mayor riesgo de morbimortalidad perinatal, muerte fetal, malformaciones,
EMH, macrosomía, RCIU y complicaciones metabólicas ▪ Diabétes gestacional
• Se inicia durante el embarazo (90% de las embarazadas diabéticas)

2
3
Temas clave de Pediatría y Cirugía Infantil

• Determina mayor riesgo obstétrico asociándose a macrosomía, sufrimiento fetal y


complicaciones metabolicas del RN
• Problemas de RN HMD o Parto Complicaciones por macrosomía
 Distocia y traumatismo
 Sufrimiento fetal agudo
 Mayor incidencia de cesárea o Período NN
 Hipoglicemia
• Problema más frecuente
• Incidencia de hasta el 50% de todos los HMD
• Se debe a hiperinsulinismo, disminución de la entrega de glucagón y depresión de la respuesta
de catecolaminas.
• Período de mayor riesgo hasta 48 horas y las primeras 6 horas son las más críticas
 Hipocalcemia
• Hasta en el 50% de los HMD insulinodependientes
• Se da en las primeras 24 horas
• Disminución de la respuesta a la PTH y/o hipomagnasemia asociada o Si no se corrige con calcio,
debe administrarse paralelamente magnesio ▪ Hipomagnasemia
• Debe sospecharse en hipocalcemia refractaria a tratamiento
 EMH
• Hiperinsulinismo fetal interfiere en la síntesis de surfactante
• HMD tiene una maduración más tardía de neumocitos tipo II y la insulina antagoniza el
efecto de maduración del cortisol. ▪ Poliglobulia
• Incidencia: 5 – 30%
• Se ha demostrado un aumento de la EPO circulante debido a menor entrega de O2 desde la
madre al feto, aumento del consumo, hiperinsulinismo y EPO ineficaz
 Hiperbilirrubinemia
• Más frecuente que en la población general determinada por poliglobulia, EPO ineficaz aumentada e inmadurez enzimática hepática.
 Miocardiopatía hipertrófica
• Engrosamiento del septum interventricular con obstrucción del tracto de salida aórtico,
secundario a hipertrofia miocárdica por hiperinsulinismo
• Se debe a depósito de glucógeno en el septum
• Sospechar en HMD con IC o SDR de etiología poco clara
• Diagnóstico es por ecocardiograma
• Pronóstico es bueno, la mayoría está asintomático al mes y la hipertrofia revierte ▪
Malformaciones
• Incidencia 2 a 3 veces mayor
• Etiologia: Mayor predisposición genética y el efecto adverso de la hiperglicemia sobre la
multiplicación celular el primer trimestre

2
3
Temas clave de Pediatría y Cirugía Infantil

• Buen control de la glicemia en el embarazo disminuye el riesgo de malformaciones


• Las más frecuentes son o SNC: anencefalia, encefalocele, mielomeningocele, holoprosencefalia
o CV: TGA, CIV, DAP, Ventrículo único, hipoplasia VI o Renales: Agenesia, doble uréter o GI: Atresia
anorrectal, colon izquierdo pequeño Obstrucción intestinal baja.
o Pulmonares: Hipoplasia pulmonar (se asocia a agenesia renal) • Diagnóstico o De certeza se basa en los antecedentes
maternos o Sospecha
 Todo RN GEG con características típicas
• Obeso, aspecto pletórico, facie mofletuda, abdomen globuloso, aumento de la circunferencia
de hombros (CH) con diferencia CH:CC > 7 cms ▪ Ante la sospecha, siembre pedir HbAc1 a la
madre.
• Tratamiento o Hospitalización preventiva en neonatología
• < 2000 gr y/o < 34 semanas
• Insulinodependientes
• PEG
 Manejo: Iniciar glucosa parenteral (4 a 6 mg/kg/min) antes de los 30 minutos de vida o Controlar con
exámenes
 1 hematocrito
 Glicemia seriada las primeras 48 horas ▪ En hospitalizados controlar
• HGT a las 30’, 1, h, 8, 12, 24, 36 y 48 horas Si es < 45 mg/dL controlar glicemia
• Hematocrito a las 2 horas
• Calcemia alas 8, 24 y 48 horas o Ca iónico < 4,5 mg/dL o Ca total < 7 mg/dl y no responde a
tratamiento Tomar magnasemia
o Recomendaciones específicas
▪ En HMD hipoglicémico comenzar infusión de glucosa EV 4 – 6 mg/kg/min, luego titular según controles
• Las células son hipersensibles a glucosa, ante una carga exagerada liberan mucha insulina por lo que hay que aportar solo la glucosa
necesaria para mantenerla normal
▪ INICIO PRECOZ DE ALIMENTACIÓN A las 2 horas de vida con LM

2
3
Temas clave de Pediatría y Cirugía Infantil

Trastornos de la glucosa
• Introducción Adaptación neonatal o Niveles intrauterinos de glucosa son 54 – 90 mg/dL, con menores valores a mayor EG
o Al nacer se interrumpe la transferencia placentaria de glucosa lo que genera la necesidad de cambios hormonales que permitan
mover glucosa endógena
▪ Aumenta epinefrina, glucagón, GH; y disminuye la insulina o RN sano disminuirá su glicemia las primeras 2 horas de vida, que
se estabiliza con la respuesta glucogenolítica (glucógeno hepático), posteriormente la movilización de acidos grasos y la producción de
cuerpos cetónicos.
o Alteración en esta regulación puede determinar hipo o hiperglicemia

Hipoglicemia
• Definición: o Nivel límite de la concentración de glucosa que permite función cerebral normal.
o Valores:
 RN asintomático: < 45 mg/dL
 RN sintomático: 40 mg/dL
 RN asintomáticos pero con factores de riesgo primeras 48 horas: 36
mg/dL durante el primer día
o Se puede usar el HGT o La hipoglicemia tiene relevancia clínica por ser una causa
prevenible de daño del SNC
• Factores de riesgo o Pre natales: HMD, madre hipertensa, enfermedad hemolítica por Rh, uso
de beta agonistas y de hipoglicemiantes orales
o Neonatales: Prematurez, RCIU, asfixia perinatal, poliglobulia, hipotermia, infecciones,
malformaciones cardíacas de bajo flujo, alteraciones endocrinas
• Fisiopatología y etiopatogenia o La homeostasis de la glucosa en el SNC depende tanto de la
concentración como del flujo cerebral o Una caída en la concentración produce un aumento
compensatrorio del flujo manteniendo la tasa metabólica cerebral.
o Si no es suficiente para el correcto aporte de glucosa, se inicia el uso de compuestos
metabólicos alternativos como lactato que puede provocar daño neuronal o HMD
 Glicemia depende del control de la glicemia materna en el embarazo y en
especial en el parto.
 El hiperinsulinismo fetal determina macrosomía, acumulación de tejido
adiposo, poliglobulia y alto riesgo de hipoglicemia precoz.
o RCIU
 Presenta múltiples factores que pueden determinar hipoglicemia:
• Menor depósito de glucógeno
• Menor depósito de ácidos grasos (gluconeogénesis)
• Menor actividad enzimática
• Mayor uso de glucosa cerebral
• Glicólisis anaeróbica por hipoxia
• Hipermetabolismo por frío
• Aumento de extracción por poliglobulia.

2
3
Temas clave de Pediatría y Cirugía Infantil

o RNPT Mayor riesgo en < 32 semanas


 Enzimas de glicogénesis maduran antes que las glicogenolíticas
 Hipotermia e hipoxia que aumenta el consumo o Hiperinsulinismo neonatal Alteración congénita de la
secreción de insulina, con hipoglicemia persistente
o RN hospitalizado por hipoglicemia cuya causa no sea evidente o no hayan elementos para hipoglicemia
transitoria, realizar PTGO en ayuno
 Si < 45 mg/dL Descartar alteración endocrina o metabólica de los CHOs como etiología o Hipoglicemia
post prandial Galactosemia (posterior al consumo de leche) .
• Clasificación o Según duración
▪ Hipoglicemia transitoria (días)
• Responde rápidamente a cargas inferiores de 12 mg/kg/min y requiriendo aporte por menos de
7 días
• Causa: limitación en el aporte endógeno (prematuros, PEG, asfixiados), HMD, hiperinsulinismo
o aumento de la extracción periférica (poliglobulia) ▪ Hipoglicemia transitoria (semanas)
• Se da en RN asfixiados y/o PEG
• Cuadro que persiste más allá de los 3 primeros días, incluso de la primera semana
• Cursan con niveles inadecuadamente altos de insulina, pero < 20 microU(ml.
• Puede tardar meses en resolverse
▪ Hipoglicemia persistente
• Requiere cargas mayores de 12 mg/kg/min o continua por > 7 días.
• Causas: Defectos continuos del metabolismo, hiperinsulinismo congénito.
o Según etiopatogenia
 Hipoglicemia por hiperinsulinismo neonatal: HMD, eritroblastosis fetal, uso prenatal de beta agonistas y
diuréticos tiacídicos, hiperinsulinismo congénito persistente (adenoma pancreático)
 Hipoglicemia por insuficiencia de hormonas contrarreguladoras: Insuficiencia suprarrenal,
hipotiroidismo, déficit de GH, panhipopituitarismo, déficit de glucagón.
 Hipoglicemia por falta de depósitos o alteración enzimática de la movilización: prematurez, RCIU, ayuno
prolongado, consumo de OH materno, enfermedades metabólicas de acidos grasos, CHOs y AAs.
 Aumento de la extracción periférica o uso celular: Poliglobulia, cardiopatía congénita,
• Tratamiento o Hipoglicemia transicional precoz asintomática
 Alimentar VO y control HGT seriado
 De persistir hipoglicemia Glucosa 4 - 6 mg/kg/min. o Hipoglicemia sintomática o severa (< 25
mg/dL)
 Bolo de 200 mg/kg (2 ml/kg de SG al 10%) y continuar con infusión continua de 4 – 8 mg/kg/min
 Controlar glicemia 30 – 60 minutos
 Estabilizado Disminuir la carga en 1 – 2 mg/kg/min cada 12 horas + control de la alimentación
progresiva
o Hipoglicemia prolongada
▪ Hidrocortisona 5 mg/kg/día IM cada 12 horas

2
3
Temas clave de Pediatría y Cirugía Infantil

o Hiperinsulinismo
▪ Carga de glucosa de 15 – 20 mg/kg/min vía central (por la osmolaridad de la mezcla e infusión continua).
• Prevención o Control adecuado de la diabetes materna y carga de glucosa
periparto o Control seriado en niños de riesgo y alimentación precoz.
o Infusión continua de glucosa en RNPT menores de 32 semanas o en los RN en que no es posible alimentar.
o Evitar la suspensión brusca de infusiones de glucosa EV
• Pronóstico o Bueno. La hipoglicemia transitoria asintomática no deja secuelas.
o El pronóstico se ensombrece en presencia de convulsiones, lesiones irreversibles del SNC, daño característcio de atrofia cortical y de sustancia blanca
subyacente.

Hiperglicemia
 Definición o Corresponde a los niveles de glucosa > 125 mg/dL en sangre
 Síntomatología (intolerancia a la glucosa) o Aumento de la osmolaridad plasmática, glucosuria, diuresis
osmótica y deshidratación hiperosmolar o RNEBPN asocia mayor riesgo de muerte, HIC grado III y IV y más días de
hospitalización
 Fisiopatología o Sobreproducción y subutilización de glucosa
 Insuficiente secreción de insulina
 Aumento de las hormonas contrarreguladoras en estrés
 Falta de inhibición de la producción endógena de hormonas contrarreguladora por resistencia de
hepatocito a insulina
 Factores de riesgo o Principal FR RNEBPN
 Inadecuada secreción de insulina, resistencia periférica a ésta y ausencia de inhibición de la producción
endógena con la infusión exógena
o Cargas superiores a 8 mg/kg/min o aumentos de la carga de glucosa mayores de 3 a 5 mg/kg/min o Cirugías Estrés quirúrgico o séptico
genera aumento de hormonas antagonistas de insulina o Uso de hipoglicemiantes, corticoides, dopamina, cafeína, teofilina y fenitoína.
 Diabetes neonatal o Forma de presentación muy poco frecuente o Hiperglicemia que requiere insulina
por más de dos semanas o Característicamente RN PEG con deshidratación, poliuria, marcada glucosuria, falta de
crecimiento y cetonuria.
o Puede ser un cuadro transitorio, persistir o recurrir posteriormente o Tratamiento con insulina
monitorizado para evitar hipoglicemia
 Tratamiento o Manejo inicial
 Disminución de la carga de glucosa en 2 mg/kg/min cada 4 – 6 horas
 Si se mantienen glicemias > 250 mg/dL, usar insulina o RNEBPN Insulina si se requieren cargas de glucosa
< 6 mg/kg/min para mantener glicemias < 200 mg/dL o Insulina
 Solución continua de insulina en albúmina (1 U de insulina por 20 g de glucosa – 1 U por 4 g) ▪ Dosis:
0.01 a 0.1 U/kg/hora
 Control estricto y suspender insulina antes de glicemia < 180 mg/dL
 Prevención o Uso de cargas de glucosa adecuadas o En RNPT extremos, la carga inicial 4 – 6 mg/kg/min y
aumentos de no más de 2 mg/kg/min o Inicio precoz de alimentación enteral Estimula la producción de insulina
y disminuye el riesgo de hiperglicemia

2
3
Temas clave de Pediatría y Cirugía Infantil

2
3
Temas clave de Pediatría y Cirugía Infantil

Trastornos del calcio, fosforo y magnesio


• Introducción o El 45% del calcio sérico está unido a proteína o EL nivel de Ca está
determinado por la PTH, la calcitonina (CT), la vitamina D y los niveles de P, Mg, pH y
albúmina en sangre
o El calcio disminuye con acidosis metabólica o respiratoria aguda o El magnesio varía según pH, albúmina, sales
de fosfato, bicarbonato y calcio o Reguladores de la mineralización son PTH, Vitamina D, CT y minerales
• Funciones de regulación del metabolismo del Ca y P o PTH “Saca calcio del
hueso”
 Aumenta niveles de calcio Aumenta la reabsorción renal y la resorción ósea
 Aumenta niveles de vitamina D
 Disminuye niveles de fósforo Disminuye la reabsorción renal o Vitamina D “Mete calcio al hueso”
 Aumenta niveles de calcio
 Aumenta niveles de fósforo
 Disminuye niveles de PTH
 Aumenta el remodelamiento óseo o Calcitonina “Deja de sacar calcio del hueso”
 Disminuye los niveles de Ca y P Disminuye su reabsorción renal o GH “Aporta calcio y fósforo al hueso”
 Aumenta niveles de calcio
 Aumenta niveles de fósforo

Hipocalcemia
• Definición o Niveles séricos de Ca total > 7 mg/dL o Niveles séricos de Ca iónico < 3.5 mg/dL
• Síntomas o Aumenta la excitabilidad neuromuscular irritabilidad, temblores, hipertonía,
hiperexcitabilidad, osteotendíea, prolongación del QT y convulsiones
• Fisiopatología o Puede ser precoz o tardía
▪ Precoz del RNPT
• Disminución mantenida de los niveles de Ca sérico en los primeros días
• Asintomática
• Hay respuesta hormonal adecuada pero no hay receptores hormonales para elevar
rápidamente la calcemia
• Calcio se sigue depositando en el hueso a pesar de la interrupción del aporte activo placentrario
• Factores de riesgo: diabetes materna, SHE, hipomagnasemia, uso de furosemida, alcalosis
respiratoria o metabólica, asfixia neonatal, exanguinotransfusión. ▪ Tardía Tanto del RNPT como RNT
• Aporte inadecuado de Ca y P
• Alto aporte de P lleva a hiperfosfemia con disminución de PTH y vitamina D + aumento de CT,
lo que lleva a hipocalcemia
• Otras causas

2
4
Temas clave de Pediatría y Cirugía Infantil

o Hipomagnasemia
o Hipoparatiroidismo transitorio idiopático
o Hipoparatiroidismo congénito o
Hiperparatiroidismo materno o
Anticonvulsivante materno o Patología renal o
hepática o Malabsorción intestinal o Uso
prolongado de diuréticos de asa o
Exanguinotransfusión

• Estudio diagnóstico
o Si es refractaria, prolongada o recurrente Medir hormonas regulatorias
• Tratamiento o RNPT sano, asintomático No requiere tratamiento o RN sintomático o enfermo Calcio EV 45 mg/kg/día hasta
normalización o Convulsiones o tetania Calcio 10 – 18 mg/kg de EV LENTO (la infusión rápida de Ca EV puede provocar bradicardia y
arritmias.

Hipercalcemia
• Definición o Niveles séricos de Ca > 11 mg/dL
• Sintomatología o Neurológico: Hipotonía y
letargia o Digestivo: Mala tolerancia alimentaria, vómitos,
constipación o Nefrourológico: Poliuria, deshidratación,
HTA, nefrocalcinosis o Cardiovascular: Bradicardia
• Fisiopatología Excluir aporte excesivo deCa o
Hiperparatiroidismo neonatal debido a hiperplasia de
células paratiroideas
 Secundario a hipoparatiroidismo materno o Necrosis de grasa subcutánea
 RN asfixiados, hipotérmicos o traumatismos del parto puede presentarse
(semanas después del golpe)
o Hipercalcemia infantil idiopática o Hipercalcemia
por déficit de P o Otras causas
 Uso de diuréticos tiazidicos e IRA
 Intoxicación por vitamina D
• Tratamiento o Corregir hipofosfemia
▪ Fosfato VO o EV o SI toma LM de RNPT administrar Ca o Aporte de P por riesgo de
calcificaciones de tejidos no óseos o Disminuir absorción intestinal de Ca
o Aumentar la excreción urinaria con diuréticos de
asa (furosemida) o Disminuir la liberación ósea con CR 5
– 8/kg cada 12 horas o Diálisis peritoneal o hemodiálisis
si IRA.

2
4
Temas clave de Pediatría y Cirugía Infantil

Hipofosfemia
• Definición o RNT: Niveles
séricos < 4,5 mg/dL o RNPT: 5 – 8 mg/dL
• Sintomatología o Hipotonía,
letargia, CEG
• Fisiopatología o Frecuente por
aporte insuficiente de P o Signo precoz
de la enfermedad metabólica ósea o
Otras causas
 Hiperparatiroidismo NN, transitorio o primario
 Tubulopatías perdedorasde fosfato
 Síndrome de realimentación hipofosfemia después de proceso hipercatabólico •
Tratamiento Aporte de P VO o EV

Hiperfosfemia
• Definición o P > 9
mg/dL
• Sintomatología o
Solo si hay hipocalcemia
• Fisiopatología o
Inhibición de PTH y
vitamina D Baja de Ca
sérico Tetania NN y
convulsiones o Niño
asfixiado Hiperfosfemia
por lisis celular, acidosis y
glicogenólisis que lleva a la
hipocalcemia precoz.
o Leche de vaca no adaptada Aporte excesivo de P
• Tratamiento o
Reducir aporte de P
o Tratamiento de hipocalcemia si es sintomática

Hipomagnesemia
• Definición o Niveles de Mg < 1,6 mg/dL o Sintomatología con niveles
< 1,2 mg/dL
• Sintomatología o Hiperexcitabilidad
o Se asocia a hipocalcemia sintomática que no responde a aporte de Ca hipomagnasemia
• Fisiopatología
o Inhibición de la secreción y acción de PTH
 Niño con NPT sin Mg
 HMD

2
4
Temas clave de Pediatría y Cirugía Infantil

 SHE
 RCIU
• Tratamiento o Sulfato de Mg al 50% 0.1 a 0.2 ml/kg EV cada 12 horas
hasta normalizar o Efecto de Mg sobre calcemia es diferente según
niveles de Mg
 Bajos Calcemia sube
 Normales Calcemia baja.

Hipermagnesemia
• Definición o Niveles > 2,8 mg/dL
• Sintomatología o Hipotonía, letargia o Depresión
respiratoria, apnea o Íleo intestinal o Hipotensión
arterial
• Fisiopatología o El uso de sulfato de Mg en la madre es
la principal causa o Uso de sulfato de Mg en el manejo
de la hipertensión pulmonar persistente
• Tratamiento o Se espera excreción renal, usar
diuréticos de ser necesario.

2
4
Temas clave de Pediatría y Cirugía Infantil

Sepsis neonatal y meningitis


• Introducción o Las infecciones sistémicas y locales son frecuentes en el período NN.
o Pueden adquirirse in útero, vía transplacentaria o trancervical, produciendo amnionitis, infecciones del cordon,
neumonía congénita y sepsis. o Aunque la incidencia de sepsis en RNT y RNPT tardío es baja, el potencial de
sucesos adversos, incluyendo la muerte, es importante.
• Definición o Sepsis neonatal: Es el síndrome clínicos en RN (<28 días) manifestado por signos sistémicos de infección y
aislamiento de patógeno bacteriano desde el torrente sanguíneo
• Clasificación De acuerdo a la edad de presentación
 Sepsis neonatal precoz: Antes de 7 días de edad (algunas literaturas hablan de 72 horas)
 Sepsis neonatal tardía: Después de 7 días de edad con un inicio entre las 72 horas a los 7 días.
o Niños con SNP presentan síntomas durante la hospitalización de nacimiento o Niños con SNT se presentan en
el domicilio o en urgencias a menos que una comorbilidad haya prolongado la hospitalización de nacimiento
• Epidemiología o La incidencia total es de casi 1/1000 RNV o Los índices de infección se elevan a medida que disminuye la
edad gestacional (afecta a RNPT mayormente)
o La incidencia de SNP hay disminuido debido a la reducción del SGB por la profilaxis antibiótica intraparto
o La incidencia estimada en general de SN es de 1 – 2 /1000 RNV o Es mayor en RNPT que en RNT
• Patogénesis
▪ SNP
• Se debe principalmente a transmisión vertical por contaminación ascendente del líquido amniótico o durante el parto vaginal
con bacterias del TGU materno.
• La corioamnionitis es un factor de riesgo, así como la colonización por SGB
▪ SNT
• Vía de transmisión o Vertical: Resulta en colonización inicial que evoluciona a infección posterior o Horizontal: Directamente
por contacto con fuentes ambientales
o Factores metabólicos (hipoxia, acidosis, hipotermia, desordenes metabólicos congénitos) contribuyen al riesgo y
severdad de la sepsis
o Factores de riesgo maternos
 Corioamnionitis: Refleja infección intrauterina
 Tº intraparto > 38 ºC
 Parto en < 37 semanas (RNPT)
 Colonización materna por SGB
 Ruptura prematura de membranas (> 18 horas)
• Clínica o Generalidades
 Va desde síntomas sutiles a shock séptico.
 Los signos y síntomas son poco específicos, se incluyen
• Síntomas infecciosos: Inestabilidad térmica, irritabilidad, letargia, rechazo alimentario
• Síntomas respiratorios: Taquipnea, hipoxia
• Síntomas CV: taquicardia, hipoperfusión e hipotensión

2
4
Temas clave de Pediatría y Cirugía Infantil

▪ Es importante identificar factores de riesgo y tener un alto índice de sospecha cuando un niño se desvía de su patrón habitual de
alimentación
o Síntomas y signos más comunes ▪ Sufrimiento fetal:
• Taquicardia fetal intraparto por infección intraamniótica
• Fluido teñido de meconio dobla el riesgo de sepsis
• Apgar < 6 a los 5’ ▪ Inestabilidad térmica
• Puede elevarse, disminuir o estar normal
• RNT generalmente están febriles; RNPT, hipotérmicos. ▪ Síntomas respiratorios y CV
frecuentes
• Distrés respiratorio 85% SNP
• Apnea o Menos común y más probable en RNPT o Clásico de la sépsis tardía por SGB
• Hipertensión pulmonar persistente SNP
• Taquicardia y bradicardia
• Hipoperfusión e hipotensión Más tardíos pero más sensibles
▪ Síntomas neurológicos: Letargia, tono disminuido, rechazo alimentario, irritabilidad y convulsiones (menos comunes, pero
aumentan la probabilidad de infección) ▪ Otros: Ictericia y hepatomegalia (35%)
• Evaluación y manejo inicial (ATB empírica) o Generalidades
 Se requiere una evaluación pronta y el inicio de terapia ATB
 Debido a que los síntomas son poco específicos, se deben realizar exámenes de
laboratorio a cualquier niño con factores de riesgo o signos y síntomas incluidos en
sepsis. o SNP
 Anamnesis perinatal: Buscar factores de riesgo y uso de ATB profiláctico para SGB ▪
Exámenes de laboratorio como se listan abajo. ▪ Evaluar
• En niños sintomáticos Evaluación diagnóstica completa (Permite el seguimiento de la
terapia) o Hemocultivo, PL, hemograma completo, radiografía de tórax y cultivos
endotraqueales (si está intubado)
• En niños de buen aspecto Observación por mínimo 48 horas
o SNT
▪ Evaluación diagnóstica completa + Urocultivo y cultivo de otros potenciales focos (traqueal, secreción ocular, drenaje, pústulas) ▪
Iniciar ATB empírico
• Indicaciones de tratamiento ATB empírico o Aspecto tóxico o Síntomas activos
o Leucocitosis en LCR o Sospecha o confirmación de corioamnionitis
• Debe incluir agentes contra SGB y otros o Ampicilina + gentamicina se prefiere antes que ampicilina + cefotaxima o Agregar
cefalosporina de 3ra si se sospecha meningitis
o Laboratorio: estudio de infección
• Hemocultivo: Establece el diagnóstico definitivo
• PL: o Debe realizarse en todo neonato bajo evaluación por sepsis debido a que los signos de
meningitis pueden estar ocultos.
o Puede diferirse en compromiso CV o pulmonar hasta la estabilización o Se debe
enviar la muestra a Gram, cultivo y citoquímico o Recomendación de PL de la AAP

2
4
Temas clave de Pediatría y Cirugía Infantil

 Hemocultivo positivo
 Hallazgos altamente sugestivos de sepsis
 Laboratorio altamente sugestivo de sepsis
 Deterioro clínico durante la terapia ATB
• Urocultivo: Para niños > 1 semana, no de rutina para < 6 días debido a la alta posibilidad de positividad
por reflejo de la bacteremia.
• Hemograma
o SNP: Todos los pacientes o SNT: Apoya el diagnóstico o RAN anormal (aumentado o
disminuido) se asocia a sepsis, sin embargo, es más útil para identificar quienes NO
tienen sepsis.
• PCR: Ayuda a guiar la duración del ATB
• PCT: Aumenta especificidad de PCR en la detección de infecciones bacterianas
• Diagnóstico: Se establece únicamente con un hemocultivo positivo o Hemocultivo positivo: Es
el gold estándar que confirma el diagnóstico
• Diagnósticos diferenciales o Infecciones no bacterianas: Tomar cultivos antes de ATB empírico.
• Tratamiento o Terapia de soporte
 Monitor CP en sintomáticos ya que el curso clínico se deteriora rápidamente
 Mantener adecuada oxigenación y perfusión
 Prevención de hipoglicemia y acidosis metabólica
 Mantener un estado HE adecuado o Terapia antibiótica

A quien tratar Decidir en base a factores de riesgo, evaluación clínica y test de
laboratorio. • Indicaciones o Aspecto tóxico o Síntomas activos o Leucocitosis en LCR
o Sospecha o confirmación de corioamnionitis o Algun cultivo positivo
• ATB empírico inicial Basado en la edad, patógenos probables, foco y susceptibilidad al ATB o
SNP
▪ Ampicilina 150 mg/kg/dosis EV cada 12 horas + Gentamicina
4mg/kg/día (tomar función renal por genta)
• Son sinérgicos, y debido a la emergencia de resistencia a cefalosporinas, se prefiere
gentamicina antes que cefotaxima.
▪ Agregar una cefalosporina de tercera a ampi + genta en sospecha de meningitis y en neonatos críticos con factores de riesgo de resistencia
a ampicilina (RPM o tratamiento previo materno con ampicilina)
o SNT Depende de si proviene de la comunidad o del intrahospitalario ▪ De la comunidad: Menor riesgo de multirresistencia
• Ampicilina 75 mg/kg/dosis EV cada 6 horas + Gentamicina 4 mg/kg/día.
• Ampicilina + Cefotaxima
▪ Intrahospitalarios: Se reemplaza ampicilina por vancomicina (mayor riesgo de multirresistencia)
o Circunstancias especiales (en chile la resistencia es baja, por lo que no es tan criterioso usar vancomicina)
 En SNT agregar cefotaxima al régimen si la PL sugiere meningitis (leucocitosis de
LCR) Espectro amplio incluyendo enterobacterias gran negativas y neumococo.

2
4
Temas clave de Pediatría y Cirugía Infantil

 Si el foco es cutáneo, articular u óseo, considerar vancomicina en reemplazo de


ampicilina (alta probabilidad de S. aureus como agente etiológico)
 Si el foco es catéter intravascular debe iniciarse vancomicina y gentamicina para
cubrir SGB, S. aureus y bacterias gramnegativas
 Si el foco es del tracto gastrointestinal, el uso de clindamicina o metronidazol está
indicado
• ATB tras hemocultivo positivo Administrar según antibiograma por 10 días.
• Pronóstico o La mortalidad es del 2 – 4% y depende mucho de la EG del infante (a menor EG
mayor mortalidad) y del patógeno (E.coli es más mortal que SGB).

Infecciones congénitas. Síndrome de TORCH


• Generalidades o Definición
 Infección congénita: Infección adquirida vía transplacentaria durante la gestación o Etiología
 Grupo bien delimitado de patógenos: Toxoplasma, Rubéola, CMV, VHS, Virus varicela zoster, sífilis,
parvovirus, VIH, hepatitis B, Gonococo, Chlamydia y Mycobacterium tuberculosis
o Clínica
 Las manifestaciones son similares entre los distintos agentes CIU retrasado,
anasarca, anemia, trombocitopenia, ictericia, hepatoesplenomegalia,
coriorretinitis y malformaciones congénitas
o Diagnóstico
 La sospecha debe incluir aislar el MO
• Cultivo Rubéola, CMV, VHS, gonococo y M. tuberculosis
• Identificar el antígeno Hepatitis B y Chlamydia
• Identificar el genoma (RPC o IgM/IgG específicos) Toxoplasma, sífilis, parvovirus, VIH, Borrelia.

Toxoplasmosis
• Definición: Infección por Toxoplasma gondii
• Transmisión o Vertical transplacentaria después de primoinfección materna aguda
 30 – 40% de los casos
 Riesgo de infección aumenta a lo largo de la gestación 90% cerca del término o Rara vez en
casos de reactivación (embarazadas inmunodeprimidas)

2
4
Temas clave de Pediatría y Cirugía Infantil

• Clínica o La gravedad del cuadro varía de manera inversa con la EG a la que ocurre lainfección o La mayor parte son
infecciones subclínicas sin infección evidente al nacimiento
 Se puede encontrar evidencia de anormalidades en el examen oftalmológico y del SNC o Signos y síntomas
clásicos
 Hidrocefalia
 Coriorretinitis
 Calcificaciones intracerebrales o Neonatos:
 PEG, ictericia precoz, hepatoesplenomegalia, exantema maculopapuloso generalizado.
 Convulsiones frecuentes
 Rx de cráneo Calcificaciones corticales difusas (diferentes a periventriculares del CMV)
• Mayor riesgo de complicaciones neurológicas
• Diagnóstico o PCR
o Serología
 IgG específico: Máximo a los 2 meses, luego persisten elevados
 IgM Lactantes o Estudio clínico
 Evaluación oftalmológica, auditiva y neurológica TAC y PL
• Tratamiento
o Derivar para tratamiento
 Inicial Primetamina + ácido fólico + Sulfadiazina por 1 año

Rubéola
• Definición o Infección por virus de la rubéola, rara actualmente debido a la vacuna o Tiene gran
morbimortalidad si se adquiere intrauterino durante la gestación precoz ▪ < 4 semanas
• 85% defectos congénitos
• 40% aborto o mortinato
 13 – 16 semanas
• 35% malformaciones
 > 4 meses No causa enfermedad
• Transmisión Vertical transplacentaria tras primoinfección materna
durante el embarazo.
• Clínica o Cuadros oftalmológicos: cataratas, retinopatía, galucoma o
Cuadro cardíaco: CIA, estenosis de la arteria pulmonar Síndrome de
rubéola congénita o Cuadro auditivo: hipoacusia multisensorial o Cuadro
neurológico: Alteraciones de la conducta, meningoencefalitis, retraso
mental, microcefalia o Cuadro constitutivo: hepatoesplenomegalia,
ictericia precoz, trombocitopenia, osteopatías, púrpura
trombocitopénico
• Diagnóstico o Serología
 Detección de IgM específico Infección reciente

2
4
Temas clave de Pediatría y Cirugía Infantil

 Medición períodica y seriada de IgG Confirmación diagnóstica o Aislamiento viral


en sangre, orina, LCR y frotis faríngeo
• Tratamiento o Aislamiento
▪ Evitar contacto con mujeres embarazadas o No hay tratamiento específico
o Prevención: Vacuna antirubéola de virus vivo atenuado

Citomegalovirus
• Definición o Es la infección congénita más frecuente o Es la principal causa de hipoacusia neurosensorial, retraso mental,
enfermedad retiniana y parálisis cerebral
• Epidemiología o Infección en el 1% de los nacimientos
• Transmisión Riesgo de transmisión vertical depende de la EG durante la infección materna: cuando más precoz, más
sintomático será el niño o Primoinfección durante el embarazo
 Transmisión vertical en 35% de los casos
 Secuelas en 25-40% de los casos o Infección previa al embarazo Transmisión vertical 5 – 8% de los casos
▪ Reactivación: 10 – 20%

2
4
Temas clave de Pediatría y Cirugía Infantil

Infección crónica
 Reinfección o Infección adquirida durante el parto o por la leche materna no se asocia a
enfermedad neonatal o secuelas del SNC
• Clínica o Asintomáticos 90% al nacer o Sintomáticos y PEG 10%
 Microcefalia
 Trombocitopenia
 Hepatoesplenomegalia
 Hepatitis
 Calcificaciones intracraneales
 Corioretinitis
 Anomalías auditivas o Radiografía de cráneo calcificaciones periventriculares o Secuelas
 Hipoacusia
 RDSM
• Diagnóstico detección de CMV en las primeras 3 semanas de vida indica infección congénita o Aislamiento del
virus en orina o saliva o Cultivo viral o PCR de muestra de orina o Serología: IgM
• Tratamiento Ganciclovir
• Pronostico Mortalidad 10 – 15%

VHS
• Definición:
o Infección por VHS1 o VHS -2 (85%) o Son cuadros graves que pueden llevar a la muerte si se retrasa
el tratamiento
• Epidemiología o Incidencia 1 cada 3000 – 5000 RNV o Mayor riesgo de parto prematuro
• Transmisión
 Vía transplacentaria 5%
 Vía ascendetne: Inmediatamente antes del parto o durante el parto (por paso a través del canal
de parto) 85%
 Post parto (lesiones no genitales) 10% o Primoinfección materna
 Riesgo de infección 35 – 50%
 Mortalidad 70% o Reactivación
 Riesgo de infección 1 – 3 %
• Clínica o La mayor parte son asintomáticos durante el nacimiento, mostrando síntomas de infección 5 – 10 días de vida
Enfermedad diseminada a órganos y sistemas (diagnóstico diferencial de sepsis neonatal)
 Infección del SNC Encefalitis herpética
 Infección cutánea/ojo/boca Queratoconjuntivitis herpética y estomatitis herpética
• Lesiones cutáneas son más tardías
o Sospechar en neonato con fiebre, irritabilidad, hallazgos anormales en el LCR y convulsiones
• Diagnóstico o Cultivo de líquido de vesícula
▪ Si es positivo en muestras posteriores a 48 horas de vida indica infección intraparto o PCR de muestra de sangre, orina y LCR.
• Tratamiento o Medicamento: Aciclovir EV 14 – 21 días. o Prevención: Cesárea si hay lesiones vaginales
• Pronóstico Bueno en lactantes con enfermedad cutánea/ojo/boca.

Sífilis congénita
• Definición o Infección por Treponema pallidum o Cuanto mayor
sea el tiempo entre la infección de la madre y el embarazo, menor
posibilidad de transmisión.
o Debe realizarse una pesquisa activa en la embarazada
 Primer control
 28 semanas
 Entre las 28 y 34 semanas
 En el parto

25
0
Temas clave de Pediatría y Cirugía Infantil

• Transmisión En cualquier etapa del embarazo o Vía
transplacentaria: La más frecuente o Durante el parto: Por
contacto con chancro primario
• Clínica o Cuadros clínicos
 Sífilis congénita precoz (< 2 años) 60% asintomáticos al nacer
 Neurosífilis
 Sífilis congénita tardía (> 2 años): Afección de huesos, articulaciones, dientes, ojos y piel.
o Síntomas
 Hepatoesplenomegalia
 Obstrucción nasal
 Adenopatías
 Lesiones mucocutáneas
 Osteocondritis
 Exantema
 Anemia hemolítica y trombocitopenia
 Triada de Hutchinson: queratitis intersticial, sordera por afectación del VIII PC, dientes de Hutchinson.
• Diagnóstico o Prueba no treponémica
▪ VDRL o Pruebas treponémicas FTA-Abs – MHA
Confirman el diagnóstico.
• Tratamiento
o PNC sódica 100000 U/kg/día por 14 días

Enfermedad de Chagas
• Definición: Enfermedad producto de la infección por Tripanosoma cruzi, transmitido por el
Triatoma infestans (vinchuca)
• Epidemiología o La transmisión vertical es el principal mecanismo de transmisión en Chile
o Riesgo de transmisión de 5 – 8% en cada embarazo
• Clínica o 90% serán asintomáticos
• Diagnóstico o PCR
o Serología
 IgM Tamizaje
 IgG Confirmación
• Tratamiento Nifurtimox VO por 60 días

Hepatitis B
• Definición: Infección por virus hepatitis B, capaz de generar hepatitis crónica y
hepatocarcinoma
• Epidemiología o De baja prevalencia en chile 0.15%
• Transmisión o Pre natal
 Madre que se contagia con hepatitis B en el tercer trimestre
 Madre portadora crónica de VHB
• Diagnóstico o Serología: HBsAg
• Tratamiento No existe
• Prevención Vacuna antihepatitis B

Parvovirus B 19
• Definición: Infección por virus Parvovirus B 19, virus DNA que invade precursores eritrocitarios, destruyendo las células en
división activa
• Transmisión Transmisión vertical (33%) solo en primoinfección. o Primoinfección de embarazada ocurre en el 3 – 20% de
los embarazos, de los cuales el 33% genera transmisión vertical
o Solo el 10% de los fetos se infectan.

25
1
Temas clave de Pediatría y Cirugía Infantil

• Clínica
 Aborto, mortinato
 Hepatitis, miocarditis
 Anemia, hidrops fetal o Crisis de anemia aplásica Falla cardíaca Ascitis, derrame, edema,
PHA.
• Diagnóstico o Serología

25
2
Temas clave de Pediatría y Cirugía Infantil
o PCR
• Tratamiento No tiene
VIH
• Definición: Infección por virus VIH que genera efecto citopático en linfocitos CD4
• Diagnóstico o Serología: IgG o PCR
o Seguimiento: Carga viral y conteo CD4
• Tratamiento Zidovudina (AZT)

25
3
Temas clave de Pediatría y Cirugía Infantil
Síndrome de Down
• Definición o Es la anomalía de números de cromosomas más frecuente o Aparece en 1 de cada
1000 RNV
• Epidemiología o La mayoría es por no disyunción del cromosoma 21 o 1 – 2% aparece
mosaicismo Dos poblaciones celulares, una con la trisomía y otra normal.
▪ Aunque se cree que su afectación es más leves, los hallazgos clínicos son variables o En Chile 1: 670 NV (ECLAM 1:405 NV)
o Fuerte relación a edad materna (> 35 años)
• Clínica o La mayoría se diagnostica en período neonatal o Peso y talla normales, pero son
hipotónicos o Características faciales típicas
 Braquicefalia
 Occipucio aplanado
 Hipoplasia del tercio medio facial
 Aplanamiento del puente nasal
 Hendiduras palpebrales
 Pliegue epicántico
 Macroglosia con protrusión lingual
 Manos anchas y cortas
 Pliegue palmar transverso
 Hipotonía grave problemas de alimentación y disminución de la actividad o Cardiopatías
congénitas (50%)
 Defectos del Tabique aurículo ventricular
 CIV o CIA
 Valvulopatías o Anomalías gastrointestinales (10%)
 Atresia duodenal
 Páncreas anular
 Ano imperforado o Hipotiroidismo congénito ( 4 – 18%) o Policitemia del RN
 Hematocrito > 70% o Reacciones leucemoides con leucocitosis marcada Transitorias o Mayor
riesgo de leucemia (10 – 20 veces más)
 < 2 años Megacarioblástico agudo
 > 3 años Leucemia linfoblástica aguda o Mayor susceptibilidad a la infección o Cataratas
o Inestabilidad atlanto axoídea
• Manejo
o Siempre realizar cariograma o Uso de tablas específicas o Educación
asistida, pero integrada o Prevención de patologías asociadas o Prevención
del daño oxidativo

25
4
Temas clave de Pediatría y Cirugía Infantil
Consultas del RN en urgencias
• Orina naranja o rojiza Eliminación de cristales de urato
• Lesiones traumáticas de nervios periféricos o Falta de movilidad de boca o cara o de extremidades superiores o
Requiere evaluación neurológica
• Caída del cordón umbilical o Ocurre entre los 7 – 15 días o Si no ha caído, investigar uraco persistente con ecografía
o Mal olor o secreción
 SI hay eritema, induración Onfalitis. Debe hospitalizarse
 Granuloma umbilical tocaciones con nitrato de plata
• Llanto y cólicos del lactante o Definición: llanto de al menos 3 horas, tres veces a la semana, 1 de cada 3 semanas o
Situación normal
• Conjuntivitis neonatal o RN en partos vaginales o Tomar cultivo de secreción ocular, realizar aseo con SF y ATB tópicos
por 7 días (ungüento CAF) o Recordar causas complejas gonococo y chlamydia
• Piodermitis Estafilococo o Impétigo bulloso
 Lesiones cutáneas con eritema, formación de costras y secreción
 Realizar cultivo bacteriano, curación y ATB tópicos (mupirocina o cloxacilina) o Síndrome de piel
escaldada
 Por la toxina bacteriana
 Requiere Hospitalización y terapia antiestafilocócica EV
• Onfaltitis o Eritema periumbilical con o sin secreción o Hospitalizar pues da compromiso sistémico y requiere
tratamiento EV contra gran positivo y negativo
• Cianosis o Descartar sepsis si asocia CEG o Cardiopatía congénita o Síndrome de Down
• Apnea
o Definición: crisis de cese de respiración por mas de 15 segundos, asociado a bradicardia, palidez y/o cianosis
 Hospitalizar para monitoreo de FC, FR y Saturación
 Chequeo básico de exámenes: Hemograma, PCR, ELP, calcemia, glicemia, gases ▪
Interconsulta a neurólogo, cardiólogo.
• ALTE: Episodio apneico que impresiona amenaza a la vida y que requirió reanimación o Hospitalizar y realizar manejo
similar a apneas
• Enfermedades metabólicas o Errores innatos del metabolismo: vómitos, letargia, mayor compromiso neurológico,
hepatomegalia, hipoglicemia, ictericia.
• Enfermedades endocrinas o La más común Hipotiroidismo congñenito
 Síndrome ictérico
 Hipoalimentación con letargia o Hiperplasia suprarrenal
 Hombre, después de la 2da semana
 Vómitos profusos, deshidratado, mal incremento ponderal, hiponatremia e hiperkalemia, shock similar a
sepsis.
 Requiere manejo intensivo
• O2 al 100%
• Bolos de SF
• Corticoides
• DVA

25
5
Temas clave de Pediatría y Cirugía Infantil

PEDIATRÍA AMBULATORIA

Control de salud infantil (niño sano) II

Controles de salud infantil por edad


• Controles por edad

• Control del RN del primer mes Médico o Período


 Ideal 28 días y 1 mes 5 días
 Tardío 1 mes 6 días y 1 mes 20 días.
o Objetivos
▪ Pesquisa de anomalías, malformaciones y problemas del neurodesarrollo ▪ Otros
• Evaluación integral del DSM y crecimiento
• Educación sobre crecimiento y crianza
• Evaluar el bienestar familiar post bebé
• Fomentar LM hasta mínimo los 6 meses
o Instrumentos a aplicar
 Protocolo de evaluación neurosensorial
 Score de riesgo de morir por neumonía IRA.
o Pauta

25
6
Temas clave de Pediatría y Cirugía Infantil
 Anamnesis
• Antecedentes perinatales o Antecedentes gineco-
obstétricos o Complicaciones del embarazo o
Complicaciones del parto
• Alimentación o Del niño Educar sobre preparación de LPF
o De la madre (Usa o no LPM) o Revisar el incremento
ponderal respecto al control de la diada
• Cuidados del niño o Posición y hábitos de sueño Educar
sobre posición y prevención de MS o Deposiciones
 Frecuencia y apariencia de deposiciones y orina
 Número de mudas y peso del pañal con cada muda o Baño Lugar,
calefacción, uso de productos tópicos, frecuencia. o Enfermedades del niño

• Antecedentes familiares o Patologías


 Displasia de cadera
 Patologías congénitas: CC, labio leporino, estrabismo, cáncer. o
Características de la vivienda Patio, construcción.
o Integrantes del hogar + mascotas o Tabaquismo en la familia o Medio de transporte Educación sobre prevención
de accidentes o Estado de la familia y de los miembros respecto al niño o Salud de la madre
 Examen físico: Buscar dirigidamente • Antropometría: Longitud, peso, CC.
• General: o Relación madre, padre y niño: Consuelo, cariño, contacto visual, empoderamiento
del rol materno, respeto, apoyo paterno.
o Piel: Ictericia, hemangiomas y dermatitis.
o Ganglios y signos de bi o pancitopenia.
• Oftalmológico o Rojo pupilar o Test de Hirshberg
• Bucodental: Descartar algorra, evaluar frenillo sublingual.
• Cardiopulmonar: Pulsos femorales.
• Abdomen: Hernias.
• Genitoanal: Dermatitis del pañal, genitales bien diferenciados, clítoris normotrófico, testículos
descendidos, descartar hipo o epispadia, evaluar hidrocele fisiológico.
• Ortopedia o Fractura de clavícula o Disrrafia espinal o Displasia de caderas: Signo de Ortolani y
Barlow
• Neurológico o Reflejos: Tono nucal, moro, succión.
o Movilidad: Control cefálico o Signos de parálisis
cerebral: Rotación interna de extremidades, extensión
de extremidades, asimetría.
• Auditivo: Alerta al sonido ▪ Diagnósticos
• Nutricional Eutrófico o no, buen o mal incremento ponderal
• DSM Normal o anormal
• Problemas de salud encontrados
• Problemas PS familiares
• Otros problemas
 Indicaciones
• LME libre demanda. Si usa FI, intentar disminuirla y aportar LM.
• Educación o Anticipar reacción a vacuna BCG (Brote posterior al mes de la vacuna) o
Educación sobre el posición y hábitos de sueño o Educación sobre cuidados de la piel
 Baño con agua o jabón hipoalergénico escaso
 Lavar ropa con jabón en barra blanco o detergente hipoalergénico ▪
No usar suavizantes. Usar ropa 100% algodón, sin etiquetas.
 No exponer directamente al sol ni usar bloqueador o Educación sobre salud
bucal: Limpiar post mama con gasa.
o Educación sobre prevención de accidentes: Silla del auto y posición al dormir (ALTE)

25
7
Temas clave de Pediatría y Cirugía Infantil
• Guía anticipatoria sobre la crisis hipertónica del ano a los 2 meses aprox. •
Suplementar vitamina D 400 UI (10 gotitas/día) • Control en 1 mes
más (CSI de los 2 meses).
o Banderas rojas
 Incremento ponderal < 20 gr/día Fomentar LME, educación en técnica de lactancia, educación sobre
abrigo adecuado al lactar, posición caballito y succión vigorosa (sonda al dedo) si hay succión débil
 Ausencia de LM Apoyar biopsicosocialmente a la madre, control en clínica de LM
 Problemas de lactancia Educación sobre la técnica, control en clínica de LM en 48 horas, sugerir grupos
de apoyo
 Ictericia Solicitar perfil hepático, urgencias si hay ictericia bajo las rodillas, control para evaluar
progresión y seguimiento si se hospitaliza
 Angiomas de la línea media Derivar a dermatólogo o neurólogo (posibles malformaciones)
 Evaluación neurosensorial de 1 a 3 puntos Variación fisiológica normal, control a los 3 meses.
 Evaluación neurosensorial > 3 puntos, macro o microcefalia Derivar a neurólogo infantil
 Signos de parálisis cerebral Derivar a neurólogo
 Sospecha de hipoacusia o FR familiares o personales Derivar a ORL
 Rojo pupilar alterado o leucocoria IC a oftalmólogo urgente, ingreso a AUGE.
 Sospecha de CC IC a cardiólogo o pediatra, ingreso a AUGE.
 Hemangiomas Derivar a dermatólogo si es de gran tamaño, crece rápido, es periorificial, está complicado
o son más de 5.
 Criptorquídia, hernia inguinal, hipospadia o epispadia Derivar a cirugía infantil ▪ Score de IRA
moderado o grave
• Moderado derivar a talleres IRA, consejería antitabaco, intervención en factores de riesgo
modificables
• Grave Visita domiciliaria para ver factores de riesgo respiratorios, seguimiento telefónico.
 Estrabismo fijo IC a oftalmólogo, ingreso a AUGE
 Signo de Ortolani y Barlow positivo o FR de displasia Derivar a traumatólogo, ingreso AUGE
 Frenillo sublingual corto que dificulte LM Derivar a cirugía infantil.
 Dientes neonatales Derivar a odontólogo
 Algorra Consejería sobre buen aseo de boca
 Sospecha de FLP Derivar a pediatra, ingresar a AUGE
 Sospecha de disrafia espinal oculta Derivar urgente a neurocirujano
 Dificil consuelo Descartar patología médica, educar a los padres sobre técnicas de manejo del llanto
 Adenopatías > 2 cms, supraclavicular, sin síntomas de IRA, más de un grupo, signos de compromiso
hematológico, visceromegalia Estudio con exámenes y derivar a nivel secundario.
• Control del lactante a los 3 meses Médico o Período
 Ideal 2 mes 21 días – 3 meses 10 días
 Tardío 3 meses 11 días – 3 meses 20 días o
Objetivos: Revisión de resultados de Rx de pelvis ▪ Otros
• Evaluación integral del desarrollo y crecimiento
• Resolver dudas sobre salud y crianza
• Fomentar LME hasta los 6 meses
o Instrumentos Score de riesgo de morir por neumonía.
o Pauta
▪ Anamnesis
• Antecedentes perinatales o Antecedentes gineco-obstétricos o
Complicaciones del embarazo o Complicaciones del parto
• Alimentación o Del niño Educar sobre preparación de LPF o De la madre
(Usa o no LPM) o Revisar el incremento ponderal respecto al control
anterior
• Cuidados del niño o Posición y hábitos de sueño Educar sobre posición y
prevención de MS o Deposiciones
 Frecuencia y apariencia de deposiciones y orina

25
8
Temas clave de Pediatría y Cirugía Infantil
 Número de mudas y peso del pañal con cada muda o Baño
Lugar, calefacción, uso de productos tópicos, frecuencia.
o Enfermedades del niño
• Antecedentes familiares o Patologías
 Displasia de cadera
 Antecedentes en la familia de alteraciones auditivas y
visuales ▪ Patologías congénitas: CC, labio leporino, estrabismo, cáncer. o
Características de la vivienda Patio, construcción.
o Integrantes del hogar + mascotas o Tabaquismo en la familia o Medio de transporte Educación
sobre prevención de accidentes o Estado de la familia y de los miembros respecto al niño
• Antecedentes de la madre o Post natal
 Extensión por 84 días: Red de apoyo,
técnica de extracción de LM
 Fin de los 84 días: Red de apoyo ▪
Examen físico
• Antropometría
• General o Relación madre, padre, niño.
o Piel: Hemangiomas (perioirificiales), nevos, manchas café
con leche, dermatitis seborreica, dermatitis de contacto y del
pañal, evaluar reacción BCG. o Ganglios: Si es positivo, buscar
visceromegalias y palidez, petequias o equimosis.
• Oftalmológico o Rojo pupilar y Test de Hirshprung o Epífora o secreción pupilar o Fijación y
seguimiento con la mirada
• Cardiopulmonar: Descartar soplos, arritmias, cardiopatía cianótica.
• Abdomen: Visceromegalias y hernias inguinales, distensión abdominal
• Genitoanal: Genitales claramente diferenciados, dermatitis del pañal, examen genital (descenso
testicular y sinequias vulvares).
• Neurológico: Hipotonía, simetría en la movilización de extremidades, posición de los pulgares.
o Reflejo cóceo palpebral o Moro a ruidos fuertes o
Descartar dificultad en la succión
• Auditivo: Otoscopía.
• Ortopedia: Maniobras de Ortolani y Barlow
▪ Diagnósticos
• Nutricional Eutrofia, riesgo de desnutrición, desnutrición, sobrepeso, obesidad.
• DSM Normal, anormal
• Problemas de salud encontrados
• Problemas psicosociales familiares
• Otros problemas de salud
o Banderas rojas
 Incremento ponderal inadecuado Educación en LM, extracción y control en clínica de lactancia para
asegurar buen incremento ponderal.
 Eutróficos con 2 o + FR de malnutrición por exceso Taller de prevención de MPE ▪ MPE Educación
sobre señales de hambre, reforzar LME y AC desde los 6 meses.
 Riesgo de desnutrición (P/E -1 DS) Descartar síntomas depresivos de la madre y LM
 Desnutrición (P/E – 2 DS)
 Ausencia de LM o LM parcial Evaluar motivos, apoyar a la madre, control en clínica de LM.
 Problemas de lactancia Educación en técnica, control en clínica de LM en 48 horas, sugerir grupos de
apoyo.
 Algorra Consejería sobre buen aseo de boca
 Madre con síntomas depresivos
• > 10 Eventual ingreso a GES, fomentar descanso y actividad física, red de apoyo.
• 0 – 10 (síntomas suicidas) Evaluar riesgo vital (ideación, planificación, ejecución), atención médica de urgencia,
activar red de apoyo.

25
9
Temas clave de Pediatría y Cirugía Infantil
 Cuidadores con dificultades en el cuidado Fortalecer vínculo, presentar caso para plan de trabajo y visita
domiciliaria, activar red comunal chile crece contigo, derivar a nadie es perfecto, derivar a salud mental.
 Difícil consuelo Descartar causa médica con examen físico, educar sobre técnicas de manejo del llanto,
analizar estresores a nivel familiar y consejería, revisar resultados de escala de relación vincular alterada y
velar por el cumplimiento de la intervención.
 Señales de maltrato o abuso sexual Solicitar apoyo para diagnóstico, determinar riesgo vital o delito.
 Examen neurológico alterado Derivar a neurólogo infantil ▪ Score de IRA moderado o grave
• Moderado derivar a talleres IRA, consejería antitabaco, intervención en factores de riesgo
modificables
• Grave Visita domiciliaria para ver factores de riesgo respiratorios, seguimiento telefónico.
 Rx de pelvis alterada o signos positivos Derivar a traumatología para diagnóstico, ingresar a AUGE.
 Rojo pupilar alterado o leucocoria IC a oftalmólogo urgente, ingreso a AUGE.
 Estrabismo fijo IC a oftalmólogo, ingreso a AUGE
 Sospecha de CC IC a cardiólogo o pediatra, ingreso a AUGE.
 Hemangiomas Derivar a dermatólogo si es de gran tamaño, crece rápido, es periorificial, está
complicado, son más de 5.
 Criptorquídia, hernia inguinal, hipospadia o epispadia Derivar a cirugía infantil
 Constipación orgánica Ante deposiciones duras o caprinas, derivar a gastroenterólogo. ▪
Hipoacusia o FR familiares o personales Derivar a ORL

Desarrollo psicomotor
• Definiciones o Crecimiento: Incremento de la masa corporal, traducido en el peso, talla y CC o Maduración: Proceso
genéticamente determinado de organización progresiva de estructuras corporales
o Desarrollo: Incremento de las habilidades funcionales de un sujeto que obedece a su crecimiento, maduración e
interacción con el ambiente. Tiene gran estabilidad intraespecie y gran variabilidad interindividual
o Desarrollo psicomotor
 Destrezas o habilidades cognitivas, motoras, comunicativas y sociales
 Desde el nacimiento hasta los 5 años
 Proceso dinámico, resultando de la maduración del SNC que implica cambios de la función neuromuscular
• Características del DSM o Resulta en la adquisición de habilidades y respuestas (lenguaje, motricidad, pensamiento
y adaptación)
o Ocurre por la proliferación de dendritas y mielinización de los axones.
o Tiene una progresión céfalo-caudal en tronco y proximal a distal en extremidades o Existen rangos de normalidad
(recordar corregir edad en RNPT) o Evaluaciones objetivas clasifican a los pacientes en normales, de riesgo o
anormales

26
0
Temas clave de Pediatría y Cirugía Infantil

Alimentación infantil

Alimentación del menor de 2 años


• Introducción o La mayor parte de las preferencias y aversiones se establecen antes de los 2 años
o Es importante que el tamaño de las porciones sea adoptado, de acuerdo a las señales de
saciedad del niño, evitando sobreconsumo
o Las respuestas aprendidas incluyen preferencias por estímulos experimentados dentro del útero, como el sabor y
los olores de los alimentos consumidos por la madre y que se transmiten a través del líquido amniótico
o Las preferencias por lo dulce y la aversión a lo amargo son innatas; a los 4 meses surge la preferencia por lo sabores
salados
o Los sabores y olores de alimentos que consume la madre se transmiten a través de la LM y los lactantes aprenden a
preferir dichos sabores por lo que es importante tener una dieta variada. o Una forma eficaz de introducir un nuevo alimento
es “imitación o modelaje”. Si los niños ven a su padre, madre o hermanos disfrutar de un alimento, estarán más interesados
en probarlo.
o Es importante considerar el nivel de madurez del niño para adecuar la alimentación
 La pérdida del reflejo de extrusión ocurre a los 4 meses y permite la aceptación de alimentos no
líquidos
 Sentarse con apoyo cerca de los 4 meses permite el control de la cabeza y el cuello, necesario
para deglutir sólidos
 La aparición de movimientos verticales en la boca o “saboreo” se observa desde los 4 meses
 La masticación aparece a los 8 – 10 meses, lo que permite pasar de texturas blandas (puré) a más
sólidas

26
1
Temas clave de Pediatría y Cirugía Infantil
 Es normal que rechacen un nuevo alimento (neofobia). Los lactantes aceptarán alimentos
azucarados pero pueden rechazar amargos (algunas verduras verdes). La exposición repetida y contextos
sociales positivos contribuyen a moldear esta característica.
o La correcta posición al comer es con el niño sentado, tronco en línea media y cabeza sobre el tórax o La evolución en la AC se asocia a la erupción
dentaria
 Dientes primarios aparecen a los 6 meses, son 20 y terminan de aparecer a los 3 añs
 Dientes permanentes aparecen entre los 5 – 6 años de edad o A los 6 meses se puede sentar
e iniciar la masticación vertical lo que permite comenzar con la AC
• Alimentación del niño de 0 – 6 meses o Objetivo Mantener LME hasta los 6 meses o Definición
▪ LME: Leche del pecho de su madre o extraída o de un banco de leche humana y no otro tipo de líquidos ni sólidos, con excepción de
SRO, gotas o jarabes.
o Beneficios
 LM brinda beneficios al niño y a la madre
 Los primeros 6 meses cubre las necesidades energéticas y nutricionales.
 Los líquidos adicionales desplazan la leche materna y reducen su ingesta total
o LME
▪ Proceso
• El niño es capaz de mamar y tragar solamente líquidos por la presencia del reflejo de succión y
deglución y el de extrusión, que determina que el sólido sea expulsado
• El calostro es la leche de los primeros 2 – 3 días. Producido en pequeña cantidad
(40-50 ml/día) o La succión precoz y frecuente mantiene la glicemia adecuada, evita la deshidratación y la pérdida exagerada de peso, contribuye
a la coordinación succión-respiración-deglución.
o El calostro proporciona además protección inmunológica y facilita la reproducción de lactobacilo bífido en el lumen intestinal, promoviendo la
flora protectora
• Leche aumenta su producción entre los días 23 y 4 del parto, provocando que los pechos se
sientan llenos (“subida” o “Bajada” de la leche). En el tercer día un lactante toma 300-400 ml en
24 horas y al 5to día 500-800 ml
• La composición de la leche no es igual al principio y al final de la mamada, ni en los primeros
días o a los 6 meses.
o Los primeros días contiene más proteínas o Al final de la mamada tiene más calorías, grasa y vitaminas. ▪ Frecuencia y
duración
• 15-20 primeros días tiene 8 tomas en 24 horas (no más de 3 horas sin lactar)
• Signos de alimentación suficiente: tranquilo, peso adecuado, orinando.
• El niño regula el final de la mamada, soltando espontáneamente el pecho
• Para asegurar adecuada producción y flujo se requiere amamantar de día y de noche, a libre
demanda
• LMLD disminuye complicaciones (ej. Mastitis) y asegura producción futura o Es importante la
lactancia nocturna dado el alza de la prlactina nocturna
• La ingesta en 24 horas varía entre 440 ml – 1220 ml, con promedio de 800 ml los primeros 6
meses ▪ Posición y técnica
• Deben seguir la posición más comoda
• Recomendaciones para lactancia exitosa o Inicio precoz, dentro de la primera hora de vida. De
no estar presente la madre, iniciar extracción de calostro la primera hora post parto
o Amamantar en un lugar tranquilo o Mirar a los ojos al niño o Recordar que a
mayor succión, mayor producción o No establecer horario entre tomas o tiempo
de cada pecho
o Es normal que puedan pedir frecuentemente, lo que no es señal de hambre o
La mama debe estar bien alimentada, comer de todo y saludable, consumir
bastante agua, no OH, tabaco o drogas, ni exceso de té o café.
o En la noche, priorizar LMLD porque aumenta la producción de leche ▪ Señales
de hambre
• Tempranas o Despertar
o Agitación – apertura de boca
o Giros de cabeza – buscar el pecho con la cabeza
• Medias o Estiramiento o Incremento de agitación y movimientos físicos

26
2
Temas clave de Pediatría y Cirugía Infantil
o Chupar sus manos
• Tardías o Llanto o Agitación intensa o Enrojecimiento
▪ Indicaciones de amamantamiento correcto
• Posición de acople: nariz y mentón tocando la mama
• Deglución audible
• Actitud mamando tranquilamente, rítmico, relajado.
• Sueño tranquilo /1.5 – 3 horas entre mamadas
• Aumento de peso normal
• Promedio 6 pañales mojados al día
• Promedio 4 deposiciones dia, desde el día 5, disminución paulatina de la frecuencia. Al mes una
deposición y a los 2 meses, 1 cada 5 – 7 días.
o LMNE Fórmulas de inicio
 Se puede adelantar la AC en niños a los 5 meses para evitar introducir fórmulas ▪
Tipos de fórmulas
• Fórmulas de inicio o Aportan 67 Kcal/100 ml o Cubren los requerimientos
nutricionales de lactantes sanos, RNT, el primer año
• Fórmulas de continuación o A partir de los 6 meses o Sin evidencia científica
• LPF (PNAC) ▪ Preparación
• Fórmulas de inicio: Indicada en cada envase (en general, 13 – 14%
• LPF Diluir al 7.5% + maltodextrina 3% (sacarosa 3%) + aceite vegetal 2% (canola o
soya)
 Volumen y fraccionamiento
• Se inicia con 60 ml por vez los primeros días y escalar progresivamente hasta 200 ml
por vez a los cinco meses 150 cc/kg/día
• Fraccionamiento o Cada 3 horas (8 veces) hasta los 3 meses o Cada 4 horas (6 veces)
hasta los 6 meses
o La mamadera nocturna quitar en la medida que deje de despertarse en la
noche con hambre • Alimentación del niño de 6 meses - 1 año o Objetivo
 Incorporar la alimentación solida
 Debe ocurrir a los 6 meses para cubrir requerimientos nutricionales y estimular el DSM o
Definición
 AC incluye alimentos sólidos y semisólidos (papillas o purés) y también líquidos (agua)
 Se recomienda continuar LM o Fórmula
 La segunda papilla se inicia a los 6 meses o Preparación de fórmulas de continuación
 Preparar según indicación del envase (14 – 15%)
 LPF 7.5 % + cereal al 5% (no requiere azúcar)
 El uso de FC desde los 6 meses no es indispensable. La fórmula de inicio es útil todo el primer
año cuando hay que reemplazar la leche materna.
o Proceso
 La madurez morfo funcional se caracteriza por digestión adecuada de sólidos, control de la cabeza y
tronco, uso de la musculatura masticatoria, inicio de la erupción dentaria, incremento de las percepciones
sensoriales, extinción del reflejo de extrusión
 Puede ocurrir neofobia, esperable. Ofrecer en nuevas ocasiones y con insistencia suave hasta su
aceptación.
o Consistencia y cantidad
 Papilla o sopa-puré, sin grumos ni fibra que estimulen la extrusión.
 Cuando maduran los incisivos superiores (8-9 meses) dar puré más grueso y luego molido con tenedor,
cuando erupcionen los primeros molares (12 meses)
 A los 8 meses ofrecer 150 ml de comida (3/4 taza) + 100 ml de fruta (1/2 taza); y a los 9 meses 200 ml de
papilla (1 taza) y 100 ml de fruta (1/2 taza).
o Composición
 La papilla debe contener cereales, vegetales y carnes bajas en grasa, vacuno, pollo o pescado ▪
Agregar 2.5 – 3 ml de aceite vegatal crudo (canola o soya).

26
3
Temas clave de Pediatría y Cirugía Infantil
 Postre: Puré de fruta, crudas o cocidas, sin adición de azúcares ni miel (riesgo de botulismo)
 Puede consumir cualquier fruta
 Debe aportar misma densidad energética que la LM: 65 – 70 Kcal/100 gramos ▪ Las 2 papillas no
deben aportar más del 50%, el resto lo aporta la LM o fórmula.
 No agregar azúcar, sal ni edulcorantes en < 2 años
 Se recomiendan cereales a partir de los 6 meses (trigo, avena, centeno y cebada) incluso en antecedentes
familiares de enfermedad celíaca
 Agregar legumbres entre los 7 – 8 meses, una porción reemplaza al puré de verduras con carne; 2 veces
por semana
 Pescado entre los 6 – 7 meses, criterio válido también para niños con atopia o alergia alimentaria; 2 veces
por semana
• Preferir pescados grasos con alto contenido de DHA, que ayuda al DSM ▪ Huevo desde los 9 – 10 meses.
 Agregar agua 20 – 50 ml 2 - 3 veces al día, separándola de la leche. Si rechaza, mantener ofrecimiento, no
cambiar por jugos.
o Ofrecimiento
 Cada vez que incorpore un nuevo alimento, entregarlo uno por vez y en pequeñas cantidades.
 No mezclar postre con comida
 No soplar alimentos
 Respetar al niño si no quiere comer más o Horario
 Debe acomodarse a las costrumbres familiares y progresivamente incorporarse al horario familiar.
• Alimentación del niño de 1 año – 3 años o Objetivo: Incorporarlo a la alimentación del hogar o
Proceso
 Velocidad de crecimiento disminuye, por lo que hay un descenso relativo de las necesidades
nutricionales y una disminución fisiológica del apetito
 Es recomendable dejarlo comer con sus propias manos y enseñarle a beber de un vaso ▪
Incorporar el uso de cepillo de dientes, sin pasta dental.
o Alimentación láctea
 Continuar con LM
 Leche en polvo diluirla al 7,5%
 Leche purita cereal fortificada con vitaminas y minerales o Horarios
 Debe incorporarse a los hábitos y características de la alimentación familiar,
sin reproches, castigos ni imposiciones.
 Incorporar cuatro tiempos de comida Desayuno, almuerzo, once y cena
 Se puede incorporar colación a media mañana solo si hay espacio > 4 horas
o Alimentación sólida Ofrecer ensalada de verduras en los tiempos de
almuerzo y cena desde los 12 meses.
o Destete respetuoso
 Entre los 2 y 4 años. No hay razones para determinar una edad específica.
▪ Planificación del proceso
• Dejar de ofrecer el pecho
• Espaciar las tomas
• Ofrecer otras comidas cuando pida mamar, evitar el conflicto
• Cuando pida mamar, darle un poco y luego distraerlo con otras cosas
• Destete parcial: Dejar de darle durante el día y mantener una toma antes de dormir
• Mujer debe sacarse leche para evitar la mastitis, no es necesario el vaciamiento completo
• Suplementación en LME o Vitamina D
 Suplementar con vitamina D 400 UI diarias durante el primer
año de o Hierro
 1 mg/kg/día desde los 4 meses hasta 1 año. • Beneficios de
la LME
Para el niño Para la madre Para la sociedad

26
4
Temas clave de Pediatría y Cirugía Infantil
• Suple todos los requerimientos • Retracción del útero • Prevención del maltrato infantil
• Protección inmunológica • Recupera el peso post • Espaciamiento de nacimientos
• Mejora el desarrollo cognitivo embarazo • Ahorros en alimentos y gastos
• Evita trastornos vinculares • Previene Ca mama y ovario por enfermedad
• Mejor desarrollo maxilofacial • Apego y satisfacción emocional
• Menos riesgo de obesidad • Menos riesgo de depresión PP
• Menor riesgo de DMI y II • Menor costo
• Menor riesgo de atopías en
niños con FR

• Extracción y almacenamiento de la leche materna o Previo a la extracción, realizar masaje suave circular sobre pezón
y areola para preparar.
o La cantidad extraída puede variar según el estado psicológico de la madre, hora, ambiente y tiempo dedicado.
o Evitar el trasvasije.
o Conservación Refrigerador o congelador
 A temperatura ambiente 12 horas
 Refrigerador 3 a 5 días
 Congelador dentro del refrigerador 14 días
 Congelador independiente 3 meses
 Descongelar lentamente la noche anterior. Entibiar a baño maría una vez descongelada, sin hervirla.
• Contraindicaciones de la LM o Infección por VIH, TBC activa no tratada o Uso de drogas AAS en dosis altas,
amiodarona, ACO, antilipémicos, bromuro, drogas de abuso, metamizol, metronidazol, salicitalos, sales de oro, yodo
en dosis altas.
o Galactosemia
o Tratamiento oncológico en curso

Alimentación del mayor de 2 años


• Guía alimentaria (general para la población chilena) o 3 lácteos al día o 2 verduras y 3 frutas
o Legumbres 2 veces por semana en vez de carne o Pescado 2 veces por semana (no frito) o
Reducir azúcar y sal
• Recomendaciones
o Comer 4 comidas en preescolares y escolares

Evaluación nutricional
• Introducción o El objetivo de la EN es establecer el grado de riesgo asociado al compromiso
nutricional de adquirir una enfermedad interrecurrente o agravar una ya en curso
o La evaluación se hace a través de la medición de parámetros antropométricos donde
los más usados son el peso y la talla
o El peso debe aproximarse en el lactante a los 10 gr más cercanos, y en el escolar, a los
100 gr más cercanos.
o La talla se mide en el infantómetro hasta los 3 años, momento en el que se habla de
longitud. En los niños mayores, se mide de pie y hablamos de estatura.
o El perímetro craneano no es útil en la evaluación nutricional, pero es importante para
vigilar el crecimiento y desarrollo. Se debe medir alrededor de la cabeza por delante
encima del borde supraorbitario y detrás por el occipucio.
o Para establecer la normalidad de los parámetros, se compara con medidas de
referencia ▪ Tablas
• < 5 años: Tablas OMS
• > 5 años: NCHS ▪ Evaluación
• Se busca P/E, T/E y P/T, índices que aportan mejor información que el parámetro por si solo.
• Al haber déficit, el parámetro que se altera más rápido es el peso, por lo que el P/E nos dirá si
existe un compromiso agudo

26
5
Temas clave de Pediatría y Cirugía Infantil
• T/E informa del tiempo de evolución de este compromiso nutricional (La talla requiere más
tiempo para afectarse)
• P/T informa sobre la compensación de la desnutrición (Un peso bajo se compensa con una talla
baja, manteniendo proporcionalidad)

P/E T/E P/T


N N N Eutrofia
⬇ N ⬇ Desnutrición aguda
⬇ ⬇ ⬇ Desnutrición crónica
⬇ ⬇ N Desnutrición crónica recuperada, retraso del crecimiento o enano nutricional

 Otros parámetros funcionales Se usan para evaluar situaciones de exceso


• Circunferencia braquial
• Pliegue cutáneo
• Circunferencia muscular braquial
• Área muscular braquial
• Área grasa braquial
• Evaluación por etapas o Recién nacido
 En la primera semana puede disminuir su peso en un 5 – 10% (menor peso 3 – 4to día)
• Patológico si pierde > 15%
• Debe recuperarse máximo al 10º día de vida
 En el primer mes se evalúa el incremento ponderal siendo adecuado entre 23 y 28 grs/día
(óptimo a 40 grs/día)
o RNPT
 Hasta las 40 semanas de edad corregida
• Velocidad de crecimiento > 20 gr/kg/día
Edad corregida = Edad cronológica – (40 – edad
gestacional)
• Talla: 1 cm por semana
 De 40 semanas a 3 meses de EC
• Incremento de 30-40 gr/día
• Aplicar curvas OMS
 Desde los 3 meses hasta 6 meses de EC
• Incremento 15-20 gr/día
• Aplicar curvas OMS
 Desde los 6 meses hasta los 12 meses de edad corregida
• Aplicar curvas OMS o Si el PN < 1500 gr, corregir edad
hasta 24 meses, usando tablas OMS de RNT o Si el PN 1500 – 2500 gr,
corregir hasta 12 meses, usando tablas OMS de RNT ▪ RNPT
hospitalizado
• Tablas de crecimiento intrauterino de Alarcón y Pittaluga
o Menores de un año El más importante es el P/E
 P/E para eutrofia y déficit (riesgo de desnutrir y desnutrición)
 P/T para exceso (sobrepeso y obesidad)
 T/E para talla baja (< 2 DS) y desnutrición crónica si se asocia a P/E
< 2 DS o Niños de 1 a 6 años No se ocupa el P/E, siendo el índice más importante el
P/T
 P/T ara eutrofia, exceso y déficit
 T/E para talla baja y desnutrición crónica si se asocia a P/T < 2 DS o
Escolar y adolescente
 Se utiliza el IMC y T/E
• IMC

26
6
Temas clave de Pediatría y Cirugía Infantil
o Existen diferencias en el IMC para una misma edad y estatura y entre una y otra etapa de
Tanner, por ello se debe ajustar el IMC en función del desarrollo puberal, que sirven para
evaluar la edad biológica
o Se usará edad biológica en hombres entre 10 y 15 años y mujeres de 8 a 14 años.
▪ Si la diferencia de la EB y la EC es < 1 año, usar IMC por EC ▪ Si la diferencia EB y EC > 1 año, usar IMC por EB según
• Hombres: desarrollo genitales externos
• Mujeres: desarrollo mamario
▪ La tabla se presenta cada 6 meses, las edades intermedias deben aproximarse a la edad más cercana
o Calificación
 IMC < p10: Bajo peso
 IMC p10 – p85: normal
 IMC p85 y <p95: Riesgo de obesidad ▪ IMC > p95: Obesidad
• Dado que la normalidad del peso es amplia, algunas señales de alarma son o Cambio de canal de crecimiento del IMC en 2 controles
o Aumento del IMC > 1,5 puntos en 6 meses • T/E o Calificación
 Talla baja < p3
 Talla normal p3 y <p95 o Señales de alarma
 Incremento en talla <2.5 cm en 6 meses (menor velocidad de crecimiento de lo
esperado para la edad)
 Incremento en talla > 3,5 cms en 6 meses (pudiera ser pubertad precoz)
 Cambio de canal de crecimiento entre dos controles
 T/E entre p5 y p10 • Diagnóstico nutricional integrado o Incluye
 Calificación nutricional dada por índices
 Información alimentaria
 Antecedentes personales y familiares
• Curva pondoestatural
• Velocidad de crecimiento en el tiempo
• Peso y talla de los padres
• Patologias y antecedentes familiares de RCV y otros.

26
7
Temas clave de Pediatría y Cirugía Infantil
Lesiones dermatológicas frecuentes del lactante

Dermatitis
• Definición
o Sinónimo de eccema, inflamación de la piel o Puede ser
▪ Evolución
• Aguda Vesículas, rezumación
• Crónica Descamación y liquenificación
• Subaguda
 Agentes
• Exógenas
• Endógenas
• Dermatitis del pañal o Definición: Aquellas relacionadas con el área del pañal, agravadas por el
pañal o en el área del pañal, sin relación con él o Epidemiología
 Frencuente
 La clave es la topografía y localización de la lesión o Etiopatogenia
 Dermatitis de contacto
• Dermatitis irritativa del área del pañal
• Dermatitis alérgica ▪ Dermatitis infecciosas
• Candidiasis
• Impétigo buloso
 Dermatitis inflamatorias
• Dermatitis seborreica
• Psoriasis
 Dermatosis en el área del pañal
• Acrodermatitis enteropática
• Granuloma glúteo infantum
o Etiología
 Nutrición: Alimentación con fórmula tiene mayor incidencia (acidificación de
las heces) ▪ Sobrehidratación
• Agua, orina y heces deterioran la función de barrera
• pH urinario y heces tienen amoníaco, el cual es irritativo
 Fricción
 MO o Fisiopatología
 La fricción favorece el inicio de la erupción
 La inflamación se mantiene por los componentes de la orina y deposiciones
 Hay degradación microbiana que predismone a la infección o Cuadros
clínicos
 Dermatitis de contacto
• Dermatitis de contacto irritativa
o Ubicación: zonas prominentes respetando pliegues o Agente: Factores mecánicos irritativos
(orina, heces), pañal plástico o Clínica
 Disuria. Ocurre antes de los 12 meses
 Intértrigo: Cuando compromete pliegues. Eritema, maceración y exudado.
• Dermatitis de contacto alérgica o Definición
 Poco frecuente en lactantes
 Sensibilización al pañal o Prevención
 Pañal desechable
 Educación sobre muda, aseo con agua, uso de vaselina y pasta
Lassar.
 NO USAR CORTICOIDES riesgo de Cushing. ▪ Dermatitis
infecciosas

26
8
Temas clave de Pediatría y Cirugía Infantil
• Candidiasis o Definición
 Sobreinfección de dermatitis
irritativa de > 72 horas o Clínica
 Eritema intenso, satinado, brillante ▪
Compromiso del fondo de pliegues
 Bordes nítidos, escamas blancas,
lesiones pápulo-vesiculares
 Características: Lesiones satélite o
Tratamiento
 Antimicóticos tópicos y orales ▪
Dermatitis inflamatorias
• Dermatitis seborreica o Definición
▪ Lesiones eritemato-escamosas color anaranjado con escamas seborreicas
o Clínica
 Compromiso de pliegues
 Ausencia de satélites
 Compromiso de otras áreas corporales
• Psoriasis o Definición
 Eritema rojo vivo de límites abruptos, sin
satélites o Clínica
 Mayor compromiso de pliegues
 Sin respuesta a tratamiento habitual
 Historia familiar ▪ Dermatosis en el área del
pañal
• Acrodermatitis enteropática y Granuloma glúteo o
Raras o Severas y recurrentes
• Dermatitis atópica o Definición
 Dermatosis inflamatoria pruriginosa, crónica y recidivante
 Tienen distribución típica
 Afecta con mayor frecuencia a la infancia pero puede persistir hasta la
adolescencia
 Se produce por hiperreactividad cutánea frente a factores desencadenantes
o Etiología Multifactorial
 Base genética y desencadenante ambiental
• Genética o 70% tiene antecedentes familiares o
Piel seca por menor retención de agua y
disminución de sebo o Alteración de la función de
barrera o Mayor susceptibilidad al prurito o
Altreraciones inmunológicas
• Desencadenantes o Alérgenos Alimentos y
aeroalérgenos o Agentes microbianos S. Aureus,
Malassezia o Clima
o Irritantes Lana, nylon, jabón, humo de cigarro o Otros Medicamentos, estrés,
sudor.
o Epidemiología
 Prevalencia 15 – 30% nños
 45% antes de los 6 meses de edad, el 69% antes del año.
 70% remite en la adolescencia o Evolución natural
 60 % persiste hasta la vida adulta
 Se sigue de asma o rinitis alérgica
 El tratamiento podría evitar esta marcha o Clínica
 Distribución y morfología típica
• Lactante: Dermatitis aguda en cara, tronco y
extremidades

26
9
Temas clave de Pediatría y Cirugía Infantil
• Niño < 12 años: Dermatitis crónica en pliegues,
cuello, fosa cubital y poplítea
• Adolescente: Dermatitis crónica en superficies de
flexión (párpados y dorso de manos)
o Criterios diagnósticos
 Prúrito + al menos 3 de las siguientes
• Compromiso de pliegues cutáneos o mejillas en < 10 años
• Historia familiar o personal de atopía
• Historia de piel seca el último año
• Eczema flexural o de mejillas en < 4 años
• Inicio antes de los 2 años
 Búsqueda de elementos extrínsecos
• Test cutáneo (prick test)
• Test de parche (Patch test)
• IgE específica
o Datos importantes
 El atópico se sobreinfecta con S. aureus (90% de las placas está presente)
 La presencia del S. aureus favorece el progreso y mantención de la dermatitis
 Se deben utilizar ATB anti S. aureus
 Las 2 mejores alternativas tópicas son la mupirocina y el ácido fusídico o Tratamiento
 Medidas generales
• Educación
• Evitar irritantes
• Baños cortos y lubricación posterior
• LM
• Supresión de los alimentos que probadamente causan reacción ▪ Medicamentos
• Antihistamínicos
• Corticoides tópicos
• Corticoides orales EN AGUDO
• ATB tópicos o sistémicos Sobreinfección comprobada ▪ Derivar
• DA severas y moderadas
• DA leve y moderada cada cierto tiempo para control
• Cuando sospeche elementos extrínsecos y no se disponga de exámenes para aclararlos.
• Prúrigo agudo o Definición
 Pápulas eritematosas coronadas por vesícula
 Muy pruriginosas
 Se producen por hipersensibilidad a picada de insecto, que replica la lesión original o Tratamiento
 Antihistamínicos
 Corticoides tópicos
 Medidas generales
• Dermatitis seborreica de la indancia o Definición
 Costra láctea en cuero cabelludo de etiología no precisada
 Ocurre entre el 3 y 8 mes de vida
 Se generaliza a cejas, cara, pliegue retroauricular, axilar e inguinal, además de la zona del pañal.
o Tratamiento
 Lubricación con aceites vegetales
 Ketoconazol
 Hidrocortisona al 1%
 Shampoo antiseborreico
• Sudamina o Definición

27
0
Temas clave de Pediatría y Cirugía Infantil
 Alteración transitoria y fisiológica de los conductos sudoríparos en respuesta a sudoración excesiva
 Es más frecuente en RN y lactantes
 Son vesículas de 2 – 3 mm, cristalinas, en pliegues y zonas de roce o Tratamiento
 Disminuir la temperatura ambiental
• Lesiones vasculares o Hemangioma
▪ Definición
• Tumor benigno pedíatrico más común
• Puede ser único o múltiple
• Proviene del endotelio capital
• Los de mayor tamaño pueden comprometer estructuras vitales
y causar plaquetopenia
• Los periorificiales comprometen la función del órgano
• En la línea media pueden indicar disrrafias espinales ▪
Epidemiología
• 2% RNT
• Mas frecuentes en mujeres 3:1; y en RNPT
 Clinica
• Localización principal Cabeza y cuello 60%
• Presentación riesgosa: periorificiales, perioculares, segmentarios (asocian
otras anormalidades) ▪ Complicaciones
• Ulceración Más frecuente
• Pliegues, trauma, lesiones segmentarias se pueden lesionar
• Dolor, infección, hemorragia y cicatriz. ▪ Diagnóstico Clínico
• Imágenes sirven para diagnóstico diferencial o ECO Doppler: Diferencia
hemangioma de malformación vascular o Otros DD
 Fibrosarcoma
 Rabdomiosarcoma ▪
Tratamiento
• Objetivos o Prevenir el daño irreversible de órganos o Prevenir la
desfiguración permanente o Minimizar el estrés psicosocial o Evitar
procedimientos invasivos o Tratar adecuadamente las ulceraciones
• Manejo
o Corticoides sistémicos, tópicos e intralesionales o Embolización e intervención o Propanolol

PATOLOGÍA RESPIRATORIA

Infecciones respiratorias altas

Rinofaringitis aguda
• Definición o Resfirado común. Infección vírica con síntomas de rinorrea y
obstrucción nasal, fiebre baja o ausente y falta de manifestaciones
sistémicas
• Etiología o Rinovirus Más frecuente o Coronavirus Menos frecuente o
Otros
▪ VRS, Influenza, parainfluenza, ADV
• Epidemiología o Incidencia máxima desde otoño hasta el final de la
primavera
▪ Refleja la prevalencia estacional y vida confinada durante los meses más fríos o Niños jóvenes se resfrían 6 – 7 veces al año, lo que
disminuye con la edad hasta 2 – 3 veces en la edad adulta
• Clínica o Síntomas 1 – 3 días del contagio
▪ Obstrucción nasal, rinorrea, faringitis, tos seca o Dura 1 semana (90%) o hasta 2 semanas (10%) o Existe cambio de color o consistencia
de las secreciones nasales (mucopurulentas) lo que no indica sinusitis ni sobreinfección
• Diagnostico diferencial

27
1
Temas clave de Pediatría y Cirugía Infantil
o Rinitis alérgica o Cuerpo extraño o Sinusitis o
Coqueluche
o Faringitis estreptocócica
• Tratamiento o No hay tratamiento específico o Medidas sintomáticas o No
dar antiH2, descongestionantes ni combinacioes en < 6 años por efectos
adversos y falta de beneficio comprobado
o No dar antitusivos (acumulación de secreciones) o
Fiebre Antipiréticos
• Complicaciones y pronóstico o OMA es la más frecuente complicación (5 –
20%) o Sinusitis bacteriana

Faringoamigdalitis aguda
• Resumen inicial o La etiología viral es la más frecuente o La sospecha clínica no basta para el diagnóstico
o Debe confirmarse con algún examen rápido o cultivo faríngeo
o El tratamiento antibiótico es fundamental si hay FA estreptocócica para evitar complicaciones posteriores
o La PNC y la Amoxicilina oral son el tratamiento de primera línea
• Epidemiología o La gran mayoría son virales en < 3 años o > 3 años el más importante factor bacteriano es el S. pyogenes o
Gran importancia radica en sus potenciales complicaciones no supurativas: enfermedad reumática y glomerulonefritis
postestreptocócica
o Adicionalmente el tratamiento reduce la transmisión de S. pyogenes luego de 24 horas.
• Etiología o Viral Lo más frecuente en < 2 - 3 años
 ADV (fiebre faringoconjuntival), VEB (mononucleosis infecciosa), PI (laringitis aguda),
Rinovirus (resfrío común), Coronavirus (resfrío común), VHS (gingivoestomatitis), Coxsackie
(herpangina) y CMV o Bacteriana
 S. pyogenes Lo más frecuente
 Gonococo
 Mycoplasma
• Clínica o Se presenta como fiebre + odinofagia, irritabilidad y rechazo alimentario o En niños peude presentarse con síntomas
poco sugerentes de compromiso faríngeo como dolor abdominal, vómitos, CEG con fiebre baja, ausencia de odinofagia.
o Eritema faríngeo y tonsilar o Vesículas, exudado y petequias en paladar blando o Síntomas de complicación
 Disfagia, sialorrea, dificultad respiratoria (obstrucción de VA en mononucleosis infecciosa)
 Odinofagia intesa, trismus y dolor a la movilización cervical en absceso o flegmón periamigdalino.
o Signos y síntomas sugerentes de FA Estreptocócica
 Odinofagia
 Fiebre de inicio brusco
 Exposición a infectados
 Adenopatías
 Inflamación faríngea y/o faringoamigdaliana
 Exudados faringoamigdalianos
 Petequias en paladar y rash escarlatiniforma
 Ausencia de tos o Evolución
 Suele iniciarse como cuadro de VAS con síntomas variables dependiendo de la etiología
• Viral: Fiebre, odinofagia , coriza
• Bacteriana: odinofagia, adenopatías, dolor abdominal, petequias en paladar blando
• Estudio de urgencia o Objetivo Establecer diagnóstico etiológico (descartar S. pyogenes) o Dos corrientes de
recomendaciones de estudip
 Grupo 1: Establecen que la confirmación MO es fundamental previo al tratamiento
 Grupo 2: Considera enfermedad benigna y autolimitada, con baja tasa de complicaciones no supurativas,
ATB solo para casos seleccionados.
• AAP recomienda un test confirmatorio a todo paciente con riesgo de tener faringitis estreptocócica.
o Riesgo se establece según criterios cínicos y epidemiológicos
• El riesgo de no tratar con ATB vs tratar en relación al costo/beneficio, sustenta el confirmar el diagnóstico antes de los ATB.
• En aquellos lugares donde se recomienda tratar la FAE deberían realizarse esfuerzos para acceder a test confirmatorios

27
2
Temas clave de Pediatría y Cirugía Infantil
o Cultivo faríngeo GS o Test pack faríngeo Buen rendimiento y rapidez. Un TP negativo debe confirmarse con cultivo faríngeo.
•Tratamiento o ATB
▪ Objetivo: Prevenir las complicaciones no supurativas y supurativas, disminuir transmisibilidad y reducir síntomas
agudos ▪ Elección del ATB
• PNC es de elección (no hay resistencia de S. pyogenes) o <
27 kg 600.000 U IM x 1 vez o >27 kg 1.200.000 IM x 1 vez
• Amoxicilina es mejor tolerada con misma efectividad o 50
mg/kg/día c/24 horas VO x 10 días
• Alergicos Azitromicina o 12 mg/kg/día (Max 500 mg) VO
x 5 días (único tratamiento acortado aprobado)
o Corticoides
 Para aliviar odinofagia intensa
 Sospecha de obstrucción VA por adenopatías en mononucleosis

Sinusitis
• Definición o Es la infección supurada de los senos
paranasales que complica al resfriado común y la rinitis
alérgica o Los senos etmoidales se neumatizan al nacer,
los maxilares a los 4 años, los frontales a los 7 y los
esfenoidales a los 5.
• Etiología o La obstrucción del flujo mucociliar en el resfrío,
impide el drenaje sinusal y predispone al a proliferación
bacteriana
o El 90% es por neumococo, Moraxella, S. aureus y S. pyogenes.
• Epidemiología o El resfrío común es el principal
predisponente.
o Otros predisponentes Alergia, fibrosis quística, inmunodeficiencia por VIH, intubación, pólipos nasales y cuerpo
extraño nasal
• Clínica o Rinorrea persistente, mucopurulenta.
o Taponamiento nasal o Tos nocturna o Halitosis, dolor facial, cefalea Menos habituales.
• Estudios
o Imágenes
 Radiografía simple y TAC Nebulosidad sinusal, engrosamiento de la mucosa o nivel hidroaéreo
 Las radiografías normales tienen un valor predictivo negativo alto para sinusitis bacteriana
• Tratamiento o ATB
▪ Amoxicilina/Ácido clavulánico por 10 -14 días Primera línea
• Complicaciones o Celulitis orbitaria o Celulitis
periorbitaria

Otitis media
• Definicion: Infección supurada del oído medio o OM recurrente: 6 o más OMA antes de los 6
años de edad. Se asocia a anomalías craneofaciales
• Etiología o Bacterias acceden cuando la permeabilidad de la trompa de Eustaquio está
bloqueada por infección de la VAS o hipertrofia adenoídea
o El aire atrapado se reabsorbe creando presión negativa facilitando el reflujo de
bacterias nasofaríngeas, llevando a un derrame infectado del OM
o Tanto bacterias como virus pueden causar OM
 Bacterias Neumococo, Moraxella, S. pyogenes (menos frecuente)
 Virus Rinovirus, influenza, VRS
• Epidemiología o Incidencia máxima entre los 6 – 15 meses o Factores de riesgo
 Edad joven
 Alimentación sin LM
 Exposición pasiva al tabaco
 Exposición aumentada a patógenos Jardin infantil.
• Clínica o Lactantes Síntomas inespecíficos: Fiebre, irritabilidad y alimentación deficiente.

27
3
Temas clave de Pediatría y Cirugía Infantil
o Mayores y adolescentes Fiebre y otalgia o Otorrea o Signos de resfrío común
o Otoscopía Abombamiento de la membrana, visualización de material purulento
• Diagnóstico o Requiere
 Comienzo agudo de signos y síntomas
 Derrame en el oído medio
• Abombamiento de la membrana
• Movilidad limitada o ausente del tímpano
• Nivel hidroaéreo
• Otorrea
▪ Signos de inflamación de oído medio
• Eritema
• Otalgia definida
• Tratamiento o Primera línea en < 2 años Amoxicilina (80-90 mg/kg/día cada 12 horas
▪ Fracaso de esta terapia sugiere H. influenzae, Moraxella, neumococo • Amoxicilina /Ac. Clavulánico
(amoxicilina 80-90 mg/kg/día
• Ceftriaxona (50 mg/kg/ IM 1 dosis día por 1 – 3 días.
o En < 3 años causa vómitos
o Paracetamol e ibuprofeno para la fiebre

Otitis externa
• Definición o Inflamación y exudación en el CAE en ausencia de otros trastornos como OM y mastoiditis
• Etiología o Patógenos bacterianos más frecuentes Pseudomonas y S. aureus (natación)
• Epidemiología o Frecuencia máxima en vernao, en contraste con OM (meses más fríos y en relación a
infecciones virales de la VAS)
o La limpieza del conducto, la natación alteran la integridad del revestimiento cutáneo y las defensas locales.
• Clínica o Dolor, hipersensibilidad, exudado ótico o Fiebre ausente o Audición no se afecta o
Hipersensibilidad al movimiento del pabellón, en especial del trago, y masticación o Inspección
Inflamación del revestimiento del CAE con eritema y edema • Diagnostico CLINICO, no requiere
laboratorio
• Tratamiento o ATB y corticoides
 Ciprofloxacino + HCT
 Polimixina B-neosporina + HCT o Medidas generales en casa
 Eliminar exceso de agua tras baño
 Secar el CAE con secador
 Evitar actividades predisponentes hasta resolución del cuadro.

Generalidades de IRAs bajas: Resumen guía MINSAL

Laringitis aguda obstructiva


• Definición: o Inflamación aguda de la laringe con diversos grados de
obstrucción o Afecta epiglotis, cuerdas vocales (glotis), y región subglótica
• Epidemiología o Edad más frecuente 6 meses – 5 años (máximo entre 1,5 y 2
años) o Ocurre en otoño – invierno
• Fisiopatología o Infección viral produce inflamación difusa de la VA con punto
de mayor obstrucción en el espacio subglótico
o Edema disminuye el tamaño del lumen incrementando la resistencia de la VA o Mayor trabajo respiratorio Estridor
(síntoma cardinal).
• Etiología o Viral: Parainfluenza 1 y 3, VRS, ADV (lo más frecuente o Alergias:
Edema angioneurótico o Agentes físicos y químicos
• Clínica o Síntomas de inicio nocturno y evolución rápida
 Coriza (precede al resto de los síntomas)
 Disfonía, tos disfónica o llanto disfónico
 Estridor inspiratorio

27
4
Temas clave de Pediatría y Cirugía Infantil
 Dificultad respiratoria
 Fiebre moderada o Signos
 Estridor
 Dificultad respiratoria: polipnea, retracciones costales, cianosis, desaturación ▪ Signos de
infección respiratoria alta
• Diagnóstico CLINICO! o Criterios según escala de valoración (Severidad de
Downes)
 Grado I (Leve): Disfonía, estridor inspiratorio leve e intermitente que se acentúa con el llanto
 Grado II (Moderado): Disfonía, estridor inspiratorio y espiratorio, tiraje intenso, signos de hipoxemia
(palidez, inquietud, sudoración, polipnea), disminución del murmullo pulmonar
 Grado III (Severo): Disfonía, estridor inspiratorio y espiratorio, tiraje intenso, signos de hipoxemia,
disminución del murmullo pulmonar
 Grado IV (Agotamiento): Disfonía, estridor, tiraje intenso, palidez, alteraciones de conciencia, cianosis,
aparente disminución de la dificultad respiratoria (signos de falla respiratoria inminente Incapacidad
para llorar o hablar, compromiso de conciencia profundo, ausencia de entrada de aire, movimiento
paradojal del tórax, cianosis central y signos de hipoperfusión)
o Identifiación del agente etiológico (solo en hospitalización) Panel viral
o Fibrobroncoscopía Casos graves sin respuesta o persistencia post 48 horas de terapia (confirma y descarta otras
patologías)
• Diagnóstico diferencial o Supraglóticas
 Infecciosas: Epiglotitis, abscesos (retrofaringeo y periamigdalino), síndrome mononucleósico.
 No infecciosas: Cuerpo extraño, trauma, angiedema, neoplasias o Subglóticas
 Infecciosas: Traqueítis bacteriana
 No infecciosas: Cuerpo extraño, laringotraqueomalacia, comprensión extrínseca, croup espasmódico
• Tratamiento o Medidas generales
 Paracetamol 15 mg/kg/dosis o Ibuprofeno 10 mg/kg/dosis si dolor o fiebre
 Monitorización de saturación y FR Evaluar progresión de DR ▪ Educación
• Sobre la evolución esperable de la enfermedad
• Sobre medidas de prevención
o Tratamiento específico
▪ Grado I Ambulatorio
• Corticoides si es que es segunda consulta o Betametasona 0,4 mg/kg VO o Dexametasona 0.15
– 0.3 mg/kg EV
▪ Alternativa: Prednisona 2 mg/kg por 1 – 3 días ▪ Grado II y III Observación ambulatoria hasta que baje de
grado
• O2 para Sat > 93%
• Corticoides o Dexametasona 0.15 – 0.3 mg/kg/dosis EV o Betametasona 0.4 mg/kg VO x 1 vez
(máximo 10 mg en < 8 kg)
• Adrenalina o Nebulizar con adrenalina corriente 4 ml (0.9 mg/kg/dosis en > 4,5 kg) x 10 –
15 minutos a 4 – 6 lts por minuto o Repetir cada 20 minutos máximo 3 veces
o Observar durante 2 horas por posible efecto rebote ▪ Grado IV Estabilizar y
trasladar
• Manejar como grado III y una vez estable, trasladar
• Intubación y hospitalización
o Indicaciones para manejo ambulatorio ▪ Medidas generales
• Reposo relativo y alimentación a tolerancia
• Líquidos abundantes
 Enfermería
• Aseo nasal frecuente
• Control de la temperatura
• Evitar sobre abrigo ▪ Medicamentos
• Paracetamol 15 mg/kg/Dosis cada 6 horas o Ibuprofeno 10 mg/kg/dosis cada 8 horas si hay dolor
o fiebre > 38º C

27
5
Temas clave de Pediatría y Cirugía Infantil
 Otros
• Consultar si progresan los síntomas, fiebre > 38ºC por más de 3 días, DR, rechazo alimentario
o Indicaciones de hospitalización
 Empeora o no mejora a las 2 horas de observación post tratamiento
 Grado III desde el ingreso a urgencias
 Deshidratación o mala tolerancia oral
 Relativas: Dificultad de acceso, reconsulta o presentación atípida (edad fuera del rango o fuera de
temporada)
• Prevención o LME hasta los 6 meses o Evitar exposición a
tabaco o Lavado de manos o Limitar contactos con otros
niños.

Bronquitis aguda catarral (no obstructiva)


• Definición: Inflamación aguda
de la mucosa bronquial, viral y
autolimitada.
• Etiología o VIRAL Rinovirus,
VRS, parainfluenza, Influenza,
Adenovirus, Coronavirus,
metaneumovirus.
• Clínica o Tos productiva o
Síntomas de IRA alta viral:
fiebre, coriza, odinofagia,
anorexia, decaimiento. o
Examen pulmonar normal. A
veces crépitos gruesos
(secreción en la vía aérea) pero sin DR ni sibilancias.
• Diagnóstico CLINICO!
• Diagnóstico diferencial:
Laringotraqueitis aguda,
coqueluche, bronquitis
obstructiva, aspiración de
cuerpo extraño
• Tratamiento o Medidas
generales

 Paracetamol 15 mg/kg/dosis cada 6 horas o Ibuprofeno 10 mg/kg/dosis cada 8 horas si fiebre


> 38 o dolor
 Educación sobre evolución y prevención o Tratamiento específico
 No existe. Es un cuadro autolimitado. Si hay hipersecreción, derivar a KNT respiratoria. o
Indicaciones al alta
 Medidas generales
• Reposo relativo y alimentación a tolerancia
• Líquido abundante
 Enfermería
• Aseo nasal frecuente
• Control de la temperatura
• Evitar sobre abrigo ▪ Medicamentos
• Paracetamol 15 mg/kg/dosis cada 6 horas o Ibuprofeno 10 mg/kg/dosis cada 8 horas si dolor o fiebre > 38º C
 Otros Consulta si fiebre > 38 por más de 3 días, DR o rechazo alimentario. o Indicaciones de
hispitalización Si se complica o si hay DR

27
6
Temas clave de Pediatría y Cirugía Infantil
Bronquiolitis
• Introducción o Infección respiratoria baja viral con síntomas iniciales catarrales y tos que progresan a
sibilancias, crépitos y variable dificultad respiratoria
o Es casua frecuente de hospitalización en el < 1 año o Tiene baja mortalidad
o No existe tratamiento específico
• Definición o Corresponde a la primoinfección por VRS en el menor de 2 años
• Epidemiología o 80% es por VRS en brotes a principios de otoño hasta primavera o 15% de los primoinfectados
por VRS desarrollan bronquiolitis y el 3% requiere hospitalización o EL VRS se excreta entre 5 – 8 días o Se
transmite a través del contacto con secreciones por las manos o superficies. Inoculan la mucosa conjuntival o
nasal
• Fisiopatología o EL VRS penetra en las conjuntivas y fosas nasales. Se replica en las células y rápidamente va
hacia la vía aérea distal
o En ese momento la respuesta inmune se activa. Hay reclutamiento celular que produce gran
inflamación, daño epitelial y edema de la pared de la vía aérea, además de hipersecreción
o La broncoconstricción es secundaria a la acumulación de moco y detritus celulares, lo que explica la
nula respuesta a B2 agonistas y corticoides
o La obstrucción de la vía aérea pequeña genera la signología de sibilancias o crépitos. o El aire
atrapado hacia distal de la obstucción se reabsorbe a nivel alveolar generando Atelectasias
• Clínica o Inicialmente: coriza, estornudos, fiebre baja o 2 – 3 día: Tos, rechazo alimentario, taquipnea
o Al examen pulmonar: Espiración audible, retracciones costales, sibilancias, crépitos
• Laboratorio o Hemograma y PCR: Relizarlas en cuadro febriles persistentes, en < de 1 mes o si hay signos de
gravedad. o Estudio virológico ▪ Tipos
• Test Pack (rápido)
• IFD
• PCR
▪ Cuando tomarlos
• No en urgencias, realizar al subir a sala
• Primer caso de sala cuna
• Paciente con factores de riesgo
▪ Confirma el diagnostico, limita el uso de ATB y permite tomar medidas de aislamiento.
• Imágenes o No se recomienda de rutina o Tomar cuando haya deterioro clínico evidente o dudas diagnósticas
o Radiografía de tórax: Hiperinsuflación, Atelectasias subsegmentarias o lineales, compromiso intersticial
perihiliar bilateral, condensación (20%)
• Diagnóstico diferencial o Crisis de asma, coqueluche, cardiopatías, cuerpo extraño, displasia broncopulmonar
• Tratamiento de urgencias o Hidratación EV si no tiene VO disponible o Oxigenoterapia si saturación < 94%,
hay polipnea o retracción o NO USAR broncodilatadores ni corticoides o Paciente con gravedad moderada –
severa
▪ Prueba terapéutica con broncodilatadores
• > 6 meses y antecedentes de atopía familiar y personal
o Adrenalina Alfa adrenérgico (vasoconstrictor y reduce edema) y B adrenérgico (broncodilatador)
 Ensayo terapéutico en < 6 meses
 Evidencia de que disminuye la admisión hospitalaria o Suero hipertónico al 3% 2 – 4
ml NBZ no tiene suficiente evidencia, pero se ocupa o KNT-R no se recomienda
• Indicaciones para la casa o Alimentación normal + hidratación abundante (> 6 meses) o Aseo nasal
frecuente o Antipiréticos si hay fiebre o KTR solo si hay atelectasia importante o Control con pediatra
en 48 horas para evaluar evolución o Control en SU S.O.S.
• Indicaciones de hospitalización
o Apneas
o Requerimientos de oxígeno por desaturación o Score de Tal > 9 o Sin
respuesta luego de 2 horas en urgencias
o Factores de riesgo (cardiópatas, prematuros inmunodeficientes) o
Imposibilidad de alimentarse o Causas sociales o Visitas repetidas al
servicio

27
7
Temas clave de Pediatría y Cirugía Infantil
Bronquitis obstructiva aguda
• Definición o Obstrucción bronquial aguda con sibilancias, crépitos y roncus de etiología viral en meses fríos.
o Se denomina SBO en menores de 2-3 años.
o Se diferencia de la bronquiolitis en que ésta no responde a broncodilatadores y corticoides (ya que no se produce
por broncoespasmo o hiperreactividad bronquial, si no por hipersecreción y edema)
• Etiología Viral o VRS, Rinovirus, Metaneumovirus, Parainfluenza, Adenovirus, Influenza, Bocavirus
• Clínica o Síntomas de IRA viral: Fiebre, CEG, coriza, tos, sibilancias audibles.
o Síntomas de dificultad respiratoria o Menor de 3 meses Apnea o Examen físico
 Sibilancias Signo cardinal (pueden estar ausentes en obstrucción severa)
 Roncus y crépitos gruesos, disminución del MP, espiración prolongada
 Aumento del díametro AP del tórax (casos severos) e hipersonoridad
 Signos de hipoxemia: Palidez, polipnea, compromiso de conciencia, signos de DR. o Complicaciones:
neumonía, atelectasia, escapes aéreos (neumotórax, neumomediastino, enfisema subcutáneo),
insuficiencia respiratoria.
• Diagnóstico CLINCO o Escala de valoración Signos de insuficiencia respiratoria (Score de Tal para < 3
años)
 < 5 Leve
 6 – 8 Moderada
 > 9 Severa o Factores de riesgo (clasificar un grado mayor)
 Hospitalización
de causa
respiratoria el
ultimo año
 Uso previo de
ventilación
mecánica por
causa obstructiva
 Requerimiento
de 2
medicamentos
controladores
con buena
adherencia
 Control de base
insuficiente
o Exámenes de apoyo
 Oximetría de
pulso Normal
no determina
estado ya que
puede estar
compensando
con la FR y
comprometerse
por agotamiento
• Tratamiento
o Medidas generales
 Posición
semisentada
 Manejo
de la fiebre con
paracetamol
 Derivación a sala IRA si presenta 3 o más episodios en 1 año o Farmacoterapia
 O2 si saturación < 93% o SCORE de tal > 9
 Corticoide
• Prednisona 1 – 2 mg/kg dosis única VO
• Betametasona 0.4 mg/kg/dosis

27
8
Temas clave de Pediatría y Cirugía Infantil
• Dexametasona 0.3 mg/kg/dosis
• Hidrocortisona 10 mg/kg/dosis ▪ Broncodilatadores
• SABA (acción corta): SBT o Bromuro de Ipatropio
• NBZ: SBT 2.5 mg(0.5 ml) completar con SF hasta 4 ml a flujo 6 – 8 litros por 10 minutos
o Tratamiento específico Crisis de asma
 Crisis leve (> 5 puntos)
• Manejo ambulatorio
• Salbutamol 2 puff cada 4 – 6 horas por 5 – 7 días con aerocámara
• KNTR
• Sala IRA si > 3 cuadros obstructivos
 Crisis moderada (6-8 puntos)
• SBT 2 puff cada 10 minutos por 5 veces
• KNT R
• Reevaluación 1 hora o > 9 O2, corticoides sistémicos y hospitalizar o 6 – 8 Segundo curso de
SBT 2 puff cada 10 minutos por 5 veces + corticoide sistémico (prednisona 1 – 2 mg/kg/dosis
VO)
o < 5 Control en sala IRA en 24 horas
• Reevaluación 2 hora o > 6 O2 y hospitalizar o < 5 Control en sala IRA en 24 horas y
completar 5 dìas con corticoide sistémico (prednisona) ▪ Crisis severa
• > 11 O2 y corticoides sistémicos (prednisona) + NBZ c/ SBT + Hospitalizar
• 9 – 10 O2 + SBT 2 puff cada 10 mintuos por 5 veces + corticoide sistémico
(prednisona)
o Indicaciones al alta
 Paracetamol 15mg/kg/dosis cada 6 horas o Ibuprofeno 10 mg/kg/dosis cada 8 horas si dolor o fiebre
 SBT 2 puff cada 4 – 6 horas por 5 – 7 días
 Prednisona 1 – 2 mg/kg/día cada 12 horas por 5 días (si paso a la 2da hora de tratamiento)
 KNT respiratoria o Criterios de hospitalización
 Obstrucción severa (puntaje 11 y 12)
 Obstrucción moderada que no responde a la segunda hora
 Consultas repetidas por obstrucción moderada en tratamiento con corticoides que pasa a una segunda
hora
 Escasa red de apoyo

Coqueluche
• Definición o Infección por Bordetella pertussis de la VA alta y baja de curso prolongado (más
de 6 semanas) y eventual compromiso sistémico
o Paroxismos de tos característicos y/o apnea en el
menor de 3meses.
o Riesgo vital en el menor de 6 meses o Notificación
obligatoria
• Clínica o Tos progresiva que se hace paroxística, emetizante, de gran intensidad, con episodios
en salva y gallito instpiratorio. Puede durar de 1 a 3 meses
o Congestión facial (rubicundez), petequias,
hemorragias subconjuntivales, epistaxis.
o Examen pulmonar sin agregados o 3 períodos
▪ Período catarral (1 – 2 semanas)
• Coriza, malestar general, anorexia, tos nocturna ▪ Período paroxístico (4 semanas)
• Paroxismos de tos que terminan en gallito inspiratorio
• Puede tener cianosis y vómitos
• Se pueden manifestar las complicaciones o Neurológicas: encefalopatía, pérdida de
conciencia, hipoglicemia, HIC y convulsiones.
o Respiratorias: Atelectasias, neumonía, bronquiectasias ▪ Período convaleciente: 2 – 4 semanas
• Tos menos intensa. Desaparecen los signos

27
9
Temas clave de Pediatría y Cirugía Infantil
• Diagnóstico CLINICO y epidemiológico o Antecedentes epidemiológicos: Brote, contacto y
esquema de vacunación.
o Laboratorio:
 Hemograma: Leucocitosis (>20.000) con predominio linfocitario (>70%) o RAL > 10.000.
 IFD o PCR para Bordetella positivo
• Diagnóstico diferencial o Síndrome coqueluchoideo: Similar a coqueluche pero de evolución
más corta y benigna, sin efectos sistémicos, producidos por otros agentes: VRS, adenovirus,
chlamydia, mycoplasma
• Tratamiento o Medidas generales
 Paracetamol o ibuprofeno en caso de fiebre o
Tratamiento específico
 ATB (Corta la cadena de transmisión en 5 días,
pero no acorta el cuadro) Macrólidos
• Azitromicina 10mg/kg/día 1 dosis diaria x 5 días
• Eritromicina 50-60 mg/kg/día cada 6 horas por 7 días
• Claritromicina 15mg/kg/día cada 12 por 7 días
• Clotrimoxazol (alergia a macrólidos) Sulfametoxazol mg/kg/día cada 12 por 14 días
▪ Completar 5 días de ATB antes de retornar al colegio
o Manejo de contactos (persona que duerme bajo el mismo techo ▪ Contactos de riesgo
• Lactantes menores de 1 año
• Embarazadas en el 3er trimestre
• Mayores de 65 años
• Niños y adultos con enfermedades pulmonares o cardiovasculares crónicas ▪ Indicaciones
• QP solo en contactos de riesgo
• Puede ser administrada hasta 21 días del contacto
• Contactos sintomáticos tratar como casos
• Menores de 6 años no vacunados o menos de 5 dosis de vacuna DPT iniciar o completar el
esquema
o Hospitalización
 RN y lactante menor de 3 meses (riesgo de apneas y paro)
 Lactante menor con coqueluche grave
 Neumonia, IR, complicaciones sistémicas (convulsiones, encefalitis)

Infección respiratoria por Influenza


• Definición o Enfermedad respiratoria aguda de origen viral o Elevado potencial epidémico asociado a
transmisibilidad y variabilidad antigénica con intercambio genético entre los virus.
o Presenta una demanda elevada y fallecimientos en los grupos de riesgo.
• Etiología: Virus influenza serotipos A, B y C o Tipos A y B Epidemias o Subtipos HA y NA Pandémicos
• Clínica o Fiebre elevada de comienzo súbito de 2 a 5 días asociado a calofríos, malestar general, mialgias, cefalea,
artralgias, dolor ocular y fotofobia.
o Progresa a síntomas respiratorios: Odinofagia, congestion nasal, rinitis y tos
o Otros síntomas menos frecuentes: Inyección conjuntival, dolor abdominal, náuseas, vómitos y diarrea
o Menores de 3 meses apneas
• Complicaciones o Respiratorias: OMA, NAC (sobreinfección bacteriana) o Extrarespiratorias: Rabdomiolisis,
miocarditis, pericarditis, Sd de Guillain Barré y encefalitis
• Diangóstico CLINICO y epidemiológico o Laboratorio: IFI de confirmación (negativo no descarta)
• Tratamiento o Medidas generales
▪ Paracetamol o ibuprofeno en caso de fiebre o Tratamiento específico ▪
Antiviral

28
0
Temas clave de Pediatría y Cirugía Infantil
• Solo pacientes de riesgo para IRA grave antes de 48
horas desde la aparición de los síntomas
• No indicar en menores de 3 meses salvo en situación
crítica por patología asociada o riesgo grave
• Condiciones de riesgo o Menor de 2 años o
Inmunodeprimidos, diabéticos o Daño pulmonar
crónico o SBOR / Asma o Cardiopatías congénitas o IRC
o Epilepsia
o Manejo de contactos
 Contacto cercano: Menos de 1 mt
por mas de 15 minutos ▪ Quimioprofilaxis
• Embarazada
• Personal de salud no vacunado sin barrera de protección
• No dar en menores de 3 meses salvo alto riesgo
o Hospitalizar
 Hipoxemia
 Deshidratado o rechazo alimentario
 Dificultad respiratoria
 Compromiso hemodinámico
 Consulta repetida por deterioro clínico

28
1
Temas clave de Pediatría y Cirugía Infantil

Neumonía
• Introducción o En los menores de 2 años predominan los virus como etiología principal o Orienta a la etiología y por tanto
al tratamiento: la clínica, la radiología, el momento epidemiológico y la edad
o Se deben hospitalizar todos los pacientes con dificultad respiratoria, requerimientos de O2, deshidratados,
intolerancia a ATB orales o neumonía con derrame.
o Los menores de 3 meses deben hospitalizarse de forma precoz por riesgo de insuficiencia respiratoria

28
2
Temas clave de Pediatría y Cirugía Infantil
• Definición: Es la infección del tracto respiratorio inferior que afecta a las vías aéreas y al parénquima, con condensación de
los espacios alveolares. Causa fiebre, síntomas respiratorios agudos (tos, taquipnea y/o retracción costal) y se asocia a
cambios radiológicos pulmonares o Neumonía lobar: Localizada a uno o más lóbulos o Neumonía atípica: Cursa con patrones
intersticiales más difusos que la NAC lobar.
o Bronconeumonia: Término histológico, no debe usarse en clínica. Se refiere a la inflamación del pulmón centrada en
los bronquiolos lo que lleva a producción de exudado mucopurulento que obstruye las vías aéreas péquelas y causa
condensación focal de lóbulos adyacentes.
o Neumonía intersticial: Inflamación del intersticio o Neumonitis: Inflamación pulmonar que puede o no cursar con
condensación o Neumonitis intersticial: Característica de las infecciones víricas agudas.
• Etiopatogenia o Generalidades de la patogenia
 Los defectos en la defensa del huésped aumentan el riesgo de neumonía vacunación recibida, la
radiografía y  Los contaminantes son
la severidad
del cuadro apresados en el moco,
clínico siendo batidos
normalmente por los
cilios del epitelio en
sentido ascendente
hacia la faringe,
produciendo la
expectoración.
o Etiología
 Orienta a la etiología la
edad, el momento
epidemiológico, la
 Los patógenos que causan NAC varían con la edad
• Lactantes: El más frecuente es el VRS
• Menores de 5 años: Otros virus respiratorios (Parainfluenza, influenza, MNV, ADV)
• Mayores de 5 años: Mycoplasma pneumoniae
 Neumococo es la causa bacteriana más frecuente de neumonía lobar a cualquier edad excepto en
neonatos
 M. pneumoniae y Chlamydia son las principales causas de neumonía atípica.

28
3
Temas clave de Pediatría y Cirugía Infantil

Chlamydia provoca un síndrome respiratorio similar en los lactantes con 1 a 3 meses de edad, con tos e hiperinsuflación, por contagio perinatal.
 En inmunocomprometidos: enterobacterias gramnegativas, micobacterias, Aspergilllus, CMV y
Pneumocystis jiroveci
 En fibrosis quística está causada por S. aureus y Pseudomonas

Epidemiología o Factores de riesgo para infecciones bajas: RGE, alteraciones neurológicas (aspiración),
inmunocompromiso, anomalías anatómicas del tracto respiratorio, hospitalizados
o Transmisión Depende del agente
 Virus: Por contacto y gotitas
• VRS por gotitas gruesas
 Mycoplasma: Microaspiración de la vía aérea (transmisibilidad mínima)
• Clínica o Edad es determinante
 Neonatos: Solo fiebre o hipoxia
 Lactante menor: Apnea puede ser el primer signo
 Lactante mayor y niños: Fiebre, calofríos, taquipnea, tos, malestar general, dolor pleurítico, retracciones,
dificultad respiratoria, disnea, respiración asimétrica (dolor)
o Cuadros
▪ Neumonia viral (Generalmente en menor de 2 años)
• Clínica o Inicio con síntomas catarrales, congestión de mucosas o
En horas o días agrega fiebre bajo 38,5 ºC
o Tos, sibilancias, estridor, crépitos finos y gruesos variables, grado variable de DR
• Radiografía de tórax o Infiltrados intersticiales, hiperinsuflación y
atelectasias.
• Laboratorio: No de rutina o Hemograma y PCR no se alteran
(patrón viral) ▪ Neumonía bacteriana
• Clínica o Cuadro de inicio brusco de decaimiento, anorexia.
o Fiebre alta (>39 ºC) o Escalofríos, tos, disnea, signos de condensación alveolar o Dolor torácico y dolor abdominal (en
neumonía de las bases) o Posición antiálgica, disminución de la excursión del HT afecto. o Matidez
• Radiografía de tórax o Condensación alveolar, broncograma
aéreo (visualización de bronquios pequeños y bronquiolos con
aire interior debido a contraste con zona de condensación),
derrame pleural, neumonía redonda, abscesos.
• Laboratorio: Leucocitosis y PCR elevada (>70 mg/l) ▪ Neumonía
atípica Mycoplasma pneumoniae
• Clínica
o Inicio subagudo, principalmente escolares
o Taquipnea, tos seca (predomina), disnea (Síntomas son peores
que los signos)
o Roncus, sibilancias, crépitos bibasales, estertores o Síntomas
extrapulmonares: Cefalea, artralgias, rush cutáneo o Conjuntivitis
(por chlamydia)
• Radiografía o Hiperinsuflación, infiltrados intersticiales bibasales,
condensación atelectásica del LM

Edad Patógenos comunes (Bacterias y virus) Tratamiento ATB ambulatorio (7- Tratamiento ATB
10 días) hospitalizado (10-14 días)

Neonatos SGB, E.coli, Lysteria No se recomienda Ampicilina + cefotaxima


1 a 3 meses
• NAC febril Neumococo VRS, parainfluenza, No se recomienda Amoxicilina o ampicilina
influenza, ADV,

28
4
Temas clave de Pediatría y Cirugía Infantil

• NAC afrebril Chlamydia, CMV Azitromicina, eritromicina o Azitromicina, eritromicina o
Mycoplasma, claritromicina claritromicina
Ureaplasma.Bordetella
3 meses a 5 años Neumococo VRS, parainfluenza, Amoxicilina + azitromicina (si se Ampicilina
influenza, MNV, ADV sopecha atípico)
5 a 18 años Mycoplasma, neumococo Influenza Amoxicilina o azitromicina (si se Ceftriaxona + Azitromicina
sospecha atípico)

• Estudio diagnóstico o Consideraciones


 Rara vez se puede obtener esputo, sin embargo, no están indicados procedimientos invasivos para
obtenerlos ▪ Hemocultivos
• No deben tomarse de forma rutinaria en pacientes de aspecto no tóxico y con vacunas al día.
• Si tomarlos en deterioro clínico luego de tratamiento ATB o cuando se va a hospitalizar al
paciente
 Hemograma y PCR
• No es necesario de rutina en pacientes ambulatorios
 Test virales: Deben usarse para disminuir el uso de antibióticos
• Disponibles o Test pack (<1h) VRS, Influenza A – B, y ADV
o Panel viral (<24 hrs) VRS, Influenza A-B, parainfluenza 1, 2 y 3, ADV y
MNV
 Radiografía de tórax AP y Lat
• El uso rutinario en pacientes que se ven bien no se justifica, sobre todo en menores de 2 años,
donde la mayoría es de etiología viral
• En DR significativo, hipoxemia o falla de tratamiento ATB debe tomarse para verificar
complicaciones
o Exámenes
 Neumonía viral
Recuento de blancos normal o levemente elevado
• RPC o Test pack positivo (no descarta coinfección bacteriana)
 Neumonía bacteriana
• Leucocitosis marcada (>20.000) con predominio de segmentados.
• Hemocultivos positivos (20%) ▪ Neumonia por Chlamydia
• Eosinofilia ligera
 Neumonía por TBC PPD (+)
 Derrame pleural Toracocentesis
• Gram y cultivo para bacterias, micobacterias, hongos y virus • Es terapéutica: reduce la toxicidad y mejora
la mecánica pulmonar
o Radiografías de tórax (AP y LAT)
 No son necesarias para confirmar el diagnóstico (el diagnóstico es clínico)
 Se recomienda para diagnóstico en niños hospitalizados ▪ Patrones
• NAC bacteriana Consolidación lobar o neumonía redonda. Derrame en el 30%
• NAC viral Infiltrados lineales, broncograma aéreo, hiperinsuflación.
• NAC atípica: Aumento de marcas intersticiales o bronconeumonía
• Derrame Evaluar con ecografía para ver tamaño y comprobar que el líquido no esté tabicado

28
5
Temas clave de Pediatría y Cirugía Infantil

• Diagnóstico diferencial o Clínica


▪ Enfermedades pulmonares agudas
• Neumonitis alérgica
• Asma
• Fibrosis quística
▪ Enfermedades cardíacas Edema de pulmón por IC ▪ Enfermedades autoinmunes
• Vasculitis
• LES.
o Radiología
 Traumatismo, contusión pulmonar, hemorragia y aspiración.
• Tratamiento o Indicaciones de hospitalización (en sala)
 < 3 meses con sospecha de NAC bacteriana
 < 6 semanas con sospecha de NAC viral
 Rechazo alimentario, hidratación inadecuada
 Desaturación y dificultad respiratoria
 Falta de respuesta al tratamiento luego de 48 horas
 Persistencia de la fiebre, deterioro clínico, progresión radiológica o sospecha de complicaciones
 Pacientes con enfermedad de base
 Visitas repetidas a urgencias
 Apnea, sepsis e inaestabilidad hemodinámica o Indicaciones de hospitalización en UCI
 Necesidad de ventilación invasiva o no invasiva
 Necesidad de monitorización CR por inminente insuficiencia respiratoria
 Necesidad de monitorización CR por compromiso CV: hipotensión, taquicardia.
 Necesidad de monitorización de apneas, crisis de cianosis
 Desaturación con FiO2 de 40-50%
 Necesidad de monitorización con estado mental alterado, hipercapnia o hipoxemia o
Broncodilatadores
 Salbutamol puede ser de utilidad en obstrucción bronquial asociada
• IDM 2 – 8 puff. Mantener si respuesta es positiva
o Oxígeno
• En caso de hipoxemia o signos de DR a pesar de saturación < 93%,
 Bigotera: Hasta 3 litos
 Venturi: Sobre 3 litros (aporta hasta 50% FiO2)

28
6
Temas clave de Pediatría y Cirugía Infantil

 Mascarilla de recirculación (al 100%) o Hidratación parenteral
 En caso de deshidratación, intolerancia a medicamentos o gran dificultad respiratoria o Antibióticos en
paciente de tratamiento ambulatorio
 No dejar de rutina en > de 3 meses hasta pre escolar ya que la mayor causa en este rango son virus.
 Amoxicilina es la primera línea en lactante, pre escolar, escolar y adolescente. Alternativa es una
cefalosporina de tercera como cefuroximo.
 Dar por 10 – 7 días
 Macrólidos cuando es sospecha de NAC atípica
• Azitromicina por 5 días
• Alternativas: Claritromicina o eritromicina por 10 días
• Adolescentes: Moxifloxacino o levofloxacino por 7 días. ▪ Hospitalizados
• < de 6 semanas: Ampicilina + cefotaxima EV
• > 6 semanas – escolares: Ampicilina o PNC G
• Neumonías complicadas con empiemas: Cefalosporina de 3º (no banco por alta resistencia)
Neumonía grave: Iniciar ATB antes de la 1 hora o Lactante y pre escolar: Cloxacilina + cefotaxima o Escolar: Cloxacilina +
cefotaxima o ceftriaxona + macrólidos
• Influenza: NO debe retrasarse el inicio del antiviral en neumonía moderada a grave.
Administrar aún luego de 48 horas de iniciado el cuadro o Oseltamivir 4 mg/kg/día (Máximo 150 mg/día) en 2 dosis por 5 días •
Complicaciones o Derrame pleural y empiema: NAC bacterianas originan acumulación de líquido inflamatorio en el espacio pleural
adyacente, con derrame paraneumónico o empiema si es pus. Ambos deben drenarse.
o Neumatocele: Disección de aire dentro del tejido pulmonar o Bronquiectasias: Por cicatrización de las vías aéreas
que provoca dilatación crónica de los bronquios, aumentando el riesgo de infección
o Absceso pulmonar: Por necrosis del tejido pulmonar. Es infrecuente en niños
• Pronóstico o La mayoría se recupera con rapidez y por completo o Las anomalías radiográficas pueden
tardar 6 – 8 semanas en normalizarse o En pocos puede recidivar. En esos casos se debe investigar
trastorno subyacente con PPD, test del sudor, broncoscopía, etc.
o La neumonía grave por ADV puede llevar a bronquiolitis obliterante Sustitución de tejido bronquial en bronquiolos por tejido cicatricial con
reducción del volumen y distensibilidad de los pulmones
• Dosis de antibióticos o Amoxicilina: 75 – 100 mg/kg/día en 2 dosis VO o Azitromicina: 10 mg/kg/día 1 y
luego 5 mg/kg/día 2 – 5 en una dosis VO o Cefuroximo 30 – 50 mg/kg/día en 2 dosis VO o Claritromicina
15 mg/kg/día en 2 dosis VO o Eritromicina 50 mg/kg/día en 4 dosis VO o Moxifloxacino 400 mg/día VO o
Levofloxacino 500 mg/día VO o Cloxacilina 25-50 mg/kg/dosis cada 6 horas EV o Ampicilina: 100-200
mg/kg/día en 4 dosis EV o Cefotaxima 100-150 mg/kg/día en 3-4 dosis EV o Ceftriaxona 50-100 mg/kg/12-
24 horas EV o PNC sódica 200.000 UI/kg/día en 4 dosis EV

Fibrosis quística
• Definición y etiología o Es un trastorno autosómico recesivo donde el gen de la FQ codifica un
regulador de la conductancia transmembrana (canal de cloro) en las células epiteliales.
o Dicho canal es importante en el movimiento adecuado de sal y agua a través de las
membranas y por lo tanto, para mantener la composición de secreciones, sobre todo
en las vías respiratorias, hígado y páncreas.
o Se altera la conductancia del cloro en la glándula sudorípara que produce niveles
excesivos de sodio y cloro en el sudor Base para la prueba de cloro en el sudor
(prueba diagnóstica estándar)
▪ Es positiva cuando cloro en sudor > 60 mEq/l (99% de los pacientes) o Las secreciones anómalas de las vías respiratorias las hacen más
propensas a colonización bacteriana promoviendo la adhesión bacteriana al epitelio.
o Todo lo anterior conduce a infecciones crónicas de las vías respiratorias y, finalmente,
al daño bronquial crónico (bronquiectasias)
• Clínica o Malabsorción de proteínas Fallo de medro, hipoalbuminemia, esteatorrea o
Enfermedad hepática Ictericia colestásica o Manifestaciones pulmonares
▪ Infección respiratoria crónica
• La impermeabilidad al cloro produce excesiva reabsorción de sodio, con deshidratación relativa
de las secreciones que obstruye las vías, deteriorando el transporte mucociliar llevando a
colonización, principalmente de S. aureus y Pseudomonas.

28
7
Temas clave de Pediatría y Cirugía Infantil

• Tos productiva recurrente
• Bronquiectasias por obstrucción Insuficiencia pulmonar muerte
• Acropaquias
• Sinusitis crónica
• Poliposis nasal
• Aspergilosis broncopulmonar alérgica: Hipersensibilidad a Aspergillus que produce
inflamación/obstrucción de las VR agravando la enfermedad pulmonar o Tratamiento:
Corticoides EV (prednisona) + antifúngicos (itraconazol)
• Hemorragia de la VR: Rotura de vasos colaterales de la arteria bronquial en porciones del
pulmón infectadas de forma crónica.
• Neumotórax: En enfermedad pulmonar avanzada.
• Dificultades en el control del asma
o Manifestaciones pancreáticas
 Insuficiencia pancreática exocrina (90%): Se produce por espesamiento del moco que provoca destrucción
de los conductos pancreáticos imposibilitando excretar las enzimas pancreáticas al intestino
 Esteatorrea, deficiencias vitaminas ADEK y fallo de medro: Producto de la malabsorción de proteínas,
azúcares y grasas debido a la falta de enzimas pancreáticas
 Hipoproteinemia y edema periférico en lactantes: por falta de proteínas
 Deficiencia de insulina, intolerancia a la glucosa, diabetes: Destrucción de islotes pancreáticos.
o Manifestaciones digestivas
 Ileo meconial: Por meconio espeso

28
8
Temas clave de Pediatría y Cirugía Infantil
 Obstrucción intestinal baja: Moco espeso denso en la luz (pacientes mayores) o Manifestaciones
genitoreproductivas
 Hombres: Ausencia del conducto deferente y azoospermia en varones (casi universal) Esterilidad
 Mujeres: Amenorrea, secreciones anormales en las trompas y cuello No esteriles o Manifestaciones
hidroelectroliticas
 Hiponatremia y alcalosis metabólica hipoclorémica Dada por la incapacidad de la glándula sudorípara de
conservar cloro y sodio.
• Estudios diagnósticos o Gold estándar: Test del sudor positivo Establece el diagnóstico o
Criterios:
▪ Una o más características clínicas típicas de FQ Enfermedad pulmonar crónica, anomalías gastrointestinales, síndrome de pérdida de sal
y azoospermia obstructiva +
• Dos resultados positivos de la prueba del sudor
• Dos mutaciones conocidas causantes de FQ
• Una anomalía característica del transporte iónico en el epitelio nasal demostrada in vivo
(diferencia del potencial nasal).
o Niños con características clínicas atípicas y pruebas del sudor limítrofes pueden ser variante de fQ leve o no clásica
• Tratamiento Multifactorial (principalmente a las complicaciones GI y pulmonares) o
Respiratorio: Facilitar la eliminación de secreciones y minimizar los efectos de la infección crónica
 KNT – R
 NBZ con SSH 7% (disminuir la viscosidad de las secreciones
 ATB para infección crónica o Páncreas: Insuficiencia pancréatica exocrina
 Creon
 Dietas en alto contenido calórico
• Régimen hiper-hiper
• No retirar las grasas de la dieta
 Vitaminas ADEK
 Diabetes Insulina o Digestivo
 NN con íleo meconial Intervención quirúrgica o enemas con contraste
 Obstrucción intestinal Laxantes orales: PEG
 Colestasis Ácido ursodeoxicólico

Asma
• Definición: o Es una inflamación crónica de las vías aéreas asociada a hiperreactividad bronquial o Se caracterizaa
clínicamente por episodios recurrentes de sibilancias, disnea, opresión torácica y tos. o Existe un grado variable
de obstrucción que es reversible o En preescolares no se habla de asma, sino de episodios de sibilancias
recurrentes y/o tos persistente en donde el asma es probable.
• Epidemiología o Prevalencia en escolares de 15 a 18%
• Fisiopatología o Exacerbaciones
 La inflamación crónica aumenta en respuesta a un desencadenante
 Hay disminución del calibre de la VA por edema, hipersecreción, broncoconstricción, y acumulación de
detritus
 Lo anterior provoca alteración V/Q que revierte con aumento del trabajo respiratorio pero lleva a la falla
respiratoria por agotamiento
• Clínica
o Síntomas principales:
• Sibilancias
• Disnea
• Tos: Irritativa, en accesos y de predominio nocturno o matinal
• Opresión torácica
 Se presentan de forma episódica, espontánea o tras exposición a ciertos agentes (virus, alérgenos, humo,
ejercicio, aire frío, etc)
 Presentan variabilidad diaria, acentuándose en la noche y al despertar
 Los síntomas alivian con broncodilatador

28
9
Temas clave de Pediatría y Cirugía Infantil
 Hay antecedentes familiares de asma e historia personal de dermatitis atópica y/o rinitis alérgica
o Examen físico
 Exacerbaciones: Tos, sibilancias espiratorias, espiración prolongada, disminución del MP, signos de
hiperinsuflación (aumento del diámetro AP e hipersonoridad)
 Exacerbaciones graves: Dificultad respiratoria, quejido, aleteo nasal, retracciones, polipnea, dificultad para
hablar y alimentarse
 Signos de dermatitis atópica y rinitis alérgica.
• Diagnóstico diferencial: Fibrosis quística, malformaciones congénitas de la VA (RN), RGE (tos),
cuerpo extraño, disquinesia ciliar, tuberculosis, cardiopatías congénitas.
• Diagnóstico CLINICO o Se apoya en
 Historia clínica y examen físico sugerente: Sintomas episódicos de obstrucción
 Demostración de obstrucción con reversibilidad a broncodilatadores
 Exclusión de otros diagnósticos
o Exámenes
▪ Espirometría Se realiza con broncodilatador
• Evalúa obstrucción (a partir de los 6 años)
• Resultados o Normal: Sobre el p5
o Alteración: VEF1/CVF disminuida, VEF1 disminuido, FEF 25-75% disminuido,
CVF normal o Respuesta broncodilatadora (apoya el diagnóstico) (tras 4 puffs de SBT IDM
15 antes de espirometría)
 Aumento del VEF1 > 12%
 FEF 25-75% 30% sin modificación de la CVF (menor valor) o En niño estable, la espirometría es
normal. Si la respuesta broncodilatadora es negativa realizar test cutáneo y test de ejercicio-
metacolina.
 Pruebas de provocación bronquial con ejercicio y/o metacolina Evalúa hiperreactividad
bronquial
• Metacolina Alta sensibilidad, baja especificidad (descarta asma si es negativo)
• Test de provocación con ejercicio Alta especificidad, baja sensibilidad (confirma si es positivo)
 Evaluación de alergias Test cutáneo o pruebas in vitro
• Determinan sensibilidad a alérgenos inhalantes (los alimentarios no son precipitantes de asma)
• Limitacion Test positivo no indica que la causa del asma es la alergia
 Radiografía de tórax Solo si se sospecha complicación (infección, ruptura alveolar)
• Clasificación Según
nivel de control
o El control del asma considera dos componentes
 Control
actual
Síntomas,
nivel o
limitación
de
actividad,
terapia de
rescate y
función
pulmonar
 Riesgo futuro Probabilidad de deterioro, exacerbaciones y declinación de la función
pulmonar
• Tratamiento
▪ Descansa en 4 pilares
• Educación y autocuidado
• Control de los agravantes
• Farmacoterapia
• Manejo de las exacerbaciones
o Educación y autocuidado

29
0
Temas clave de Pediatría y Cirugía Infantil
 Nivel de control del asma
 Técnica inhalatoria
 Auto-evaluación
 Reconocimiento precoz de la descompensación
 Adherencia al tratamiento o Control de los factores agravantes ▪ Prevención
• Control ambiental: Evitar el humo de tabaco (principal desencadenante), alérgenos, irritantes
• Beneficio es controversial pero el costo beneficio le hace recomendable
▪ Medidas intradomiciliarias
• Eliminar el tabaquismo
• Manejo de ácaros o Uso de fundas y cubre colchones (reduce la hiperreactividad) o Lavar
sábanas y frazadas cada 6 semanas o No tener objetos que acumulen polvo (peluches, libros, etc) o
Aseo con paños húmedos o No hacer aseo o Ventilar y disminuir la humedad o Evitar mascotas con
pelos o plumas o Evitar contaminantes: aerosoles, olores irritantes y químicos, ▪ Medidas
extradomiciliarias
• No ventilar la casa en días de alta contaminación ambiental
• Mantener ventanas del auto cerradas en carreteras
• Evitar ir al campo en primavera, por la concentración de polen ▪ Evitar infecciones
respiratorias
• Evitar contacto con
personas con IRAs
• No ir a lugares muy
concurridos
• Cumplir con las
inmunizaciones
o Farmacoterapia
• Clasificar al paciente según el grado
de control y otorgar el tratamiento
escalonado en 5 pasos
• Medicamentos pueden ser
controladores (uso diario) o aliviadores (SOS) ▪ Síntomas ocasionales
• Paso 1 Medicación de rescate o
SABA: SBT 2 puffs SOS
▪ Alternativa: Bromuro de ipatropio
• Paso 2 Uso de SBT > 3 x semana y/o exacerbación
en los últimos 2 años que haya requerido
corticoides sistémicos o Inicial Corticoide inhalado
en dosis baja (CIDB)
▪ Alternativa: ALT ▪ Sin control después de 2 – 3 meses
• Paso 3 o > 4 años CIDB o CIDM + LABA
(Salmeterol)
 Alternativa: Subir dosis de CI a DM
 Alternativa: ALT + CI o < 4 años Subir dosis de CI
 Alternativa: CIDB + ALT
• Paso 4 o Subir dosis a CIDM + LABA
 Alternativa en < 4 años CIDM + ALT o Derivar al especialista

29
1
Temas clave de Pediatría y Cirugía Infantil

 Uso de corticoides orales


• Paso 5 o Agregar corticoide oral
o En > 6 asños con altos niveles de IgE y frecuentes hospitalizaciones Omalizumab
(anticuerpo anti IgE) ▪ Consideraciones
• Evaluaciones o Al inicio del tratamiento o Control
mensual para cambio de tratamiento si fuera
necesario o Una vez controlado visita cada 3 meses
• Cambios en el tratamiento o A los 3 meses
 Buen control con CIDM o CIDA reducir en un 50% la dosis.
 Buen control con CIDM o CIDA + LABA reducir en 50% el CI o suspender LABA
o Asintomático por 1 año Suspender CIDB
• Asma por ejercicio o Indicador de mal control, por lo que se debe subir un paso en el
tratamiento o SBT (2 puff) o ALT antes del ejercicio.
• Sospecha de asma de difícil control o Paso 4 sin control a pesar de cumplimiento
óptimo Derivar
o Manejo de las exacerbaciones
 Clasificar las crisis en leves, moderadas o severas: GINA (> 3 años) y TAL (< 3 años)

29
2
Temas clave de Pediatría y Cirugía Infantil

Leve: 0-5 puntos; Moderada: 6-8 puntos; Grave: 9 – 12 puntos

 Evidencias
• Hipoxemia O2 por mascarilla o bigotera para Sat > 94%
• SABA es la terapia SOS
• BA de dosis medida (IDM) son más eficientes que las NBZ en ausencia de hipoxemia
• Uso precoz de corticoides en las primeras 48 horas y los corticoides sitémicos en la primera hora
disminuyen el riesgo de hospitalización
• Corticoides VO tienen similar eficacia que los EV (se prefiere VO)
•Anticolinérgicos o Bromuro de ipatropio tiene efecto aditivo a los BA en crisis severa
Mejoran la función pulmonar o Usar dosis múltiples (dosis únicas no son efectivas)
• Crisis severa usar sulfato de magnesio EV Disminuye el riesgo de hospitalización ▪ Manejo
de las crisis leves
• Clínica: Aumento de la FR, sibilancias al final de espiración, sin retracción ni desaturación
• Tratamiento o SABA en IDM 2-8 puff x 3 veces cada 10 minutos o Reevaluar
o Agregar Prednisona 2 mg/kg (max 40 mg) ▪ Manejo de la
crisis moderada
• Clínica: Aumento de la FC y FR, retracción y sibilancias intensas. Saturación
entre 90 – 93%.
• Tratamiento o O2 para saturar > 93%
o SABA en IDM 2-8 puff x 3 veces o Bip 2 – 6 puff o NBZ de Bip + SBT o Berodual + S.
fisiológico (3,5 ml en < 20 kg; 1 ml en > 20 kg)
o Corticoides
 Prednisona oral 2 mg/kg
 Si no hay VO: Hidrocortisona 10
mg/kg/dosis EV o Reevaluar en 1 hora
 Hospitalizar si
• Sat 90-93% Sala
• Sat < 90% UTI

 Manejo de las crisis severas


• Clínica: Muy agitado, taquicardico, polipneico, intensa retracción y sibilancias audibles a
oído, habla con palabras sueltas y satura < 90%. Si se agota hay asincronía
toracoabdominal y disminución de la FR y retracción o torax silencioso o cianosis.
• Tratamiento o Hospitalizar en UCI o Monitorización o O2 por mascarilla al 100% o Via
venosa permeable o GSA: PCO2 normal o alta es signo ominoso o ELP: Controlar K que
baja con BA o S. Fisiológico en bolo o NBZ con SBT 2,5 – 5 mg en dosis repetidas cada
20 minutos.
o NBZ con anticolinergico + BA: si no hay respuesta con BA solo.

29
3
Temas clave de Pediatría y Cirugía Infantil
 Berodual 0,5 ml + S. fisiológico (3,5 ml en < 20 kg; 1 ml > 20 kg) o Corticoides EV: Hidrocortisona
7 – 10 mg/kg / Metilprednisolona
1mg/kg/Dosis o Sulfato de magnesio (broncodilatador) 50 mg/kg EV
 Criterios de alta Estable en SV usando IDM, saturando > 93% sin retracción
• Indicaciones o Prednisona 2 mg/kg/día por 48 horas, luego 1 mg/kg/día por 3-5 días o Uso frecuente de SABA cada 3 – 4 horas o
Consulta en urgencia si requiere > 10-12 puff de BA en 1 hora o Control con pediatra en 48 horas o Control con broncopulmonar en crisis
severas

Bronquitis obstructiva aguda


• Definición o Obstrucción bronquial aguda con sibilancias, crépitos y roncus de etiología viral en
meses fríos. o Se denomina SBO en menores de 2-3 años.
o Cuando ha habido más de 3 episodios, se habla de Síndrome Bronquial Obstructivo Recurrente
(SBOR) o Se diferencia de la bronquiolitis en que ésta no responde a broncodilatadores y corticoides (ya que no se produce por broncoespasmo
o hiperreactividad bronquial, si no por hipersecreción y edema)
• Etiología Viral o VRS, Rinovirus, Metaneumovirus, Parainfluenza, Adenovirus, Influenza, Bocavirus
• Clínica o Síntomas de IRA viral: Fiebre, CEG, coriza, tos, sibilancias audibles.
o Síntomas de dificultad respiratoria o Menor de 3 meses Apnea o Examen físico
 Sibilancias Signo cardinal (pueden estar ausentes en obstrucción severa)
 Roncus y crépitos gruesos, disminución del MP, espiración prolongada
 Aumento del díametro AP del tórax (casos severos) e hipersonoridad
 Signos de hipoxemia: Palidez, polipnea, compromiso de conciencia, signos de DR. o
Complicaciones: neumonía, atelectasia, escapes aéreos (neumotórax, neumomediastino, enfisema subcutáneo),
insuficiencia respiratoria.
• Diagnóstico CLINCO o Escala de valoración Signos de insuficiencia respiratoria (Score de Tal
para < 3 años)
 < 5 Leve
 6 – 8 Moderada

29
4
Temas clave de Pediatría y Cirugía Infantil
 > 9 Severa o Factores de riesgo (clasificar un grado mayor)
 Hospitalización
de causa
respiratoria el
ultimo año
 Uso previo de
ventilación
mecánica por
causa obstructiva
 Requerimiento
de 2
medicamentos
controladores
con buena
adherencia
 Control de base
insuficiente

o Exámenes de apoyo
▪ Oximetría de pulso Normal no determina estado ya que puede estar compensando
con la FR y
comprometerse por agotamiento
• Tratamiento o Medidas generales
 Posición semisentada
 Manejo de la fiebre con paracetamol
 Derivación a sala IRA si presenta 3 o más episodios en 1 año o Farmacoterapia
 O2 si saturación < 93% o SCORE de tal > 9 ▪ Corticoide
• Prednisona 1 – 2 mg/kg dosis única VO
• Betametasona 0.4 mg/kg/dosis
• Dexametasona 0.3 mg/kg/dosis
• Hidrocortisona 10 mg/kg/dosis ▪ Broncodilatadores
• SABA (acción corta): SBT o Bromuro de Ipatropio
• NBZ: SBT 2.5 mg(0.5 ml) completar con SF hasta 4 ml a flujo 6 – 8 litros por 10 minutos
o Tratamiento específico Crisis de asma
 Crisis leve (> 5 puntos)
• Manejo ambulatorio
• Salbutamol 2 puff cada 4 – 6 horas por 5 – 7 días con aerocámara
• KNTR
• Sala IRA si > 3 cuadros obstructivos
 Crisis moderada (6-8 puntos)

29
5
Temas clave de Pediatría y Cirugía Infantil
• SBT 2 puff cada 10 minutos por 5 veces
• KNT R
• Reevaluación 1 hora o > 9 O2, corticoides sistémicos y hospitalizar o 6 – 8 Segundo curso de
SBT 2 puff cada 10 minutos por 5 veces + corticoide sistémico (prednisona 1 – 2 mg/kg/dosis VO)
o < 5 Control en sala IRA en 24 horas
• Reevaluación 2 hora o > 6 O2 y hospitalizar o < 5 Control en sala IRA en 24 horas y
completar 5 dìas con corticoide sistémico (prednisona) ▪ Crisis severa
• > 11 O2 y corticoides sistémicos (prednisona) + NBZ c/ SBT + Hospitalizar
• 9 – 10 O2 + SBT 2 puff cada 10 mintuos por 5 veces + corticoide sistémico
(prednisona)
o Indicaciones al alta
 Paracetamol 15mg/kg/dosis cada 6 horas o Ibuprofeno 10 mg/kg/dosis cada 8 horas si dolor o fiebre
 SBT 2 puff cada 4 – 6 horas por 5 – 7 días
 Prednisona 1 – 2 mg/kg/día cada 12 horas por 5 días (si paso a la 2da hora de tratamiento)
 KNT respiratoria o Criterios de hospitalización ▪ Obstrucción severa (puntaje 11 y 12)
 Obstrucción moderada que no responde a la segunda hora
 Consultas repetidas por obstrucción moderada en tratamiento con corticoides que pasa a una segunda hora
 Escasa red de apoyo

29
6
Temas clave de Pediatría y Cirugía Infantil

PATOLOGÍA CARDIOVASCULAR

Cardiopatías congénitas

Cardiopatías congénitas acianóticas


• Etiología y epidemiología o Aparece en 8 de cada 1.000 RNV o Se asocian a trastornos cromosómicos, defectos
genéticos únicos, teratógenos o enfermedad metabólica materna
o Tres grupos fisiopatológicos
 Shunt de izquierda a derecha
 Shunt de derecha a izquierda
 Lesiones estenóticas obstructivas
• Definición o Abarcan a los shunt de izquierda a derecha con aumento del flujo sanguíneo pulmonar y lesiones
obstructivas CAP, CIV, CIA y estenosis aórtica, estenosis pulmonar, coartación de aorta.

Comunicación interventricular
• Definición Ausencia de formación adecuada del tabique ventricular
• Etiología y epidemiología o Es el defecto congénito más frecuente (25%), siendo
las más frecuentes las CIV perimembranosas
o El flujo que la atraviesa depende del tamaño del defecto y de la
RVP
 Las CIV grandes no son sintomáticas al nacer porque la RVP
está elevada en ese momento
 A medida que disminuye la RVP, mientras el RN crece,
aumenta el shunt y pueden haber síntomas.
• Clínica o CIV pequeñas son frecuentemente asintomáticas salvo un soplo fuerte
o CIV grandes producen circulación pulmonar excesiva e IC o Hallazgo típico
Soplo holosistólico
• Estudios de imagen
o ECG y Radiografía de tórax hallazgos dependen de la CIV
 Pequeñas normales
 Grandes Sobrecarga, dilatación del VI, aumento de la silueta de la arteria pulmonar, aumento del flujo
pulmonar.
 Hipertrofia del VD por aumento de la RVP (hipertensión pulmonar)
• Tratamiento 1/3 cierra de forma espontánea o Tratamiento inicial (moderadas-grandes)
▪ Diuréticos y reducción de la poscarga o Tratamiento quirúrgico cierre del
defecto (la mayoría)

Comunicación interauricular
• Etiología y epidemiología o Fallo en el crecimiento del tabique o reabsorción excesiva o 10% de
todos los defectos congénitos o Tipos
 Ostium secundum (agujero en la región del agujero oval) es la CIA más frecuente
 Ostium primum: Defecto completo del canal auriculoventricular
 Defecto del seno venoso (el menos frecuente), asocia retorno venoso pulmonar anómalo

29
7
Temas clave de Pediatría y Cirugía Infantil
• Clínica o El shunt depende del tamaño del defecto y de la distensibilidad de ambos ventrículos.
o Rara vez los niños y lactantes son sintomáticos o Soplo sistólico de
eyección o S2 desdoblado fijo (por sobrecarga del VD con eyección
prolongada al circuito pulmonar)
• Imagen
o Radiografía de tórax
 Aumento del flujo a través de la AD, VD, AP y pulmones.
 Cardiomegalia, dilataci o ECG Desviación del eje a la derecha
e hipertrofia del ventrículo derecho
• Tratamiento o NO está indicado el tratamiento farmacológico o SI a los 3 años
persiste, se recomienda el cierre
 Ostium secundum puede cerrarse por cateteristmo
 Ostium primum y seno venoso requieren cierre
quirúrgico

Conducto arterioso persistente


• Etiología y epidemiología o Permite a la sangre pasar de la arteria pulmonar a la aorta durante la vida fetal o
Con la caída de la RVP tras el nacimiento, aparece cortocircuito de izquierda a derecha y aumento del flujo
pulmonar
o 5 – 10% de las CC
• Clínica o Síntomas dependen de la cantidad de flujo
pulmonar o La magnitud depende del tamaño del CAP y
de la RVP
 CAP pequeños Sintomáticos
 CAP grandes IC a medida que disminuye la
RVP
o Examen físico
 Presión de pulso ampliada (paso de
la sangre a la circulación pulmonar
en diástole)
 Soplo continuo (como máquina) en
área infraclavicular izquierda que se
irradia a arterias pulmonares
 Soplo mesosistólico en el ápex o Imagen
 CAP pequeños
• ECG y radiografía Normales

Moderados a graves o Silueta completa de la AP
o Aumento de los vasos sanguíneos pulmonares o ECG puede mostrar hipertrofia ventricular izquierda o
Tratamiento
 Cierre espontáneo después de algunas semanas es infrecuente
 CAP moderados Diuréticos, pero requieren cierre
 CAP pequeños cierre programado o por cateterismo
Defectos del canal AV
• Etiología y epidemiología o Pueden ser completos o parciales o Pueden haber válvulas auriculoventriculares
anormales o Defecto completo produce CIA tipo ostium primum, una CIV posterior y hendiduras en la valva anterior
de la mitral.

29
8
Temas clave de Pediatría y Cirugía Infantil
• Clínica o IC a medida que la RVP disminuye (hasta las 6 – 8 semanas de vida)
o Síntomas más precoces y más graves si hay insuficiencia de la valcula
AV
o Hipertensión pulmonar por la circulación pulmonar incrementada
o Crecimiento escaso o Síndrome de Down
• Imágenes o ECOcardiografía Diagnóstica o Radiografía de tórax
Cardiomegalia con dilatación de todas las cavidades y aumento de la
vascularidad
o ECG desviación del eje a la izquerda e hipertrofia auricular
combinada
• Tratamiento o Diuréticos + digoxina
o Reparación quirúrgica es la última instancia

Estenosis pulmonar
• Etiología o El 10% de todas las CC o Puede ser
 Valvular
 Subvalvular
 Supravalvular o Resultado del fallo de desarrollo de las valvas de la válvula
• Clínica o Síntomas dependen del grado de obstrucción o
Estenosis leve es asintomática o Grave Disnea con el
ejercicio, fatigabilidad fácil
▪ NN: Pueden presentarse cianóticos por el shunt D – I o Soplo sistólico de eyección en 2do
EIC izquierdo
• Imagen
o Estenosis leve ECG y Rx de tórax: normales
o Estenosis moderada-grave Desviación del eje a la derecha e HVD o Dilatación postestenótica de la AP principal
o ECO: Evaluar el sitio de estenosis, grado de hipertrofia y morfología valvular
• Tratamiento o No suele progresar o Estenosis sintomáticas
Valvuloplastía con globo
▪ Si no es satisfactoria reparación quirúrgica

Estenosis aórtica
• Etiología y epidemiología o 5% de todas las CC o Puede ser valvular, subvalvular o supravalvular
• Manifestaciones clínicas o Estenosis ligeras no producen síntomas o Estenos graves
Fatigabilidad fácil, dolor torácico con ejercicio y síncope o Lactantes con estenosis aórtica crítica
debut con IC o Soplo sistólico de eyección en 2do EIC
▪ Duración es mayor entre mayor sea el grado de estenosis o Chasquido sistólico de eyección
• Imágenes o Grados leves ECG y Rx de tórax son normales o Moderada – grave ECG y Rx de
tórax: HVI o Radiografía de tórax dilatación postestenótica de la aorta ascendente o cayado o
Ecocardiograma Muestra la estenosis, la morfología valvular e HVI
• Tratamiento Grado va creciendo con la edad, pudiendo progresar a insuficiencia aórtica o Seguir
con ecocardiogramas o Valvuloplastía con globo en estenosis importante o Tratamiento
quirúrgico Si falla valvuloplastía

Coartación de aorta
• Etiología y epidemiología o 10% de todos los defectos CC o El área cercana a la inserción del
conducto arterioso no se desarrolla correctamente produciendo estrechamiento de la luz
aórtica
• Clínica Presentación depende de la gravedad de obstrucción y defectos asociados o Arcos
aórticos hipoplásicos, válvulas aórticas anómalas y CIV o Pueden ser dependientes de CAP
que proporciona el flujo aórtico descendente.
o Síntomas solo cuando el CAP se cierra
 Escasa alimentación, DR, Shock en < 2 semanas
 Pulsos femorales débiles y retrasados en comparación con el radial
 Función cardíaca es deficiente o Niños mayores Asintomáticos
 Molestias en las piernas con el ejercicio, cefalea o epistaxis

29
9
Temas clave de Pediatría y Cirugía Infantil
 Pulsos disminuidos o ausentes en las EEII, hipertensión o soplo (interescapular izquierdo)
 Chasquido sistólico de eyección en válvula aórtica anómala
• Imagen o ECG y radiografía HVD, cardiomegialia marcada, edema pulmonar
o Ecocardiografía Lugar de la coartación y lesiones asociadas, HVI y morfología y función de la válvula aórtica
• Tratamiento o En descompensación cardíaca PG E1 (abre el conducto arterioso) o
Inotrópicos, diuréticos.
o Reparación quirúrgica Lo más frecuente

Cardiopatía congénita cianótica


• Introducción o El retorno venoso cruza de derecha a izquierda y regresa al organismo sin pasar por los pulmones
(shunt D –I) o Cianosis, signo visible del shunt, aparece con 5g/100 ml de Hb reducida en sangre.
o Defectos cianóticos congénitos más frecuentes
 Tetralogía de Fallot
 Transposición de grandes arterias
 Atresia tricúspide
 Tronco arterial
 RVP anómalo o Muchas aparecen en el período neonatal

Tetralogía de Fallot
• Etiología y epidemiología o Es la CC cianótica más frecuente (10% de todas las
CC) o Cuatro defectos estructurales
 CIV, Estenosis pulmonar, acabalgamiento de la aorta, HVD
 La CIV es grande
• Clínica o Inicialmente pueden estar acianóticos o Hallazgos típicos
 Soplo de estenosis pulmonar
• Si la estenosis es grave o se agrava aumenta la cianosis (aumenta el shunt)
acortando el soplo
 S2 único
 Impulso ventricular derecho o Crisis hipóxicas o
Tromboembolia cerebral o Abscesos cerebrales derivados
del shunt D-I
• Imágenes o ECG Desviación del eje a derecha e HVD
o Rx de tórax Corazón en bota (punta hacia arriba por HVD) o Ecocardiograma Estenosis
pulmonar
• Tratamiento o Crisis hipóxicas Oxígeno, posición genupectoral (aumenta el RV) o Aumentar la RVS
Agonista alfa adrenérgico (fenilefrina) o Definitivo Reparación quirúrgica completa o paliativa de
derivación.
Transposición de grandes arterias
• Epidemiología o Solo 5 % de todas las CC o Es la lesión cianótica más
frecuente en RN
• Definición o Discordancia ventriculoarterial secundaria a anomalías en
la formación del tabique del tronco arterial
o Derecha: la aorta se origina en el ventrículo derecho y la
arteria pulmonar en el ventrículo izquierdo desatauración
de sangre que regresa al corazón derecho, bombeándose la
sangre oxigenada de los pulmones al VI y de nuevo a los
pulmones.
▪ Solo sobrevive si hay mezcla en agujero oval persistente, CIV, CIA,
CAP.
• Clínica o Cianosis o Taquipnea ligera y S2 único o CIV grande Menos
cianosis o IC o Corazón hiperdinámico
• Imagen
o ECG Desviación del eje a derecha, HVD o Rx de tórax Incremento de los vasos sanguíneos pulonares,
sombra cardíaca ahuevada. o Ecocardiografía Transposición y la cantidad de mezcla
• Tratamiento o Inicial PG E1 o Si persiste en hipoxia Septostomía auricular con globo: mejora la mezcla

30
0
Temas clave de Pediatría y Cirugía Infantil
o Inversión arterial Reparación quirúrgica completa (en las primeras 2 semanas de vida)
Atresia tricúspide
• Etiología y epidemiología o Es
el 2% de las CC
o Ausencia de válvula tricúspide que genera VD hipoplásico o Todo el RV cruza hacia la AI
o Es necesario CAP o CIV para mantener flujo pulmonar y supervivencia
• Clínica o Cianosis intensa o S2
única
o Si hay CIV soplo
• Imagen o ECG HVI y eje QRS
superior
o Rx de tórax Silueta cardíaca normal o ligeramente ensanchada con flujo pulmonar disminuido o Ecocardiograma
Lesiones asociadas
• Tratamiento Depende de la
presencia de CIV y del flujo
hacia los pulmones o Si no
existe CIV o es pequeña
PGE2 mantiene flujo hasta
cirugía

Tronco arterial
• Etiología y epidemiología
o Menos del 1%
o Fallo en la formación del tabique del tronco Sale un tronco arterial único con CIV grande debajo del tronco
o Aparece en las 3 - 4 primeras semanas de gestación
• Clínica Grados variables de cianosis dependen de cantidad de flujo pulmonar o IC a medida que la RVP disminuye
o Taquipnea y tos o Pulsos periféricos saltones por llenado diastólico de las arterias pulmonares que se originan en
el tronco único
o Chasquido de eyección o Soplo sistólico en el borde esternal izquierdo
• Imagen o ECG Hipertrofia ventricular combinada y cardiomegalia o Rx de tórax Flujo pulmonar aumentado,
arterias pulmonares desplazadas o Ecocadiograma Anatomía, CIV, Función de la válvula del tronco y origen de las
arterias pulmonares
• Tratamiento o Médico Fármacos descongestivos o Quirúrgico Cierre de la CIV y conducto entre VD y AP.

Shock y síndrome séptico


• Introducción o La terapia agresiva y rápida con oxígeno, fluidos e inótropos durante la
primera hora es fundamental o En el shock séptico debe realizarse tratamiento ATB en la
primera hora
• Definición o Shock: Estado en el que peligra la vida debido a entrega insuficiente de oxígeno
y nutrientes a los tejidos
▪ Puede ser y existe sin hipotensión, especialmente en niños
• Epidemiología o El más frecuente es el shock séptico (57%$), hipovolémico (24%),
distributivo (14%) y cardiogénico (5%)
• Fisiopatología o La función circulatoria está determinada por el corazón y el flujo sanguíneo
o El entrega de oxígeno (delivery) está determinada por el gasto cardíaco y el contenido
arterial de oxígeno
 EO2 = GC x CaO2 o El gasto cardíaco es el producto del volumen sistólico y la frecuencia
cardíaca
 GC = VE x FC o El volumen sistólico (VE) depende de la precarga, la contractilidad
miocárdica y la resistencia
(poscarga) o En el lactante hay menor contractilidad y mayores requerimientos por lo que el aumento del GC depende del incremento de
la FC mas que del VE

30
1
Temas clave de Pediatría y Cirugía Infantil
o La variabilidad de la FC depende del SNA y sistema de conducción sano o El CaO2 está
determinado por la [Hb], la SaO2 y la PaO2
 Cao2 = [([Hb] x 1.34 x SaO2) + (PaO2 x 0.003)] x 10 o La modulación de la Resistencia vascular
sistémica (RVS) en los diferentes lechos es de los primeros mecanismos de compensación para derivar sangre a los
órganos vitales (corazón y cerebro).
 Un aumento de la RVS ayuda a mantener PA normal o Shock es un estado dinámico
 Primera etapa: órganos vitales se mantienen por mecanismos compensatorios.
 Schock terminal: daño irreversible a los órganos
• Etiología y tipos de shock o Shock hipovolémico:
 Disminución del volumen circulante.
 Origen más frecuente pérdida de agua por diarrea.
 Otras: Pérdida de sangre, de plasma (quemaduras, peritonitis) o Shock cardiogénico
 Por falla de bomba generando disminución del GC
 Origen arritmias, sobrecarga de volumen, disfunción sistólica
 Otras: miocarditis virales, arritmias, drogas, complicaciones de cirugía cardíaca, alteraciones metabólicas,
cardiopatías congénitas
o Shock distributivo
▪ Vasodilatación periférica y acúmulo de la sangre en la vasculatura periférica ▪ Origen anafilaxia, injuria espinal o
del SNC, drogas, sepsis.
o Shock séptico
▪ Shock + SIRS con precarga baja y RVS baja, por disminución del tono vascular y aumento de la permeabilidad (shock caliente) o alta (shock
frío)
• Clínica del shock séptico o Triada: Fiebre, vasodilatación y taquicardia o Cambios en el estado
mental: irritabilidad, llanto inapropiado, confusión, poca interacción con los padres, letargia,
sospechar shock séptico.
• Diagnóstico de shock séptico o 1. Sospecha de infección hipotermia o fiebre o 2. Signos de
perfusión inadecuada
 Estado mental disminuido
 Llene capilar > 2 segundos
 Pulsos disminuidos
 Extremidades frías o moteadas
 Llene capilar flash y pulsos marcados
amplios (shock caliente)
 Diuresis < 1 ml/kg/hora o 3. Hipotensión
+ signos evidentes de infección Confirman.
• Sospecha en el RN o Signos y síntomas
 Taquicardia
 Dificultad respiratoria
 Mala alimentación
 Tono disminuido
 Mal color
 Taquipnea
 Diarrea
 Mala perfusión
 Historia de corioamnionitis
 RPM o Diagnóstico diferencial
 Shock por cierre de DA en cardiópata ductus dependiente
 Errores del metabolismo: hiperamonemia o hipoglicemia • Tratamiento o Hora
de oro
 Reconocimiento precoz, terapia agresiva y rápida con O2, fluidos e inótropos
 Reconocer el shock y su gravedad
 Signos de shock inicial: taquicardia, taquipnea discreta, llene capilar lento, leve
irritabilidad. o Fase inicial

30
2
Temas clave de Pediatría y Cirugía Infantil
 Objetivos: mejorar entrega de oxígeno, asegurar ventilación, corregir hipovolemia y
mejorar la función cardíaca
 Meta: FC normal, PA normal, LC < 2 seg, pulsos normales, diuresis de 1 ml/kg/h, estado
mental normal, glicemia normal, calcio normal ▪ Pasos
• Asegurar vía aérea
• Ventilar con oxígeno al 100%
• Establecer vía venosa, si no se logra en 90 segundos o 3 intentos Osteoclisis
• HEMOGLUCOTEST
• Iniciar reposición de volumen o Ringer Lactato 20 ml/kg en bolo tan rápido como se puedea
o De ser necesario dar hasta 3 bolos de 20 ml/kg entre 10 y 15 minutos o Signos de sobre
carga: crépitos pulmonares, dificultad respiratoria, galope cardíaco, hepatomegalia.
o Después de 1 hora continuar con cristaloides si Hb > 10 g/dl y transfusión si Hb < 10 g/dl
•Shock séptico o En shock séptico, si luego de 60 ml/kg no mejora, iniciar inótropos antes de
la primera hora
▪ Dopamina hasta 10 mcg/kg/min o ATB antes de la primera hora según sospecha, previa muestra para HC
• Exámenes de laboratorio inicialmente SOLO HIPOGLICEMIA o Resto de los exámenes: solo
cuando se pueda obtener una segunda vía.
▪ GSA, lactato, calcemia, Hb, hematocrito, pruebas de coagulación,
PCR, hemograma, creatinina, grupo y Rh

30
3
Temas clave de Pediatría y Cirugía Infantil

Enfermedad de Kawasaki
• Definición o Es una vasculitis de causa desconocida, caracterizada por afectación multisistémica
e inflamación de arterias de tamaño pequeño o mediano dando lugar a aneurismas
• Epidemiología o Es la segunda vasculitis más frecuente de la niñez o Es más habitual en niños de
origen asiático o Ocurre más a menudo en menores de 5 años con peak a los 2- 3 años o Rara
después de los 7 años
• Clínica o La afectación aneurismática de las arterias coronarias es la más importante o Cursa en 3
fases
▪ Fase aguda
• Dura 1-2 semanas
• Comienzo súbito de fiebre elevada (>40 ºC) sin foco aparente seguida de inyección conjuntival
bilateral no supurada y cambios mucosos (labios agrietados y secos, lengua de frambuesa)
• Otros síntomas y signos o Adenopatías cervicales (mayores de 1,5 cms a fines diagnósticosy
tumefacción de manos y pies

30
4
Temas clave de Pediatría y Cirugía Infantil
o Exantema morbiliforme (80%) acentuado en la zona inguinal y tórax o Irritabilidad extrema en lactantes o Artritis o Piuria esteril, pleocitosis
del LCR o Carditis Taquicardia, disnea, insuficiencia cardíaca congestiva o Aneurismas gigantes de las arterias coronarias (Infrecuentes en esta
fase, pero más frecuentes en niños muy pequeños) ▪ Fase subaguda
• Dura hasta 4 semanas
• Se caracteriza por resolución gradual de la fiebre en ausencia de tratamiento y otros síntomas
• Aparece descamación de la piel de manos y pies
• Plaquetas, otrora normales, aumentan mucho (> 1 millón/mm3)
• Aparecen los aneurismas de las arterias coronarias Mayor riesgo de morbimortalidad o Factores
de riesgo
 Fiebre prolongada
 Elevación mantenida de los parámetros
inflamatorios (VHS) ▪ Fase de convalecencia
• Se caracteriza por la desaparición de los síntomas clínicos y continúa hasta que la VHS se vuelve
normal (6 – 8 semanas después del comienzo de la enfermedad)
• Pueden aparecer líneas de Beau en las uñas de las manos (depresiones en la uña)
• Estudio diagnóstico o Laboratorio e imágenes
 Sirven para excluir otras causas de fiebre, sobre todo infección
• Cultivos de sangre, orina y radiografía de tórax
 En Fase aguda están alterados los parámetros inflamatorios y la VHS puede mostrar elevación intensa (>80
mm/h)
 Los recuentos de plaquetas son inapropiadamente bajos o normales
• En fase subaguda aumenta muhco la cifra de plaquetas
 La PL se realiza para excluir una infección aunque puede revelar leucocitosis aséptica.
 Pruebas hepáticas pueden estar alteradas por alteración vascular
 Ecocardiograma sirve para vigilar aneurismas arteriales coronarios (a las 2 – 3 semanas y a las 6 – 8 semanas)
• Diagnóstico CLINICO o Se basa en fiebre de más de 5 días de evolución sin foco y el hallazgo de
4 de 5 criterios
 Conjuntivitis bilateral no supurada
 Cambios de las membranas mucosas del tracto respiratorio superior
• Inyección faríngea
• Labios secos agrietados
• Labios inyectados
• Lengua en fresa
 Cambios en las extremidades
• Eritema periférico
• Edema periférico
• Descamación periungueal, líneas de Beau
• Descamación generalizada
 Exantema polimorfo principalmente troncal
 Adenopatías cervicales > 1,5 cms de diámetro o El diagnóstico de EK incompleta (atípica) se da por
fiebre de al menos 5 días anque solo hayan 2 o 3 criterios clínicos, pero en presencia de aneurismas de arterias
coronarias
o EK debe considerarse en lactantes menores de 6 meses con fiebre de al menos 7 días aun sin otros criterios
• Complicaciones o La mayoría resuelve sin
secuelas.
o Se han documentado infartos al miocardio probablemente por estenosis de una arteria coronaria en el sitio de un
aneurisma
o Otras complicaciones
 Trombosis de arteria coronaria
 Aneurisma periférico
 Aneurismas arteriales coronarios
 Infarto de miocardio
 Insuficiencia cardíaca

30
5
Temas clave de Pediatría y Cirugía Infantil
 Hidropesía de la vesícula biliar
 Meningitis aséptica
• Tratamiento o Se basa en la administración
de IGIV
o Se utiliza AAS en dosis antiinflamatorias y antitrombóticas hasta que el ecocardiograma muestre ausencia de
aneurismas
• Pronóstico
o La IGIV y la AAS reducen la prevalencia de la enfermedad arterial coronaria
Reanimación cardiopulmonar
• Resumen inicial o La primera causa en pediatría de PCR es la hipoxia o Si un paciente no respira y
no se mueve a pesar de estimularlo, se debe iniciar la reanimación empezando por compresiones
torácicas
o Con dos reanimadores, la secuencia es 15 compresiones x 2 ventilaciones con bolsa
mascarilla 100% o Instalar monitor cardíaco y observar ritmo. Según el ritmo, iniciar
adrenalina o desfibrilar o Luego de 2 fases, considerar intubar pero no como
prioridad si el paciente está ventilando con bolsa y mascarilla
o Considerar causas reversibles de paro 6H y 5T o Objetivo postresucitación es
preservar la función neurológica, adecuada función cardíaca, respiratoria y renal
• Introducción o Cuando el lactante o niño no respira, no se mueve ni reacciona ante un estimulo
doloroso se debe iniciar de inmediato la RCP básica que debe ser seguida de RCP avanzada
• Epidemiología o La causa más frecuente de PCR es por hipoxia debido a insuficiencia respiratoria
o shock hipoxemia, bradicardia e hipotensión
o 5 – 15% puede corresponder a un ritmo desfibrilable Fibrilación ventricular o
Taquicardia ventricular
▪ Ocurre principalmente en cardiópatas o sanos que sufren electrocución o miocarditis virales.
• Pronóstico malo
• RCP básico y avanzado o RCP básico
▪ C: Compresiones cardíacas
• Si un paciente no respira y no se mueve a pesar de estímulo doloroso, inicie RCP de inmediato,
empezando por compresiones torácicas o Ritmo: 100 x minuto o Comprimir 1/3 del diámetro AP
del tórax.
o < 1 año englobar el tórax con las manos y comprimir con los pulgares el esternón bajo
las tetillas
o > 1 año Comprimir con el talón de la mano derecha
• El masaje logrará llevar flujo al cerebro y a nivel coronario
 A: Vía aérea permeable
 B: Ventilación
• Coordinarse con un segundo rescatador para luego de 15 compresiones aportar 2 ventilaciones
con bolsa mascarilla al 100%
• Ventile con fuerza suficiente para que el tórax se expanda, evitando la ventilación excesiva porque
aumenta la presión intratorácica y disminuye el retorno venoso, bajando el GC y por ende el flujo
coronario.
• Poner el cuello en posición de olfateo
• Bolsa de 500 ml en el RN
o RCP avanzado
 Instale el monitor y pida 2 enfermeros y un auxiliar de traslado (para envío de muestras)
 Solicite que alguien interrogue a los padres sobre posibles usos de fármacos o tóxicos
 Evaluar pulso luego de 2 minutos de reanimación (no más de 10 segundos) < 1 año:
braquial o femoral; > 1 año: carotídeo o femoral
• Paciente sin tiene pulso en asistolía, FC < 60 lpm o AESP o Solicitar una vía venosa o
instalar osteoclisis para pasar Adrenalina 0.1 ml/kg
(1:10.000)
 La dilución se obtiene mezclando 1 ampolla (1 ml a 1:1.000) con 9 ml de SF para
completar 10 ml

30
6
Temas clave de Pediatría y Cirugía Infantil
 Repetir dosis cada 3 – 5 minutos (2 ciclos) si mantiene ese ritmo ▪ No parar el
masaje
• Paciente sin pulso en FV o TV o Desfibrilar
 Ubicación: paraesternal derecha, bajo la clavícula y en el ápex ▪ Dosis
• 1° descarga 2 joules/kg
• 2° descarga 4 joules/kg
• 3° descarga 4 – 10 joules/kg + amiodarona 5 mg/kg si no hay
respuesta
 Reiniciar compresiones después de cada descarga y mantener administración
de adrenalina cada 3 - 5 minutos ▪ Intubación
• Evaluarla luego de 2 ciclos o Permitirá manejo de VA independiente del masaje
cardíaco o 1 ventilación cada 6 – 8 segundos; Masaje a 100 x minuto
• No es primordial si se está ventilando con bolsa y mascarilla
• Número del TET o RN 3.5 mm o 1 – 2 años 4 mm o > 2 años (Edad/4) + 4
• No instalar SNG por riesgo a estimular vómitos y aspirar
• Confirme la ubicación del tubo o Expansión simétrica del tórax o Aire en ambos
campos o No escuchar ruidos gástricos en tórax o SI ha conseguido ritmo
 Capnografía o capnometría
 Control con radiografía de tórax
• Alternativa Máscara laríngea
 Luego de 4 ciclos, evaluar condiciones reversibles de paro (6H y 5T)
• • Hipoxemia • Tensión neumotórax
• • Hipovolemia • Toxinas
• Hidrogeniones (acidosis)
• Trombosis
• Hipoglicemia
pulmonar/coronaria
Hipo/hiperkalemia
Hipotermia • Trauma (hipovolemia, HIC)

PATOLOGÍA DIGESTIVA

Síndrome diarreico agudo


• Introducción o Causa frecuente de consulta de urgencias o El manejo inicial debe centrarse en la
rehidratación y prevención de deshidratación o Habitualmente no es necesario la búsqueda etiológica y no
está indicado el tratamiento antibiótico ni sintomático
• Definiciones o Síndrome diarreico agudo: Aumento de la frecuencia y/o volumen (> 10 ml/kg/día) de
deposiciones, con disminución de consistencia y que dura < 14 días
o Síndrome disentérico: Si hay sangre en las deposiciones o Enfermedad transmitida por
alimentos (ETA): Afección aguda gastrointestinal en dos o más personas que han comido en
una fuente común en las últimas 72 horas
• Epidemiología o Es la principal causa de mortalidad en < 5 años
• Etiología y fisiopatología o Virus: 40% de todos los casos o Bacterias: Shigella, E. coli enteropatógena, E. coli
enterohemorrágica, Salmonella, Campylobacter.
o Síndrome disentérico habitualmente por ECEH y Shigella, muy ocasionalmente amebas
o ETA: S. enteritidis, S. aureus o El síndrome bacteriano se clasifica en secretorio o invasivo,
de acuerdo a la patogenia o Diarreas intrahospitalarias: Rotaviru, clostridium difficile.
• Clínica o Anamnesis
• Vómitos
• Fiebre

30
7
Temas clave de Pediatría y Cirugía Infantil
• Signos de deshidratación: irritabilidad, decaimiento, sed, ausencia de lágrimas, mucosa oral seca, pliegue
cutáneo (+), diuresis
• Síndrome disentérico: Consultar por palidez, equimosis, diuresis (sospecha de SHU)
• Antecedentes epidemiológicos de ingesta de alimentos o agua contaminada.
• Orienta a cuadro viral
• Vómitos más intensos que preceden a la diarrea.
• Diarrea líquida, sin sangre ni pus.
• Duración breve: 2 – 3 días; o prolongada: 9 días (ADV)
• Fiebre tiende a ser más baja que en infección bacteriana.
o Examen físico
• Peso: Cambio agudo en el peso es buen indicador de
deshidratación
• Constatar grado de deshidratación
• Leve o Pérdida de agua < 50 ml/kg o < 5% del peso o
Condición general: buena
• Moderada o Pérdida de agua 50 – 100 ml/kg o 5-10% del peso
o Condición general: irritable o Signos
 Lágrimas ausentes
 Ojos hundidos
 Mucosa oral seca
 Sed que hace que beba ávidamente
 Pliegue cutáneo de retracción lenta
• Grave o Pérdida de agua > 100 ml/kg o > 10% del peso o
Condición general: letárgico o Signos
 Ojos muy hundidos
 Mucosa oral muy seca ▪ Bebe mal o no
logra hacerlo
o Laboratorio: en la mayoría no se requiere
• Estudio diagnóstico etiológico o Detección de rotavirus (test pack o Elisa)
• Cuadro grave o con deshidratación
• Cuando requiere hospitalización
• Lactante en sala cuna o jardín (primer caso o brote)
• Casos intrahospitalarios
• Patología GI crónica
• Inmunocomprometido o Comprocultivo: Pobre rendimiento, por lo que debe restringirse solo a:
• Síndrome disentérico
• Inmunocomprometidos
• Sospecha de ETA
• SHU con diarrea
• Rotavirus negativo con persistencia de los síntomas
• Patología gastrointestinal crónica
• Neonatos
• Viaje reciente a zona endémica
• Brotes o Examen parasitológico directo y seriado
• Útil si el coprocultivo es negativo en un síndrome disentérico o en antecedentes de viajes a zonas de riesgo
• Solicitar muestra directa (PAF directo). Si es negativo, solicitar estudio seriado (tres muestras más)
o En urgencias
• Deshidratación grave: ELP, GSA, función renal.
• Síndrome disentérico: Hemograma Sepsis o SHU (anemia hemolítica, trombopenia)

30
8
Temas clave de Pediatría y Cirugía Infantil
• Tratamiento inicial Objetivos: Prevenir y tratar la deshidratación, mantener la alimentación, uso selectivo
de medicamentos.
o Rehidratación
• Hidratación EV solo en deshidratación severa o fracaso en el uso de SRO
• En la fase inicial, debe usarse soluciones isotónicas como SF o RL a 20 – 30 ml/kg peso, a pasar en 20 – 30
minutos, repetir hasta 3 veces de ser necesario.
• Tan pronto como las condiciones lo permitan (mejoría de conciencia y diuresis), iniciar SRO
o Fármacos
• Probióticos:
• Concentrados de bacterias o levaduras. En lo normal la flora saprófita es responsable de bajar el
pH en el intestino y con ello facilitar la producción de acidos grasos de cadena corta, los que
favorecen la absorción de agua en el colon • En la diarrea aguda la flora está disminuida y los
probióticos ayudan a reconstituirla
• Disminuirían la duración de la diarrea y su frecuencia.
• Útiles en diarrea por rotavirus
• Son seguros, pero están contraindicados en inmunocomprometidos
• Rececadotrilo o acetorfan
• Inhibidor de la encefalinasa que aumenta la encefalina libre en la membrana basal de los
enterocitos y asi disminuir la actividad del AMPc en diarrea secretora.
• Disminuye el volumen de las deposiciones y la duración de la diarrea
• Requiere más investigación
• Ondansetrón
• Reduce los vómitos y la necesidad de hidratación EV • Produce leve aumento de la diarrea sin
relevancia clínica
• Hay evidencia para su uso VO, sublingual o EV.
• Antibióticos
• No se recomiendan de rutina
• Deben restringirse a disentería, especialmente en inmunocomprometidos (tratamiento empírico)
y en Cólera
• Situaciones específicas o SDA por Salmonella c/bacteremia o Disentería por ECEI, Campylobacter.
o Diarrea prolongada por Yersinia, grave por ECET o ECEP o Diarrea por Clorstridium difficile o
Amebiasis o Inmunosuprimidos
o Planes
• Plan A: < 5% de deshidratación
• 1) Rehidratar con SRO (Vol = (peso gr x % deshidratación)/100, de a poco, por 2 – 4 horas
• 2) Reevaluar o Rehidratado Alimentación normal + SRO 10 ml x peso kg por cada diarrea o vómito
o NO rehidratado SRO para reemplazar déficit e iniciar alimentación
• Plan B: 5 – 10% de deshidratación
• 1) Rehidratar en 4 – 6 horas con SRO (puede ser por SNG si hay mala tolerancia)
• 2) Reevaluar o Rehidratado Iniciar alimentación normal + SRO 10 ml x peso kg por cada diarrea
o vómito o No rehidratado Pasar a Plan C
• Plan C: > 10% de deshidratación
• 1) Reanimar con SF + reemplazar déficit y mantención por las próximas 4 – 8 horas
• 2) Reevaluar: Mantención o Líquidos EV para mantención + pérdidas Iniciar alimentación + SRO
10 ml x peso kg por cada diarrea o vómito o SRO por VO o SNG para mantención + pérdidas
o Consideraciones
• Deshidratación moderada-severa que no responde a tratamiento inicial HOSPITALIZAR
• Aquellos que logran resolver su estado pueden continuar con manejo ambulatorio con control
precoz (24-48 horas)
• Signos de urgencia
• Intolerancia a las SRO, agua o alimentos
• Más de 4 episodios de vómitos por hora

30
9
Temas clave de Pediatría y Cirugía Infantil
• Incapacidad para reponer las pérdidas por tasa fecal muy elevada
• Signos de deshidratación
• Sangre en deposiciones
• Cuadro febril > 72 horas

31
0
Temas clave de Pediatría y Cirugía Infantil
Síndrome emético
• Introducción o Tres elementos para un buen diagnóstico
 Edad del paciente
 Existencia o no de obstrucción
 Búsqueda de síntomas y signos de enfermedad extra abdominal o El RN debe
ser hospitalizado para estudio de sepsis, enfermedad neurológica, renal o metabólica.
• Definición o Vómito
 Expulsión forzada de contenido gástrico con uso de la musculatura torácica y
abdominal
 Implica movimientos peristálticos retrógrados
 Suele precederse por nauseas y acompañarse de síntomas autonómicos o
Reflujo: NO va acompañado ni de uso de musculatura ni de síntomas
• Fisiopatología o Cuatro vías
 Vías aferentes vagales: Sobre distensión gástrica, intoxicación, irritación de la
mucosa gástrica, drogas, radiación, dolor intenso
 Área postrema: Zona gatillo con quimiorreceptores estimulada por drogas,
acidosis, hipoxia
 Sistema vestibular: Información vestibular entre en conflicto con lo visual o por
irritación del laberinto
 Centros cerebrales superiores: Estrés, temor. o Centro del vómito inicia y
coordina las reacciones de estas vías o Efectos del vómito: Pérdida de líquido y ácido
 Deshidratación y alteraciones AB: Alcalosis hipoclorémica e hiperkalémica
 Aspiración de material vomitado: compromiso de conciencia
 Hemorragia digestiva alta: Síndrome de Mallory Weiss
 Vomitadores persistentes: esofagitis y problemas nutricionales
• Etiología o Por edad
 RN
• RGE
• Obstrucción, anomalías congénitas
• ECN
• Estado infeccioso: meningitis, sepsis
• Trastornos innatos del metabolismo: galactosemia, acidosis renal, hiperplasia suprarrenal ▪
Lactantes menores
• RGE
• Obstrucción intestinal: invaginación, EHP, hernia incarcerada
• Gastroenteritis aguda Causa más frecuente
• Infecciones
• Tos: hiperreactividad bronquial, cuerpo extraño, coqueluche (tos emetizante)
• Sobredosis de medicamentos ▪ > 1 año
• Gastrointestinales: Gastroenteritis, RGE, apendicitis
• Obstrucción GI, Invaginación, hernia atascada
• Secundaria a tos: Asma, infección, cuerpo extraño
• Metabólico: CAD

▪ Adolescentes
• Toxinas/drogas
• Embarazo
• Apendicitis aguda
• Patología biliar
• Úlceras gástricas y duodenales
• Patología ovárica: Torsión, folículo roto

31
1
Temas clave de Pediatría y Cirugía Infantil
• Intolerancias y alergias alimentarias: Sangre en deposiciones + dermatitis atópica + urticaria +
dificultad respiratoria • ETA: historia grupal
o Por tipo de vómito
 Post prandial no bilioso Estenosis hipertrófica del píloro
• Se puede palpar oliva pilórica
• Diagnóstico ECO abdominal ▪ Bilioso Obstrucción intestinal
• Mal rotación
• Vólvulos
• Invaginación (diarrea en jalea de grosellas y masa palpable en flanco derecho)
• Cuerpo extraño
• Hernias atascadas
• Enfermedad de Hirschsprung
• Divertículo de Meckel
• Bridas intestinales
 Hemático, poco volumen Esofagitis, gastritis
 Hemático, alto volumen SMW, úlcera, várices esofágicas
 Fecaloídeo Obstrucción intestinal o Trastornos neurológicos
 HIC
 Hidrocefalia
 Hematoma subdural
 Tumores
 Alteraciones del oído medio
 Migraña con aura o Otras causas
 Embarazo
 Trastornos alimentarios
 Causas psicógenas
• Diagnóstico Clínico + exámenes o Evaluar estabilidad HD e hidratación o Descartar abdomen
agudo quirúrgico o Descartar procesos infecciosos, embarazo o toxicidad o Examenes
 Hemograma, PCR, hemocultivos, glicemia, GSV, perfil bioquímico, cetonemia, ELP,
citoquímico, cultivo LCR
 Radiografía de abdomen simple
 Eco abdominal
 Endoscopía
• Tratamiento o A) Paciente estable e hidratado o con deshidratación leve a moderada sin abdomen
agudo
(sugerente de GEA)
 Probar tolerancia oral con agua fraccionada ▪ Rehidratación oral
• SRO (ReHsal 60) ▪ Antieméticos
• Ondansetrón - mg/kg VO, SL o EV
• Dar una dosis y a los 15 minutos, iniciar tolerancia oral fraccionada
• Si hay fracaso, iniciar hidratación parenteral y probar nuevamente tolerancia oral.
• Si vuelve a fracasar Hospitalizar, si no, dar el alta.
o B) Paciente inestable, aspecto grave
 Shock (pálido, compromiso de conciencia, tono disminuido, mala perfusión, llene capilar > 2 segundos,
hipotenso)
• O al 100%
• Vías venosas permeables u osteoclisis
• Ringer Lactato a cc/kg en bolo (repetir hasta 3 veces si se requiere)
• Tomar exámenes sanguíneos y radiológicos una vez estable
• Corregir estado AB • Hospitalziar en UCI
o C) paciente con sospecha de patología quirúrgica

31
2
Temas clave de Pediatría y Cirugía Infantil
 Régimen cero
 Hidratación parenteral
 Examenes de sangre y radiológicos pertinentes
 IC a cirujano infantil
 Hospitalizar
o D) Vómitos persistentes pese a hidratación parenteral y antieméticos HOSPITALIZAR

Constipación y encopresis
• Definición o Estreñimiento: 2 o menos deposiciones por semana o heces duras, como bolas,
durante al menos 2 semanas
▪ Puede ser funcional
• 2 o menos deposiciones a la semana, supresión voluntaria de la defecación y emisión infrecuente
de heces de gran diámtro, con mucho dolor
• Postura de retención: Piernas extendidas, rígidas o cruzadas
• Encopresis: Incontinencia fecal por fuga de heces retenidas
• Diagnóstico diferencial o Constipación funcional:
 Con frecuencia durante el entrenamiento de la evacuación, cuando no desea defecar.
 Heces retenidas se hacen más duras y grandes con el tiempo y llevan a defecación dolorosa
 Esto agrava la supresión voluntaria de la defecación, con perpetuación del estreñimiento o
Enfermedad de Hirschsprung
 Se caracteriza por retraso en la eliminación de meconio en NN, distensión abdominal, vómitos y
heces mal olientes.
 Causado por fracaso en migración de células ganglionares hasta el intestino distal
Espasmo y obstrucción de los segmentos aganglionares
 La mayoría desarrolla sñíntomas de EC y obstrucción
 Niños mayores no evacúan heces de gran calibre por espasmo rectal y no tienen encopresis. o Otras
causas: Hipotiroidismo, fibrosis quística, malformaciones anorrectales, retraso o discapacidades del DSM
• Características distintivas o Las malformaciones congénitas causan síntomas desde el
nacimiento o El estreñimiento funcional es lo más frecuente en niños mayores
 Comienza después de ingresar al colegio (limitado acceso libre y privado al baño) o
Hirschsprung
 Biopsia rectal confirma la ausencia de células ganglionares
 Manometría anorrectal prueba la ausencia de relajación del esfínter anal interno o
Hipotirioidismo Pruebas tiroídas o Malformaciones anorrectales Examen físico o Fibrosis quística Test
del sudor
• Evaluación y tratamiento de la constipación funcional o En la mayoría de los casos la
anamnesis sugiere constipación funcional
 Ausencia de constipación NN
 Retención fetal activa
 Heces grandes infrecuentes o Ablandadores de heces PEG
o Refuerzo positivo a la hora de crear hábitos
▪ Que se siente solo durante unos minutos e inmediatamente después de las comidas.

PATOLOGÍA INFECCIOSA

Exantemas
• Definición: Erupciones cutáneas localizadas o generalizadas por virus, bacterias o medicamentos; o bien
asociados a enfermedades sistémicas.
• Introducción o Históricamente se registraron en orden numérico: La primera enfermedad fue la escarlatina, la
segunda el sarampión, la tercera la rubéola, la cuarta el exantema estafilocócico, la quinta fue el eritema
infeccioso por parvovirus y la sexta el exantema súbito por HV 6

31
3
Temas clave de Pediatría y Cirugía Infantil
o En la edad pediátrica, lo más frecuente son los enterovirus.
o Las manifestaciones exantemáticas son erupciones cutáneas que pueden variar desde máculas,
pápulas, vesículas, pústulas o petequias de color rojo o rosado. o Generalmente son cuadros
autolimitados o El enfrentamiento debe incluir una historia clínica que indague:
 Morfología de la lesión individual
 Localización y distribución de las lesiones
 Duración del cuadro ▪ Presencia de prurito
 Enantema asociado
 Síntomas generales y prodrómicos
 Antecedentes epidemiológicos periodo de incubación, vacunaciones, asistencia a jardín o
viajes recientes.
• Clasificación
Según morfología Según Patrón:
 Maculares • Anular
 Eritematosos • Reticular
 Vesiculares
 Papulares
 Pustulares
 Petequiales

• Fisiopatología general o Diseminación Hematógena y siembra de la dermis, epidermis o endotelio vascular


(varicela, meningococcemia) o reacción inmune manifestada en la piel (sarampión y rubéola)
o Acción de toxinas bacterianas que diseminan por la sangre hasta la piel (escarlatina, síndrome de piel
escaldada estafilocócica)
o Mecanismo inmunológico (síndrome de Stevens Johnson, eritema nodoso, enfermedad de Kawasaki)
• Principales exantemas o Sarampión
▪ Clínica
• Enfermedad aguda febril (fiebre alta)
• Síntomas superiores: tos, coriza, conjuntivitis.
• Exantema Máculo papular preauricular de extensión centrífuga, no pruriginoso, dura de 4 a 7 días y que
descama en láminas finas.
• Enantema asociado Manchas de Koplik: Pápulas blanco azuladas adyacentes al 2do molar superior.
o Patognomónico.
o Preceden al exantema y desaparecen dos días después de la aparición de éste por lo
que podrían no ser vistos en la evaluación.
• Complicaciones o Respiratorias: neumonía, laringitis, OMA (por sobreinfección) o SNC: Convulsiones, cambios de
conducta, deterioro intelectual.
 Agente: Virus sarampión ARN ▪ Epidemiología
• Transmisión por contacto y via respiratoria
• Período de incubación de 8 a 12 días.
 Diagnóstico: Clínico. Confirmar con IgM.
 Aislamiento: Contacto y respiratorio por 4 días (mientras dura el exantema)
 Tratamiento: Sintomático
• Vitamina A 400.000 UI VO en individuos en grupo de riesgo (lactantes, inmunodeprimidos) para
evitar complicaciones.
o Escarlatina (Fiebre escarlatina) ▪ Clínica
• Enfermedad aguda febril, rápida instalación (hasta 40º C)
• Síntomas generales de FA: Odinofagia, cefalea, calofríos, vómitos, dolor
abdominal.
• Exantema o Papulas pequeñas eritematosas tipo quemadura de sol,
superficie rugosa y áspera (papel de lija), más intenso en pliegues, axilas e
ingle.
o Pruriginosas.

31
4
Temas clave de Pediatría y Cirugía Infantil
o Petequias en pliegues Líneas de Pastia o Aparecen 12 a 48 horas post fiebre,
principalmente en las orejas, tórax y axilas. Se generaliza en 24 horas.
o Respeta el triángulo de Filatov o Duración: 1 semana. Luego descamación severa
de manos y pies durante 3 semanas.
• Enantema asociado Enantema de Forchheimer (petequias en paladar) y lengua en fresa (roja brillante con aumento de tamaño papilar)
que evoluciona desde una lengua blanca a los 2 - 3 días de enfermedad. Obliga a descartar enfermedad de Kawasaki.
 Agente: Streptococcus BHGA y su toxina eritrogénica.

 Epidemiología
• Transmisión por contacto con paciente con FA estreptocócica (1 – 10) a través de secreciones
respiratorias
• Afecta a escolares
• Incubación 2 – 4 días.
 Diagnóstico: Clínico y bacteriológico (rescatar cultivo de faringe o de la lesión cutánea)
• Hemograma: recuento de blancos y de plaquetas (diferencial con Kawasaki)
• Antiestreptolisina – O para confirmar sospecha de complicación postestreptocócica como la fiebre
reumática.
• PCR para monitorizar complicaciones no supurativas como la fiebre reumática.
 Aislamiento: Mascarilla para evitar contacto con secreciones + contacto para heridas infectadas en las
primeras 24 horas de ATB. ▪ Tratamiento:
• Penicilina VO x 10 días o PNC Benzatina 600.000 UI/Kg IM si < 30 kg; 1.200.000 UI/Kg IM si > 30
kg.
• Macrólidos: Eritromicina o claritromicina x 10 días (alergia a PNC).
o Rubeola
▪ Clínica:
• Enfermedad leve, subclínica y oligosintomática en el 50% de los niños.
• Síntomas prodrómicos: Linfadenopatía dolorosa retroauricular, cervical
posterior y occipital.
• Exantema
o Maculas y pápulas rosadas pálidas en cara y cuello inicialmente, que se generaliza en 2
– 3 días.
o
Puede ser eritematoso y descamarse fácilmente
• Enantema asociado Enantema de Forschheimer: Máculas eritematosas y petequias en paladar blando.
 Agente: Virus rubéola (ARN)
 Epidemiología: Incubación de 14 a 23 días.
 Diagnóstico: Clínico + epidemiológico.
• Estudio a las 72 horas IgM específico (marca infección reciente)
 Aislamiento: Mascarilla por 7 días pesquisado el exantema. Evitar contacto con embarazada por riesgo de
rubeola congénita.
 Tratamiento: Sintomático.
o Eritema infeccioso (Enfermedad de la cachetada) ▪ Clínica
• Cuadro febril leve (1/3 de los pacientes) de escolares y artralgias
• En la tercera semana comienza el exantema lo que coincide con la aparición de IgG.
• Exantema
o Primera fase: Rojo en ambas mejillas (Signo de la cachetada). o Segunda fase:
Exantema reticulado en tronco y extremidades o Prurito (+) o Tronco, nalgas
y extremidades es confluente. o Se exacerba con cambios de temperatura o
Puede causar crisis aplásica en pacientes con anemia hemolítica crónica. ▪
Agente: Parvovirus B19 (ADN) ▪ Epidemiología
• Afecta más a escolares y adolescentes • Estacionario, durante invierno y primavera
• Transmisión via respiratoria.
• Incubación: 4 – 14 días.
• Mayor contagiosidad es durante el pródromo, dos semanas antes del exantema.
 Diagnóstico: IgM específica.

31
5
Temas clave de Pediatría y Cirugía Infantil
• Alternativa: PCR
 Tratamiento: Es autolimitado, aunque puede exacerbar enfermedades autoinmunes pre existentes, como la
fiebre reumática.
• Inmunocompetentes: Sintomático
• Inmunocomprometidos: Ig IV
o Exantema súbito (fiebre de los tres días)
▪ Clínica
• Enfermedad febril alta (hasta 40,5ºC) pero que conservan buen estado
general. El exantema aparece apenas desaparecida la fiebre.
• Consulta en SU por convulsión febril.
• Exantema
o Aparece luego de 3 – 4 días de fiebre. o Maculo papular, granuloso al tacto, rosado
claro, no pruriginoso. o Dura de 2 – 3 días.
o Aparece inmediatamente terminada la fiebre.
• Enantema asociado Manchas de Nagayama: Manchas eritematosas en paladar blando y úvula.
 Agente: Virus herpes tipo 6 (ADN)
• El virus infecta las células T activadas, causando aumento en la actividad de linfocitos
citolíticos y la inducción de citocinas. El virus se vuelve latente y puede reactivarse si se compromete la
inmunidad. ▪ Epidemiología
• Afecta lactantes de 6 a 18 meses
• Transmisión via respiratoria y contacto (saliva)
• Incubación de 5 - 15 días.
 Diagnóstico CLINICO y de exclusión.
 Aislamiento: Precauciones estándar.
 Tratamiento: Sintomático de la fiebre y de los episodios convulsivos.
o Varicela
▪ Clínica
• Cuadro febril moderado (pródromo) que da paso al exantema a las 24 – 48 horas.
• Exantema
o Vesicular generalizado y pruriginoso con lesiones en distintos estados (mácula, pápula, vesícula y costras)
que duran aproximadamente una semana.
o Afecta principalmente a piel cabelluda, cara o tronco; luego extremidades.
o Dejan cicatrices deprimidas, hiper o hipopigmentadas. • Enantema asociado En mucosa oral:
Vesículas o úlceras
• Reactivación por HZoster.
• Puede haber linfadenitis regional.
 Agente etiológico: Virus varicela zoster (ADN)
 Epidemiología
• Transmisión contacto y via respiratoria por gotitas de saliva (muy contagioso).
• Brotes a fines de invierno y en primavera.
• Incubación de 10 – 21 días, estando la mayor contagiosidad dos días antes del exantema hasta
que se forma la costra.
• No se considera resuelto si no hasta todas las lesiones se hayan secado y haya aparecido costra.
 Diagnóstico: Clínico.
• Test de Tzanck (rápido) Celulas gigantes multinucleadas.
• Diferencial con prurigo agudo
 Aislamiento: Respiratorio y de contacto por 5 días hasta que comienza el exantema o hasta que esté costra.
 Compliaciones: Sepsis, afectación neurológica (encefalitis, meningoencefalitis, ataxia cerebelosa, mielitis
transversa, SGBLS y púrpura fulminans.
 Tratamiento: Sintomático pues es autolimitado. Sin embargo suele sobreinfectarse (con S.
aureus o SBHGA)
• Inmunocompetente: Sintomático + antipiréticos SOS (solo paracetamol, AAS puede causar Sd. De
Reye)

31
6
Temas clave de Pediatría y Cirugía Infantil
• Prevenir sobreinfección evitando el prurito y el grataje con uñas cortas, baño diario y antiH2.
• Aciclovir indicado en riesgo de complicaciones (inmunodeprimidos, adolescentes, adultos.
Iniciarse máximo a 24 hrs del exantema.
• IgG antes de las 48 horas de la exposición y hasta 96 horas después.
o Niños inmunodeprimidos o Embarazadas o RN con madre con varicela
o RNPT hospitalizado.
o Sindrome pie, mano, boca.
 Cuadro febril de cuantia variable con amplia gama de síntomas respiratorios, neurológicos (encefalitis,
meningitis) gastrointestinales (diarrea) y cardíacos (pericarditis, miocarditis).
 Exantema
• Vesículas en la lengua y mucosa oral, pápulas eritematosas en palmas y plantas.
 Agentes: Enterovirus distintos al polio y Coxsackie. ▪ Epidemiología
• Transmisión orofecal y respiratoria
• Factores de riesgo: Niños pequeños, bajo NSE.
• Excreción fecal y respiratoria dura semanas.
 Diagnóstico
• Aislamiento por cultivo viral rutina.
• En muestras de LCR PCR
 Aislamiento: De contacto en lactantes y niños pequeños que no controlan esfínter.
 Tratamiento: Sintomático
• Ig con altos títulos de AC en pacientes graves (neonatos, inmunodeficientes con infección crónica, miocarditis).
o Sindrome papulopurpúrico en guante y
calcetin ▪ Clínica
• Cuadro febril con síntomas respiratorios y gastrointestinales
• Exantema
o Pápulas y púrpura sobre eritema en
manos y pies.
o MUY pruriginoso o Autolimitado a las 2
semanas
• Enantema asociado Púrpura en paladar duro, blando, faringe y lengua.
 Agente: Parvovirus B19
 Epidemiología
• Más común en adolescentes y adultos jóvenes.
 Tratamiento: Sintomático

31
7
Temas clave de Pediatría y Cirugía Infantil

Síndrome febril agudo sin foco


• Resumen inicial o Síndrome febril agudo foco comprenden el 14% de las consultas por fiebre en
menores de 2 años o La etiología más frecuente es infección viral banal de vías respiratorias altas
o El RN constituye el grupo de mayor riesgo porque localiza menos síntomas y signos, y tienen
enfermedades bacterianas graves ocultas. Requieren estudio completo y generalmente
hospitalización
o En el mayor de 2 – 3 meses el tratamiento dependerá de la evaluación clínica y del
grado de fiebre o En cualquier grupo etario la principal etiología bacteriana es la ITU
o Solo 1 – 2% tienen hemocultivos positivos, y la gran mayoría por neumococo o Si se

31
8
Temas clave de Pediatría y Cirugía Infantil
decide enviar a casa, requiere seguimiento y evaluación en 24 horas si la fiebre
persiste o En > 3 meses con fiebre < 39.5 solo basta observar
• Definiciones o Síndrome febril agudo sin foco
 Fiebre de 39 °C en niño < 3 años cuya historia o examen clínico no encuentra foco o Aspecto tóxico
 Apariencia clínica que sugiere enfermedad grave por uno o más signos: letargia, mala perfusión, hipo o
hiperventilación, cianosis.
o Bacteremia oculta
 Bacteria en sangre sin aspecto tóxico y sin foco
evidente o Sepsis
 Bacteria patógena en sangre en niño con
aspecto tóxico
• Epidemiología o Etiología viral La mayoría en < 2 años, fiebre > 39 °C y sin foco o Etiología
bacteriana grave 7 – 10%: meningitis, ITU, artritis, OML, neumonía o sepsis
▪ ITU: Es la causa bacteriana más frecuente en varones < 6 meses y niñas < 1 año ▪ Bacteremia oculta
• 92% neumococo, 5% Salmonella, 1% Meningococo, 1% SGB
• La mayoría evoluciona bien sin tratamiento
▪ RN
• 15 – 30% tienen infección seria por bacterias más agresivas SGB (meningitis), E.
coli, Listeria
• Más frecuentes: ITU y bacteremia oculta
o Causa no infecciosa Grupo menor: Enfermedad de Kawasaki
• Clínica, diagnóstico diferencial y laboratorio
 Se debe distinguir a quienes causan infección bacteriana grave o bacteremia
que se puede focalizar en meninges y cerebro o RN
 Mayor susceptibilidad
• Poca capacidad para localizar síntomas y signos
• Inmadurez inmunológica
• Se puede infectar en relación a malformaciones anatómicas
• Sistema inmune es virgen a los agentes infecciosos comunes (VRS, rotavirus)
• Adquiere agentes en el paso por el canal del parto ▪ Realizar batería completa de
exámenes
• Hemograma, PCR y PCT
• Hemocultivos
• Orina completa y urocultivo
• PL citoquímico, Gram, cultivo, PCR
• Radiografía de tórax: solo si hay síntomas respiratorios ▪ Pre asumir infección
bacteriana grave, sobre todo en < 2 semanas.
o Lactantes hasta los 3 meses
 Persiste la posibilidad de infección seria sin aspecto tóxico ▪ Deben evaluarse
igual que un RN salvo excepciones
• PL solo si se indica ATB por sospecha de infección grave
• Radiografía de tórax solo si taquipnea, ruidos agregados, retracción intercostal ▪ Niños de
bajo riesgo de infección grave
• Sanos antes de la fiebre
• Sin evidencias de infección bacteriana focal
• Buen aspecto general
• Laboratorio normal o negativos en la evaluación inicial
o Niños desde los 3 meses hasta los 3 años ▪ Generalidades
• Apreciación clínica es importante. SI se ve bien tiene bajas posibilidades de tener infección
bacteriana grave (3%)
• Punto de corte de temperatura para estudio de enfermedad bacteriana es de 39.5 °C en > de 6
meses

31
9
Temas clave de Pediatría y Cirugía Infantil
• Las infecciones ocultas más frecuentes son: ITU (primera a descartar o confirmar), bacteremia
oculta, neumonía
• Durante el brote de influenza, solicitar detección del virus para empezar, si se dispone del test.
• Lo más importante es el seguimiento a las 24 horas si la fiebre persiste para evitar enfermedad
grave
 Exámenes Ningún examen es indicador absoluto de bacteremia. El que más sirve es
el RAN (> 10.000) y la PCT
• PCR o > 70 mg/l indica infección bacteriana grave o < 50 mg/l indica baja probabilidad de infección
bacteriana grave
• PCT
o Más sensibilidad y especificidad que la PCR en las primeras horas o Buen valor predictivo en sepsis, para control de
tratamiento y pronóstico o Ventaja: Se eleva antes que la PCR, con máxima utilidad antes de 8 horas de evolución
(buen VPN) ▪ Radiografía de tórax
• < 90 días, sin síntomas ni signos respiratorios no se justifica (probabilidad de neumonía casi nula)
• 26% de niños con temperatura > 39 °C, < 5 años, sin síntomas ni signos respiratorios y con
leucocitosis de 20-000, tienen neumonía oculta por lo que se les debe tomar una placa.
• Tratamiento o RN
 Presumir infección grave
 Realizar batería completa de exámenes
 Hospítalizar e iniciar ATB empírico hasta cultivos
• Ampicilina + cefalosporina de 3ra
 Evaluar exámenes diagnósticos de infección viral (se realizan en sala normalmente) o Lactantes <
3 meses de bajo riesgo (criterios de Rochester) ▪ Tratar de forma ambulatoria
• Manejo ambulatorio con ATB o Si tienen acceso fácil al hospital o Una dosis de ceftriaxona
IM o IV 50 mg/kg con control en 24 – 48 horas
• Manejo ambulatorio sin ATB o Observación cuidadosa con instrucción a los padres de
detectar empeoramiento (letargia, cambios de coloración de la piel, irritabilidad,
dificultad respiratoria) o Controlar en 24 horas
o Lactantes < 3 meses de alto riesgo
 Hospitalizar e indicar ATB empírico a espera de cultivos o Niños de 3 – 36
meses sin aspecto tóxico
 Vacuna antineumocócica (al menos una dosis):
• Solo tomar OC y urocultivo. o Si sugiere ITU, iniciar tratamiento.
o SI es negativo, test de detección viral ▪ Tratamiento ATB
• Si la sospecha de bacteremia es alta o si hay dudas o Ceftriaxona 50 mg/kg/día
en 1 dosis EV o IM a espera de cultivos
• Controlar evolución
• Criterios de Rochester para bajo riesgo de infección bacteriana grave o 1. Buen
aspecto o 2. Previamente sano o 3. Sin evidencia de infección en piel, tejidos
blandos, articulaciones y oído o 4. Laboratorio
 Leucocitos 5.000 – 15.000/mm con < 1500 x mm3 de baciliformes
 Orina con < 10 leucocitos por campo
 Si hay diarrea < 5 leucocitos/campo en extendido fecal.

Meningitis y encefalitis

Meningitis
• Definición o Inflamación de las meninges que puede ser bacteriana, viral o por hongos.
o La meningitis bacteriana aguda es una emergencia médica o Aun con tratamiento precoz puede dejar secuelas graves o llevar a la muerte,
sobretodo en infección por neumococo o meningococo.

32
0
Temas clave de Pediatría y Cirugía Infantil
• Etiología o Meningitis aséptica: Meningitis vírica que puede contener otros agentes infecciosos
(sífilis, tuberculosis, enfermedad de Lyme), infecciones parameníngeas (absceso encefálico,
peridural o empiema del seno venoso) ▪ Meningitis vírica:
• Enterovirus (más frecuente), parechovirus y arbovirus.
• Se excretan vía fecal y la transmisión persiste por semanas
• Son la principal causa de meningoencefalitis
• Otros: VHS, VEB, CMV y VIH
o Meningitis bacteriana MO más frecuentemente aislados son el neumococo y meningococo (Neisseria
meningitidis)
 Neonatos
• Más habitual: SGB, E.Coli, Klebsiella, Enterobacterias
• Menos habitual: Listeria ▪ > 1 mes
• Neumococo
• Meningococo
 Factores predisponentes
• Exposición reciente a persona con meningitis por meningococo
• Infección respiratoria u ótica reciente
• Viaje a zona meningocócica entérica
• TEC penetrante
• Otorrea o rinorrea
• Implantes coclearos
• Defectos anatómicos
• Déficit de Ac antineumocócicos
o Meningitis neonatal son los mismos que sepsis neonatal (SGB, E. Coli, Lysteria) o Meningitis estafilocócica ocurre
en pacientes neuroquirúrgicos o tras TEC penetrante o Meningitis parcialmente tratada es una meningitis
bacteriana complicada con tratamiento ATB antes de la PL, lo que puede llevar a obtener LCR negativo
• Epidemiología o Incidencia es más alta en menores de 1 año o Factores de riesgo:
 Inmunodeficiencia
 Hacinamiento
 Fuga de LCR por anomalía o fractura Aumenta el riesgo de neumococo o Meningitis viral
 Enterovirus y Parechovirus
• Mayor incidencia en verano y otoño
• Mas prevalente en grupo socioeconómico bajo, niños pequeños e inmunocomprometidos • Clínica o Pródromo de
síntomas respiratorios superiores (frecuente) o Inicio rápido de los síntomas neurológicos (neumococo y meningococo) o
Signos de inflamación meníngea: cefalea, irritabilidad, náuseas, rigidez de nuca, letargo, fotofobia y vómitos (m
o Signos de irritación meníngea (Kernig y Brudzinski): Solo en > de 12 meses (antes no están presentes), siendo mínimos
en lactantes jóvenes.
o Signos en niños mayores: Signos neurológicos focales y convulsiones; Signos de HIC: cefalea, diplopía, vómitos,
abomamiento de fontanela; Signos de HIC herniada: ptosis, paralisis del VI par, anisocoria, bradicardia con
hipertensión, apnea.
o Signos en lactantes y niños: Inespecíficos Fiebre, Artralgias y mialgias, Irritabilidad, alimentación deficiente.
o Lesiones petequiales o purpúricas (meningococo), sepsis, shock y coma
• Estudio diagnóstico o Si se sospecha meningitis bacteriana
 Realizar PL (excepto si hay signos de HIC por riesgo de hernia)
• Recuento de blancos, fórmula, proteínas, glucosa, Gram y cultivo
 RPC para enterovirus, parechovirus y VHS o Estudio en urgencias
 Examenes generales
• Hemograma y PCR Orienta a la etiología (viral o bacteriana)
• Pruebas de coagulación Muy alteradas contraindican PL
• Glicemia Para compararla con la glucorraquia
• Hemocultivo Sobretodo si no se puede realizar PL. (+) 90% de meningitis bacteriana
• GSA, lactato, función renal, ELP.

32
1
Temas clave de Pediatría y Cirugía Infantil
o Hallazgos del LCR

Condición Leucocitos Proteínas Glucosa Comentarios


 Meningitis bacteriana Hasta 60.000, predominio 100 - 500 < 40 o <40% de la Gram: tiñe MO aguda
PMN glicemia
 Meningitis bacteriana Hasta 10.000 predominio MN >100 Baja o normal Se pueden ver MO. Si se
tratada dieron ATB pre PL se
pueden detectar por
RPC
 Meningitis tuberculosa 10-500 con predominio PMN; 100 -500 <50, baja PPD y Rx de tórax (+) luego
predominio linfocitos y progresivamente monocitos
 Meningitis micótica 25 – 500 predominio PMN, 20-500 <50, baja Tinta china (+)
luego MN progresivamente Criptococo
 Meningitis viral < 1000, predominio PMN, <200 Normal (hasta 40) RPC: Enterovirus y VHS luego MN
 Absceso 0-100 PMN 20-200 Normal

o Imágenes: En ocasiones es necesario TAC antes de la PL para descartar HIC.


• Tratamiento Esterilizar el LCR con ATB o Estabilizar al paciente (ABC)

32
2
Temas clave de Pediatría y Cirugía Infantil o
Sospecha de MBA: ATB empírico
• Lactantes y pre escolares: Cefotaxima o ceftriaxona + Vancomicina (por alta
resistencia del neumococo a PNC y porque también afecta al meningococo) o
Neumococo: 14 días o Meningococo: 7 días
• Escolares y adolescentes: Ceftriaxona o cedotaxima + vancomicina
• Neonatos y lactantes < de 3 meses: Agregar ampicilina para listeria. ▪
Agregar corticoides en sospecha de neumococo
• Dexametasona (0.6 -0.8 mg/kg/día) cada 12/8 horas x 2 días o Iniciado junto a ATB
disminuye incidencia de pérdida de audición y defectos neurológicos.
 Hospitalizar UTI
 Profilaxis antibiótica de contactos
• Meningococo: o Ciprofloxacino 500 mg x 1 vez o Ceftriaxona 250 mg IM en embarazadas y lactantes o
Rifampicina 20 mg/kg/día cada 12 horas x 2 días
o Meningitis viral: Tratamiento de soporte, reposo, no requiere hospitalización.
 PL puede aliviar la cefalea ▪ Hospitalizar (sala)
• CEG o signos de encefalitis
• Requerimientos de hidratación o analgesia EV
• Inmunocomprometidos
• < 1 año
• Diagnostico dudoso que requiere ATB empírico
• Complicaciones
o SIADH: Monitorear diuresis y administración de líquidos o Derrames subdurales:
neumococo
 Derrames son estériles y asintomáticos, no requieren drenaje
a menos que haya HIC o Considerar repetir PL tras 48 horas si no hay
mejoría con el tratamiento o en aquellos que hayan recibido corticoides.
• Pronóstico
o Mortalidad de meningitis bacteriana es significativa
 25% para neumococo
 15% para meningococo o 35% tendrá secuelas (principalmente por neumococo)
 Sordera, convulsiones, dificultades de aprendizaje, ceguera, paresia, ataxia, hidrocefalia. o Mal
pronóstico
 Menor edad, duración prolongada, enfermedad previa, convulsiones, coma, shock, leucopenia
en LCR o bacterias visibles al gran, inmunocomprometidos.
o Recidiva: poco probable. De ocurrir, ocurre 3-14 días después del tratamiento por focos parameningeos o
gérmenes resistentes.
o Recurrencia: indicaría defecto subyacente anatómico o inmunológico

Encefalitis
 Definición
Proceso inflamatorio del parénquima con disfunción cerebral, por lo general agudo, que puede ser difuso o localizado
 Etiopatogenia o Dos mecanismos
1. Infección directa del parénquima
2. Respuesta inmune en el SNC varios días después de las manifestaciones extraneurales de la infección
o Agentes
 Virus son la principal causa: Enterovirus, arbovirus, VHS y VIH
• Los más frecuentes: Enterovirus, parechovirus, VHS, VEB y
Adenovirus.
• VIH causa encefalitis subaguda en niños y adolescentes que se
presenta como síndrome febril agudo de comienzo insidioso. ▪ Clínica o Pródromo de síntomas
inespecíficos: faringitis, fiebre, cefalea, molestias abdominales o Aparición de síntomas
característicos: letargo progresivo, cambios conductuales y defectos neurológicos.

32
3
Temas clave de Pediatría y Cirugía Infantil o
o Convulsiones (frecuentemente) o Exantema maculopapular o
Complicaciones graves: coma fulminante, mielitis transversa, neuropatía
periférica.
 Diagnóstico
o Clínica: Signos y síntomas característicos o Exámenes
 LCR con patrón viral
• Pleocitosis linfocítica, aumento proteínas y glucosa normal.
 EEG Confirma el diagnóstico: Actividad difusa de ondas lentas. o Diagnóstico etiológico
 Serología
 Biopsia cerebral: si hay signos focales en encefalopatía grave refractaria (encefalitis rábica y enfermedad
de Creutzfeld Jakob), arbovirus, micóticas y vasculopatías (no infecciosa)
 Tratamiento NO hay terapia específica o Tratamiento sintomático en UCI para manejar convulsión y
detectar anomalías HE, proteger la VA y reducir la presión IC. o Antivirales
 VHS: Aciclovir EV
 CMV: Ganciclovir
 VIH: TARV
 Pronóstico o Síntomas remiten en 2 o 3 semanas
o 2/3 se recupera por completo, el resto queda con residuos clínicos como paresia,
espasticidad, afectación cognitiva, debilidad, ataxia y convulsiones recurrentes
o La mortalidad global es del 5% o VHS tiene peor pronóstico, sobre todo en < de 1 año.

Convulsión febril
• Introducción
Son las convulsiones más frecuentes en la infancia o Ocurren el el 2 – 5% de los niños entre 6 meses y 5 años o Ocurre en ausencia de infección
intracraneal, alteración metabolica, historia de convulsión afebril.
o Ocurren antes o poco después del inicio de la fiebre, incrementándose con la temperatura del niñi y no con el
índice de aumento de la temperatura
• Clasificación o Crisis febriles simples (65-90%)
▪ Duración < 15 minutos, generalizadas, un episodio en 24 horas, sin patología neurológica previa
o Crisis debriles complejas
▪ Duración de más de 15 minutos, focales, recurrente en 24 horas
• Factores de riesgo o Retraso del DSM o Infecciones virales o Historia familiar de convulsiones febriles o
Convulciones o Algunas vacunas o Deficiencias de hierro y zinc
• Etiología o Las infecciones virales son una causa común de fiebre que desata convulsiones febriles ▪
Mayor riesgo en VHS 6 (se adquiere en los primeros 2 años de vida) ▪ Otros: Influenza,
ADV, parainfluenza.
• Evaluación o Generalmente se presentan a urgencias cuando ya se ha resuelto la convulsión o La evaluación
inicial se enfoca en determinar el foco de la fiebre
 Preguntar antecedentes familiares de convulsiones febriles, epilepsia, inmunizaciones, ATB de uso
reciente, duración de la convulsión, fase post ictal prolongada y signos focales.
 Examinar signos meníngeos y nivel de conciencia del niño o Los exámenes de laboratorio en crisis febriles
simples no se recomiendan por que las alteraciones
ELP e infecciones graves son poco frecuentes o PL
 Es una opción en niños de 6 – 12 meses cuya inmunización para Haemophilus y neumococo esté
incompleta o que hayan usado antibiótico previamente
 Se recomienda fuertemente en signos menigneos o e aquellos con hallazgos sugerentes de infección
intracraneal
o EEG no predice recurrencia de crisis febriles simples
o Neuroimágenes de rutina tampoco se recomiendan en crisis febriles simples por no tener valor pronóstico, incluso
luego de una primera convulsion compleja no sería de ayuda en niños de buen aspecto.
o EEG y neuroimagenes deben considerarse en niños con anormalidades neurológicas en el examen y aquellos con
crisis febriles recurrentes
• Factores de riesgo de recurrencia de crisis febriles o > 18 añis o Duración de la fiebre < 1 hora antes del inicio
de la convulsión o Temperatura < 40 ºC

32
4
Temas clave de Pediatría y Cirugía Infantil o
• Tratamiento agudo
Como la mayoría de las convulsiones febriles se resuelven antes de la presentación, los médicos deben estar preparados para tratar los
estatus epilépticos o Con vía venosa
 Lorazepam 01 mg/kg/dosis
 Diazepam 0.3 mg/kg/dosis o Sin via venosa
 Diazepam 2 – 10 mg VR
 Lorazepam 0.2 mg/kg VR
 Midazolam 0.25 mg/kg/dosis VO o nasal
• Vía oral es más efectivo que VR e igual que el Diazepam EV
• Si no cede la crisis en 5 minutos o Repetir BZP (máximo 2 dosis)
• Si no cede con 2da dosis de BZP a los 15 minutos o Fenitoína carga 20/mg/dosis EV con monitorización
cardíaca a 1 mg/kg/min
(lento) o Fenobarbital carga 20 mg/kg/dosis
 No tiene igual recomendación que fenitoína por sus efectos adversos a
nivel de conciencia
 De elección cuando no hay vía venosa disponible, monitor, usuario crónico
de fenitoína o portador de patología CV
o Tercera línea Ac valproico, levetiracetam
 Solo en UCI, por lo que hay que
trasladarlo en este punto
 Cuando hay refractariedad •
Pronóstico o Niños no tienen secuelas neurológicas a partir de una crisis febril o La
mortalidad es muy rara o Hay un leve aumento en la mortalidad durante los dos años luego
de una convulsión febril compleja o Padres deben ser advertidos que esta patología
recurren frecuentemente o La recurrencia es similar en crisis simples y complejas
• Prevención o El uso continuo de fenobarbital y acido valproico es efectivo en reducir la recurrencia, pero en
el costo/beneficio, por los efectos adversos, no están indicados.
o El uso intermitente de antipiréticos o anticonvulsivantes en el inicio de la fiebre no se recomienda
 No han demostrado reducción en la recurrencia de las convulsiones simples o El uso
intermitente de diazepam al inicio de la fiebre es efectivo en reducir la recurrencia de las convulsiones febriles
simples, pero la AAP no la recomienda. Sin embargo, si hay ansiedad parental, se puede considerar dar al inicio de
la fiebre.
o La administración rectal de diazepam para abortar la convulsión en casa se debe considerar en convulsiones
prolongadas y en aquellos con alto riesgo de recurrencia.
o Estos pacientes tienen un ligero mayor riesgo de epilepsia (2%), que se incrementa en convulsiones febriles
complejas
 Factores de riesgo de epilepsia futura después de una convulsión febril
• Convulsión febril compleja
• Historia familiar de epilepsia
• Duración de la fiebre < 1 hora antes del inicio de la convulsión
• Anormalidad del neurodesarrollo (hidrocefalia, paralisis cerebral)

32
5
Temas clave de Pediatría y Cirugía Infantil
• Recomendaciones prácticas o Cuando hay un foco febril claro, los exámenes de laboratorio, EEG
y Neuroimagen no se recomiendan en crisis febriles simples.
o EL uso a largo plazo de antiepilépticos intermitentes o continuos después de una primera convulsion febril no se
recomienda
o El uso de antipiréticos al inicio de la fiebre no es efectivo para reducir la recurrencia de convulsiones febriles simples.

32
6
Temas clave de Pediatría y Cirugía Infantil
Vacunas
• Generalidades o Inmunidad: Protección de un individuo contra enfermedades infecciosas a
través de 2 mecanismos ▪ Pasiva: Transplacentaria por Ig
 Activa: Vacunas o Factores que influyen en la respuesta inmune a una vacuna
 Vacuna: Tipo, cantidad de antígeno, adyuvante y la vía de administración
 Huésped: Edad, estado nutricional, enfermedades coexistentes (estado inmune) y presencia de
anticuerpos maternos
o Reactogenicidad: Son los efectos colaterales, que pueden ser
 Locales: Dolor, eritema, induración
 Sistémicos: fiebre, cefalea, molestias GI. o Seguridad: Implica que
una vacuna tenga una reactogenicidad aceptable, sin eventos adversos serios o
Inmunogenicidad: A la capacidad de inducir la respuesta inmune o Estabilidad:
La capacidad de mantención de los componentes activos durante el transporte
y almacenaje
o Eficacia: Capacidad de proteger contra la enfermedad o Efectividad: Eficacia en una
población determinada o
Eficiencia: Implica la Vacuna Composición
relación costo-beneficio. Tuberculosis Bacteria atenuada
o Booster: Aumento de (Mycobacterium bovis)
eficacia con cada dosis de Polio Virus atenuado (Polio 1, 2 y
refuerzo 3)
o Efecto rebaño: Vacunación Difteria Proteica (Toxoide diftérico)
de la población protege a Tétanos Proteica (Toxoide tetánico)
los nos vacunados por B. Pertussis Lactantes: Inactivada
ausencia de la enfermedad (Bacteria completa)
• Clasificación Escolares: Inactivada
▪ Por tipo (Fracciones antigénicas)
• Vivas atenuadas: Tienen mayor Hib Inactivada (PRP de Hib
reactogenicidad conjuado con Toxoide
tetánico o diftérico)
• Inactivadas: Presentan menor Neumocócica conjugada (10 Conjugada (13 serotipos de
inmunogenicidad o Fraccionada o
v) neumococo)
antígeno purificado o MO completo
Hepatitis B Inactivada (Ag de superficie
muerto o inactivado ▪ Por vía
de administración recombinante)
Sarampión, rubéola, Virus vivos atenuados
• Oral parotiditis (trisvírica)
• Parenteral Meningocócica (ACWY) Conjugada
• Mucosa VPH VLP
 Según su uso en la salud pública
• Sistémicas: Vacunas PAI
• No sistémicas: Que se usan en brotes, grupos de riesgo, viajeros (Ej. Vacuna contra
Influenza A) • Vacunas vivas atenuadas o
Generalidades
 Portan el agente vivo que condiciona una respuesta inmune sólida y duradera
 Pueden generar infección de individuos no vacunados al excretar el virus
 Podría generarse reversión a cepa purulenta
 Interfiere con los anticuerpos circulantes
o Vacunas
▪ Trisvírica: Sarampión, rubéola, parotiditis
• Anti-rubéola
o Efectos adversos: Fiebre, adenopatías, artralgia y artritis transitoria o
Precauciones: No administrar a embarazadas o Contraindicaciones:
Inmunodeprimidos, alergia a vacuna, neomicina o kanamicina.
• Anti-sarampión o Generalidades: Se puede usar hasta 72 horas post exposición o Inmunogenicidad y eficacia: 95% a los 12 meses y 98%
a los 18 meses. Con la segunda dosis sube a 99%, con una eficacia del 90-95%.
o Efectos adversos: Fiebre, exantema, trombocitopenia transitoria o
Contraindicaciones:

32
7
Temas clave de Pediatría y Cirugía Infantil
 Reacción anafiláctica a dosis previa, huevo, neomicina o gelatina
 Inmunosuprimido grave: leucemia, linfoma, VIH avanzado, radio o
quimioterapia, corticoides altas dosis.
o Precauciones
▪ Esperar 3 meses post vacunación para embarazarse (por riesgo teórico de daño fetal)
• Anti-parotiditis o Generalidades: existen 3 cepas
 Norteamérica
 Suiza
 Japón o Eficacia: Mayor con cepa norteamericana o Efectos
adversos: (raro) fiebre, parotiditis, orquitis (puede generar infertilidad) y pancreatitis
 Anti-poliomielitis
• Generalidades: o Incluye 3 serotipos de polio: VP 1 y 3 (el
2012 se eliminó el 2 por riesgo de polio paralítica)
o Se excreta el virus por deposiciones por lo que es importante el efecto rebaño.
• Contraindicaciones:
o Embarazas
o Inmunodeprimidos y sus contactos ▪ Anti-varicela
• Generalidades o Indicada desde los 12 meses
 1 dosis en < 12 años
 2 dosis en > 12 años (separadas por 4 semanas) o
Duración hasta 20 años
• Inmunofenicidad o En niños sanos 95% o En adolescentes 78-
99%
• Efectos adversos
o Abscesos en el lugar de la vacuna o Exantema leve y fiebre (2% desarrolla la enfermedad)
• Contraindicaciones o Inmunodeficiencia grave o VIH
o Leucemia linfoblástica aguda o Quimioterapia o Embarazo o Enfermedad aguda con fiebre o
Alergia a la neomicina y a la gelatina o Tratamiento crónico con salicilatos ▪ Anti-rotavirus •
Generalidades o Dos tipos
 Rotarix: Monovalente 2 dosis
separadas de 1 mes
 Rotalec: Prentavalente o Protege hasta
2 años
• Inmunogenicidad: Buena a los 2 meses
• Eficacia o 70% en todo tipo de diarreas o 85% en diarreas
asociadas a hospitalizaciones o 100% en cuadros graves
• Efectos adversos o Leves molestias GI
• Contraindicaciones o Anafilaxia previa, alergia al látex (en la
monovalente)
• Precaución: Inmunosuprimidos, historia de invaginación
intestinal, enfermedad gastrointestinal crónica
 BCG
• Generalidades o Vacuna contra TBC meníngea o miliar producida por Mycobacterium bovis.
o Protege contra bacteremias graves
o Genera reacción cutánea en el deltoides (sitio de vacunación)
• Eficacia: Variable (50-85%)
• Contraindicaciones o Inmunosuprimidos o Embarazadas o RNPT < 2.000 gr
• Efectos colaterales o Adenitis o Osteítis por BCG (sitio de vacuna es el deltoides,
cercano al húmero) o Supuración local y fístula • Vacunas inactivadas o
Generalidades
 No pueden replicarse (sin riesgo en inmunodeprimidos)
 NO son tan efectivas como las vacunas vivas
 Hay mínima interferencia con anticuerpos circulantes

32
8
Temas clave de Pediatría y Cirugía Infantil
 Respuesta es principalmente humoral
 Anticuerpos caen con el tiempo
 Requieren 3 – 5 dosis y adyuvantes.
• Adyuvante Sustancia que ayuda a estimular una respuesta inmune más intensa requiriendo menos antígeno.
o Mecanismo de acción
 Presentación de antígeno
 Localización del antígeno al sitio de inoculación ▪ Estimulación directa de la respuesta inmune
o Tipos
• Enteras: Polio EV, rabia, hepatitis A, Influenza, B. pertussis completa.
• Fraccionadas: Influenza, hepatitis B, VPH, B. pertussis acelular ▪ Anti-
pertussis
• Generalidades o Se administra combinada con el toxoide diftérico y tetánico (DPT)
o 2 tipos
 Células enteras (bacteria inactivada) DPT
 Fraccionada (pertussis acelular) DPTa (PAI)
• Es menos reactogénica
• Previene las complicaciones
• Inmunogenicidad y eficacia o 74-98%
• Efectos adversos o Locales: eritema, edema, induración o Generales:
fiebre, decaimiento, irritabilidad. Muy reactogénica en > 7 años
(NO DAR) o Graves: Hipertermia, llanto pesistente, convulsión febril, síndrome hipotonía-hiporespuesta, ecefalopatía.
• Contraindicaciones absolutas o Reacción anafiláctica previa o
Encefalopatía 7 días pre vacuna o Trastorno neurológico progresivo ▪
Anti-hepatitis A
• Generalidades o El control de infección se logra a través de la vacuna y
de mejores condiciones sanitarias
o Dura hasta 10 años
• Inmunogenicidad o 1 dosis: 90-95% o 2 dosis: 99%
• Eficacia: 95-100%
• Efectos adversos: No tiene
• Contraindicaciones o Alergia al aluminio, hipersensibilidad previa a la vacuna
▪ Anti-hepatitis B
• Generalidades o Primera vacuna que reduce la frecuencia de cáncer
(hepatocarcinoma) o Contiene el antígeno de superficie que se inocula IM o
Población de alto riesgo de Hepatitis B
 Hijo de madre y parejas sexuales de HBsAg positivo
 Heterosexuales con más de 1 pareja en 6 meses o con antecedentes de ETS
 Homosexuales activos o varones bisexuales
 Adictos a drogas EV
 Contacto de portador de HBsAg
 Trabajadores de la salud
 ERC EV en hemodiálisis o peritoneodiálisis
 Receptor de productos sanguíneos
 DHC
 Infección por VIH
• Inmunogenicidad o 3 dosis (genera anticuerpos protectores)
 Población pediátrica: 95%
• Eficacia: 90-95%
• Efectos adversos: Escasos ▪ Anti-VPH
• Generalidades o 2 tipos
 Gardasil

32
9
Temas clave de Pediatría y Cirugía Infantil
• Cuadrivalente 16 y 18 (CaCU), 6 y 11 (Condilomas)
• Adyuvante Aluminio
 Cervarix
• Bivalente 16 y 18
o Las vacunadas deben seguir con su screening habitual
• Inmunogenicidad o 99%
o < 16 años desarrollan mas anticuerpos que las mujeres adultas
• Eficacia o Elevada en prevenir infecciones persistentes y enfermedades no
previamente infectadas por dicho genotipo
• Efectos adversos o Locales y fiebre
• Contraindicaciones o Anafilaxia o Embarazadas o Inmunosuprimidos ▪ Anti-
influenza
• Generalidades o La vacuna es la forma más efectiva para el control de la
infección o La vacuna cambia anualmente debido a la variabilidad genética del
virus o 2 tipos
 Inactivas parenteral es la que ocupa el MINSAL
 Viva atenuada de uso nasal o Se elabora con 3 cepas 2
tipos A y 1 tipo B o Vacunación programada en individuos de riesgo
 Mayores de 50-65 años
 Niños > 6 meses
 Niños portadores de patologías crónicas
 Embarazadas 2 o 3 trimestre
 Personal de salud
 Personas en contacto con grupo de alto riesgo
• Inmunogenicidad: Es menor en niños pequeños e inmunodeficientes
• Eficacia: Variable
o 70-90%
o Lactantes 6 meses a 2 años: 57-84%
• Efectos adversos (poco frecuentes) o Síndrome de Guillain Barré o Lactantes
Convulsiones febriles
• Contraindicaciones o Diferir en cuadros febriles, anafilaxia o angioedema al
huevo o al pollo. • Vacunas conjugadas y polisacáridas o
Generalidades
 Polisacáridas: Respuesta T independiente
 Conjugada: Respuesta T dependiente o Vacunas
 Anti-neumocócica conjugada
• Generalidades o Vacuna heptavalente
 Disminuye enfermedad por neumococo resistente
 Efecto rebaño: Por disminución de la portación nasofaríngea de los lactantes inmunizados
o Tipos
 Prevenar 13
• Es la que se ocupa actualmente en el PNI (tridecavalente)
• Inmunogenicidad y eficacia o > 2 años: 90%
o No induce anticuerpos protectores en menores de 2 años
• Efectos adversos: Baja tasa
• Contraindicaciones: Anafilaxia a dosis previas ▪ Anti-neumocócica polisacárida
• Generalidades o Vacuna polivalente o Contiene timerosal
• Indicaciones o Niños mayores de 2 años o Asplenia o Síndrome nefrótico o IRC
o Anemia de células falciformes o
Inmunosupresión o Defectos con pérdida
de LCR o VIH

33
0
Temas clave de Pediatría y Cirugía Infantil
o Otitis recurrentes o Crónicos CV, DM,
cirrosis o Mieloma múltiple, linfoma,
leucemia. ▪ Anti-meningocócica
• Generalidades o Hay tanto conjugadas (tetravalente A,C,Y,W135 y monovalente C)
como polisacáridos (A, C, Y, W135 y monovalente C)
o Fracaso se debe a predominio de
serotipos B del cual no hay vacuna o
Duración: NO más de 4 años
(polisacárida)
• Indicación tetravalente o > 2 años de alto riesgo (asplenia, deficiencia del
complemento) o Viajeros a zonas endémicas o Brotes y epidemias por serotipos
inlcuidos
• Inmunogenicidad y eficacia o Mala inmunogenicidad en menores de 2 años, excepto
componente A que es inmunogénica desde los 3 meses.
• Efectos adversos: o Síntomas locales, fiebre, irritabilidad, cefalea y fatiga (más
frecuente con vacuna conjugada) • Vacunas proteicas o Vacuna toxoide tetánico
 Inmunogenicidad y eficacia
• Protección del 80% 1 dosis; 2 dosis sube a 90%; 3 dosis a 99%
• 5 dosis duran para toda la vida.
 Efectos adversos
• Se pone en el muslo por riesgo a osteítis en el deltoides. ▪ Indicaciones
• Vacunación desconocida o menor de 3 dosis
• 3 o más dosis y booster > 10 años
o Vacuna toxoide diftérico
▪ Generalidades
• Dura 10 años
• Entre los 2 meses – 7 años se dan 5 dosis
• En > 7 años se dan cada 10 años
• Datos prácticos o Lapsos entre vacunas

33
1
Temas clave de Pediatría y Cirugía Infantil

Interferencia: vacunas vivas con la misma via de administración (esperar 4 semanas) ▪ Sin interferencia: vacunas inactivadas,
fraccionadas, cirosomales.
o Timerosal vs autismo
 El timerosal
contiene casi 50%
de etilmercurio
 Es un preservante
para evitar
contaminación con
hongos y bacterias
 NO hay asociación
con trastorno
autista.
 Las vacunas en chile
presentan bajos
niveles de
etinilmercurio, que
no sobrepasan las
normas
internacionales
estrictas para
metilmercurio (que
sí es tóxico)

Hepatitis viral
• Etiología o Seis virus primarios A, B, C,
D, E, G o El VHD es un virus defectuoso que requiere coinfección
con VHB o sobreinfección en portadores crónicos HBsAg
o VHB, VHC y VHD Hepatitis crónica
• Epidiemiología o VHA es el más frecuente (la mitad de los casos) o
Factores de riesgo para VHB y VHC
 Drogas IV
 Exposición a productos hematológicos
 Transmisión perinatal de infección materna o VHB y VHC causan infección crónica que
puede llevar a cirrosis y hepatocarcinoma • Etiología
VHA VHB VHC
Transmisión Fecal – oral Transfusión, sexual, perinatal Parenteral, transfusión,
perinatal

Período de incubación 15- 30 días 60-180 días 30-60 días


Marcadores séricos Anti VHA HBsAg, HBcAg, HBeAg, antiHBs, Anti VHC (IgG, IgM)
antiHBc

• Clínica Superposición de los cursos clínicos de hepatitis A, B y C.


o Fase preictérica
 Dura 1 semana
 Cefalea, anorexia, malestar general, dolor abdominal, náuseas, vómitos (precede a enfermedad
detectable)
 Lactantes con VHB Urticaria, artritis antes de ictericia o Fase ictérica
 Ictericia
 Hepatomegalia dolorosa

33
2
Temas clave de Pediatría y Cirugía Infantil

 Aumento de las enzimas hepáticas 15 – 20 veces
• Laboratorio o GPT y GOT elevadas Inflamación parenquimatosa o
FA y bili total y directa Colestasis (daño hepatocelular y de los
conductos biliares)
o Tiempo de protrombina predictor de lesión hepática grave y progresión a insuficiencia hepática fulminante
• Diagnóstico o Confirmación serológica
 IgM anti VHA + IgG anti VHA bajos o ausentes
 HBsAg Infección aguda o crónica por VHB
 HBeAg Infección aguda por VHB
 HBeAg y HBsAg constante en ausencia de antiHBe Riesgo alto de transmisión (replicación viral
continuada)
 Desaparición HBsAg y emergencia de antiHBs Período ventana
 antiHBs Inmunidad de por vida
 antiHBc Infección por VHB en fase ventana
 antiHBe grado bajo de capacidad infecciosa en estado portador
ARNVHS infección activa (positivo 3 días después de la inoculación.
• Tratamiento o Sintomático
 Reposo, hidratación, nutrición adecuada o Hospitalizar a
 Vómitos intensos y deshidratación
 TP prolongado
 Signos de encefalopatía hepática o Prevenir contagio a contactos íntimos
 VHA Medidas higiénicas o VHB Interferón alfa 2 b o lamivudina o VHC Interferón alfa o ribavirina o
Contagio de VHB por transmisión vertical o al principio de la vida da lugar a infección crónica donde el interferón no es
efectivo
• Complicaciones o Hepatitis fulminante Encefalopatía, hemorragia
GI por várices esofágicas o coagulopatía, ictericia
(mas mortalidad)
 La mayoría de las hepatitis víricas se resuelven sin tratamiento
 Menos del 0.1% progresa a necrosis hepática fulminante o 90% de VHB perinatal infección
persistente
 Persistencia de HBsAg > 6 meses
 No presentan indicios clínicos de hepatits activa (al menos que haya sobreinfección con
VHD)
o 85% VHC Infecciones crónicas
 Niveles fluctuantes de transaminasas
 20% llega a cirrosis
 25% carcinoma hepatocelular
• Prevención o VHA:
 Buenas prácticas higiénicas
 Vacuna o VHB-VHC: Cribado de donantes de sangre.
 VHB: Vacuna a todos los lactantes y adolescentes hasta los 18 años
 Cribado de HBsAg antenantal a todas las embarazadas

Infección urinaria
• Introducción o El estudio de un síndrome febril sin foco, aún en ausencia de síntomas urinarios, debe incluir
siempre sedimento de orina y urocultivo
o La muestra en niños sin control de esfínter debe obtenerse por cateterismo uretral o punción suprapúbica
• Definición o ITU se define como la presencia de síntomas y signos sugerentes asociado a invasión y
multiplicación en la vía urinaria de organismos patógenos (bacterias) de la región perineal (vía ascendente)

33
3
Temas clave de Pediatría y Cirugía Infantil

o Otras vías menos frecuentes: hematógena, directa (cirugías urológicas, traumas abdominales)
• Epidemiología o Primera causa de síndrome febril sin foco, de origen bacteriano, en menores de 3 años.
o Predomina en varones hasta los 3 meses de vida, posteriormente en niñas o Recurrencia en mujeres es muy
frecuente (30%) o En varones está circunscrita solo al primer año de vida
• Etiología o Uropatógenos de origen intestinal
 E. coli 90%
 Klebsiella, Proteus, Enterobacter, Enterococo y Pseudomonas 10% • Se asocian a
malformaciones de la vía urinaria, vejiga neurogénica e instrumentalización urológica
 SGB RN
 S. saprophyticus Niñas sexualmente activas
• Clínica o Síntomas
• Desde bacteriuria asintomática hasta urosepsis • A menor edad los síntomas son más
inespecíficos ▪ RN y lactantes:
• Signos de enfermedad sistémica (rechazo alimentario, vómitos, dolor abdominal, diarrea, irritabilidad, peso
estacionario, ictericia) + fiebre.
• Neonatos generalmente tienen aspecto séptico ▪ Pre escolares, escolares y adolescentes
• Síntomas urinarios: disuria, poliaquiuria, urgencia, tenesmo, enuresis, alteraciones del color y olor de la
orina
• Dolor en fosa lumbar, fiebre y vómitos PNA
• ITU atípica: Evolución tórpida (síntomas que sugieren alteraciones anatómicas) o Chorro débil o Masa
abdominal o vesical o Aumento de creatinina o Sepsis
o Falla de respuesta a ATB o infección por germen no E. coli
o Factores de riesgo de primo infección y recurrencia
 ITU previa
 Fiebre sin foco
 Diagnóstico antenatal de anomalía rectal
 Antecedente familiar de RVU o enfermedad renal
Constipación
 Disfunción miccional
 Mal desarrollo pondoestatural o Examen físico Evaluar
 Deshidratación
 Alteración del crecimiento pondoestatural
 Masa abdominal y/o globo vesical
 Signos de disrrafia (nevos, fositas pilonidales, hemangiomas, desviación del pliegue interglúteo) que den
cuenta de vejiga neurogénica
 Presencia de epi o hipospadias, fimosis, balanitis; vulvitis, vaginitis, sinequia de labios menores
• Diagnóstico Sospecha clínica y laboratorio. Confirmar con urocultivo.
o Laboratorio
▪ Urocultivo
• Requiere mínimo 18 horas pero confirma el diagnóstico
• En < 2 años que no controlan esfínter tomar muestra por sondeo o punción
(urocultivo (+) por bolsa no se considera positivo)
• En > 2 años se toma orina de segundo chorro
• Tomar siempre antes de iniciar tratamiento ATB
• Positivo cuando o > 1 ufc/ml por punción o > 10.000 ufc/ml por cateterismo
o > 100.000 ufc/ml por bolsa o segundo chorro
 Sedimento de orina
• > 5 leucocitos por campo o > 10 leucocitos por microlitro
• Nitritos positivos
• Bacterias positivas
• Tratamiento o Indicaciones de hospitalización

33
4
Temas clave de Pediatría y Cirugía Infantil

 RN y lactantes < 3 meses
 Infección febril con importante CEG a cualquier edad
 Deshidratación
 Sospecha de urosepsis SIRS + OC sugerente
 Hiperemesis que impida tratamiento VO
 Fracaso del tratamiento ambulatorio Fiebre después de 48 horas de ATB efectivo ▪
Antecedentes de malformaciones de las vías urinarias (obstructivas) o sospecha.
 Inmunodeprimidos
 Riesgo social
 ITU en adolescente embarazada o Tratamiento
 Objetivos: Evitar la diseminación y evitar las complicaciones a largo plazo ▪ Medidas
generales
• Hidratación adecuada
• Educar sobre hábitos miccionales: Micción cada 3 horas y no posponer hábito
• Educar sobre hábitos defecatorios: Evitar constipación y aseo adecuado
 Antibioterapia
• Elección o < 3 meses e inmunocomprometidos
 Hospitalizar
 Amikacina 15 mg/kg/día EV o > 3 meses que requieren hospitalización
 Cefotaxima 100-150 mg/kg/día c/6-8 horas o > 3 meses ambulatorios
 Ciprofloxacino 20-30 mg/kg/día
 Cefadroxilo 50 mg/kg/día c/ 8 – 12 horas
• Duración o ITU febril 7 – 10 días o ITU afebril 7 días o RN 10 – 14 días
• Datos clave o ATB debe adecuarse a antibiograma intentando usar el ATB de menor
espectro lo antes posible
o Aminoglucósidos restringirlos a cuando no hay otra opción y por períodos
no mayores a 72 horas por toxicidad renal y ótica
o Buena respuesta al tratamiento: Mejoría clínica a las 48 horas o Mala
respuesta al tratamiento: Mantención de los síntomas o mayor compromiso
del estado general a las 48 horas.
 Sospechar ITU atípica
 Tomar OC + urocultivo de
control + Imágenes de forma precoz o Profilaxis se indica
desde el día siguiente del termino de la terapia para reducir
las recaídas. ▪ Urocultivo de control
• Si la evolución es favorable, el UC de control se debe realizar 3 – 5 días después de
suspendido el antimicrobiano.´
• Antes solo si persiste febril después de 48-72 horas de terapia ▪ Imágenes
• ECO renal: Todos en el primer episodio
• Uretrocistografía miccional o < 1 año o ITU atípica
o Antecedentes familiares de RVU o Dilatación de las vías urinarias en ECO
renal o Cintigrama DMSA alterado
• Cintigrama renal DMSA o Fase aguda: En ITU febril e ITU dudosa (cuadro febril
sospechoso con UC o
ATB previo) o Fase tardía: Toda ITU febril entre 6-12 meses post episodio • Indicaciones al alta de urgencias o
Tratamiento ATB 7 – 10 días o Educación sobre profilaxis y tratamiento o Reforzar hábitos: Hidratación, evitar retención y
constipación o Corregir técnica de higiene

33
5
Temas clave de Pediatría y Cirugía Infantil
o Control de pediatra con UC para chequear sensibilidad o Profilaxis si el paciente < 2 años con ITU febril y hasta completar estudio, RVU G III o
ITU recurrente o disfunción vesical
• Derivar o ITU complicada
 RN
 Lactantes y niños mayores con ECO renovesical alterada o sospecha de alteración orgánica o funcional de
vejiga
 Presencia de reflejo vesicoureteral y otra malformación

33
6
Temas clave de Pediatría y Cirugía Infantil

33
7
Temas clave de Pediatría y Cirugía Infantil
Infecciones osteoarticulares
• Definición: Incluye a la OML aguda OML sub aguda y artritis séptica. o OML: Infección del tejido óseo o
A. Séptica: Infección de una articulación
• Epidemiología o Las IOA en general tienen una incidencia de aproximadamente 1/5000 – 1/10000 < 12
años afectados anualmente
o La mayoría son hombres (3:1) o Artritis séptica es mucho más frecuente (50%) que la OML o Generalmente
menores de 5 años o Localización más frecuente de AS Cadera y rodilla; OML Fémur y tibia
• Etiología o Bacterias
 S. aureus el más frecuente en todas las edades ▪ Otros:
SGB, S. pneumoniae, S. pyogenes y Salmonella.
 0 – 3 meses Bacilos gramnegativos
 Adolescentes con actividad sexual Neisseria gonhorroeae ▪
Consumo de lacteos no pasteurizados Brucella sp. ▪
Actualmente ha ganado fuerza Kingella kingae
• Cocobacilo gramnegativo
• Saprófito respiratorio
• Provoca comúnmente artritis séptica
o Vía de transmisión
 Hematógena La más importante debido a abundante irrigación sinovial
 Contiguidad Contaminación directa por heridas o cuerpos extraños
• Fisiopatología o OML Aguda
 Multiplicación bacteriana Respuesta inflamatoria Edema y pus
 Aumento de la presión intraósea en un tejido inextensible Colapso de la
microcirculación isquemia y necrosis Mayor inflamación
 Pus busca salida por canales Despega el periostio Abscesos subperiósticos Mayor
compromiso circulatorio Necrosis de distintos segmentos (colonizados, fuera del
alcance de los ATB) Secuestros Cronicidad (defecto óseo permanente) ▪
Diseminación local
• Al cartílago de crecimiento
• En la cadera, por cercanía con la cápsula articular Artritis séptica
o OML Sub aguda
 Reacción inflamatoria menor y delimitada Lesión osteolítica
 No gran aumento de la presión IO Compromiso circulatorio delimitado
 Lesión osteolítica puede afectar el cartílago de crecimiento (absceso de Brodie) o Atritis séptica
 Sinovitis aguda purulenta Aumento del líquido sinovial y de la presión intra articular
 Enzimas proteolíticas del pus atacan el cartílago hialino Degeneración Artrosis
 En RN y lactantes menores Pus destruye las epífisis cartilaginosas
 Artritis séptica del RN Luxación e isquemia de la cabeza femoral
• Clínica Triada: Dolor, fiebre, impotencia funcional
o Sospecha fundamental debido a que es oligosintomático y a la tendencia de los padres de buscar mecanismos traumatológicos erráticos ▪
Orienta
• Intervalo asintomático entre el trauma y el síntoma
• Empeoramiento progresivo del dolor
o OML aguda
 Dolor Signo más constante
• Inicio gradual (días a semanas)
• Intenso, en la metáfisis de los huesos largos
• Afecta principalmente a rodilla, hombro, cadera y muñeca (huesos de mayor crecimiento)
 Fiebre y compromiso funcional se hace constante con la evolución del cuadro
 Niños pequeños: Posiciones antiálgicas, negativa de caminar o claudicación de la marcha o
Artritis séptica
 Dolor constante
 Aumento de volumen local

33
8
Temas clave de Pediatría y Cirugía Infantil
 Articulaciones profundas (cadera y hombro)
• Dolor a la movilización pasiva
• Cadera: contractura en flexión antiálgica
o OML Sub aguda
 Dolor leve, persistente de 2 o más semanas de evolución
 Sin fiebre ni CEG
 Historia de claudicación de la marcha
• Diagnóstico diferencial
o OML
 TMT
 Otras infecciones: Celulitis, AS
 Tumores: S. Ewing
 Infartos óseos por hemoglobinopatías (raro en Chile)
o AS
 Infecciones de partes blandas y sistémicas
 Otras causas de dolor articular: Perthes, trauma, etc.
• Diagnóstico Sospecha clínica + Confirmación por laboratorio e imágenes o Laboratorio
▪ Recuento de blancos y PCR
• Repercusión sistémica. Orienta.
• Normales en OML Sub a guda
 Cultivo o Artrocentesis (AS)
 Identifica al agente causante
 Descomprime la articulación
 Análisis del líquido: GB, Gram, cultivo.
o Imágenes
▪ Radiografía simple Pedir siempre de manera inicial en toda IOA. o No es útil en la OML inicial ni en la artritis séptica, pues no muestra
cambios.
o Ayuda al diagnóstico diferencial y seguimiento
• Artritis séptica de cadera Separación entre el fémur y el cotilo por pus.
• OML aguda tendiente a la cronicidad Secuestros y defectos óseos secundarios
• OML Sub aguda (> 10 días) Lesión osteolítica en la metáfisis. ▪ Ecografía
• Artritis séptica en articulaciones profundas (cadera y hombro) Derrame articular
• OML aguda Limitada. Útil para evaluar absceso subperiósticos
 Gammagrafía ósea con Tc 99: Proceso inflamatorios en pelvis y columna (lugares de difícil acceso) Dudas
o sospecha de múltiples focos sin disponibilidad de escáner ni resonancia.
 TC
• OML Sub aguda Lesiones osteolíticas, compromiso del cartílago de crecimiento.
• OML Aguda Secuestros
• OML crónica Mejor que la RM ▪ RM: Gran utilidad
• OML aguda temprana Edema óseo y focos metafisiarios antes del pus
• Artritis séptica de cadera Pesquisa de OML aguda metafisiaria asociada
• Artritis séptica de muñeca y tarso
o Cultivo: Tomar antes del tratamiento ATB
 Tejido óseo
 Membrana sinovial
• Tratamiento o HOSPITALIZAR o Antibioterapia
▪ Iniciar ATB EV a penas se hayan tomado los cultivos (retraso determina secuelas) ▪ Considerar el germen más
frecuente S. aureus y combinar según edad.
• RN considerar bacilos gramnegativos Cefalosporina de 3ª
• Niños sin inmunización considerar Salmonella Cefalosporina de 3ª

33
9
Temas clave de Pediatría y Cirugía Infantil
• Niños mayores considerar, además del S. aureus, SGA y neumococo Cloxacilina o cefazolina
no asociada.
▪ Esquema • < 5 años o Cefazolina o Cloxacilina + ceftriaxona
• > 5 años o Cefazolina o Cloxacilina
o Tratamiento quirúrgico
▪ Control local de la infección y prevención de secuelas esqueléticas ▪ Drenaje del pus
• AS Drenaje completo de la articulación por artrotomía (evitar múltiples punciones)
• OML aguda Drenaje intraoseo o subperióstico + drenaje al exterior
• OML Sub aguda No es necesario abordaje invasivo, solo realizar pequeña ventana si está cerca
de la cortical, si no, solo drenaje por trocar de biopsia durante el procedimiento.

PATOLOGÍA ENDOCRINA

Talla baja
• Introducción o La estatura tiene una herencia multifactorial modulada por hormonas y
factores de crecimiento o Se ve influenciada por factores ambientales y un adecuado entorno
afectivo y psicosocial o El crecimiento longitudinal se va modificando en ganancia absoluta
como en velocidad en distintas etapas de la vida
• Definición o Talla baja: Longitud o estatura menor al p3 o menor a -2 DS para la edad y el sexo
• Etiología o Talla baja primaria (intrínsecas del cartílago de crecimiento)
 Síndromes definidos: Síndrome de Turner, síndrome de Noonan, Síndrome de Down, etc.
 PEG sin crecimiento compensatorio
 Displasias esqueléticas: Acondroplasia, hipocondroplasia, discondrosteosis, osteogénesis imperfecta,
mucopolisacaridosis
 Displasias con defectos en la mineralización: Raquitismo o Talla baja secundaria (alteración del cartílago
de crecimiento)
 Desnutricón
 Enfermedades sistémicas: Cardiopatía, enfermedad pulmonar crónica, enfermedad hepática, enfermedad
intestinal (malabsorción, enfermedad inflamatoria intestinal), síndrome de intestino corto, ERC, anemia
crónica
 Desórdenes del eje GH/IGF-1 y resistencia a GH ▪ Endocrinopatías
• Síndrome de Cushing
• Hipotiroidismo
• DM sin control metabólico ▪ Enfermedades metabólicas
• Metabolismo Ca/P
• Errores innatos del metabolismo (HdC, lípidos, proteínas ▪ Psicosocial: Depresión,
anorexia nerviosa
 Iatrogenias: Corticoides (local o sistémico), radio/quimioterapia o Talla baja idiopática
 Con/sin baja estatura familiar
 Con/sin maduración lenta (Retraso del crecimiento)
• Evaluación inicial o Indicadores de crecimiento: Se deben evaluar los indicadores de crecimiento que
nos permiten hacer estimación de los cambios somáticos del paciente en el tiempo ▪ Curva
de crecimiento
• < 5 años: Curvas OMS (hechas en base a lactantes alimentados con LM)
• > 5 años: Curvas NCHS (hechas en base a población norteamericana, por lo que pueden generar
subestimaciones de la talla) ▪ Medición de la estatura
• > 2 años: Infantómetro
• > 2 años: Estadiómetro de pared ▪ Talla absoluta
• Una talla menor a – 3 DS es siempre patológica

34
0
Temas clave de Pediatría y Cirugía Infantil

La mayoría de los que crecen entre el p3 y p5 son variantes normales (talla baja familiar y/o retraso constitucional) ▪ Velocidad del crecimiento
• Se expresa en cm/año
• Una VC normal es buen y precoz indicador de
salud
• Debe establecerse en período no menor a 3
meses en el lactante y a 6 meses en el niño mayor
• Considerar que en los primeros dos años se
adquiere el carril correspondiente a la carga
genética, período en el que pueden ocurrir
cambios en la curva (canalización)
• Hasta el inicio de la pubertad no es habitual
cambiar de carril por lo que siempre amerita evaluación
• Luego de los 2 años no debieran haber cambios en la curva de crecimiento
• La velocidad de crecimiento varía en las distintas etapas o Valor mínimo en la edad escolar previa
al inicio puberal o No debe ser inferior a 4 cm/año ▪ Relación Peso/Talla
• Se debe evaluar al niño en el contexto de su curva de peso o Por ejemplo si un niño comprometió
primero el peso y luego la talla orienta a enfermedad sistémica (enfermedades renales,
cardíacas, pulmonares, malabsorción)
o Si una talla baja se asocia a incremento de peso debe descartarse patología endocrina (déficit de hormona de crecimiento, hipotiroidismo,
hipercortisolimo)
• Talla diana o carga genética: o La talla final del niño tiene relación directa con la estatura de los
padres
 Niñas [(talla padre – 13 cms) + talla madre]/2
 Niños [talla padre + (talla madre + 13 cms)/2 o Rango de 1 DS para hombre
corresponde a +/- 7 cms y en mujeres +/- 5 cms
o Clasificación: Establecido el diagnóstico, se debe precisar la causa
▪ Talla baja idiopática (sin causa reconocible) ▪ Talla baja patológica
• Primarias: Trastornos del crecimiento que afecta el cartílago de crecimiento
•Secundarias: Condiciones ambientales o patología sistémica
o Anamnesis y examen físico ▪ Anamnesis:
• Antecedentes perinatales: enfermedades del embarazo, crecimiento IU, edad gestacional, peso
y talla
• Antecedentes personales: tiempo de evolución, DSM, estado nutricional, desarrollo puberal, uso
de fármaco y/o drogas, comorbilidades, actividad deportiva
• Antecedentes familiares: Talla parental, edad de desarrollo puberal de ambos padres,
menarquía de la madre, consanguinidad, enfermedades familiares genéticas. ▪ Examen
físico:
• Peso, talla, relación P/T
Proporciones corporales CC, envergadura, talla sentado, segmento superior, segmento inferior, relación SS/SI, distancia acromion
olecranon, olecranon radio. o Segmentos
 Segmento inferior: Desde la sínfisis del pubis hasta el suelo
 Segmento superior: Talla – SI

34
1
Temas clave de Pediatría y Cirugía Infantil

 Relación SS/SI
• Va declinando
con la edad: 1.7
en neonatos y 1
en > 10 años.
• Evergadura: es
más corta que la
estatura en
prepúberes y
ligeramente superior en púberes.
o Este estudio es relevante porque las patologías se caracterizan por
un crecimiento disarmónico (ej. Displasias esqueléticas)
 Acondroplasias e
hipocondroplasias:
Acortamiento rizomélico de las extremidades
 Haploinsuficiencia del gen SHOX: Acortamiento mesomélico o Talla baja
proporcionada y P/T normal o aumentada Endocrinopatías
 Déficit de la GH
 Hipotiroidismo
 Síndrome de Cushing o Talla baja proporcionada y P/T disminuido
 Mayor gasto metabólico (cardiopatía, insuficiencia hepática o renal) ▪
Malabsorción
• Examen físico segmentario debe buscar dismorfias y fenotipos que orienten a síndrome genético asociado a talla baja o Síndrome de Turner:
Cuello alado, cúbito valgo, implantación baja del cabello, coartación aórtica, malformaciones renales
o Síndrome de Noonan: Cuello corto, orientación antimongoloide de hendiduras palpebrales, estenosis pulmonar,
malformaciones torácicas y esternales o Síndrome de Silver-Rusell: Facie triangular, historia de RCIU
o Radiografía de carpo: Determinación de la edad ósea, predicción de la talla adulta
• Estudio complementario o Pacientes donde la historia y el examen físico no orienten a causa en
particular ▪ Exámenes generales
• Hemograma/PCR: anemia, infecciones subagudas, EEII
• Creatinina y ELP, Ca/P plasmático, GV, OC: ERC, acidosis tubular renal (< 3 años)
• Albúmina, glicemia, perfil hepático: Enfermedad hepática
• IgA total, anticuerpo antitransflutaminasa y antiendomisio (IgA): Enfermedad celíaca (2 – 8 %)
• Parasitológico seriado de deposiciones: malabsorción secundaria
Cariograma: Dismorfias o Considerar Síndrome de Turner en toda niña con talla baja sin causa aparente
o Varones con anomalías genitales, retraso del DSM, RCIU severo sin compensación posterior.
• Estudio del eje somatotropo: Déficit de GH o GH se secreta en peaks en el día por lo tanto se
debe solicitar determinación de niveles plasmáticos de factores de crecimiento de síntesis
hepática dependientes de la GH
 IGF 1
 IGFBP-3 (proteína transportadora de IGF 1)
• Radiografía de esqueleto: Siempre en talla baja desproporcionada o Descartar displasia
esquelética: Craneosinostosis, osteogénesis imperfecta, estrechamiento de la distancia
interpeduncular caudal de la columna (hipocondroplasia), retraso de la
osificación de huesos púbicos (condrodistrofia neonatal), deformidad de
Madelung, etc.
• Neuroimagen: RNM de cerebro en sospecha de lesión intracraneana, defectos de la línea media,
déficit de GH o hipopituitarismo • Orientación diagnóstica o Talla baja idiopática
 Sin alteraciones sistémicas, nutricionales, endocrinas o cromosómicas
 P/T normal
 Sin déficit de GH
 60-80% talla < 2 DS ▪ Incluye
• Talla baja familiar o Talla < p3 pero con crecimiento y desarrollo normal, edad ósea normal, estatura
acorde a la talla parental

34
2
Temas clave de Pediatría y Cirugía Infantil

• Retraso constitucional o Maduración física lenta o P/T normales pero velocidad de crecimiento cae
en algún momento o Edad ósea atrasada > 2DS o Inicio de pubertad tardía (90% antecedentes de
desarrollo puberal tardío en padres o familiares) o Buen pronóstico de talla final, crecen por tiempo
más prolongado ▪ La talla será menor que su talla diana
o PEG
 Definición: peso y/o talla de nacimiento < 2DS según curvas de Alarcón y
Pittaluga
 90% logrará crecimiento compensatorio con patrón de incremento en la
velocidad de crecimiento en los primeros 2 a 3 años de vida, seguido por
talla estable en la infancia, edad normal de inicio puberal y talla adulta
menor a su carga genética.
 Aquellos sin crecimiento los primeros 6 meses requieren mayor evaluación
 Aquellos < 2 DS a los 2 años requieren tratamiento con GH o
Endocrinopatías: 5%. El eje más importante es el de la GH. Otros sistemas
son cortisol, tiroides y esteroides.
 Déficit de la GH:
Congénito:
o RN con hipoglicemia, ictericia prolongada, micropene, defectos de la línea media,
criptorquídea y facie característica
o Peso y talla de nacimiento normal y velocidad de crecimiento cae a los 6 meses.
• Adquirido o Único signo puede ser la caída en la velocidad del crecimiento seguido de talla bajo
asociada a aumento de peso concomitante (signo de alarma, se espera que un paciente con
sobrepeso aumente su velocidad de crecimiento y avance en la edad ósea)
o Indagar historia de traumatismo o infección del SNC y síntomas de tumor como cefalea y alteraciones visuales. ▪ Hipotiroidismo
• Adquirido o Pacientes normales al examen físico o Caída de la talla puede ser el único signo o
Síntomas: Crecimiento de fanereos, sequedad de la piel, constipación, intolerancia al frío,
desánimo, alteraciones menstruales. ▪ Hipogonadismo
• Más frecuente en varones
• Pubertad ausente (ausencia de caracteres secundarios a los 13 años en mujeres y 14 años en
hombres)
• No presentan estirón puberal
• Pueden tener historia de micropene o criptorquídea ▪ Hipercortisolismo
• La mayoría de las veces es iatrogénico
• Clínica: Cara de luna, relleno temporal, giba dorsal, acné, hirsutismo, estrías violáceas, HTA,
hiperglicemia, talla baja asociada a obesidad centrípeta.
• Manejo o Derivación
 Talla/Edad < 3 DS
 Talla/Edad repetidos controles < 2,5 DS
 Diferencia entre carga genética y talla actual > 2 DS
 Velocidad de crecimiento < 4 cm/año
 Caída sostenida de percentiles de talla luego de 2 años de edad
 Talla baja asociada a dismorfias o desproporcionada
 PEG sin crecimiento compensatorio o Tratamiento orientado a la causa
 Puede incluir GH, andrógenos e inhibidores de la aromatasa
• GH:
o Inyectable, de uso diario y de alto costo.
o Indicaciones
 Déficit de GH
 Síndrome de Turner
 Síndrome de Prader Willi
 IRC

34
3
Temas clave de Pediatría y Cirugía Infantil
 Síndrome de Noonan
 Talla baja idiopática (T < 2,25 DS)
 PEG sin compensación después de 2 años o Objetivos: normalizar la talla en la niñez para obtener
talla adulta lo más cercano a lo normal
o Debe mantenerse hasta el final, cuando se han fusionado los cartílagos del crecimiento o la velocidad sea menor a 2 cm/año
• Andrógeno: Oxandrolona o Andrógeno poco potente, bajo costo o Incrementa la velocidad en 2-
5 cms/año o Usar en edad ósea < 11 años
• Inhibidores de la aromatasa: o Reducen estrógenos y retrasan la maduración ósea o Prolonga el
crecimiento puberal para incrementar la talla (menos efectivo que andrógenos)
o Uso prolongado lleva a deformidades vertebrales
• Zinc: Solo en caso de deficiencia de zinc (pacientes desnutridos o con malabsorción severa)

34
4
Temas clave de Pediatría y Cirugía Infantil
Obesidad
• Definición o Exceso de grasa.
o Se puede estimar con la relación P/T en < 2 años y con el IMC en > 2 años. ▪ El IMC varía con la edad y
el sexo
• Bajo peso: IMC < p5
• Normal: IMC entre p5 – p85
• Sobrepeso: IMC > p85 – p95
• Obeso: IMC > p95
• Obeso severo: IMC > 120% del p95 o IMC > 35 kg/m2 (p99)
• Epidemiología o La prevalencia de la obesidad en la infancia es mayor en países desarrollados o
La mayoría de los niños obesos se convertirán en adultos obesos, lo que está relacionado a la
edad obesidad parental y severidad de la obesidad. Esto otorga peso a las intervenciones en la
edad temprana
o La severidad de la obesidad es un importante predictor de la persistencia en la edad adulta.
o El género también es importante: El 80% de las niñas obesas persistirá vs el 30% de los niños obesos, probablemente
dado por cambios en la composición de grasa durante la pubertad.
o La obesidad en adolescentes marca mayor riesgo de persistencia.
o Algunos estudios revelan que un componente importante de la obesidad del adolescente se establece antes de los
5 años de edad
o Situación en Chile
 Menores de 6 años
• 9.6% de obesos (P/T > a 2 DS)
• 22.6% con sobrepeso (P/T entre 1 y 2 DS)
• La obesidad ha ido aumentando en la última década con tendencia a la estabilización
• La prevalencia se ha mantenido constante desde los primeros meses hasta los 3 años (cerca
del 8%). A partir de esa edad ha habido un aumento importante (casi triplicación) a los 6 años.
▪ Escolares
• El sobrepeso y la obesidad supera el 40% (tendencia creciente en los últimos años)
• La prevalencia es similar por sexo, sin mayor prevalencia en mujeres ▪ Adolescentes
• Dificil determinar la prevalencia por pocos estudios • Hay mayor prevalencia en el sexo
femenino.
• Etiología o Clasificación etiológica
 Obesidad exógena o nutricional: 95-99% de todos los caos
 Obesidad endógena o de causa orgánica: Parte de un síndrome o Factores ambientales:
• Casi todos los tipos de obesidad son influenciados por factores medioambientales como la vida
sedentaria y la ingesta calórica mayor que el gasto.
• Estrategias de intervención a esta edad son por lo general efectivas.
• Las mejores estrategias son los programas escolares que aumentan la actividad física, educación
nutricional, y la calidad de la comida servida en el colegio; así como las estrategias enfocadas en
los padres diseñadas a aumentar la actividad en niños y disminuir el uso de pantallas. ▪
Bebidas azucaradas
• Son un importante contribuyente a la obesidad
• La APA recomienda limitar el acceso a bebidas azucaradas como estrategia.
• Representan 10-15% del total de ingesta calórica
• La relación parece estar mediada en parte por factores genéticos cuantificados como una
predisposición genética
• Estudios han encontrado que la ingesta de sal en la dieta podría asociarse a aumento del
consumo de bebidas azucaradas por aumento de la sed, por lo que estrategias de restricción de
consumo de sal podrían ser útiles ▪ Televisión
• Es la influencia medioambiental mejor establecida
• Se relaciona directamente con la cantidad de tiempo gastada en televisión
• Mecanismos propuestos o Desplazamiento de la actividad física o Depresión del gasto
metabólico o Efectos adversos en la calidad de la dieta o Efectos de la televisión en el dormir
• Los efectos están mediados primariamente por cambios en la ingesta energética

34
5
Temas clave de Pediatría y Cirugía Infantil
▪ Videojuegos de ejercicio
• Inducen un pequeño a moderado aumento del gasto energético pero no tan alto como el
deporte mismo ▪ Sueño
• Hay una asociación entre acortamiento del sueño y la obesidad
• El dormir podría tener relación además con la resistencia a la insulina, independiente de su
asociación a obesidad
• El mecanismo podría incluir alteraciones en la leptina y grelina sérica, ambas hormonas
implicadas en la regulación del apetito, además de proporcionar mayor tiempo para la ingesta
de comida.
o Factores genéticos
 Juegan un rol permisivo e interaccional con los factores medioambientales
que producen la obesidad
 Se ha visto que los factores hereditarios son responsables del 40-85% de la
variación en la grasa corporal
 Riesgo de padecer obesidad en hijos de padres obesos
• 1 padre: 40%
• Ambos padres: 80%
• Alta correlación en gemelos homocigóticos
 Una variedad de síndromes específicos y defectos genéticos simples
relacionados con la obesidad infantil han sido identificados
• Niños con síndromes genéticos tienen un inicio temprano de la
enfermedad y hallazgos físicos característicos al examen físico
como dismorfias, talla baja, retraso del crecimiento o retraso
mental.
• El síndrome de Prader-Willi es el más común. Se caracteriza por
hipotonía y dificultades para la alimentación durante la infancia
(frecuentemente con fallo de medro), con hiperfagia y obesidad
que de desarrolla en la niñez con retraso del desarrollo.
o Enfermedades endocrinas
 Representan menos del 1% de los niños y adolescentes con obesidad
 Usualmente se asocian con sobrepeso u obesidad moderada más que con
obesidad severa.
 La mayoría tiene talla baja o hipogonadismo. ▪ Las principales
consideraciones son
• Exceso de cortisol: Por ejemplo en uso exógeno de corticoides o el
síndrome de Cushing)
• Hipotiroidismo
• Deficiencia de la hormona del crecimiento
o Obesidad hipotalámica
 Aparece más frecuentemente luego de tratamiento quirúrgico de
craniofaringioma
 Se asocia a panhipopituitarismo
 Regulación del apetito a nivel hipotalámico asociado a niveles de leptina
(aumento) y grelina
(disminución)
• La leptina produce disminución del apetito, aumento del gasto
energético, aumenta la angiogénesis y la oxidación de ácidos
grasos.
o Programación metabólica
 Las influencias medioambientales y nutricionales durante períodos críticos
del desarrollo pueden tener efectos permanentes en la predisposición a la
obesidad y síndrome metabólico, fenómeno llamado “programación
metabólica”.
 Puede explicar la transmisión intergeneracional de la obesidad
 Períodos críticos

34
6
Temas clave de Pediatría y Cirugía Infantil
• Gestación y primer año de vida:
o PEG, GEG o RNPT tienen mayores índices de resistencia a la insulina durante la infancia
o Hiperglicemia materna asociada a hiperinsulinismo fetal y lipogénesis
(macrosomía fetal) o Hábitos de alimentación
• Infancia y niñez temprana:
o Hay asociación entre índices de ganancia de peso durante la infancia y obesidad o
síndrome metabólico durante la niñez.
o Alta ingesta proteica durante los dos primeros años de vida podría promover la
ganancia de peso
o Fórmulas con concentraciones estándar de proteínas se asocian con mayores IMC en
escolares en comparación con fórmulas con baja concentración de proteínas y
lactancia materna.
o Rebote de adiposidad entre los 5 – 7 años
 Segunda fase de crecimiento rápido de tejido adiposo
 Elevación más precoz se asocia con mayor riesgo de obesidad posterior
• Adolescencia o Mayor vulnerabilidad en niñas (aumento de grasa corporal)
o Aumento de grasa abdominal (> en niños) > riesgo cardiovascular
 Implicancia clínica
• La intervención puede ser una útil herramienta en prevención de la obesidad
• Metas apropiadas son el control glicémico en mujeres embarazadas e índices moderados de
ganancia de peso en lactantes y niños
• La lactancia materna tiene un efecto protector en el desarrollo de obesidad
• Lactantes PEG quienes sufren el crecimiento compensatorio, se ven asociados con mejoras en
resultados del neurodesarrollo, pero también con aumento del riesgo de síndrome metabólico.
• Diagnóstico o Antropometría
 Menores de 6 años: P/T > 2 DS (OMS)
 Mayores de 6 años: IMC > p95 (NCHS)
• Complicaciones o Psicosociales
▪ Alteraciones psíquicas que llevan a aislamiento social, problemas de rendimiento, disfunción familiar e ideación suicida
• Baja autoestima, trastornos de conducta, depresión,
ansiedad, angustia, fobia social, bullying.
o Respiratorias
 Alteración de la función pulmonar (espirometría)
 Empeoramiento del asma ▪ Apneas del sueño
• Menor tono de la musculatura faríngea
• Disminución del diámetro de vía aérea superior
• Hiperinsulinemia
o Digestivas
 Litiasis biliar (50% de la colecistitis en adolescentes asociado a obesidad) ▪ Esteatosis hepática
Cirrosis
• 50% de los niños obesos sufre esteatosis hepática
• Mayor riesgo en hispanos y hombres
o Epifisiolisis o Síndrome metabólico
 Agrupamiento de diferentes factores de riesgo cardiovascular y metabólicos en un mismo individuo
 Prevalencia en niños y adolescentes obesos quintuplica a la de eutróficos 32% vs 6,4%
 No hay consenso para diagnóstico en niños y adolescentes
 Se asocia con sensibilidad insulínica y con magnitud y distribución de la obesidad ▪ Presencia de 3 de
los 5 componentes de
• Cook: PC > p90, PA > p90, TG > 110 mg/dl, HDL < 40 mg/dl o hiperglicemia de ayuno
• Ferranti: PC > p75, PA > p90, TG > 100 mg/dl, HDL < 45 mg/dl o glicemia de ayuno > 100 mg/dl
 Etiología: Se ha relacionado a una serie de citoquinas producidas por el propio tejido graso
(resistina, IL-1, IL-6 y TNF alfa) generando un estado de inflamación continuo • Tratamiento

34
7
Temas clave de Pediatría y Cirugía Infantil
o El objetivo es disminuir el peso corporal y la masa grasa, asegurando un crecimiento normal en el tiempo y modifical
la conducta alimenticia y de actividad física
o Las intervenciones más exitosas incluyen dietas bajas en calorías, educación en nutrición, modificación de la conducta
y actividad física o Dieta
 En niños no se pueden realizar grandes restricciones calóricas (generalmente 1200 – 1300 kcal)
 En adolescentes con obesidad moderada es suficiente una restricción del 30-40% de los requerimientos
 Se administra en forma equilibrada con 25-35% de grasa, 50-55% de HdC y 15-20% de proteínas
repartiéndose en 5 – 6 comidas al día
 Conviene beber abundante agua
 Pueden lograrse pérdidas de 0.5 kg por semana
 En niños muy obesos se pueden usar dietas bajas en calorías si se controlan adecuadamente (600-900
kcal/día). Es obligatorio el control del crecimiento, EKG (posibles alteraciones del ritmo y acortamiento QT)
o Actividad física
 Actividad moderada al menos 30 minutos al día (caminata, ciclismo, etc) o Terapia
conductual o Prevención
 Programa vida sana

Cetoacidosis diabética
• Introducción o CAD resulta de una deficiencia relativa o absoluta de niveles circulantes de
insulina + aumento de las hormonas de contrarregulación como catecolaminas, glucagón,
cortisol y GH
o Esto lleva a un estado catabólico por aumento de la producción de glucosa
(glucogenolisis y gluconeogénesis) y uso periférico de glucosa disminuido resultando
en hiperglicemia, hiperosmolaridad, aumento de la lipólisis y cetogénesis (cetonemia
y acidosis metabólica)
o Puede ocurrir en el debut de una DM tipo 1 o pacientes diagnosticados que no se
inyectan insulina o cursan enfermedades interrecurrentes
• Definición
o CAD
 Glicemia > 200 mg/dl
 GV: pH < 7.3 y/o HCO3 < 15
 Cetonemia o cetonuria
• Clasificación de severidad o Leve: pH < 7.3 y/o HCO3 < 15 o Moderado: pH < 7,2 y/o HCO3 < 10
o Severo: pH < 7.1 y/o HCO· < 5
• Clínica o Deshidratación, respiración de Kussmaul (rápida y profunda) o Síntomas GI: Nauseas,
vómitos, dolor abdominal (simula abdomen agudo), progresiva pérdida de conciencia, fiebre
(gatillante infeccioso)
o Laboratorio: leucocitosis con desviación izquierda, glicemia alta, acidosis metabólica,
ELP normales o bajo
• Tratamiento o Manejo inicial en urgencias Corregir deshidratación, cetoacidosis, glicemia
normal, evitar complicaciones, identificar precipitantes ▪ Evaluación clínica
• Pesar (para todos los cálculos)
• Evaluar severidad de deshidratación y/o shock

Evaluar conciencia
▪ Estudio de laboratorio inicial
• Glicemia, cetonemia, cetonuria, GV, ELP, BUN, creatinina, hematocrito, calcio, fósforo, magnesio,
HbA1c
 EKG Alteraciones ELP
 Cultivos de sangre y orina Sospecha de infección como gatillante o Medidas de soporte
adicionales
 VA, vía venosa (2, para hidratar y pasar insulina)
 Monitor cardíaco

34
8
Temas clave de Pediatría y Cirugía Infantil
 Oxígeno si hay compromiso respiratorio, shock y CAD grave
 Sonda vesical en inconsciente que no controla esfínter
 SNG en niño inconsciente o Monitoreo
 SV horarios, Glasgow, signos de edema cerebral
 HGT horario
 Glicemia venosa cada 2 – 4 horas
 Balance hídrico ▪ Exámenes
• Glicemia, ELP, GSV, Ca, P, Mg a las 2 – 4 horas, luego según evolución
• Cetonemia a las 12 horas, luego cada 24 horas
• BUN, creatinina y Hto cada 8 horas
o Hidratación y corrección HE restaurar volumen, restablecer Na, restablecer déficit de agua, mejorar filtración
glomerular, reducir glicemia y cetonas, reducir riesgos de edema cerebral ▪ En caso de shock
• Bolo SF 10-20 cc/kg en 1 – 2 horas, repetirse de ser necesario ▪
En ausencia de shock
• SF para requerimientos basales + reposición de déficit en 48 horas
(lento)
• Si se aportó volumen por shock, no se descontará de la reposición
del déficit ▪ Hiperglicemia sin acidosis ni cetosis < 700 mg/dl
• Hospitalizar en sala
• No es necesario reponer rápidamente el suero ▪ Estimar
deshidratación
• Según severidad de la diabetes o Moderada pH 71 – 7.2 5–7%o
Severa pH < 7.1 7 – 10%
 Ej. Paciente de 20 kg: SC 0.79 m2. Requerimientos basales 1500 ml/m2/dia,
deshidratación leve (aprox 5%)
 5% de 20 kilos = 1 litro a reponer en 48 horas
• 1 dia o 1500 ml/M2/día + 500 ml (50% déficit) o 1500 x 0.79 + 500 o 1185 + 500 = 1685 ml/día =
70 cc/hora
• 2 día Igual ▪ Potasio
• Habitualmente déficit total independiente de la concentración sérica
• Iniciar aporte después de la expansión de volumen a pesar de normokalemia, agregando cloruro
de potasio a 20-40 mEq/l
• Si está recibiendo volúmenes sobre 10 ml/kg/hra, comenzar con infusiones de 20 mmol/L
• En hipokalemia, el reemplazo del déficit con cloruro de potasio a la solución a 20-40 mmEq/l
• Ante hiperkalemia inicial, aportar potasio únicamente después de diuresis
 Fósforo
• Solo aportar en caso de hipofosfemia severa y debilidad muscular (P < 1mg/dl)
• 1 ml = 1.1 mEq de K y P ▪ Bicarbonato
• Solo en caso de hiperkalemia que pone en peligro la vida a 1 – 2 mm/Kg en 60 minutos
 Objetivos de la glicemia
• Comenzar aporte con SF
• Cambiar a glucosado al 5% y mantener volumen prefijado cuando glicemia < 250
• Mantener glicemias entre 150 – 200
• Si fuera necesario aumentar la glucosa (al 10%), necesita insulina, por lo que no es recomendable
disminuirle el aporte ▪ Insulina
• Objetivo Normalizar la glicemia y suprimir la lipólisis y cetogénesis
• Iniciar al diagnóstico 1 – 2 horas post iniciada reposición de líquidos
• Dosis 0.1 u/kg/h en BIC (No bolo inicial)
• Niños menores: 0.05 u/kg/h
• Objetivo, glicemia debe bajar 50 – 100 mg/dl por hora. Si desciende más rápido, aportar glucosa
intentando no bajar insulina
o Complicaciones

34
9
Temas clave de Pediatría y Cirugía Infantil
▪ Edema cerebral
• Epidemiología o Incidencia 0.9 % o
Mortalidad 24 % o Secuelas
neurológicas 26% o Muerte en CAD
60 – 90%
• Signos y síntomas o Cefalea o Baja
frecuencia cardíaca o Elevación de
la PA o Baja SaO2 o Cambio del
estado neurológico
 Somnoliencia, irritabilidad, incontinencia o Signos neurológicos específicos
 Parálisis de nervio craneal o Vómitos repetidos o Ascenso progresivo y rápido del Na sérico sugiere
pérdida de agua libre como manifestación de diabetes insípida (por disminución de flujo a la pituitaria por
herniación cerebral)
 Hipoglicemia
 Hipokalemia
 Acidosis hiperclorémica
 Inadecuada hidratación
• Factores de riesgo o Menor edad de
debut o Hipocapnia o BUN elevado
o Tratamiento agresivo con HCO3 o Persistencia de la hiponatremia a pesar de insulina o
Acidosis severa o Mayores aportes de insulina y volumen en el manejo inicial

PATOLOGÍA NEFROUROLÓGICA

Síndrome hemolítico urémico


• Definición o Entidad caracterizada por anemia hemolítica microangipática, plaquetopenia y daño
multiorgánico predominantemente del riñón, tubo digestivo y sistema nervioso.
o Cursa frecuentemente con HTA
• Epidemiología o 2.9 por cada 100.000 menores de 5 años en Chile o Brotes producto de ingestión
de carne de vacuno pobremente cocida o leche no hervida (el reservorio de enterobacterias que
provocan SHU es en el TGI de los vacunos)
o Casos esporádicos producto de ingesta de agua, jugos de fruta, vegetales y frutas
contamindas.
o Contagio es persona a persona por contacto con niños portadores sanos que
almacenan la toxina en la materia fecal por más de 7 días
• Etiología
▪ Dos formas según asocien o no diarrea
• SHU D+ o típico o Asociado a diarrea mucosanguinolenta. o E coli es el agente más frecuente o
Afecta a niños entre 4 meses y 5 años
• SHU D- reúne varios subgrupos o SHU neonatal por trastornos del metabolismo de la vitamina
B12 o SHU hereditario o SHU por anormalidades del complemento o SHU por deficiencia de
proteasa para degradar el factor von Willebrand o SHU por neumococo o SHU por medicamentos
▪ Asociaciones clínicas del SHU
• Infecciones con clara asociación clara a SHU: E. coli productoras de shigatoxina, Shigella
disenteriae tipo 1, neumococo productor de neuraminidasa.
• Infecciones bacterianas y virales con asociación circunstancial a SHU: Salmonella typhi,
Campylobacter yeyuni, Yersinia pseudotuberculosa
• Formas hereditarias
• Asociado a drogas: Ciclosporina, ACOs, cocaína, clopidogrel
• Post trasplante
• Asociado a embarazo-postparto
• Asociado a otras enfermedades: LES, SAF, leucemia.
o SHU inducido por bacterias productoras de shigatoxina (VT)

35
0
Temas clave de Pediatría y Cirugía Infantil
 Las VT son exotoxinas con efecto citopatogénico irreversible ▪ La E. coli posee dos VT: VT 1 y VT 2.
 En los seres humanos, las cepas enteropatógenas de Shigella y E coli causan colitis hemorrágica por efecto
directo sobre la microvasculatura del colon provocando un daño que permite entrada de las VT a la
circulación
 EL receptor de las VT 1 y VT 2 en la membrana celular juega un papel importante en la susceptibilidad al
SHU cuando son expuestos a VTs.
 En caso de sospecharlo en un niño con diarrea sanguinolenta se debe confirmar la presencia de E coli
productor de VT a través de coprocultivo o detección de antígeno E coli, VT libre VT por RPC o Ac
anticitotoxina.
 Se produce un aumento de IgG e IgM en la primera semana con niveles máximos a los 10-15 días desde la
infección.
o SHU atípico
 No hay diarrea, pero pueden presentar fiebre y síntomas gripales
 Se presenta de forma esporádica y puede ser muy grave
 Predomina la insuficiencia renal y las manifestaciones neurológicas o SHU por neumococo
 Se presenta en infecciones graves, sepsis o meningitos por neumococo.
 La neuraminidasa del neumococo elimina el ácido de la membrana de las células endoteliales, plaquetas y
eritrocitos, exponiéndolos a antígeno, reaccionando y provocando aglutinación, lisis de eritrocitos y daño
endotelial.
o SHU neonatal
 En asociación a trastornos del metabolismo de la vitamina B12.
 Muy grave: Cursa con acidosis metabólica refractaria, hemorragias GI, daño cerebral, pancitopenia y
muerte (antes del 5to mes)
o SHU por alteración del complemento: Mal pronóstico o SHU familiar
 Comienzo lento
 Historia familiar esclarece el diagnóstico: Es autosómica dominante, ocure en hermanos; o autosómica
recesiva, ocurre en padre o abuelo.
• Fisiopatología o Daño endotelial
 Producto de la liberación de VT con reacción inflamatoria que genera microtrombosis e inhibición de la
fibrinólisis que afectan célula endotelial susceptible
 En el intestino se asocia cion presencia de leucocitos abundantes en la luz como respuesta inflamatoria de
la lámina propia intestinal. Se producen citocinas lo que acentúa el daño endotelial con pérdida de la
función de barrera, permitiendo el paso de VT a la circulación.
 VT es tóxica porque inhibe la síntesis proteica o Plaquetas
 Plaquetopenia por consumo debido a daño endotelial protrombótico y efecto directo de la
VT que se une a receptor de plaqueta que aumenta agregabilidad
 Daño endotelial induce liberación de sustancias protrombóticas como factor vW y fibrinógeno.
 Se forman microtrombos en órganos diana (riñón, páncreas, sistema nervioso, hígado) que provocan
isquema
 Estado protrombótico precede y causa la plaquetopenia y la microangiopatía trombótica renal
o Otros factores endoteliales
 Formación de microtrombos y agregación plaquetaria está favorecida por fuerzas
mecánicas de presión y tracción en el endotelio que generan NO (vasodilatador)
 El NO al vasodilatar amplifica los fenómenos inflamatorios endoteliales.
 PMN inducen daño celular en el SHU por infiltración leucocitaria prominente en las
lesiones.
 Los procesos inflamatorios ayudan a sensibilizar el endotelio a la acción de la VT
• Clínica o Período prodrómico
▪ SHU típico
• Inicio con diarrea con sangre, cólicos, distensión y vómitos
• Puede durar 3 o 4 días y desaparecer o complicarse con infarto colónico o necrosis intestinales
parciales
• Diagnóstico diferencial: invaginación, vólvulos.

35
1
Temas clave de Pediatría y Cirugía Infantil
• En la mayoría la diarrea cede (es rara la deshidratación) y cuando parece ser gastroenteritis en
remisión se desarrolla la IRAg, alteraciones hematológicas y neurológicas, HTA y a veces daño de
páncreas, corazón e hígado.
o Manifestaciones hematológicas ▪ Anemia hemolítica
• De comienzo brusco, con repercusión del estado general (motivo de hospitalización)
• Se manifiesta con intensa palidez e ictericia
• Característica principal: Esquizocitos (sugiere fuertemente el diagnóstico)
• Hay también anisocitosis y poiquilocitosis, reticulocitosos, policromatofilia y
eritroblastos circulantes. ▪ Trombocitopenia
• Entre 10.000 y 100.000 Disminución cuantitativa y cualitativa
• Tiende a normalizarse entre los 8 y 15 dìas, seguida por trombocitosis sin repercusión
clínica
• Se manifiesta con equimosis en zonas de roce y petequias ▪ Leucocitosis con
aumeneto del % de segmentados.
o Insuficiencia renal aguda
 Todos los pacientes presentan algún grado
 La mayor parte presenta cuadro grave con oligoanuria de duración variable
 Hemograma: Ingresan con retención hidrosalina leve, a veces hipervolémicos con repercusión HD Lleva
a diálisis.
 Examen de orina: hematuria microscópica en todos y macroscópica en el 30%, proteinuria variable,
alteraciones tubulares (glucosuria e hiperproteinuria)
 BUN y Crea: Elevación rápida. La función renal se recupera lentamente presentando alteraciones HE y
acidosis metabólica grave
 ELP: Hiperkalemia es la complicación más peligrosa y causa habitual de muerte. Depende de la causa propia
de la IRAg, de la hemólisis y de las transfusiones. o HTA
 Secundaria a la sobrecarga de volumen
 Agravada por la microangiopatía
 Transitoria pero de difícil manejo o Manifestaciones neurológicas
 Varían entre ligera irritabilidad hasta cuadros convulsivos
 Raramente coma y rigidez de descerebración
 Daño atribuible a microtrombosis del SNC, pero debe descartarse previamente hipo e hipernatremia o
hiperglicemia extrema que provoque hiperosmolaridad (por el daño pancreático)
o Alteraciones pancreáticas
 Ocurre por trombosis de la microvasculatura a nivel de los islotes (la función exocrina permanece intacta)
 Las manifestaciones varían entre anormalidades en la PTGO hasta diabetes.
 Hiperglicemia es la manifestación cardinal y se agrava en diálisis peritoneal por el alto contendio de glucosa
de las soluciones que se ocupan.
o Alteraciones cardíacas
▪ Secundarias a alteraciones en
• Equilibrio de ELP
• Daño intrínseco del miocardio por trombosis de la microvasculatura
• Miocarditis por el agente infeccioso
• Diagnóstico o Es clínico o Se certifica con hallazgo de la toxina o anticuerpos específicos o
Imágenes
 Eco renal y vesical: constatar si riñones eran sanos previamente
 Ecocardiograma: compromiso cardíaco
 Ecografía de abdomen: compromiso pancreático
• Pronóstico o En el período agudo la mortalidad va entre el 2 y 4%. Bajo debido a diálisis peritoneal
o Mortalidad solamente en formas arteriales con daño renal y compromiso neurológico e
intestinal graves
o En la mayoría, la anemia hemolítica y la trombocitopenia mejoran en una o dos
semanas. La función plaquetaria se demora más.
o El 70- 75 % cura sin secuelas o Grupo pequeño evoluciona con disminución
moderada de la función renal, proteinuria e hipertensión

35
2
Temas clave de Pediatría y Cirugía Infantil
o Proporción variable desarrollará IRC o El pronóstico es mejor en SHU típico
• Tratamiento: Comprende medidas sintomásticas y medidas dirigidas a modificar la
etiopatogenia o Medidas sintomáticas
 Anemia Transfusión GR (10 ml/kg): hematocrito < 20.
 Trombocitopenia Transfusión de plaquetas (15 ml/kg): Solo si hay sangrado activo (evitar) ▪ IRAg
Aporte de líquidos: Depende del compromiso renal y diuresis
• IRAg con anuria: Basal de 1 ml/kg/día ▪ Hiperkalemia
• Dialisis peritoneal: Indicada en hiperpotasemia severa
• Meidas transitorias: o Glucosa al 10% + 1 UI de insulina/3g de glucosa o Bicarbonato (2 ml/kg)
o Gluconato de calcio (2 ml/kg): Estabilizar membrana o Salbutamol (NBZ 0.5 mg/dosis) o Resinas de intercambio (1g/kg diluida en 4 ml de
S. glucosada al 10% por gramo de droga)
 HTA Nifedipino VO (0.25-0.50 mg/kg/dosis) o Medias anti
etiopatogenia
 ATB NO ÚTILES
 Antiespasmódicos CONTRAINDICADOS o Trasplante
 No se asocia con recurrencia de SHU típico pero si atípico
• Educación sanitaria o Prevención depende de medidas sanitarias y posibilidad de educar sobre los riesgos de consumo de agua y
alimentos contaminados
o Recomendaciones
 Lavarse las manos
 Lavar la tabla de cortar alimentos
 No usar los mismos tenedores y cuchillos que se usan para cortar la carne cruda
 Lavar frutas y verduras
 Cocinar bien la carne de vaca y conservar la cadena de frio de los alimentos. ▪ Consumir agua
potable

35
3
Temas clave de Pediatría y Cirugía Infantil
Alteraciones del equilibrio acido base
• Introducción o Conceptos clave
▪ Ácidos: Moléculas que pueden liberar iones hidrógeno (H+) a una solución. EJ: HCl
• Ácidos fuertes: Disocian grandes cantidades de H+. EJ: HCl
• Ácidos débiles Liberan H+ en menores cantidades y más lentamente EJ: H2CO3 ▪ Bases:
Molécula que puede aceptar H+. EJ: HCO3-
• Base fuerte: Reacciona rápido y potente con el H+. EJ: OH-
• Base débil: Capta H+ de forma más lenta y menos potente. EJ: HCO3-
 Acidosis: Adición excesiva de H+ en los líquidos orgánicos
 Alcalosis: Extracción excesiva de H+ de los líquidos orgánicos
 pH: Unidades de una escala logarítmica para expresar la concentración de
H+ en la sangre, que nace para simplificar la expresión de valores muy
pequeños. o [H+] normal 0.00000004 Eq/litro (40 nEq/litro) o pH = log
1/[H+] = - log [H+] pH = -log [0.00000004] = 7.4
• pH arterial: 7.4
• pH orina: 4-5 – 8.0
• Fisiología o Defensas ante los cambios de H+: Tampones, riñones y pulmones.
 Tampones Sustancias capaces de unirse reversiblemente al H+. El principal
es el sistema bicarbonato, sin embargo, también son importantes el sistema
fosfato y el sistema amonio.
• Sistema bicarbonato H2CO3 (ácido débil) y HCO3 o Ecuación: CO2 + H2O ↔ H2CO3 ↔ H+ + HCO3. Catalizada por la anhidrasa carbónica,
abundante en alveolos y células epiteliales de los túbulos renales. o Explicación:
 En acidosis: EL HCO3 reacciona con el H+ formando H2CO3, que es un ácido
débil (se disocia poco en H+), por lo que la reacción tiende hacia la formación
de CO2 y H2O. El exceso de CO2 estimula la ventilación para su eliminación
del LEC.
 En alcalosis: El H2CO3 reacciona con la base fuerte formando más
HCO3, favoreciendo la combinación de CO2 + H2O para sustituir al H2CO3 utilizado. Los niveles de CO2 bajan inhibiendo la ventilación para reducir
la eliminación. A su vez se incrementa la excreción renal de HCO3.
o Ecuación de Henderson Hasselbach Permite calcular el pH si se conoce la concentración molar de bicarbonato y la PCO2
 Para un ácido, su concentración en relación a sus iones disociados
(constante de disociación) es: K’ = H+ x HCO3 / H2CO3
 Por tanto, en una solución de H2CO3, la cantidad de H+ libres es:
H+ = K’ x H2CO3 / HCO3
 Como no se puede medir el H2CO3 no disociado (porque se disocia
rápidamente) se estima en base al CO2, ya que este es directamente
proporcional, obteniendo: H+ = K x CO2 /HCO3 (donde K es 1/400 de
K’ debido a la relación de CO2/H2CO3)
 Los laboratorios miden CO2 disuelto en PCO2 en relación a su coeficiente de
solubilidad (0.03 mmol/mmHg), obteniendo:
H+ = K x (0.03 x PCO2) / HCO3
 Así como la concentración de H+ se puede expresar en pH, la constante de
disociación se puede expresar en pK pK = - log K. Si la ecuación se aplica el
logaritmo negativo se obtiene: -log H+ = -log K – log (0.03 x PCO2) / HCO3,
que en consecuencia es: pH = pK – log
(0.03 x PCO2) HCO3 y aplicando ley de logaritmos pH = pK + log
(HCO3 / (0.03 x PCO2)), además, siendo pK para el bicarbonato de
6.1, la ecuación final es: pH = 6.1 + log (HCO3/(0.03 x PCO2))
• Sistema fosfato y sistema amonio.
o Se utilizan para excretar los H+ de ácidos no volátiles (que no se pueden excretar por los pulmones) y además permiten generar HCO3 nuevos.
 Pulmones
• Un aumento de la PCO2 en el LEC determina una reducción del pH
(por menos formación de H2CO3) y una disminución de la PCO2 lo
contrario, un aumento del pH.
• Un incremento en la ventilación provoca disminución de la PCO2
por aumento en la eliminación de CO2 y por tanto sube el pH; y

35
4
Temas clave de Pediatría y Cirugía Infantil
una disminución de la ventilación provoca aumento de la PCO2 por
retención de CO2 y por tanto baja el pH.
 Riñones: Controlan a través de la excreción de orina ácida o básica.
• Mecanismo global Secreción de H+, reabsorción de HCO3 y
producción de nuevo HCO3.
o Hacia los túbulos se filtran grandes cantidades de
HCO3, su paso a la orina determina pérdida de bases.
o Las células epiteliales tubulares secretan grandes
cantidades de H+ hacia la luz, lo que supone su
extracción desde la sangre, pérdida de ácidos. o El
organismo regula el equilibrio mediante la secreción
de H+, titulando los niveles de HCO3, dejando que
pasen o no a la orina para su eliminación.
 En alcalosis, los riñones dejan de reabsorber todo el bicarbonato filtrado,
aumentando su secreción por la orina.
 En acidosis, los riñones reabsorben todo el bicarbonato y además producen
bicarbonato nuevo, reduciendo la [H+] en el LEC.

 Cada vez que las células epiteliales forman un H+, forman también un HCO3,
el cual devuelven a la sangre. La secreción de H+ y el índice de filtración del
HCO3 son casi iguales, por lo que se dice que se titulan entre sí.
 La secreción de H+ es algo mayor que la de HCO3 (80 mEq/litro), debido a
los ácidos no volátiles del metabolismo, por lo que terminan excretándose a
nivel distal hacia la orina, donde son tamponados por el fosfato y el amonio,
formando sales. En este sistema el H+ se une al fosfato y al amonio, sin
formar H2CO3 (al no unirse al bicarbonato), por lo que el HCO3 que se forme
en el intracelular será un bicarbonato nuevo.
 La concentración de HCO3 está regulada por los riñones y la PCO2 por la
frecuencia respiratoria. Cuando uno de estos dos mecanismos falla, hay un
desequilibrio ácido-base.
• Trastornos AB metabólicos: Cuando las alteraciones son por cambio en la concentración
de HCO3 o Una disminución primaria de HCO3 generará una acidosis metabólica o Un
aumento primario de HCO3 generará una alcalosis metabólica
• Trastornos AB respiratorios: Cuando las alteraciones son por cambios en la PCO2 o Un
aumento de la PCO2 va a generar acidosis respiratoria o Una disminución de la PCO2 va a
generar una alcalosis respiratoria.
• Trastornos mixtos: Cuando los trastornos no van acompañados de las respuestas
compensadoras adecuadas o Ejemplo: Un paciente con pH bajo tendrá acidosis. Si es
metabólica, se espera una baja de HCO3 y una disminución de la PCO2 para compensar. Si
se ve un aumento de la PCO2, se esperaría que además hay un componente respiratorio
contribuyendo a la acidosis, como en una pérdida aguda de HCO3 gastrointestinal que
asocia un enfisema y acidosis respiratoria subyacente. • Compensaciones o Según la
ecuación de H-H, cada vez que varíe el numerador, el denominador debe variar en la
misma dirección para mantener el cociente que refleja la concentración de H+.

35
5
Temas clave de Pediatría y Cirugía Infantil
o Estas son compensaciones parciales que nunca llevan el pH a cifras normales, son predecibles y por tanto permiten establecer si son trastornos
puros o mixtos.

• En una acidosis metabólica, cada vez que el HCO3 caiga a 10 mEq/l (que disminuya en 14 mEq), la
PCO2 esperable será de 23,3 ± 3 mmHg (40 – (14 x 1.2)).
• PCO2 < 20 mmHg Acidosis metabólica + alcalosis respiratoria
• PCO2 > 26 mmHg Compensación respiratoria insuficiente
• PCO2 > 40 mmHg Acidosis metabólica + acidosis respiratoria (mixta).

• Interpretación de exámenes pH N: 7.40; PCO2 N: 40 mmHg; HCO3 N: 24 mEq/l o pH Trastorno primario: Acidosis o
alcalosis o HCO3 y PCO2 Alteración metabólica o respiratoria o Anion GAP plasmático
 Acidosis metabólica por pérdida de HCO3 (AG normal disminuido o hiperclorémicas)
• Extrarrenal o renal.
 Acidosis metabólica por ganancia de H+ (AG aumentado o normoclorémicas)
• Acidosis láctica o cetoacidosis diabética (endógeno); Intoxicación por OH (exógeno)
o GAP urinario (Na u + K u) – Cl u Evaluar origen renal de la acidosis metabólica
 Expresa la capacidad de acidificación distal (midiendo de manera indirecta la excreción de amonio, ya que
se elimina como cloruro de amonio).
 En acidosis metabólica debe ser negativo. Un Gap de 0 o positivo indica alteración de la producción de
amonio.
o Delta del Anion GAP (En acidosis metabólica con AG aumentado) Acidosis por pérdida de HCO3 agregada (EJ:
acidosis láctica + acidosis por pérdidas digestivas) o Acidosis metabólica previa ▪ Delta AG / delta del HCO3 =
(AG – 12) / (24 – HCO3) VN: 1 - 2
• < 1 Pérdida de HCO3 agregada
•> 2 Acidosis metabólica previa
o Compensación respiratoria Trastorno puro o mixto

• Etiologías
o Acidosis metabólica

35
6
Temas clave de Pediatría y Cirugía Infantil

 Clinica: Manifestaciones inespecíficas y síntomas cardiovasculares


• Hipotensión, arritmias, compromiso de conciencia, hiperkalemia, disminución
del GC, hipertensión pulmonar. ▪ Tratamiento
• Evaluar gravedad o HCO3 < 5 mmol/l Acidosis muy grave o Compensación de
PCO2
o Etiología: Acidosis láctica es más grave porque producen más H+
• Evaluar alteraciones ELP Kalemia.
o Por cada 0.1 menos de pH, K aumenta en 0.6 mmol/l. En acidosis graves, valores
normales de K indica hipokalemia subyacente que empeorará durante la corrección de
acidosis, favoreciendo fatiga muscular respiratoria. Corregir de forma simultánea
• Uso de bicarbonato o AM con AG normal: Uso restrictivo. Solo si pH > 7,20.
o AM con AG aumentado:
 Objetivo es frenar la producción de ácido. Limitar el uso de HCO3 a hiperkalemia extrema o
descensos muy marcados de pH.
 Util en acidosis extremas para ganar tiempo mientras se corrige.
 El HCO3 diana debe ser no superior a 12 mEq/L.
 Fórmula
• Déficit de HCO3 = HCO3 diana – HCO3 sérico x 0.5 (0.8) x peso corporal

35
7
Temas clave de Pediatría y Cirugía Infantil

o Alcalosis metabólica
▪ Generalidades
• Importante causa de morbimortalidad intrahospitalaria
• Se acompaña de hipokalemia, hipocloremia y contracción del VEC.
▪ Fisiopatología
• Se produce por exceso de aporte de HCO3 o por pérdida de
hidrogeniones
• Causas más frecuentes: Vómitos y diuréticos
• Para que se mantenga en el tiempo debe haber una alteración
renal de la excreción de HCO3 condicionadas por o Contracción del VCE,
hipocloremia, hiperkalemia, exceso de actividad mineralocorticoide
(hiperaldosteronismo) e insuficiencia renal grave. ▪ Diagnóstico
• Lo primero a evaluar es si hay insuficiencia renal, la volemia y los
ELP o En hipovolemia, el Cl en orina distingue entre causa por vómitos o toma
de diuréticos reciente.
o La determinación de diuréticos en orina ayuda a distinguir entre ingesta crónica de
diuréticos y tubulopatías (Sindrome de Bartter o Gitelman).
o En normo-hipervolemia con HTA, estudiar renina y aldosterona para distinguir causas
de aumento en la actividad mineralocorticoidea.

35
8
Temas clave de Pediatría y Cirugía Infantil

 Clínica
• Una AlcM severa (ph > 7,6) puede producir trastornos graves sistémicos o CV:
Vasoconstricción arteriolar, disminución del flujo coronario, arritmias.
o Respiratorio: Hipoventilación con hipercapnia e hipoxemia o Metabólico:
Glicólisis anaeróbica y acidosis láctica, hipokalemia, hipomagnasemia,
hipofosfemia.
o Neurológico: Disminución del flujo cerebral, letargia, sopor, tetania,
convulsiones.
• Aciduria paradojal: AlcM asociada a hipovolemia o hipokalemia. Los H+ no se conservan en
el plasma y el pH de la orina es ácido pese a pH plasmático elevado.
 Etiología Principales causas: Vómitos y diuréticos.
• Cl < 20 mEq/l Depleción de cloro, adecuado manejo renal, buena respuesta a volumen o
Vómitos profusos (estenosis hipertrófica del píloro) o Uso de diuréticos (tardío) o Alcalosis
de contracción o Clorhidrorrea congénita o Fibrosis quística
• Cl > 20 mEq/L Depleción de CL por pérdida renal. Resistencia a volumen o
Hiperaldosteronismo primario o Uso de diuréticos (inicial) o Hipokalemia grave o Síndrome
de Cushing o Síndrome de Bartter
 Defecto genético de canales de cloro
 Presentan hiperplasia del aparato yuxtaglomerular con niveles altos de renina y aldosterona sin HTA (por
altos niveles de PG)
o Síndrome de Gitelman Hipomagnasemia-hipokalemia familiar.
o Síndrome de Liddle Aumento de actividad de canales de Na epiteliales con aumento
de la reaborción de sodio y aumento del LEC.

35
9
Temas clave de Pediatría y Cirugía Infantil

 Tratamiento Objetivo: Corrección inicial que lleve pH > 7,55 con HCO3 > 40 mEq/L

36
0
Temas clave de Pediatría y Cirugía Infantil

o Alcalosis respiratoria
 Generalidades
• Trastorno AB más frecuente en general.
• Su detección es útil para diagnóstico precoz de enfermedades subyacentes como SIRS,
Sepsis, embolia pulmonar y enfermedades psiquiátricas. ▪ Etiología: Secundaria a
hiperventilación • Hipoxemia
• Afectación pulmonar, de receptores periféricos o cerebrales.
 Diagnóstico: Gasometría

36
1
Temas clave de Pediatría y Cirugía Infantil
 Tratamiento: Identificar y tratar la causa subyacente. En casos severos: sedación con BZP y ventilación con
sistema cerrado.

o Alcidosis respiratoria: Se clasifica en aguda o crónica.


 Diagnóstico: Considerar antecedentes y resultados de los exámenes.
• Distinguir entre hipercapnia aguda y crónica.
o Aguda: HCO3 y pH en plasma inferiores 24 – 29 mmol/L. ▪ Tratamiento: Buscar la
causa subyacente y asegurar vía respiratoria libre
• O2
• VM si mecánica ventilatoria es inadecuada.

36
2
Temas clave de Pediatría y Cirugía Infantil

Síndrome edematoso
• Introducción o Es la presencia de exceso de líquido en el espacio intersticial o Requiere que haya
retención renal de agua y sodio (primaria o secundaria)
o Se puede clasificar en generalizado o localizado, lo que orienta al diagnóstico etiológico
o Edema generalizado es de origen renal

36
3
Temas clave de Pediatría y Cirugía Infantil
• Definición o Edema: Signo causado por la acumulación anormal de líquido en el espacio
intersticial
• Epidemiología o Causa más frecuente de edema generalizado es el síndrome nefrótico ▪
Incidencia 1 – 2 x 100.000
• Etiología o Sindrome edematoso localizado
▪ Secundario a trauma, proceso inflamatorio local u obstrucción del drenaje vascular o linfático
o Síndrome edematoso generalizado
 Es de origen renal, cardíaco, inmunológico o por desnutrición severa
(kwashiorkor)
• Fisiopatología o Desbalance entre la presión hidrostática y oncótica de los espacios intravascular
e intresticial o Alteración en el sistema linfático o Debido a que el intesticio reabsorbe grandes
volúmenes, es requisito que existe reabsorción renal aumentada de sodio y agua para la
formación de edema generalizado
o Puede causarse por
 Disminución de la presión oncótica intravascular
• Hipoalbuminemia (albumina < 2 g/dL) síndrome nefrótico, cirrosis, kwashiorkor
• Aumento de la presión hidrostática IV ICC, IRA, Glomerulonefritis aguda
• Aumento de la permeabilidad vascular Hipersensibilidad, angioedema, sepsis
• Aumento de la presión oncótica intersticial Mixedema (edema sin fóvea)
• Alteración del drenaje linfático intersticial Edema localizado (generalizado si se obstruye el
conducto torácico)5 • Enfrentamiento clínico de urgencias o Anamnesis Determinar la
etiología
 Caracteristicas: localización, evolución, variabilidad en el día
 Antecedentes de enfermedades renales, cardíacas, endocrinas
 Evaluar síntomas que orienten a angioedema (riesgo vital) exposición a alérgenos, edema de mucosas,
disnea, estridor
 Sintomas que orienten a patologías renales o cardiacas orina espumosa, hematuria, oliguria, anuria,
palpitaciones, disnea, cianosis
 Antecedentes familiares de enfermedades renales o cardiopatías o Examen físico
 Determinar el origen y complicaciones
• HRA, obstrucción bronquial, derrame pleural, cardiomegalia, edema pulmonar, ascitis,
hepatomegalia, hipocratismo digital, ortopnea, soplos cardíacos, arritmia, aumento de peso
• Ya que la mayoría es de causa renal, descartando lo cardíaco, nos orientamos a síndrome
nefrótico y nefrítico o Símdrome nefrótico ▪ Definición
• Edema generalizado, inicialmente periorbitario y posteriormente se generaliza.
▪ Clínica
• Edema, proteinuria masiva (> 40 mg/m2/h o índice proteinuria/creatinuria en orina > 2)
• Hipoalbuminemia
• Hipercolesterolemia ▪ Conducta
• Hospitalizar para resolver edema y evitar complicaciones
• Gran edema, oligoanuria + hipovolemia es necesario aportar albumina EV + furosemida mejora
el efecto diurético y natriurético
▪ Etiología más frecuente
• Enfermedad por cambios mínimos
• Tratamiento en sala o Prednisona 60 mg/m2/día o Dieta hiposódica o Restricción de volumen
hasta recuperar diuresis ▪ Pronóstico
• 90% benigno a largo plazo
• Informar naturaleza crónica y gatillamiento por infecciones
o Síndrome nefrítico ▪ Clínica
• Hematuria
• Edema
• HTA
• Oliguria ▪ Etiología

36
4
Temas clave de Pediatría y Cirugía Infantil
• Glomerulonefritis aguda post estreptocócica ▪ Cuadro clínico
• Niños de 6 – 8 años, 1 – 2 semanas después de infección
amigdalina o cutánea por S. pyogenes.
• Inflamación glomerular por activación de inmunidad celular y
humoral, con formación de complejos que se depositan en el
glomérulo
• Oligura e HTA son secundarias a activación del SRAA con retención
secundaria de agua y sodio
• MC: Hematuria y/o edema
• 90% HTA
 Pronóstico favorable
 Conducta en urgencias Determinar la etiología
• Laboratorio (determinar compromiso renal, documentar infección por S. pyogenes y ver
hipocomplementemia que apoya el diagnóstico) o Perfil bioquímico + creatinina o Titulos ASO o
Perfil lipídico o Orina completa o Índice proteinuria/creatininura
o Hemograma, pCR, hemocultivo, urocultivo, Rx de tórax
• Hospitalización o Sala si el paciente está estable o UPC si HTA, oliguria, anuria o si se planifica
administrar albúmina y furosemida
o Determinar volumen intravascular para determinar terpia (aumentado o disminuido)
 Rx de
torax, ELP y PA
 SI está
disminuido, no dar diuréticos o GNA post
estreptocócica
 Hospitaliz
ar en sala si está estable
 UPC si
oligoanuria o HTA • Tratamiento o
Soporte (control de la HTA) ▪ Primera
línea
• Dieta hiposódica
• Diuréticos (furosemida)
• Restricción de volumen hasta recuperar diuresis

PATOLOGÍA HEMATOLÓGICA

Síndrome purpúrico
• Introducción o El púrpura asociado a fiebre puede evidenciar un cuadro letal
• Definición o El síndrome purpúrico es el conjunto de hemorragias cutáneas, petequias,
equimosis y hemorragias mucosas como expresión de extravasación sanguínea a partir de
capilares pequeños.
o Es una patología de la hemostasia primaria o El púrpura es una
lesión cutánea > 2 mm, palpable, rojo violácreo. o Petequia es la
lesión < 2 mm.
o Si se asocia a fiebre hay que tener en consideración las
infecciones bacterianas graves, entre ellas, la meningococcemia.
• Epidemiología o La causa más frecuente es el púrpura de Schoenlein Henoch o El púrpura
trombocitopénico más frecuente es el Púrpura trombocitopénico autoinmune (PTI) o La
enfermedad meningocócica en niños con fiebre y petequias, tiene una incidencia del 0.5 al
11%

36
5
Temas clave de Pediatría y Cirugía Infantil
• Fisiopatología o Plaquetas tienen vida media en sangre de 10 días o El síndrome purpúrico
se puede clasificar en ▪ Alteración plaquetaria
• Disminución del recuento de plaquetas (alteración cuantitativa o trombocitopenia)
• Anomalías en la función (alteración cualitativa)
 Oclusión embólica de la microvasculatura
 Lesión endotelial directa
 Malformaciones estructurales de los tejidos vasculares o Trombocitopenia
 Recuento de plaquetas < 150.000
 Manifestaciones clínicas con recuentos de aprox 50.000 ▪ Sangrado espontáneo con recuentos < 10.000
• Si hay sangrado evidente con recuento mayor Alteración cualitativa
• Mecanismos de producción (3) o Disminución de la producción en la médula o Aumento en
la destrucción: vascular (SHU) o aumento del consumo (sepsis) o Aumento del secuestro
esplénico: esplenomegalia
• Etiología o Como orientar la causa
▪ Trombocitopenia severa
• Con hepatoesplenomegalia o compromiso de otras series Leucemia, anemia aplástica
• Sin hepatoesplenomegalia o compromiso de otras series PTI, Trombocitopenia
autoinmune
▪ Trombocitopenia leve – moderada
• Con sangrado importante o Con plaqueta grande
 Síndrome de Bernard Soullier o Con
plaqueta péqueña
 Síndrome de Wisckott
• Sin sangrado importante o Trombocitopenia por drogas o Patología asociada: sepsis, CID,
VIH
o Causas de trombocitopenia
 Con destrucción plaquetaria
• Mediadas por Ac o PTI o
Infecciones o Enfermedades
inmunológicas
• Coagulopatías o CID o Sepsis o
Enterocolitis necrotizante o
Hemangioma cavernoso
• Otros o SHU o PTT o DR
 Con disminución de la producción
• Congénito o Anemia de
Fanconi o Alteraciones
metanólicas
• Adquirido o Anemia
aplástica o Leucemia o
Déficit de VB12
o Causas más comunes
 RN
• Sepsis
• Asfixia
• ECN
• Trombocitopenia aloinmune ▪ Lactantes y niños
• PTI
• Trombocitopenia por medicamentos
• Asociado a infecciones virales: enterovirus, ADV
• Asociado a infecciones bacterianas: Meningococo
• Post infecciones: VEB, varicela, rubéola
• Estudio o Exámenes generales

36
6
Temas clave de Pediatría y Cirugía Infantil
 Hemograma, PCR
 Hemocultivo
 Frotis
 Pruebas de coagulación
 Función renal
• Diagnostico diferencial según gravedad
o Sepsis: Fiebre y rash no blanqueable a la presión (petequia o púrpura) Meningococcemia o Meningococcemia: Púrpura + aspecto
tóxico + llene capilar > 2 seg o hipotensión
• Tratamiento de urgencia o Reanimación y estabilización o Sospecha de
meningococcemia
 Tomar hemocultivos e inicio de ATB EV
 PL
 Derivar a alta complejidad

Púrpura de Schonlein Henoch


• Definición o Vasculitis
más frecuente en
pediatría o Angeítis
sistémica que afecta a
vasos de piel,
articulaciones, TGI y
riñones
• Epidemiología o 2 – 1
años o > hombres
• Etiología o Poco clara o
75% tiene antecedente
de IRA previa, viral o
estreptocócica
• Patogenia o Activación
de IgA complejos
inmunes vasculitis
• Clínica o Púrpura o
Artralgias
 Oligoarticular
 Precede al púrpura o Compromiso gastrointestinal
 Dolor abdominal, vómitos
 Rectorragia
 Invaginación intestinal o Compromiso renal:
 Varios meses después del cuadro inicial
 Hematuria aislada, proteinuria, síndrome nefrótico o nefrítico.
o Piel: Maculopápulas eritematosas, petequias o equimosis extensas, palpables
 Bilateral y simétrico
 Desaparecen con reposo
• Diagnóstico CLINICO o
Laboratorio
 Hemograma
 Biopsia de piel
• Tratamiento o Manejo
sintomático
 Reposo
 Paracetamol, AINES
 Corticoides: Minimizan el cuadro clínico y disminuyen el riesgo de enfermedad renal
Trombocitopenia inmune primaria (PTI)
• Definición o Enfermedad inmune caracterizada por trombocitopenia aislada < 100.000 en ausencia de causa
subyacente

36
7
Temas clave de Pediatría y Cirugía Infantil
• Epidemiología o Incidencia de 2 – 8 x 100.000 niños al año o Edad de presentación promedio 1 – 10 años (70%) o
En 2/3 es precedida por IRA, enfermedad viral como sarampión, rubéola, parotiditis, vacuna a virus vivo.
• Fisiopatología o Producción autoanticuerpos contra antígenos plaquetarios o Paqueta cubiertas de AC es
fagocitada y degradada o Mediante células T y B se produce amplificación de respuesta inmune o AC
antiplaquetarios también afecta al megacariocito, deteriora maduración e inhiben producción
• Evolución o PTI persistente 3 – 12 meses sin remisión o PTI crónico > 12 meses de evolución o PTI severo
Sangrado severo o Respuesta completa Plaquetas > 100.000 o No respondedor Plaquetas < 30.000
• Clínica o Niño previamente sano comienzo brusco de petequias y equímosis de rápida progresión o Recuperación
espontánea en > 50% o Remisión completa en 2/3 de los casos a los 6 meses, independiente del tratamiento o 70
– 90% remisión antes de los 12 meses del diagnóstico
• Laboratorio o Hemograma trombocitopenia aislada o Mielograma Aumento de megacariocitos
▪ Se realiza ante sospecha de leucemia o aplasia por fiebre, baja de peso, dolor óseo, otra citopenia) ▪ PTI persistente o
Pruebas de coagulación NORMALES (no hacer tiempo de sangría)
• Tratamiento o Tratar solo si hay sangrado significativo o riesgo de HIC (bajo 0.9%) o Factores de riesgo
 Plaquetas < 10.000
 Trauma encefálico
 Ingesta de aspirina o Se ofrecerá tratamiento a niños con riesgo de HIC, el mínimo para obtener
un nivel homeostático seguro
o Opciones
▪ Corticoides: prednisona 2 mg/kg/día por 2 semanas, luego bajar dosis ▪ Gamaglobulina EV
• Urgencias hemorragias

PATOLOGÍA NEUROLÓGICA

TEC
• Introducción o El factor de riesgo más importante para lesión intracraneana (LIC) en un TEC leve
es sospecha de fractura de cráneo o estado mental alterado
o En el lactante además orienta a LIC el hematoma no frontal > 3cm, mecanismo severo,
antecedente de pérdida de conciencia o actuar anormal
o > 2 años, el estado mental alterado, signos de fractura de cráneo, cefalea severa,
vómitos, mecanismo severo y pérdida de conciencia, orientan a LIC
o TEC leve Típicamente reacciona y presenta Glasgow 15 – 13

Definición (TEC) o TEC: lesión física o deterioro funcional del contenido craneal por intercambio
brusco de energía cinética. Comprende cerebro, tronco hasta C1.
▪ Según gravedad se clasifican en (Glasgow) o Leves: 14 – 15 GCS 80% o
Moderados: 9 – 13 GCS 15% o Graves 3 – 8 GCS 5%
• Predictor de injuria cerebral Glasgow 13 aprox 25% sufre injuria cerebral
▪ Lesiones pueden ser
• Múltiples: fractura de cráneo, lesión cerebral difusa (concusión y daño axonal difuso)
• Localizadas: hemorragias y contusiones
o Concusión: alteración transitoria del estado mental por fuerzas biomecánicas y puede o no producir inconciencia
 Signos: confusión, amnesia
 Clasificación
• Grado I: < 15 minutos
• Grado II: > 15 minutos
• Grado IIIa: pérdida de conciencia < 1 min
• Grado IIIb: pérdida de conciencia > 1 min
• Epidemiología o Después del año, trauma es la principal causa de muerte (50% por LIC)
• Etiología o Causas más frecuentes caídas (37%), accidentes de auto (18%), atropello (17%),
bicicleta (10%)
• Estudio o TAC Controverisal
 Beneficios: reconoce LICs lo que previene morbimortalidad ▪ Contras
• Solo < 10% tiene LIC y < 1 % requiere cirugía
• Dificultades para el traslado a otras urgencias y aumento de la estadía en urgencias
• Riesgo de neoplasia a futuro 1:2500-5000

36
8
Temas clave de Pediatría y Cirugía Infantil
• Lesión intracraneana (LIC) o Factores de riesgo
 Pérdida de conciencia y/o amnesia
• Frecuente 50% de TEC leve tiene algún grado de pérdida de conciencia (baja confiabilidad a los síntomas); 50 – 70% LIC
 Vómitos aislados: Rara vez se asocia a LIC. Evaluar tomar TAC
 Cefalea: No es un buen predictor aislado, cuando se asocia a vómitos, el riesgo de LIC es mayor
 Convulsiones:
• Aislada no es predictiva.
• Tardías tienen más probabilidad de asociarse a LIC ▪ Mecanismo de injuria
• Gran intercambio de energía > probabilidad de LIC
• Caída > 3 metros, accidente vehicular o alta velocidad > probabilidad de LIC
 Edad
• Lactantes menores de 2 años tienen poca signología Observar
• Hematoma de cuero cabelludo + fractura 30% riesgo de LIC
 Desorden hemtológico Poca evidencia o Riesgo muy bajo
 Pacientes con síntomas y signos aislados: cefalea, vómito, pérdida de conciencia.
 Lactantes > 3 meses solo con hematoma frontal o Predictores
• en < 2 años o Estado mental alterado o
Fractura o Hematoma de cuero cabelludo
no frontal o Pérdida de conciencia o
Mecanismo severo o Actuar anormal
• en > 2 años o Estado mental alterado o
Fractura de base de cráneo o Pérdida de
conciencia o Mecanismo severo o Vómitos
o Cefalea severa
 Si no hay ningún factor presente, el riesgo es muy bajo NO HACER TAC
• Recomendaciones de manejo en urgencias o
TEC leve en > 2 años
 Glasgow 14 u otro signo de estado mental alterado o signos de fractura de cráneo TAC
 Pérdida de conciencia, vómitos o mecanismo severo Observar y si empeora, TAC o TEC leve en < 2 años
 Glasgow 14 u otro signo de alteración mental (sueño, difícil despertar, agitación, difícil consuelo, lenta
respuesta, fractura palpable TAC
 Hematoma no frontal, pérdida de conciencia o mecanismo severo Observar y si empeora,
< 3 meses o preferencia paterna, TAC
• Criterios de hospitalización o En UTI
 Glasgow < 15
 Examen neurológico alterado
 Síntomas significativos
 TAC alterada
 Cuidadores no confiables o sospecha de abuso
• Criterios de alta con reconsulta S.O.S. si o Es
de día y duerme más de lo habitual o Dolor de
cabeza que no disminuye con analgésicos o
progresa en intensidad, con o sin vómitos o
Cambios en la conducta habitual o Salida de
sangre o LCR por nariz/oídos o Marcha
inestable o Signos focales

36
9
Temas clave de Pediatría y Cirugía Infantil
Compromiso de conciencia
• Introducción o Los primeros minutos son para evaluar ABC. Si hay alteración, dar oxígeno al 100%, peparar para intubar
si hay hipoventilación, instalar VV y si hay shock, dar bolos de SF
o Los signos vitales incluyen el HGT o La etiología es amplia y variada o Exámenes deben pedirse según la historia y el
examen físico y neurológico o Compromiso de conciencia con examen físico y neurológico no encasillable, sospechar
intoxicación o maltrato
• Definición o Falla en responder a los estímulos externos de manera adecuada según el nivel de desarrollo
• Fisiopatología o La conciencia es la sumatoria de la interacción entre corteza y SRA, desde donde hay proyecciones
hasta el tálamo
o Alteración de neuronas reticulotalámicas de proyección o de ambos hemisferios se traducen en compromiso de
conciencia
o El funcionamiento depende de sustratos para energía, flujo sanguíneo, ausencia de metabolitos anormales,
inexistencia de toxinas, temperatura normal, falta de excitación o irritación neuronal (convulsiones, infecciones)
• Etiología o Nemotecnia AEIOU TIPS
A Alcohol, alteración AB, arritmias T Trauma
E Encefalopatía, ELP, endocrinopat I Infección
I Insulina, intususcepción P Psicógeno, poisoning (intoxicación)
O Opioides S Seizures (convulsiones)
U Uremia
o Causas potencialmente mortales
 Hematoma epidural
 Edema cerebral
 Tumores
 Infartos
 Meningitis, encefalitis
 Tóxicos
 Hipotensión, hipoxia, sepsis
 Obstrucción de shunt de derivación
• Clínica o Alteración de conciencia cualitativa (estado mental activado)
 Alucinaciones
 Ilusiones
 Delirio o Alteración de conciencia cuantitativa (estado mental deprimido)
 Letargia
 Obnubilacaión enlentecido
 Estupor: Falta de respuesta o nula movilidad espontánea
 Coma: inconciencia mantenida, profunda, de > 1 hora de evolución
• Evaluación en urgencias
o ABCDE
o Historia: antecedentes de trauma, convulsiones, medicamentos, fiebre, vómitos, tránsito intestinal alterado,
maltrato, drogas
o Examen físico: Aspecto general, hidratación, color, exantema, equimosis o Evaluación de conciencia
 Glasgow
 Signos vitales o Examen neurológico
 Nivel de funcionamiento del SNC
• Lesiones supratentoriales Déficit motor, disfunción hemisférica, reflejo pupilar conservado,
tronco conservado
• Lesiones infratentoriales Rápido compromiso de conciencia, disfunción de tronco, alteración
PC, pupilas asimétricas, patrones respiratorios anómalos, sin progresión cráneo caudal
• Tóxicas y metabólicas Compromiso de conciencia precede a signos motores, déficit motor
simétrico, pupilas reactivas salvo opiáceos, barbitúricos y anticolinérgicos ▪ Patrón de
respiración
• Cheyne-Stokes Apneas e hiperpneas alternadas por compromiso bilateral o diencefálico

37
0
Temas clave de Pediatría y Cirugía Infantil
• Hiperventilación Acidosis metabólica, hipoxia, intoxicación, edema pulmonar neurogénico (CO,
organofosforados)
• Apneurístico Pausa después de completa inspiración por compromiso de puente o médula
• Hipoventilación OH, narcóticos, sedantes (SRA)
 Tamaño y reactividad pupilar El sistema simpático y parasimpático no se afecta por causas metabólicas
• Ausencia de respuesta a la luz Causa estructural
• Asimetría pupilar Causa orgánica
• Pupilas dilatadas o Unilateral: hematoma, tumor, herniación, post ictal, lesión del III par o
Bilateral: Post ictal, hipotermia, hipoxia, encefalitis, shock, barbitúricos, atropínicos.
• Pupilas contraídas o Fijas: lesión pontina o metabólica o Reactiva: lesión medular y metabólica o
Medianas y fijas: herniación o Miosis puntiforme: opiáceos ▪ Respuesta motora
• Decorticación (brazos flectados) Lesiones tracto córtico espinal sobre el núcleo rojo
• Descerebración (brazos extendidos) Lesiones en el tracto vestíbulo espinal, toxinas
• Opistótono Lesión cortical bilateral grave
o Laboratorio
 Inmediato
• Glicemia, HGT
• Hemograma, PCR
• GV, lactato
• ELP
• BUN, crea
• Perfil hepático
• Hemocultivo
• Toxicológico (orina y sangre)
• Niveles de antiepilépticos
• Otros: Ca, P, Mg ▪ Mediatos
• PL: citoquímico, cultivo (infección, HSA)
• TAC cerebral: TEC grave, tumor, infecciones con signos focales
• RNM: TEC cerrados con TAC normal y HSA con TAC normal
• EEG: ECPE
• Tratamiento o ABC
o Hidratación parenteral con S. glucosado + ELP o Tratamiento de HIC
 Cabeza 30º
 Suero hipertónico 3% 0.1 ml/kg o manitol 0.25-1 g/Kg
 Sedación, analgesia, intubación y paralización
 ATB si infección
 Corregir el balance HE
 Mantener temperatura normal
 Disminuir agitación BZP

Intoxicaciones
• Introducción o En Chile el centro de referencia es el CITUC o Las consultas suelen ser precoces (dentro de las 2 primeras
horas tras contacto con el tóxico5) lo que facilita el manejo
o La principal vía de exposición es la digestiva5
▪ AAPCC: 83.9% exposición oral, 7.25% cutánea, 5.35% inhalación y 4.5% contacto ocular.5
• Etiología o Sustancias más ingeridas son analgésicos, cosméticos, productos de cuidado personal y productos de limpieza
o Las intoxicaciones mortales son por: analgésicos, antihistamínicos, sedantes/hipnóticos y humos/gases/vapores
• Epidemiología o Variable determinada por factores económicos, culturales, geográficos y legales según cada país.

37
1
Temas clave de Pediatría y Cirugía Infantil
 España: 0.3% de las consultas en Urgencias de pediatría5 o 4 grupos: Menores de 5 años,
adolescentes, fines homicidas y en contexto de maltrato o La mayoría de las ingestiones son involuntarias
(67%) destacando ATD y BZD 1 o 40% afecta a niños menores de 5 años1 o Las voluntarias son más frecuentes
desde los 13 años
 Considerar el uso de drogas ilícitas
 Predominio del sexo femenino5 o En Chile4
 Las sustancias más involucradas son los medicamentos, donde los más importantes son
los del SNC y respiratorios, seguido de productos del hogar (cáusticos y cosméticos). Más atrás el etanol y
el CO5.
• En otros países con más casuística (España) destaca Paracetamol (por ingesta de presentación de adultos o mala
dosificación de solución)5; en 2do lugar los psicofármacos y en 3er lugar los antitusivos.
 La exposición no intencional en todos los rangos etarios alcanza un 78.6%
 La mayor cantidad de llamadas a CITUC corresponde a menores de 5 años en Chile (50%) •
Factores que influyen es el fácil acceso que este grupo tiene a sustancias tóxicas, el almacenaje
inadecuado, trasvasije a envases atractivos y ausencia de legislación que exija envases resistentes a niños
para sustancias tóxicas, especialmente medicamentos.
• Urgencias o Enfrentamiento inicial (incluso antes de la anamnesis)
 ABCD
 Énfasis en el estado de conciencia, ritmo cardíaco, trabajo respiratorio, protección de la VA
(en caso de usar carbón activado), intubar si hay falla respiratoria y haya riesgo de aspiración.
 Monitorización cardíaca, saturación de O2, compromiso de conciencia.
 Hemoglucotest o Historia clínica
 Determinar el peso: permitirá identificar la concentración del tóxico y establecer los riesgos.
 Determinar sustancia ingerida
 Determinar cantidad de la sustancia ingerida
 Olor en el niño en sospecha de solventes
 Tiempo entre ingesta y consulta
 Número de afectados en casa: Sospecha de ingestión de CO
 Síntomas o Examen físico
 Identificación de los signos clásicos de los síndromes tóxicos: Permite adecuado tratamiento
• Clínica o Sospechar en aparición de síntomas inexplicados: alteración del estado mental, crisis comiciales, compromiso
cardiovascular, anomalías metabólicas.
o Síndrome tóxico Signos y síntomas específicos de un grupo de fármacos1, 2 o
Síndromes tóxicos9
▪ Síndrome anticolinérgico
• Generalidades o Los anticolinérgicos actúan inhibiendo los receptores muscarínicos, los cuales
están asociados al SN parasimpático que inerva el ojo, el corazón, el sistema respiratorio, la
piel, el TGI y la vejiga; y al simpático a través de las glándulas sudoríparas.
• Clínica Anhidrosis, midriasis, rubicundez, hipertermia y delirium. o SNC: Irritabilidad,
alteraciones del habla, confusión, agitación, ataxia, coma. o Delirium: Contemplar el vacío,
intervalo lúcido alternada con alucinaciones. o Midriasis: fotofobia (parálisis del músculo ciliar)
o CV: Taquicardia, elevación moderada de la PA, rubicundez (descenso del tono precapilar)
o GI: Disminución del peristaltismo (RHA disminuidos), náuseas, vómitos,
sequedad de mucosa oral
o Vejiga: Retención urinaria (contribuye a la agitación) o Hipertermia (por
anhidrosis) ▪ Síndrome colinérgico
• Generalidades o Los colinérgicos activan los receptores muscarínicos de acetilcolina e
indirectamente, a través de la acetilcolina, a los receptores nicotínicos.
o Actúan como inhibidores de la acetilcolinesterasa (AChE), resultando en una
hiperestimulación por acetilcolina.
o Se activan tanto receptores muscarínicos como nicotínicos afectando el sistema
simpático y la placa neuro-muscular.
• Clínica: DUMBBELS o SLUDGE (Defecación, urinaria incontinencia, miosis, broncorrea,
broncoconstricción, emesis, lacrimación excesiva y sialorrea; Sialorrea, lacrimación, urinaria
incontinencia, defecación, gastrointestinal disfunción y emesis) o Respiratorio: Rinorrea
profusa, sialorrea, broncorrea, tos y sibilancias por broncoconstricción.

37
2
Temas clave de Pediatría y Cirugía Infantil
o CV: Bradiarritmias, hipotensión o Ocular: Lacrimación, miosis, visión borrosa.
o Glándulas sudoríparas: Diaforesis o TGI: Nauseas, vómitos, dolor tipo cólico,
tenesmo, defecación incontrolable
(Aumento del peristaltismo y relajación del tono del esfínter anal)
o Vejiga: Incontinencia urinaria (Estimulación del detrusor y relajación del trígono)
o Placa neuromuscular: Fasciculaciones que progresan a parálisis flácida. ▪
Síndrome opioide
• Generalidades o El síndrome opiode es más frecuente de lo que se piensa. En Chile, solo durante
2014 y 2015, el CITUC recibió 275 consultas por ingesta de tramadol en niños10
o Los opioides ejercen su efecto uniéndose a 3 tipos de receptores generando diferentes respuestas clínicas
• Clínica:
o Síndrome clásico Miosis + depresión respiratoria y del SNC
 GI: RHA disminuidos (disminución de la motilidad)
 Depresión respiratoria y SNC; Daño cerebral hipóxico, neumonía aspirativa,
rabdomiolisis.
o Síndrome atípico
 Tramadol: Convulsiones. Síndrome serotoninérgico si se combina con
agonistas 5HT.
 Metadona: Prolongación del intervalo QT ▪ Síndrome
simpaticomimético
• Generalidades o Norepinefrina es un NT de las fibras simpáticas postganglionares
(adrenérgicas) que inervan piel, ojos, corazón, pulmones, TGI, glándulas exocrinas y algunos
tractos neuronales del SNC
o La respuesta depende del tipo de receptor pudiendo ser excitatoria o inhibitoria.
• Clínica o Excitación del SNC: Agitación, ansiedad, delirios y paranoia.
o Otros: Taquicardia, convulsiones, hipertensión, midriasis, hipertermia, diaforesis,
arritmias y coma. ▪ Síndrome serotoninérgico
• Generalidades o Se produce por exceso de serotonina tanto en receptores centrales como
periféricos
o Puede ser por uso terapéutico de 5HT o por combinación de fármacos
• Clínica o Central: Hipertermia, alteraciones del estado mental o Periférico: rigidez,
hiperreflexia y clonus (afectación neuro-muscular)
Principales síndromes tóxicos1,2,8
Síndrome Manifestaciones Fuente
Anticolinérgico “Niño seco” Hiposecreción de glándulas exocrinas, piel Alcaloides, hongos silvestres,
eritematosa, pupilas dilatadas, hipertermia, antihistamínicos, ATD tricícliclos
retención urinaria, delirio, alucinaciones,
taquicardia, insuficiencia respiratoria

Colinérgico “Niño mojado” Hipersecreción de glándulas Insecticidas, organofosforados,


exocrinas, incontinencia urinaria, síntomas tabaco, mordedura de Loxoceles
gástricos, fasciculaciones musculares,
broncoespasmo, convulsiones, coma

Extrapiramidal Temblor, rigidez, opistótono, tortícolis, Fenotiazidas, haloperidol, metclopramida


disfonía, crisis oculorrígida

Hipermetabólico Fiebre, taquicardia, hiperpnea, agitación, Salicilatos, herbicidas


convulsiones y acidosis metabólica

Narcótico Depresión del SNC, hipotermia, hipotensión, ATD, opioides


hipoventilación, miosis

37
3
Temas clave de Pediatría y Cirugía Infantil
Simpaticomimético Excitación, psicosis, convulsiones, Anfetaminas, cocaína, pasta base,
hipertensión, taquipnea, hipertermia, pupilas cafeína, teofilina, drogas estimulantes
dilatadas

Serotoninérgico Agitación, cambios del estado mental Sobredosis ISRS, combinación IMAO + ISRS,
(confusión, hipomanía), mioclonía, temblor, éxtasis.
hiperreflexia, ataxia, diarrea, fiebre

Deprivativo Cólico abdominal, diarrea, lagrimeo, Estados de deprivación de drogas y alcohol


sudoración, alucinaciones, agitación

Paracetamol Náuseas, vómitos, palidez, ictericia,


insuficiencia hepática tardía

Monóxido de carbono Cefalea, mareos, coma, afectación de varios


sistemas

Hierro Vómitos (sanguinolentos), diarrea,


hipotensión, insuficiencia hepática,
leucocitosis, hiperglicemia, radiopacidades en
Rx simple

• Estudio o Evaluación en urgencias


 Considerar edad y características del paciente determinará la vulnerabilidad frente al tóxico
Características generales de la población pediátrica y riesgo de intoxicación1
Menores de 5 años Adolescentes

 Grupo más numeroso  Intencionales (recreacional en


 Intoxicaciones accidentales hombres vs suicida en mujeres)
 Consulta inmediata  Ingesta fuera del hogar
 Mayor facilidad para identificar el  Consulta tardía
tóxico  Prevenibles
 Valorar
• Posible agente causal
• Maniobras realizadas que pudieran complicar la evolución (inducción de vómito)
• Estabilización previa al traslado
• Analizar posibilidad de maltrato
o Exámenes iniciales2
▪ Laboratorio: GSA, ELP, cálculo del Anion y glicemia
• Acidosis metabólica con Anion gap aumentado9 o Generalidades
 Anion GAP representa los iones no medibles. Su valor normal es entre 8 – 16
mEq/L
 Un aumento del AG acompañado de acidosis metabólica representa un
incremento de aniones endógenos (lactato) o exógenos
(salicilatos)
o Clínica MUDILES
 Metanol, uremia, diabetica cetoacidosis, isoniazida y hierro, lactato, etilenglicol y salicilatos.
 Electrocardiograma2
• Arritmias o Prolongación del intervalo QT Fenotiazinas, antihistamínicos o Ensanchamiento del QRS ATD tricíclicos, quinidina o
Bradicardia sinusal Digoxina, cianuro, colinérgico, B bloqueante.
 Cribado de orina para drogas (sospecha de ingestión medicamentosa en el hogar) ▪
Radiografía simple de abdomen
• Sustancias radiopacas: Hidrato de cloral, metales pesados, hierro,
plastilina, pastillas con cubierta entérica, amalgama dental, pilas de
reloj.
• Tratamiento1,2: Cuatro pilares Soporte, evitar la absorción, favorecer
la adsorción, Aumentar la eliminación del tóxico, Antídoto. o
Tratamiento de soporte: Pilar fundamental! o Evitar la absorción

37
4
Temas clave de Pediatría y Cirugía Infantil
 Medidas según la vía de entrada
• Oftálmica: Lavado conjuntival prolongado (20 min.) con SF 0.9%
• Pulmonar: 02 alto flujo
• Cutánea: Retirar la ropa, lavar con agua y jabón por personal protegido. Cáusticos lavar por 20
min.
• Vía oral o Lavado gástrico: Algunos autores recomiendan no realizar por norma2. No se han
encontrado diferencias con carbón activado5. Poco recomendado ya que no ha mostrado
cambiar el pronóstico clínico salvo en intoxicaciones de gran riesgo y poco tiempo para actuar
(ingesta de arsénico o drogas de acción prolongada)
 Indicación: Menos de 60 minutos de la ingestión cuando no se pueda usar carbón
activado
 Método: Proteger la VA. Instilar SF tibia 10 – 15 ml/Kg (Máx 250 ml) hasta líquido claro.
 Contraindicaciones:
• Compromiso de conciencia (aspiración)
• Riesgo de perforación de esófago
• Cirugías locales recientes
• Ácidos o cáusticos
• Riesgo de neumonitis química por vaporización de hidrocarburos
 Efectos adversos: Perforación esófago/estómago, mayor índice de aspiración e
intubación8, neumonía aspirativa, laringoespasmo, taquicardia, desaturación
o Irrigación intestinal total:
 PEG No se absorbe. Menor efecto osmótico, mantiene el balance H-E.
 Indicaciones: Medicamentos de liberación prolongada, sobredosis de fierro,
metales (no ligan carbón activado)
 Dosis: 20 ml/kg/h (máximo 2 litros/k) hasta drenaje claro o falta de evidencia
radiológica (metales)
o Favorecer la adsorción
 Dosis única de carbón activado
• Disminuye la absorción en la hora siguiente a la ingestión
• No se ha demostrado que mejore el pronóstico2
• No es efectivo contra cáusticos o corrosivos, hidrocarburos, metales pesados, glicoles y
compuestos hidrosolubles
• Requiere SNG o vía oral
• Dosis hasta los 12 años: 1 gr/kg (máx 100 gr)
• Mecanismo de acción o Adhesión directa al tóxico o Favorece el paso desde la circulación al
lumen (diálisis GI) o Bloquea la reabsorción en la CEH
o Aumentar la eliminación del tóxico
 Inducción de diuresis alcalina Ingestión de salicilato o metotrexato ▪ Dosis múltiples de carbón
activado:
• Solo si es intoxicación por gran cantidad de carbamazepina, dapsona, fenobarbital, quinina o
teofilina (Mala evidencia)
• Dosis: 0,5 g/kg cada 4 hoiras por 24-48 horas
 Diálisis: Sustancias con bajo volumen de distribución, bajo peso molecular, escasa unión a proteínas, alta
hidrosolubilidad Metanol, etilenglicol, salicilatos, litio, teofilina, bromuro, litio, acido valproico.
o Antídotos específicos
Principales antídotos de uso en pediatría1,2
Tóxico Antídoto
Paracetamol N-acetylcisteína
EV: 150 mg/kg en una hora, luego 12,5 mg/kg/h por 4 horas y luego 6,25
mg/kg/h
Enteral: 140 mg/kg, luego 70 mg/kg cada 4 horas por 17 dosis

37
5
Temas clave de Pediatría y Cirugía Infantil
Anticolinérgicos: Atropina, ATD TCC, midriáticos oculares Fisostigmina EV: 0.5 mg lento, repetir cada 5 minutos hasta conseguir efecto
(Máx 2 mg)

Rodenticidas o anticoagulantes Vitamina K EV (forma específica) o IM: 5 mg cada 6 horas con control de TP

Benzodiacepinas Flumazenil EV: 0.01 mg/kg, máx 2 mg


Betabloqueadores Glucagón EV: 0.05 mg/kg en bolo, seguido de dosis de mantención de 0.05
mg/kg

Bloqueadores de canales de calcio Glucagón EV: 0.05 mg/kg en bolo, seguido de dosis de mantención de 0.05
mg/kg

Monóxido de carbono Oxígeno 100%


Opiacios (Morfina) Naloxona EV, IM, SC: 0.01-0.1 mg/kg

o Evidencias recientes8
 No hay estudios clínicos controlados que hayan demostrado que el uso de rutina reduzca la
morbimortalidad
 Hay evidencia que sugiere que podría reducir la absorción de toxinas en seleccionadas circunstancias
 Ensayos voluntarios han encontrado que las dosis múltiples de CA disminuyen la vida media de eliminación
de ciertas drogas, incluyendo la carbamazepina y el fenobarbital, en comparación con la dosis única de CA.
 En una declaración conjunta, la AACT y la EAPCCT sugieren sólo el uso de las dosis multiples de CA en
ingestiones que amenacen la vida de carbamazepina, fenobarbital o teofilina.
 AAPX y EAPCCT en una declaración conjunta sugieren que el lavado gástrico no se debe utilizar de rutina
debido a los efectos secundarios, sin embargo, en raros casos de ingestión reciente (< 1 hora) y
potencialmente mortal, podría considerarse evaluando costo/beneficio.
 Se ha abandonado el uso del lavado gástrico debido a la efectividad del CA.
 La EAPCCT y la AACT aconsejan en contra del uso de catárticos como el sorbitol o el sulfato de magnesio
(que buscan aumentar la evacuación rectal de las toxinas) en uso aislado por alto riesgo de deshidratación.
 La combinación de un catártico y AC NO DEBE USARSE EN NIÑOS.
• Sustancias específicas9 o Monóxido de carbono
▪ Generalidades
• Fuentes comunes: Fuego, malfuncionamiento de calefactores a gas.
• Causa toxicidad sistémica desplazando el oxígeno de la hemoglobina, disminuyendo el delivery
de O2.
 Clínica: Signos y síntomas de hipoxia
o Leve: Cefalea, cansancio, gastroenteritis. o Moderado: Taquipnea, confusión moderada o Severo: Signos neurológicos, cefalea, convulsiones,
pérdida de conciencia.
 Diagnóstico: Historia de exposición, síntomas compatibles, apoyo de laboratorio.
• Historia: Más de 1 familiar con síntomas
• Síntomas: Acidosis metabólica
• Laboratorio: Carboxihemoglobina en niveles elevados (confirma el diagnóstico) ▪
Tratamiento
• Remover al paciente de la fuente de CO
• 02 alto flujo (100%) Aumenta la eliminación de CO.
o Recomendado a presión positiva en pacientes con carboxihemoglobina muy elevada.
o Salicilatos (Aspirina)
 Generalidades
• Solo el 5% de la exposición a analgésicos
• > 150 mg/kg: Toxicidad leve
• 150 – 300 mg/kg: Toxicidad moderada
• > 300 mg/kg: Toxicidad severa ▪ Clínica
• Taquipnea, náuseas, vómitos, fiebre baja, alteración del SNC (letargia; cansancio), acidosis
metabólica.

37
6
Temas clave de Pediatría y Cirugía Infantil
• Laboratorio: GSA, ELP, urea, creatinina, glucosa
o Paracetamol
 Generalidades
• Es el antipirético más usado en niñós
• Toxicidad ocurre con 150 mg/kg en < 12 años y 7.5 gr x kg en > 12 años7
• Morbilidad seria y mortalidad es rara.
• Caracterización de la población intoxicada con PCT en Chile7 (Todas las consultas al CITUC
durante 2009)

 Clínica
• Ingestión aguda: Inicialmente náuseas y vómitos. Sin tratamiento se resuelve espontáneamente
• Hepatotoxicidad se manifiesta con elevación de las enzimas hepáticas (GOT y GPT) y del tiempo
de protrombina dentro de las 24 horas desde la ingestión.
o Causada por metabolitos que se acumulan en los hepatocitos uniéndose a las enzimas intracelulares dañando la
funcionalidad de la célula.
• Rara vez evoluciona a falla hepática fulminante. ▪ Estudio
• Concentración sérica de paracetamol se puede utilizar para identificar quienes
requerirán antídoto.
o NAC actúa estimulando la producción de glutatión, depletado por el metabolito tóxico del PCT • Prevención6 o
Considerarla según el contexto
 A excepción de los gases, los niños están expuestos a lo que está disponible. La mayoría de estos productos
fueron usados previamente en las últimas 24 horas.
 Se deben fomentar las prácticas seguras en el hogar.
 Los padres deben recibir información acerca de cuáles sustancias son consideradas de alto riesgo y cuáles
no, ayudando a reconoces cuando llamar al CITUC.
 En las familias de niños hospitalizados por esta causa sean descrito altos niveles de estrés, madre que
impresiona despreocupada y bajo apoyo social.
 Intervenir en estas familias y en aquellas con múltiples episodios de intoxicaciones no intencionales.

37
7
Temas clave de Pediatría y Cirugía Infantil
1
• Resumen de enfrentamiento general

Intoxicación aguda

ABC de la reanimación

Anamnesis y Ex físico.

Sustancia conocida Sustancia desconocida

Alta certeza Dudas ¿Sd tóxico?

Manejo Llamar CITUC


Sí No

Evaluar si manejar

• Sustancias que no son tóxicas al ser ingeridas de forma aguda (aspiradas pueden causar neumonía química)3
Adhesivos Champú (Pequeñas cantidades) Polietilenglicol
Aceites minerales Detergentes de casa Pasta dental
Aceite de motor Endulzantes artificiales Play-Doh
Anticonceptivos Fósforos (Menos de 20) Sales de bromuro
Antiácidos Glicerol Sulfato de Bario
Crema de afeitar Grafito Sales de yodo
Crayones Glicoles de polietileno Silica gel (material granular que absorbe
humedad en cajas de remedios, equipos
electrónicos, zapatos, etc.)

Chicles Juguetes de baño Tinta de 1 lápiz pasta


Cosméticos Lápiz labial Tiza para pizarrón (Carbonato de calcio)

Cloro para casa, 5 a 7% Masilla de modelar Velas

Corticoides Mercurio del termómetro (si no se ingiere con Vaselina


pedazos de vidrio, ya que casi no se absorbe
con mucosa intestinal sana)

Cola fría Oxido de titanio Vitaminas para niños (Sin fierro)

37
8
Temas clave de Pediatría y Cirugía Infantil
▪ Conclusiones
o En Chile, las sustancias más involucradas en intoxicaciones son medicamentos, lo que lleva a cuestionar el fácil acceso
a algunos de ellos, principalmente el paracetamol, que es de bajo costo y de fácil adquisición en nuestro país, y
representa uno de los medicamentos más frecuentemente asociado a intoxicaciones.
o La mayoría de las intoxicaciones ocurren en menores de 5 años. Resulta importante la prevención para proteger a
este grupo etario. o Las intoxicaciones que afectan a mayores de 13 años son mayoritariamente no accidentales y
predominantemente en niñas.
o Es importante el conocimiento de los principales síndromes tóxicos que ayudarán a orientar la conducta de forma
temprana en una situación en la cual el tiempo apremia.
o La evidencia actual nos orienta al uso de dosis múltiples de CA en desmedro de dosis únicas del medicamento para
el manejo de ciertas intoxicaciones.
o El uso de catárticos debe evitarse, sobre todo en conjunto con AC, estando contraindicado su uso en niños.
o Debe considerarse la intervención en casos donde se observe una madre con altos niveles de estrés o que tenga bajo
apoyo social, además de aquellas situaciones donde haya habido múltiples episodios de intoxicaciones no
intencionales.

Bibliografía

1. Paris E., Rojas P. Capitulo 22, Intoxicaciones. Manual de Urgencias Pediátricas. Ediciones UC. Pontificia Universidad Católica de
Chile. 2015.
2. Marcdante K., Kliegman R. Capítulo 45, Intoxicaciones. Pediatría esencial Nelson 7ma edición. Editorial ElSevier Saunders. 2015
3. Paris E. Intoxicaciones, Epidemiología clínica y tratamiento, 2 ed. Ediciones Universidad Católica, 2005.
4. Mena C., Bettini M. Epidemiología de las intoxicaciones en chile: una década de registros. Rev Med Chile 2004; 132: 493-499.
5. Mintegui S. Manual de Intoxicaciones en Pediatría. 3ra edición. Grupo de Trabajo de Intoxicaciones de la Sociedad Española de
Urgencias en Pediatría. Año 2012.
6. McGuigan M. Common Culprits in Childhood Poisoning. Epidemiology, Treatment and Parental Advice for Prevention. Pediatr
Drugs 1999 Oct-Dec; 1(4); 313-324
7. Bravo V., Román M. Caracterización de la ingestión por sobredosis de paracetamol. Reporte de un centro de información
toxicológica chileno. Rev Med Chile 2012; 140: 313-318
8. Hendrickson R., Kusin S. Gastrointestinal decontamination of the poisoned patient. UpToDate, Junio 2017.
9. Holstege CP, Borek HA. Toxidromes. Crit Care Clin. 2012 Oct;28(4):479-98
10. 24 Horas.cl [Internet] 28/06/2017 Niños Intoxicados con Tramadol: CITUC advierte que “los primeros minutos son vitales”.
[Consultado el 09/07/2017] Disponible en: http://www.24horas.cl/nacional/ninosintoxicados-con-tramadol-cituc-advierte-
que-los-primeros-minutos-son-vitales-2431209#
11. Biobiochile.cl [Internet] 27/06/207 Lactante muere por sobredosis en Rengo: Madre habría suministrado medicamento
erróneo. [Consultado el 09/07/2017] Disponible en:
http://www.biobiochile.cl/noticias/nacional/region-de-ohiggins/2017/06/27/lactante-muere-porsobredosis-en-rengo-madre-habria-
suministrado-medicamento-erroneo.shtml

Síndrome convulsivo (Eventos paroxísticos de etiología epiléptica)


• Introducción
o La mayoría de los EPE empiezan en la infancia o 4% y 6% tiene crisis epiléptica antes de los 16 añis o Clínicamente, la mayoría de las crisis
dura menos de 5 minutos; las que persisten por más es poco probable que se detengan espontáneamente
• Definición o Evento paroxístico cerebral: Puede ser epiléptico o no epiléptico
 EPC no epiléptico: Compromiso funcional cerebral con recuperación brusca e inesperada (muy frecuente
en infancia y adolescencia)
 EPC epiléptico o crisis epiléptica (antes crisis convulsiva)
• Descarga paroxística, hipersincrónica de un grupo neuronal cortical que puede llevar a pérdida transitoria de la
función cerebral, motora, autonómica o mental.
• EN ocasiones la descarga no tiene clínica y es mínima, siendo evidente solo en el EEG (crisis electrográfica)
o Estatus epiléptico
 Actividad epiléptica continua de > 30 minutos o varas crisis sin recuperación que sumen > 30 minutos.
 Puede ser

37
9
Temas clave de Pediatría y Cirugía Infantil
• Con crisis motoras o Generalizado Asociado a pérdida de conciencia o Parcial
• No convulsivo: alteración de conciencia variable con actividad motora mínima o ausente •
Clasificación o Crisis epilépticas provocadas
 Ocurre en el contexto de una enfermedad aguda (fiebre, infección del SNC, trauma, encefalopatía
metabólica) que produce inestabilidad neuronal
 Muchas causas virales sin fiebre alta pueden generar una crisis epiléptica o Crisis febriles
 Es el EPCE más frecuente en pediatría. Dos definiciones
• Entre los 3 meses y 5 años asociada a fiebre, sin evidencia de infección IC
• Entre 1 mes y 5 años, en niños sin crisis neonatales ni afebriles previas
 Clasificación
• Crisis febriles simples < 15 minutos, generalizadas, únicas en ese episodio febril. • Crisis febriles
complejas > 15 minutos, focales, más de una en 24 horas ▪ Recomendaciones
• No solicitar EEG en crisis febril simple
• No usar fenobarbital o ácido valproico diario en la primera crisis (tratamiento de mantención)
• Si hay crisis prolongada, el diazepam previene el estatus
• 1/3 tendrá una segunda o tercera crisis y el 10% desarrollara epilepsia (más frecuente en las crisis
complejas)
o Crisis neonatales
 Epidemiología Incidencia de 1.5 – 3.5 por 1000 RNV ▪ Etiología Síntomas de enfermedad subyacente
• Meningitis, hipoglicemia, trauma, síncope, enfermedad metabólica
 Clínica Escueta por menor grado de mielinización del SNC
 Diagnóstico Observación y monitoreo (solo si se justifica) ▪ Principales factores de riesgo
• Prematurez
• BPN
• EHI
 Pronóstico Aumento de morbimortalidad o Crisis epiléptica no provocada ▪ Definición:
• Grupo de crisis en 24 horas sin asociación con enfermedad aguda.
• Epilepsia (recurrencia)
o 50%
o 70% si hay alteraciones en EEG, signos neurológicos en EF o causas
anatómicas (TEC) o Incidencia 45 x 100.000 • Evaluación en urgencias o Escenario 1 El más frecuente. El
paciente llega post ictal (excluir ECP no epiléptico) ▪ Anamnesis
• Determinar el número de crisis y si es primera vez que ocurre
• Clasificarla: parcial o compleja
• Identificar signos focales: aura, afasia o parálisis de Todd (paresia post ictal)
• Buscar patología previa, drogas, intoxicaciones, fiebre, DSM, antecedentes perinatales
 Examen neurológico
• Buscar disfunción del SN como paralisis de Todd
• Disfunción sensorial
• Signos meníngeos, Babinsky
• Signos de lesión cerebral pre existente ▪ Examen físico
• Fiebre
• Signos de trauma de cráneo
• Síndromes neurocutáneos, petequias, equimosis
• Buscar mordedura de la lengua
 Laboratorio si clínica sugiere crisis febril, no realizar EEG ni imágenes, estudiar como SFA
• EEG fundamental en crisis no febriles realizar en las primeras 24 horas o Normal: no
excluye el diagnóstico o Si hay signos anormales solicitar TAC de urgencia. (eventualmente RM a largo
plazo) ▪ Tratamiento
• No es necesario tratamiento antiepiléptico EV
• Indicar tratamiento rectal ambulatorio para domicilio en caso de crisis prolongada o
Diazepam rectal 0.5 mg/kg/día

38
0
Temas clave de Pediatría y Cirugía Infantil
o Escenario 2 Llega en crisis o inicia crisis en urgencias ▪
Antes de los 5 minutos de crisis
• Asegurar VA y ventilación. Dar O2
• Evaluar circulación. Monitor. Vía venosa
• HEMOGLUCOTEST
• Tomar muestra crítica de sangre y/o orina en sospecha de enfermedad metabólica
 Estatus epiléptico precoz (crisis de > 5 minutos, poco
probable que ceda espontáneamente) o Con vía venosa
 Lorazepam 01 mg/kg/dosis
 Diazepam 0.3 mg/kg/dosis o Sin via venosa
 Diazepam 2 – 10 mg VR
 Lorazepam 0.2 mg/kg VR
 Midazolam 0.25 mg/kg/dosis VO o nasal
• Vía oral es más efectivo que VR e igual que el Diazepam EV
• Si no cede la crisis en 5 minutos o Repetir BZP (máximo 2 dosis)
• Si no cede con 2da dosis de BZP a los 15 minutos o Fenitoína carga 20/mg/dosis EV con
monitorización cardíaca a 1 mg/kg/min
(lento) o Fenobarbital carga 20 mg/kg/dosis
 No tiene igual recomendación que fenitoína por sus efectos adversos a nivel
de conciencia
 De elección cuando no hay vía venosa disponible, monitor, usuario crónico
de fenitoína o portador de patología CV
o Tercera línea Ac valproico, levetiracetam
▪ Solo en UCI, por lo que hay que trasladarlo en este punto ▪ Cuando hay refractariedad

Enuresis
• Introducción o Para lograr micción voluntaria es necesario adquirir el control de la vejiga lo que supone inhibir el reflejo innato
de micción, lo que requiere maduración fisiológica y control volunatario del esfínter externo.
o Esta adquisición es normal hasta los 4 años y se desarrolla en secuencia: Control anal diurno, control anal nocturno,
control vesical diurno y, finalmente, control vesical nocturno.
• Definición
o Enuresis (DSM IV TR)
▪ Emisión repetida de orina durante el día o la noche en la cama o en la ropa (voluntaria o intencionada)
• 2 veces a la semana x 3 meses consecutivos
• Niño > 5 años
• Distintos criterios si provoca malestar clínicamente significativo o deterioro social
 Enuresis primaria: En niños que no habían tenido episodios de continencia
 Enuresis secundaria: Tras un período de continencia (6 meses – 1 año)
• Clasificación o Nocturna Solo durante el sueño (lo más frecuente) o Diurna En horas de vigilia o Mixta
• Epidemiología o Es un trastorno común o La incidencia disminuye a mayor edad y la frecuencia es mayor
en hombres (2:1) o La enuresis primaria es más común que la secundaria, habiendo incremento en la
prevalencia de la secundaria en el último tiempo
• Etiopatogenia
o Factores biológicos
 Genéticos
 Retraso de la maduración: 30% retraso del DSM en lenguaje y motor
 Disfunción vesical: capacidad vesical disminuída, iniciando antes las ganas de orinar ▪ Alteraciones del
sueño: asociado a narcolepsia y SAOS.
 Disfunción sensoperceptiva
 Estreñimiento y encopresis

38
1
Temas clave de Pediatría y Cirugía Infantil
 Alteraciones en la secreción de la ADH
 Trastornos del tracto urinario: vejiga inestable, ITUs, RVU.
 Enuresis por fármacos o Factores psicológicos y sociales
 Estrés
 Clase social: Mayor a menor nivel socioeconómico
 Trastornos emocionales y conductuales
• Evaluación clínica o Anamnesis
 Tipo de enuresis, frecuencia, síntomas acompañantes (ej. Síntomas urinarios), antecedentes de ITUs o
anomalías del sueño.
 Medidas adoptadas hasta ahora
 Evaluación de lDSM
 Evaluación afectiva o Exploración
 Examen físico y neurológico: descartar patología orgánica
 Examen mental
 Estudios complementarios: según paciente
• OC, urocultivo, radiografias de abdomen, eco renal y vesical, etc.
• Derivación o Si no responde a medidas generales o tiene comorbilidad psiquiátrica Salud mental •
Diagnóstico Según definición del DSM
• Tratamiento o Medidas generales
 Educación sobre trastorno benigno
 Reducir ingesta de líquidos antes de ir a la cama
 Levantar al niño en la noche para que orine
 Reasegurar al niño y evitar que se sienta culpable
 Observar 2 semanas con diario de autorregistros y refuerzos positivos
 Entrenamiento de retención voluntaria o Medicamentos
 Imipramina
• Dosis 25 – 75 mg (remite el 30%)
• Desmopresina 20 – 40 ug intranasal o 200-400 ug VO

CIRUGÍA INFANTIL y ONCOLOGÍA

Patología quirúrgica del RN

Atresia de esófago
• Definición: o Falta de continuidad de la luz esofágica, puede tener o no comunicación con la vía
aérea (Fístula traqueoesofágica)
o Se acompaña de alteraciones vasculares y nerviosas dando origen a alteraciones de
la peristalsis esofágica que se traduce en distintos grados de disfagia una vez
corregida la malformación.
o
Afecta el desarrollo normal desde la gestación: Altera la circulación de líquido
amniótico generando
PHA y por ende RNBPN o Los procedimientos diagnósticos y terapéuticos retrasan la adaptación normal del RN •
Epidemiología: Incidencia de 1:3500 RNV
• Embriología o Formación del esófago y la tráquea viene del divertículo ventral del intestino
anterior

38
2
Temas clave de Pediatría y Cirugía Infantil
 Porción ventral Tráquea
 Porción dorsal Esófago o Factor productor cerca de la 4ta
semana • Diagnóstico Puede ser antenatal o post natal. ▪
Diagnóstico ante natal:
• Signo de alarma PHA
• Eco prenatal Dilatación faríngea o del esófago proximal con burbuja gástrica
• Signos de alteraciones genéticas Trisomía 18
• Debe derivarse a la madre a una institución especializada antes del parto.
 Diagnóstico post natal: Manifestaciones evidentes según el tipo de atresia
• Clínica: Sialorrea persistente, dificultad respiratoria por
aspiración, distensión abdominal.
• Imágenes o Radiografía simple cérvico-tóraco-
abdominal
 Dilatación proximal del esófago que aumenta con aire a través de sonda
 Burbuja gástrica FTE o Radiografía con contraste Fístula en H o Fibrobroncoscopía
Fístula en H
• Estudio de malformaciones asociadas

• Clasificación y clínica Clasificación basada en la frecuencia de aparición (Vogt)

Tipo III Tipo I Tipo II Tipo IV


AE con FTE inferior AE sin FTE FTE sin AE AE con FTE superior e
inferior

• PHA variable • PHA • Aspiración con ingesta • • PHA variable


• Retención de secreciones • Bajo peso al nacer • Tos y ahogo Retención de
• Distensión • Retención de secreciones • Distensión abdominal Soplo • • secreciones
• abdominal Soplo • Abdomen excavado • gástrico con la Distensión abdominal
gástrico con la • Sonda no progresa respiración Afecciones • Soplo gástrico con la
• respiración Sonda • broquiopulmonares • respiración o tos
no progresa recurrentes Sonda no progresa
Neumonías aspirativas

Rx toracoabdominal AP y L con sonda radiopaca.

38
3
Temas clave de Pediatría y Cirugía Infantil
• Fondo de saco de bolsa • Fondo de saco de bolsa superior • Permeabilidad • Fondo de saco de bolsa
superior Ausencia de aire en el abdomen esofágica superior
• Aire en el abdomen • • Distensión • Aire en abdomen
gástrica • Endoscopía muy útil
• Endoscopía (ver fístula superior)
muy útil (fístula
en H)

o Asociación con otras malformaciones es de hasta el 70%.


• Cardiovasculares 35%
• Gastrointestinales 15%;
• Neurológicas y genitourinarias 5%;
• VACTERL-H (5 – 10 %): Vertebrales, anorrectales, TE, renales, extremidades, hidrocefalia
• Alteraciones cromosómicas (7%): Trisomia 21 – 13 - 18
o Repercusión sobre peso y talla Principalmente en AE tipo I por mayor PHA
o Estos pacientes requieren atención en centro con UCIN y cirugía pediátrica Derivación y transporte en el menor
tiempo posible
▪ Para transporte Control de la temperatura, sonda doble para succión de la bolsa superior, O2 y flebloclisis.
• Pronóstico (Spitz) Factores de riesgo

• Tratamiento o Medidas generales


 El mejor tratamiento es el diagnóstico temprano y
derivación a UCIN
 Régimen cero y prevención de complicaciones neumonía por aspiración
• Posición semisentado: Disminuye paso de contenido gástrico a la vía aérea
• Sonda doble lumen en bolsa esofágica superior: Evita aspiración de saliva
o Tratamiento quirúrgico No es una emergencia quirúrgica (24 – 72 horas, puede demorar más). Estabilizar y preparar para la reparación QX.
▪ Objetivo: Separar la unión de esófago y tráquea y establecer continuidad del esófago con anastomosis términoterminal entre los cabos,
intentando conservar el esófago.
• Complicaciones o Dehiscencia de la sutura esofágica Resuelve de manera espontánea en 1 – 2 semanas o RGE 40%
o Estenosis esofágica o Traqueomalasia o Recurrencia de la fístula Manejo Qx o
Lesión del nervio laríngeo recurrente (Fístula en H)

Obstrucción intestinal neonatal


• Generalidades o En el RN se produce por malformaciones
congénitas
o Sus síntomas más frecuentes son los vómitos, la distensión abdominal y el retraso en la evacuación de meconio.
▪ Síntomas varían según el lugar de la obstrucción
• Alta (duodeno o yeyuno) Vómitos precoces, biliosos.
• Baja (Íleo o colon) Vómitos en 24 – 48 horas, biliosos o
meconiales, gran distensión abdominal (puede hasta
comprometer la ventilación).
▪ Hay dilatación de asas, acumulación de líquido y aire (niveles hidro-aéreos) y progresión al íleo.
• Retención de líquidos crea tercer espacio con compromiso hemodinámico ▪
Retraso en la evacuación de meconio es de la obstrucción baja. o Al sospechar
se debe instalar sonda nasogástrica SNG
• Causas más frecuentes o Atresia duodenal y yeyunoileal o Íleo meconial o
Malrotación intestinal o Atresia cólica
o Enfermedad de Hirschprung

38
4
Temas clave de Pediatría y Cirugía Infantil
• Diagnóstico o Prenatal
 Imágenes: Ecografía en las últimas semanas
 Clínica: PHA (menor absorción intestinal). Mayor entre más alta es la
obstrucción. o Post natal (clínico – radiológico) ▪
Radiología simple
• Doble burbuja Atresia duodenal
• Doble burbuja con aire distal Estenosis duodenal
• Doble burbuja con poco aire distal Malrotación o vólvulo
• Dos o tres asas dilatadas Atresia de yeyuno
• Múltiples asas dilatadas Atresia de íleon o colon. ▪
Enema baritado Contraindicado en
obstrucción alta
• Microcolon Atresia intestinal
• Obstrucción cólica Atresia cólica
• Dilatación cólica Aganglionismo rectal

Atresia y estenosis intestinal


• Generalidades o Causa más frecuente de obstrucción intestinal neonatal o Puede ser de
cualquier parte del intestino Más frecuente yeyunoileal o Incidencia 1:3000 RNV
• Anomalías asociadas Malformaciones congénitas o Genéticas: Trisomía 21 atresia duodenal
(30%) o Cardíacas o Renales: Atresias bajas o Prematurez: Atresias complicadas o Defectos de la
pared abdominal
• Tipos de atresia o Atresia duodenal
 Imagen: Doble burbuja
 Clínica: Vómitos biliosos y precoces o Atresia yeyuno ileal
 Tipos (Grostfeld)
• I. Atresia membranosa (20%) Sin interrupción del intestino ni de su meso, solo la luz.
• II. Atresia con dos bolsones ciegos separados por cordón fibroso (30%)
• IIIa. Atresia con separación de bolsones con defecto de meso (35%, la más frecuente)
• IIIb. Atresia tipo “Apple peel” o Atresia yeyunal y gran defecto vascular de meso, íleon muy corto
arrollado sobre arteria ileocólica.
o Se asocia a intestino corto y prematurez.
• IV. Atresia múltiple
• Otras
o Atresia cólica Muy infrecuente

38
5
Temas clave de Pediatría y Cirugía Infantil
o Atresias anorrectales

Malrotación intestinal
• Definición: Interferencia entre el proceso normal del retorno del intestino fetal desde la hernia fisiológica hacia la
cavidad abdominal durante el cual se produce la rotación y fijación de las asas a la cavidad.
• Generalidades o La rotación y fijación del intestino tiene lugar en el primer trimestre de gestación o EL tubo digestivo
se desarrolla fuera de la cavidad Celómica y debe rotarse para adaptarse dentro de la cavidad
o Alteraciones en este proceso dan origen a la malrotación intestinal
• Epidemiología o 55% se presenta en la primera semana de vida y el 80% antes del primer mes.
• Clínica o Episodios recurrentes de obstrucción intestinal sub aguda con vómitos biliosos intermitentes. o Vólvulo
estrangulado Distensión abdominal, vómitos con trazas de bilis, sangre oscura por el recto, shock.
• Diagnóstico o Imágenes
 Rx de abdomen simple Aire en el estómago y duodeno, con poco o nada de gas en el resto de los
intestinos.
 Radiografía con contraste
• Contraste alto: o Configuración anormal del duodeno y unión duodenoyeyunal hacia la derecha de la línea media.
o Vólvulo: “cinta torcida” o “en sacacorchos”.
• Enema baritado: Ciego y apéndice en una posición anormal (HD o región periumbilical)

38
6
Temas clave de Pediatría y Cirugía Infantil

• Tratamiento o La reparación de la malrotación es una emergencia quirúrgica o Estabilizar o reanimar al paciente previo
a la cirugía.

Íleo meconial
• Definición o Impactación de meconio a nivel del íleon terminal con obstrucción completa o Se
produce por meconio anormalmente espeso y adherente o Consecuencia de la fibrosis quística
del páncreas
• Clasificación: Dos tipos o Simple: Impactación de meconio intestinal sin otra patología asociada
o Complicada: Asociada a otros cuadros patológicos vólvulo, atresia de íleon, estenosis ileal,
perforación con peritonitis meconial.
• Diagnóstico o Clínica
 Obstrucción baja Vómitos biliosos a las 12-24 horas de nacido, distensión
abdominal o Imágenes
 Ecografía pre natal
• Intestino hiperecogénico asociado a dilatación de asas.
• Si evoluciona a vólvulo Loops hiperecogénicos sin peristalsis
 Radiografía simple de abdomen
• Loops distendidos llenos de gas.
• Niveles hidro-aéreos (33%)
• Cúmulo granular denso en “miga de pan” en el flanco derecho (signo de Neuhauser) o fino (signo
de Singleton) Poco específico
• Calcificaciones por peritonitis meconial debido a perforación intestinal

38
7
Temas clave de Pediatría y Cirugía Infantil

Colon siempre es microcolon (nunca recibe meconio)

• Tratamiento
o Medidas generales iniciales
 Descompresión con SNG
 Profilaxis ATB
 Estabilización hemodinámica o Tratamiento médico
 Enema con contraste hidrosoluble (diagnóstico y terapeútico) o Quirúrgico
 Falla del tratamiento médico
 Resección
 Ileostomía

Enfermedad de Hirschprung neonatal


• Definición o Ausencia de células ganglionares en el intestino distal desde el esfínter anal interno.
o 80% en rectosigmoides, pudiendo abarcar todo el colon.
• Epidemiología o Incidencia de 1:5000 RNV
• Etiología o Migración celular desde la cresta neural
 Neuroblastos migran desde la cresta neural vagal a través de la pared intestinal en dirección cráneo
caudal desde el esófago hasta el ano formando el plexo nervioso entérico
 Esto ocurre entre la 5ta y 12va semana de gestación
 Una falla en esta migración provocaría la enfermedad
 Aganglionismo es más extenso entre más temprano se produzca la alteración
• Patología o Estructural Dilatación e hipertrofia del colon proximal con transición a intestino distal estrecho.
o Histología Ausencia de células ganglionares en el plexo mientérico y submucoso + troncos amielínicos de nervios
en el espacio ocupado normalmente por los ganglios.
• Diagnóstico: Clínico – imagenológico – histológico. o Clínica: 90% produce síntomas que permiten diagnóstico en
el periodo neonatal
 Constipación, distensión abdominal y vómito en los primeros días de vida

38
8
Temas clave de Pediatría y Cirugía Infantil

 Diarrea (1/3)
 Megacolon tóxico Distensión abdominal intensa de inicio súbito.
o Imágenes
▪ Radiografía de abdomen simple
• Loops dilatados de intestino con niveles de líqudo y pelvis sin aire.
• Colon dilatado y recto no distendido con péqueña cantidad de aire
• Aganglionosis total de colon Signos de obstrucción de ileon (niveles hidro aéreos y distensión)
▪ Enema baritado confirma el diagnóstico
• Retención de bario y acentuación de la zona de transición.

 Manometría ano - rectal Disminución de la actividad del esfínter anal interno


 Biopsia rectal
• Tratamiento o Estabilización hemodinámica de requerirse o Es necesario descomprimir el intestino lo más pronto
posible
 Irrigaciones rectales
 Colo/Ileostomía

Malformación ano – rectal


• Definición o Antiguamente mal llamado ano imperforado, las malformaciones ano rectales incluyen las
comunicaciones a través de una fístula del intestino con la vía urinaria o genital. • Epidemiología
Incidencia de 1 en 4500 RNV
• Clasificación o Sexo masculino (25%) Fístulas entero urinarias
• Clinica: Orina con meconio en el pañal
Rx Simple de abdomen: niveles hidro-aéreos en la vejiga
 Fístulas cutáneas
 Fístula rectouretral: Prostática y bulbar
 Fístula rectoprostática
 Fístula rectovescial o Sexo femenino (32%) Fístulas entero genitales
• Clínica: Flujo meconial manchando el pañal
 Fístula rectovestibular 3 orificios en la vulva (uretra,
vagina y en vestíbulo)
 Fístula recto vaginal 2 orificios en la vulva (uretra y
vagina)
 Cloaca 1 orificio en vulva pequeña o Ambos sexos
(32%)
• Comunicadas al exterior Pañal con meconio (32%) o
Fístula rectoperineal o Estenosis anal (excepcional)
• No comunicadas al exterior Pañal limpio (9 %) o
Agenesia anorrectal (5%) Fondo de saco ciego o
Atresia rectal (4%) Tabique interrumpe el lumen o

38
9
Temas clave de Pediatría y Cirugía Infantil

Membrana anal (1%) • Anomalías asociadas o Urogenitales (20-
50%) Agenesia o displasia renal, riñón en herradura, RVU,
hidronefrosis, hipospadias, escroto bífido
o Esqueléticas (30%) Agenesia de columna lumbosacra, hemisacro, agenesia de coxis.
o Cardiovasculares y gastrointestinales (30%) o SNC (20%): Médula anclada, canal espinal angosto, mielomeningocele
posterior o meningocele oculto
o Anomalías cromosómicas (10%)
• Tipos o Fístula rectovestibular
 Epidemiología La más frecuente en la
mujer (17,2%)
 Clínica
• Vulva con 3 orificios Uretra, vagina y fístula
• Surco intergluteo bien marcado y foseta anal visible

• Pronóstico Muy bueno


• Logran evacuaciones voluntarias
o Fístula rectovaginal
▪ Epidemiología Poco frecuente (5,7%) ▪ Clínica
• Vulva con dos orificios Uretra y vagina
• Salida de meconio por vagina
• Orificio fistuloso en cara dorsal de la vagina
Pronóstico
Malo o Cloaca (Disgenesia
cloacal)
 Epidemiología
Exclusivo de la mujer. (9,2%)
 Clínica

39
0
Temas clave de Pediatría y Cirugía Infantil

Recto, vagina y uretra generan un solo conducto llamado cloaca
• Muchos subtipos, desde conducto pequeño hasta largo y estrecho ▪ Pronóstico: Variable según subtipo, excelente
en pequeños y malo en largos.
o Fístula rectoperineal
 Epidemiología La más frecuente que afecta a ambos sexos
(32%) ▪ Clínica
• Recto ocupa el trayecto músculo esfinteriano
• Esfinter bien desarrollado
• Surco intergluteo marcado
• Foseta anal notoria ▪ Pronóstico Muy bueno
• Continentes
fecales y
urinarios
• Problema
Constipación
o Fístula rectouretral
 Epidemiología La más frecuente en el hombre
 Clínica
• Comunicación con la uretra a nivel bulbar o
prostático
• Síntoma: Orina con meconio
• Rx de abdomen simple Aire en vejiga
• Uretrocistografía
Angulación de la uretra ▪ Pronóstico Bueno
• 70% tendrá evacuaciones voluntarias
o Fístula rectovesical
 Epidemiología Exclusiva del hombre. Rara (1,8%) ▪ Clínica
• Intestino termina en el cuello vesical
• Pelvis ósea cónica y pequeña, ausencia de más
de 3 vertebras sacras y musculatura esfinteriana
rudimentaria.
• Borramiento del surco interglúteo y foseta anal
• Incontinencia urinaria, fecal y escurrimiento
 Pronóstico El peor de todos o Atresia rectal
 Epidemiología niñas (6:8). Incidencia de 3,5%
 Clínica
• Tabique que interrumpe el lumen del recto
• Estructuras esfinterianas bien desarrolladas
 Pronóstico Excelente o Agenesia anorrectal
 Epidemiología Ambos sexos con predominio masculino e
incidencia del 5,3% ▪ Clínica
• Falta de formación distal. Intestino en fondo de
saco ciego.
Evacuaciones voluntarias y continencia urinaria
• Problema Constipación y escurrimiento
• Conducta o Descartar malformación asociada de
riesgo vital o Estudio preoperatorio o Abordaje
quirúrgico en un tiempo o en tres tiempos con
colostomía
 Colostomía Cloaca, atresia rectal,
ausencia de fístula, fístula rectovesical y fístula rectouretral.

39
1
Temas clave de Pediatría y Cirugía Infantil

 En un tiempo Anorrectoplastía sagital posterior. o Siempre
tendrán algún déficit funcional Pacientes crónicos o Mayores
problemas son la constipación y la funcionalidad de la vejiga

Hernia diafragmática congénita


• Definición o Defecto congénito del desarrollo diafragmático que permite la salida de vísceras abdominales hacia la cavidad
torácica.
o Falla en el cierre del canal pleuroperitoneal entre las 8 y 10 semanas de gestación o Más frecuente en
la región postero lateral (congénita) Hernia de Bochdalek o Hernia retroesternocostal (anterior)
Hernia de Morgani o 80% lado izquierdo, 19% derecho y 1% bilateral.
• Fisiopatología o Dificulta el desarrollo normal del pulmón por contenido intestinal en el tórax Hipoplasia pulmonar
o Menor número de bronquiolos y arterias bronquiales + hipertrofia muscular de las arteriolas
Hipertensión pulmonar
• Epidemiología o Incidencia de 1:4000 RNV
• Malformaciones asociadas o Alteraciones cromosómicas: Trisomía 18, Sd de Turner, Secuencia de Pierre-Robin, Sd de rubeóla
congénita
• Diagnóstico Clínico e imagenológico o US pre natal
 PHA severo
 Visualización directa Hernia, hipoplasia, malformaciones asociadas.
 Índice ecográfico de severidad LHR (Lung to head
ratio)
• LHR > 1,4 95% sobrevida
• LHR <1,0 75% mortalidad
o Radiografía de tórax simple – TAC tórax: Corazón y mediastino
rechazados
o ECO torácica: Visualización de integridad diafragmática
• Clínica Depende del grado de alteración pulmonar o Dificultad
respiratoria y cianosis al nacer o Hipoventilación del lado ipsilateral
o Abdomen excavado
o Desplazamiento del latido y mediastino al lado contralateral o
Ausencia de MP o RHA en el tórax
• Tratamiento o Medidas preoperatorias
 Manejo de vía aérea, intubación inmediata, evitar
ventilación con mascarilla
 Sonda orogástrica descompresiva
 UCI neonatal Manejo HT pulmonar o Manejo
quirúrgico
 Reparación puede diferirse hasta estabilidad
cardiorrespiratoria mantenida ya no es emergencia
quirúrgica
 Paciente sin HT pulmonar y HDE estable Cirugía
segura entre las 24 y 36 horas de vida.
• Pronóstico o Mortalidad entre 20 – 50% por HT pulmonar persistente o
malformaciones asociadas o Factores de buen pronóstico
 Dificultad respiratoria tardía (> 6 horas de vida)
 Ausencia de herniación hepática
 Defecto pequeño
 Presentación luego de las 25 semanas de gestación
 Ausencia de malformaciones congénitas asociadas

39
2
Temas clave de Pediatría y Cirugía Infantil

Gastrosquisis y onfalocele
• Definición: Falla del cierre embriológico de la pared abdominal durante la gestación que resulta en la protrusión
de vísceras abdominales, a nivel umbilical (onfalocele) o lateral al cordón (gastrosquisis)
• Conceptos o Onfalocele
 Defecto de la pared abdominal en la línea media a nivel del cordón umbilical
 Protrusión de asas intestinales e hígado cubiertos de peritoneo y membrana amniótica, a través del anillo
umbilical
 Se puede asociar a alteración del cierre del pliegue craneal creando defecto de la línea media superior
Pentalogía de Cantrell (Hernia diafragmática anterior, onfalocele, defecto esternal, defecto pericardio,
anomalía cardíaca) ▪ Alta incidencia de anomalías congénitas
• Todos presentan malrotación intestinal incompleta
• 1/3 tendrá alguna trisomía (13,14,15,18 y 21)
o Onfalocele roto: Complicación del onfalocele donde el peritoneo se rompe exponiendo las vísceras al medio
o Gastrosquisis
 Defecto de todos los planos de la pared a nivel para umbilical derecho con protrusión de estómago e
intestino delgado, sin revestimiento.
 Intestino ectópico edematoso y dilatado característico.
• Embriología o Onfalocele: Alteración del cierre de la pared ventral del embrión en la 2da semana o
Gastrosquisis:
 Origen más tardío por ruptura intrauterina de hernia del cordón umbilical.

39
3
OSCE EUNACOM SP |||||||||||||||||||||||||||||||||||||||||||||||||||||||||||||||||||

 Menos asociación a malformaciones


 Edema y engrosamiento del intestino es por trauma e isquemia durante el parto.
• Epidemiología o Incidencia variable 1: 3000 – 1:5000 RNV o Gastrosquisis
 Madres < 20 años
 Tabaquismo
 Drógas ilícitas y vasoactivas (pseudoefedrina)
 Toxinas ambientales o Onfalocele Madres de edad avanzada.
• Malformaciones asociadas o 35 – 65% en onfalocele; > 15% en gastrosquisis o Onfalocele Gastrointestinales, craneofaciales,
genitourinarias y cardiovasculares o Gastrosquisis Tubo digestivo (malrotación intestinal)
• Diagnóstico o Diagnóstico pre natal Antecedentes obstétricos ▪ ECO pre natal
• PHA
• Aumento de la alfafetoproteína (AFP)
• BPN (<2500 g) en gastrosquisis
o Diagnóstico post natal Clínico durante el parto
▪ Descartar malformaciones asociadas Impacto en la supervivencia
• Tratamiento o Manejo pre operatorio
 Parto en centro especializado
 Cubrir con bolsa de polietileno al nacer (desde extremidades inferiores hasta el tórax)
• Protege peritoneo y disminuye la contaminación, minimiza pérdida de líquido y calor, permite observación de las asas para vigilar
vólvulación en gastrosquisis. ▪ SNG con aspiración cada 15 minutos
• Previene vómitos
• Mantiene el tubo digestivo descomprimido
 Transporte a UCIN en incubadora ▪ Hidratación parenteral
• Gastrosquisis Líquidos isotónicos hasta 3 veces más volumen que RN normal en las primeras 24 horas.
 Corrección de trastornos hidroelectrolíticos y ácido-base previo a la cirugía
 Uso profiláctico de antibióticos o Manejo quirúrgico
 Ideal cierre primario
 Onfalocele indemne
• Cirugía diferida
• Uso del saco como silo transitorio
 Defectos en condiciones de cerrarse Emergencia quirúrgica.

39
4
OSCE EUNACOM SP |||||||||||||||||||||||||||||||||||||||||||||||||||||||||||||||||||

• Pronóstico o En ausencia de malformaciones severas es bueno Sobrevida aprox 90%.

Enterocolitis necrotizante
• Definición o Necrosis isquémica de la pared intestinal que afecta preferentemente a RNPT o Frecuencia inversamente proporcional
a la EG 10% de los RN < 1500 g
• Patogenia o Multifactorial
 Predisposición genética
 Intestino inmaduro
 Ambiente microbiano particular o Fisiopatología

39
5
OSCE EUNACOM SP |||||||||||||||||||||||||||||||||||||||||||||||||||||||||||||||||||

 Respuesta inflamatoria exagerada en el íleon terminal o colon proximal a noxas isquémicas o infecciosas
 Ruptura del epitelio intestinal TB Respuesta inmune en intestino inmaduro
 Lesiones progresan hasta necrosis total de la pared Perforación
• Clínica o Cuadro lento e insidioso o rápido y progresivo o Síntomas de enfermedad sistémica Inespecíficos
 Apnea, bradicardia, inestabilidad térmica, letargia, piel moteada o Síntomas gastrointestinales
 Distensión abdominal, sangre en deposiciones, cambio de coloración de la pared, masa palpable.
• Estudio diagnóstico o Laboratorio
 Leucocitosis con aumento baciliformes
 Trombocitopenia
 Acidosis metabólica
 PCR aumentada (si persiste elevada, sospechar complicación como absceso, estenosis o
fístula)
o Imágenes
▪ Rx abdomen simple AP y L
• Neumatosis Quística o lineal
• Gas portal enfermedad avanzada, mal pronóstico
• Neumoperitoneo Perforación intestinal
• Dilatación de asas, engrosamiento de la pared, ausencia de gas abdominal, asa dilatada en varios exámenes (asa fija) menos
específicos
• Diagnóstico o Sospecha clínica
 RNPT <1500g con más de 1 semana de hospitalización y que recibió alimentación días antes
 Signos: CEG, intolerancia digestiva
o Clasificación de Bell (diagnóstica, pronóstica y orienta al tratamiento)

39
6
OSCE EUNACOM SP |||||||||||||||||||||||||||||||||||||||||||||||||||||||||||||||||||

• Tratamiento  Médico y quirúrgico


o Tratamiento médico
▪ Objetivos Evitar la progresión, restablecer homeostasis y prevenir complicaciones ▪ Etapas
• Etapa I o Ayuno
o Sonda OG abierta (medir residuo) o Rx de abdomen
o ATB Ampicilina y amikacina o Si se descarta, realimentar desde las 72 horas.
• Etapa II o Ayuno 7 – 10 días o ATB por 10 – 14 días y ajustar a cultivo o Prevenir shock Buen volumen y DVA,
balance hídrico o Radiología cada 6 – 8 horas
• Etapa III o Manejo agresivo del shock o Discutir con cirujano posibilidad de intervención antes de que se perfore.
o Considerar ATB de múltiple espectro como meropenem
o Tratamiento quirúrgico
 Objetivos Intervenir en las complicaciones o con deterioro a pesar de tratamiendo médico adecuado ▪ Indicaciones
• Absoluta Perforación intestinal confirmada

39
7
OSCE EUNACOM SP |||||||||||||||||||||||||||||||||||||||||||||||||||||||||||||||||||

• Relativas o Deterioro progresivo a pesar de medidas médicas máximas o Gas en la porta o


Asa fija en la Rx o Eritema de pared abdominal o Masa palpable abdominal • Complicaciones o Secuelas
 SIC
 Talla baja (aún sin SIC)
 Déficit neurológicos Cognitivos, lenguaje, sensorial, social.
• Prevención o LME
o Pautas de alimentación estándar en la unidad de neonatología o Probióticos Menor ECV y mortalidad (Cochrane)

Bibliografía

1. Leiva M., Barrera F., “Emergencias quirúrgicas” Guías de práctica clínica en pediatría, VII Edición. Hospital clínico San Borja Arriarán. Fundación para la asistencia
e investigación clínica pediátrica, 2013. Pags 159 – 170.
2. Losty P., Prem P. “Esophageal atresia and trachea-esophageal fistula”. Newborn Surgery. Tercera edición. Editorial Hodder Arnold, 2011.
3. Sweed Y., Prem P. “Duodenal obstruction”. Newborn Surgery. Tercera edición. Editorial Hodder Arnold, 2011.
4. Pierro A., Prem P. “Malrotation”. Newborn Surgery. Tercera edición. Editorial Hodder Arnold, 2011.
5. Millar A., Prem P. “Jejuno-ileal atresia and stenosis”. Newborn Surgery. Tercera edición. Editorial Hodder Arnold, 2011.

39
8
OSCE EUNACOM SP |||||||||||||||||||||||||||||||||||||||||||||||||||||||||||||||||||

6. Ciprandi G., Rivosecchi M., Prem P. “Meconium ileus”. Newborn Surgery. Tercera edición. Editorial Hodder Arnold, 2011.
7. Castle S., Prem P. “Necrotizing enterocolitis”. Newborn Surgery. Tercera edición. Editorial Hodder Arnold, 2011.
8. Prem P. “Hirschsprung’s disease”. Newborn Surgery. Tercera edición. Editorial Hodder Arnold, 2011.
9. Levitt M., Peña A., Prem P. “Anorectal anomalies”. Newborn Surgery. Tercera edición. Editorial Hodder Arnold, 2011.
10. Losty P., Prem P. “Esophageal atresia and trachea-esophageal fistula”. Newborn Surgery. Tercera edición. Editorial Hodder Arnold, 2011.
11. Bruch S., Prem P. “Omphalocele and gastroschisis”. Newborn Surgery. Tercera edición. Editorial Hodder Arnold, 2011.
12. Iñón A., Meneghello J. “Atresia esofágica”. Pediatría, Quinta edición, capítulo 443. Editorial Panamericana, 1997.
13. Lassaletta L., Meneghello J. “Obstrucción intestinal neonatal” Pediatría, Quinta edición. Capítulo 444. Editorial Panamericana, 1997.
14. Varela M., Meneghello J. “Malformaciones anorrectales” Pediatría, Quinta edición. Capítulo 445. Editorial Panamericana, 1997
15. Lassaletta L., Meneghello J. “Hernia diafragmática congénita” Pediatría, Quinta edición. Capítulo 446. Editorial Panamericana, 1997
16. Krebbs C., Meneghello J. “Defectos congénitos de la pared abdominal: onfalocele y gastrosquisis” Pediatría, Quinta edición. Capítulo 445. Editorial Panamericana,
1997
17. Peña A., “Guías de tratamiento unidad cuidados intensivos neonatal HSJD de La Serena”. 2012

Reflujo gastroesofágico
• Introducción o Cerda del 85% de los niños vomitará durante su primera semana de vida y el 10% permanecerá con síntomas a las
6 semanas, sin embargo, el 60% será asintomático a los 2 años (por la posición erguida y la comida solida)
• Fisiología normal del EEI o El ángulo de His es obstuso en neonatos pero disminuye durante el desarrollo, generando una mejor
barrera contra el RGE.
o Acción contenedora del pilar derecho del diafragma o Roseta mucosa: Pliegues redundantes de mucosa están
presentes en la unión GE cuando un ángulo de HIS normal está presente. Estas se doblan y se aprietan juntas para
formar una válvula antirreflujo débil. o Zona de gran presión (esfínter manométrico): Con la maduración aumenta el
tono basal de la UGE hasta los 45 días
o Esófago intraabdominal: Si es mayor de 2 cms es sugiciente, menor de 1 cm es incompetente o Hernia hiatal puede
causar desplazamiento del esófago distan dentro del tórax. o Resistencia al vaciamiento gástrico: Aumenta la presión
intragástrica, exacerbando el RGE
• Clasificación o RGE fisiológico: Común, no problemático.
o RGE patogénico:
 También llamada enfermedad por RGE.
 Se distingue por su frecuencia (mayores episodios) y la presencia de complicaciones como malnutrición, problemas respiratorios (aspiración),
esofagitis, sangrados y esófago de Barret.
 Puede ser secundario a hernia hiatal, obstrucción de la salida gástrica, hernia diafragmática, etc.

39
9
OSCE EUNACOM SP |||||||||||||||||||||||||||||||||||||||||||||||||||||||||||||||||||

• Fisiopatología o El mecanismo en niños es la relajación transitoria del esfínter esofágico inferior (95%), proviéndose el episodio de
reflujo por la posición supina durante la relajación del esfínter
o Dietas líquidas en niños facilitan la regurgitación comparada con dietas sólidas o La disminución del vaciamiento
gástrico es un hallazgo común en RNPT o Se asocia a parálisis cerebral, retraso del crecimiento, síndrome de Down y
uso de drogas durante la gestación.
o Adoptar una posición supina inmediatamente después de comer y comidas de baja calidad (grasosas, ácidas) y las
alergias alimentarias, predisponen a sufrir episodios de reflujo.
• Clínica o Regurgitación de comida
 Muy frecuente
 Va desde el babeo a vómitos explosivos.
 Lo más frecuente es regurgitación postprandial
 Puede llevar a pérdida de peso y falla de medro. o Dolor toráxico y/o abdominal
 Se manifiesta como llanto atípico e irritabilidad ▪ Síntomas
• Problemas de la vía aérea
o La vía aérea proximal y el esófago están alineados con receptores que se activan con agua, ácido o distensión,
produciendo laringoespasmo llevando a apnea obstructiva con hipoxemia, cianosis y bradicardia (ALTE)
o Puede ser un factor que genere complicación de asma debido a microaspiración y broncoconstricción refleja Sospechar
en historia de sibilancias nocturnas
o Síndrome de Sandifer Postura con opistótono o tortícolis por dolor producido por esofagitis, que genera esa posición
antiálgica al tragar.
• Estudio diagnóstico CLINICO o Si la presentación es atípica o la respuesta a la terapia no es adecuada, se requieren mayores
estudios
▪ Estudio contrastado del TGI superior busca hernia del hiato, esofagitis y dismotilidad. ▪ pHmetría 24 horas Cuantifica el reflujo y su relación con síntomas
atípicos ▪ Endoscopía alta Se indica en niños refractarios a terapia médica.
• Visualización de la mucosa: Diagnóstico de esofagitis, úlcera péptica, infección por
HP
• Bopsia
• Tratamiento o Tratamiento médico
▪ Medidas conservadoras
• Posicionamiento después de alimentarlo
• Elevar la cabecera de la cama (30 º desde el suelo)
• Proveer bajas cantidades de comida liviana y/o leche diluida
• Sonda nasoyeyunal o gastroyeyunal pueden ser de ayuda si no se está alimentando

40
0
OSCE EUNACOM SP |||||||||||||||||||||||||||||||||||||||||||||||||||||||||||||||||||

• RGE fisiológico no requerirá tratamiento más que vigilar la técnica de lactancia ▪ Medicamentos
• Metoclopramida Aumenta el tono del EEI, aumenta el vaciamiento gástrico.
• Antiácidos aumental el pH gástrico > 4, inhibe la actividad proteolítica de la pepsina
• Antagonistas H2 (ranitidina) Inhibe la pepsina, disminuye el reflujo ácido
• IBP (omeprazol) Supresión ácida total en enfermedades respiratorias crónicas o deterioro neurológico. ▪ Cirugía
Nissen • Indicaciones o ALTE asociado a RGE confirmado o Falla en la terapia médica
o Complicaciones orofaringeas y esofágicas esófago de barret o Reflujo anatómico Hernia hiatal o Niños retrasados con RGE severo donde el tratamiento médico ha fallado (la
gastrostomía pudiese adicionarse al manejo quirúrgico)
• Muchos de estos pacientes tienen malnutrición, aspiración crónica, neumonía o disfunción pulomonar
• Pronóstico o Cerca del 60-80% de los RGE se resuelven a los 2 años (50% al año)
o Algunos niños requerirán modificaciones del estilo de vida, otros, tratamiento sintomático del RGE o La cirugía se requiere en una minoría de casos y controla el reflujo en cerca
del 90%.
Estenosis hipertrófica del píloro
• Definición y epidemiología o Es la causa quirúrgica más común de vómitos en la infancia o Afecta 1 – 3: 1000 RNV o Es más frecuente en
hombres 5 : 1 o Tipicamente afecta a la edad postnatal entre las 3 – 6 semanas y los 4 meses (raro en niños mayores)
• Fisiopatología o Componente genético
 El 7% de los niños afectados tienen a uno de sus padres afectados o Asociado a
 Atresia esofágica
 Uso de eritromicina materna (no hay suficiente evidencia)
 Edad materna temprana o Se caracteriza por hipertrofia de la capa muscular circular de modo que el canal pilórico se estrecha con
engrosamiento de todo el píloro
o La obstrucción gástrica lleva a pérdida de HCl, Na, Cl, K y agua Deshidratación, alcalosis metabólica e hipokalemia.
 La respuesta compensatoria es la preservación renal Na (inicialmente con pérdida de K, y luego pérdida de agua y de protones) Acidosis
paradojal
• Clínica o Vomitos explosivos, no biliosos, progresivos, ocasionalmente café tierra (por la sangre y gastritis) en un niño de termino.
o La clave del diagnóstico es la palpación del píloro hipertrofiado en el epigastrio (oliva pilórica) o la visualización de la peristalsis gástrica.
 Examine el abdomen mientras el niño se alimenta para favorecer el hallar la oliva (difícil de ver sin este método)
• Estudio o ELP y GSA Disminución de Na, K y Cloro; Aumento de pH y bicarbonato o Eco abdominal: Confirma el diagnóstico
 Grosor > 4 mm
 Canal pilórico > 16 mm
• Cirugía o No es una emergencia quirúrgica, sino médica en un inicio (estabilizar)
 SNG

40
1
OSCE EUNACOM SP |||||||||||||||||||||||||||||||||||||||||||||||||||||||||||||||||||

 Fluidos EV
• Glucosa al 5% + NaCl 10% 20 cc + KCl 10% 10 cc, luego de confirmada la diuresis
▪ HGT Monitorizar
o Intervención Piloromiotomía descompresiva.

Abdomen agudo
• Introducción
o RN
 Lo más frecuente son los cólicos, dolor intermitente benigno.
 Si hay vómitos biliosos repetidos, rechazo y distención Malrotación intestinal y vólvulo o < 2 años
 Lo más frecuente es la GEA
 Si hay dolor intenso, cólico o rectorragia Invaginación intestinal o Pre escolar y escolar
 Lo quirúrgico más frecuente es la apendicitis aguda
 Lo más frecuente GEA y constipación
• Epidemiología o Causas quirúrgicas son poco frecuentes (hasta 3%) o Diagnósticos más frecuentes en dolor abdominal
 IRA alta
 Faringitis
 Infección viral
 Dolor inespecífico
 GEA
• Forma de presentación y diagnóstico diferencial o Causas de dolor abdominal más frecuentes según edad de presentación ▪
Menores de 2 años
• Cólico del lactante
• Síndrome diarreico agudo
• Síndromes virales ▪ Pre escolares 2 – 5 años
• Síndrome diarreico agudo
• ITU
• Trauma
• Apendicitis ▪ Escolares
• Síndrome diarreico agudo

40
2
OSCE EUNACOM SP |||||||||||||||||||||||||||||||||||||||||||||||||||||||||||||||||||

• Trauma
• Apendicitis
• ITU
▪ Adolescentes
• Síndrome diarreico agudo
• Gastritis
• Colitis
•Constipación
o Patologías graves y/o con riesgo vital
▪ Menores de 2 años
• Malformación intestinal
• Invaginación intestinal
• Acidosis metabólica
• Intoxicaciones ▪ Pre escolares
• Trauma
• Invaginación
• Apendicitis
• Intoxicaciones
• SHU
▪ Escolares
• Megacolon
• Intoxicaciones
• DM
▪ Adolescentes
• Embarazo ectópico
• Clínica o Estado enfermo o no orienta a gravedad o Signos de sangrado, pérdida de volumen (diarrea o vómitos) o deshidratación indican
necesidad de reanimación
o Dolor mejora, empeora o se mantiene. Reevaluación debe realizarse constantemente o Características del vómito y de las deposiciones
▪ Vómitos
• Biliosos: Obstrucción, vólvulo
• Café tierra: Esofagitis, gastritis, úlcera gástrica

40
3
OSCE EUNACOM SP |||||||||||||||||||||||||||||||||||||||||||||||||||||||||||||||||||

• Roja brillante, poco volumen: Esofagitis, gastritis


• Roja brillante, alto volumen: MW, ulcera gástrica, ulcera duodenal, várices esofágicas
• Contenido gástrico o comida: GEA, Obstrucción
• Fecaloídea: Obstrucción ▪ Deposiciones
• Diarera acuosa: Infección viral, apendicitis con absceso rectal
• Duras o largas: Constipación
• Disminución de frecuencia: Constipación, obstrucción
• Mucus: colitis
• Sangre roja brillante, escaso volumen: constipación, colitis PSH, pólipos
• Sangre roja brillante, alto volumen: colitis, pólipos
• Mermelada de grosellas: Intususcepción
• Melena: Ulcera gástrica, ulcera duodenal
• Pálidas, acólicas: Enfermedad biliar o hepática
• Estudio de urgencias o Hemograma y PCR Marcadores de inflamación, en casos dudosos.
o Amilasa y lipasa Sospecha de pancreatitis aguda o GSV o GSA Acidosis metabólica o Radiografía de tórax Descartar neumonía en paciente
taquipneico, ruidos agregados, retracción subcostal y DR.
o Radiografía de abdomen simple Sospecha de perforación, obstrucción intestinal o vólvulo o Eco abdominal Util en intususcepción, patología
renal/urológica, apendicitis y patología ovárica ▪ Intususcepción: Signo de tiro al blanco o donuts. o TAC Para valorar apendicitis en
obesos o con eco no concluyente
• Tratamiento inicial o ABC, fluidos, analgesia o
Interconsulta a cirujano infantil
 Dolor intenso que aumenta en forma progresiva o con signos de deterioro clínico
 Vómitos biliosos o fecaloídeos
 Resistencia involuntaria, rigidez
 Blumberg (+)
 Distensión importante, timpanismo
 Signos de atrapamiento de volumen o sangramiento intraabdominal
 Sospecha de trauma
 Sospecha de dolor quirúrgico
 Dolor sin etiología clara
 Necesidad de hospitalizar o ATB ev

40
4
OSCE EUNACOM SP |||||||||||||||||||||||||||||||||||||||||||||||||||||||||||||||||||

 Apendicitis: Precoz, antes de la cirugía


• No perforada o Cefazolina
• Perforada o Ampicilina 150 mg/kg/día c/6 horas,
gentamicina 3 – 5 mg/kg/día x 1 vez y
metronidazol 15-30 mg/kg/día cada 8 horas.
o Ceftriaxona – metronidazol

Apendicitis aguda
• Clasificación o G I: Apendicitis edematosa o G II: Apendicits flegmonosa o GIII: Apendicitis
necrótica con peritonitis localizada o GIV: Apendicitis perforada con peritonitis difusa
• Síntomas clásicos o Dolor abdominal vago periumbilical que migra a FID o Anorexia
o Vómitos posteriores al inicio del dolor
o Fiebre baja que aumenta al retrasarse el diagnóstico
o En niños menores Signos sutiles, inespecíficos, fiebre alta, diarrea
• Examen físico o Resistencia difusa o localizada en CID o Signo de blumberg (el de mayor valor)
• Diagnóstico CLINICO o La evaluación repetida es la clave del diagnóstico
o La perforación apendicular es una complicación frecuente por un diagnóstico tardío, es mayor en niños pequeños (82% < 5 años y 100% en los de 1 año
de edad)
• Enfrentamiento o Baja sospecha Observar de forma seriada
o Sospecha intermedia ECO abdominal + marcadores de inflamación
▪ TAC de abdomen en obeso, muy distendido o eco no concluyente o Alta sospecha Derivar a cirujano infantil.
• Tratamiento en sala o Deambulación precoz o Analgesia y antiinflamatorios o Régimen líquido a
las 12 – 24 horas en GI y GII; y a las 48-72 horas en GIII y GIV o Antibioterapia
▪ Ampicilina, metronidazol, gentamicina x 7 – 10 días (GIII y GIV) o Cuidados de la herida operatoria

Malrotación intestinal y vólvulo


• Definición o Son una emergencia quirúrgica o Ocurre porque el intestino no está fijo de forma
normal a la pared o Puede ocurrir a cualquier edad, pero la mayoría ocurre en RN
• Clínica o Vómitos biliosos o verde oscuro o Dolor abdominal tipo cólico o Rectorragia o Distensión
y sensibilidad tardía
o Isquemia intestinal progresiva Hipovolemia Acidosis metabólica

40
5
OSCE EUNACOM SP |||||||||||||||||||||||||||||||||||||||||||||||||||||||||||||||||||

Adenitis mesentérica
• Definición: Dolor abdominal vago o generalizado que acompaña a infección viral del tracto respiratorio alto
• Clínica o Vómitos alimentarios, poco frecuentes o Cefalea y odinofagia o Fiebre alta (> 38,5 ºC) o Adenopatía cervical
o Sensibilidad abdominal difusa
• Diagnostico: ECO abdominal
• Tratamiento: Sintomático

Constipación
• Clínica o Dolor abdominal tipo cólico y soiling o AL examen físico: masa fecal de colon sigmoides o
descendente o Tacto rectal: Deposiciones duras
o Causa: Idiopática, masas pelvianas, patología anorrectal o déficit neurológico
• Tratamiento en urgencias o Si es aguda, enema evacuante o Si es crónico, PEG. o Si es idiopático, consejos
de dieta, baño y laxantes

Dolor perimenstrual ovulatorio


• Definición
o En niñas adolescentes preguntar historia menstrual y determinar el momento del ciclo en que ocurre el dolor (ovulación
dolorosa a mitad del ciclo)
o Preguntar por actividad sexual para descartar PIP y embarazo o AL examen físico hay sensibilidad pelviana
• Diagnóstico: Eco pelviana o transvaginal + test de embarazo

Divertículo de meckel
• Definición o Remanente embriológico o Son asintomáticos, encontrándose de forma incidental en laparotomía y autopsias o Se puede presentar con
sangrado, invaginación intestinal, obstrucción o simular apendicitis.
• Diagnóstico
o Cintigrafía Tc 99 Divertículo sangrante

40
6
OSCE EUNACOM SP |||||||||||||||||||||||||||||||||||||||||||||||||||||||||||||||||||

Escroto agudo
• Definición o Situación de dolor escrotal intenso de comienzo reciente y con sensibilidad y malestar local o Incluye torsión (testicular y de apéndices o hidátide), infección
(orquitis por virus parotiditis o orquiepididimitis por bacterias), trauma, varicocele, hernia inguinal, hidrocele agudo, edema escrotal idiopático, tumor (orquidoblastoma en jóvenes
y teratoma en adolescentes), dolor referido de patología lumbar y vasculitis (púrpura de SE)
o Diagnóstico final más frecuente es la torsión de hidátide (34%), epididimitis (30%) y torsión testicular (23%)

Torsión testicular
• Definición o Se produce por rotación del cordón sobre sí mismo, lo que compromete la irrigación del testículo o Hay una ventana de 4 – 8 horas antes del
daño isquémico
• Epidemiología o 1 en 4.000 menores de 25 años o Distribución bimodal: Peak neonatal, luego alza en adolescencia.
• Clasificación o Intravaginal: Torsión dentro de la túnica, “en badajo de campana”. Más típica de adolescentes.
o Extravaginal: Del cordón. Más típica de neonatos por falla de la fijación del testículo recién descendido.
• Clínica o Torsión neonatal: masa escrotal firme, inflamada, no sensible.
o Adolescentes:
▪ Dolor intenso y reciente, vómito reflejo. ▪ Puede referirse el dolor al a región lumbar ▪ Examen:
• Teste intensamente sensible, en posición transversal alto, sin reflejo cremasteriano.
• Eritema y edema del escroto
▪ Nauseas/vómitos + cambios en la piel escrotal y alteración del reflejo cremasteriano
Confirma el diagnóstico (VPP 100%)
• Estudio diagnóstico o Exploración quirúrgica: No es opción observar, si se sospecha se debe explorar.
o Estudio microscópico de orina y tinción de Gram: Puede encontrarse piuria y bacteriuria (cuando hay dudas diagnósticas)
o Ultrasonido y Doppler: Se ve la disminución de la perfusión al Doppler, se visualiza la torsión (imagen en concha de caracol)
o Radisótopos: Muestra disminución de la perfusión
• Diagnóstico diferencial
o Torsión de hidátide o ITU
o Edema escrotal idiopático o Orqui-epididimitis o Tumor o Trauma
• Tratamiento
o Evaluación por cirujano infantil NUNCA retrasar la cirugía, considerando que por el tiempo de evolución
el testículo podría no ser viable ya que puede ser torsión parcial o intermitente.
o Se recomienda realizar pexia contralateral.
• Pronóstico o Tiempo es clave
 Menos de 6 horas 90% de sobrevida del testículo
 Entre 6 y 18 horas 40%

40
7
OSCE EUNACOM SP |||||||||||||||||||||||||||||||||||||||||||||||||||||||||||||||||||

 Más de 18 horas 0% sobrevida

Torsión de hidátide de Morgagni


• Definición o Torsión de los remanentes embrionarios de Müller en el polo superior del testículo o en el epidídimo.
• Clínica o Dolor y aumento de volumen, menos agudo, llevando a presentación tardía (días) o Signo del punto azul: Visualización de
la hidátide torcida
• Estudio diagnóstico o ECO Doppler:
 Sensibilidad entre 68% y 95% dependiendo del tamaño de la hidátide
 Especificidad del 81 al 100% en > 4-5.6 mm y > 5.6 mm respectivamente
• Tratamiento o Analgesia hasta que retorne sensibilidad normal o Si el dolor es intenso y mantenido, se puede extirpar
quirúrgicamente.

Fimosis
• Definición o Fimosis: Incapacidad de retraer el prepucio detrás del glande
 Fimosis fisiológica: propia del RN y lactante
 Fimosis patológica: propia del escolar o Circuncisión es una de las cirugías más frecuentes
• Etiopatogenia o Patogenia
 En el pene, a nivel distal, se produce una fusión entre el epitelio prepucial y glandular en las primeras etapas de la vida.
 4 – 5% de los RN tiene el prepucio libre.
 La liberación prepucial es progresiva y lenta. Al año, la mitad no presenta fusión prepusioglandular.
 A los 3 años, el 10% persiste, y a los 4 solo un 5%.
 Cuando persiste en mayores de 4 años, ya no hay mejoría. o Fimosis fisiológica
 Liberación progresiva entre ambos epitelios que conforma las adherencias balanoprepuciales
 La descamación de ambos epitelios y la degeneración celular conforma el esmegma.
• Puede enquistarse en ocasiones, formando los quistes de esmegma que libera material caseoso estéril que
puede confundirse con pus.
o Fimosis patológica
▪ Baja frecuencia y de etiología mayormente congénita ▪ Dos etiologías menos frecuentes
• Cicatricial: Frecuente, producto de masaje prepucial en etapas tempranas que provoca fisuras con cicatrización posterior y estrechez secundaria
• Balanitis xerótica obliterante: Prepucio de evolución normal que a los 8 – 9 años muetra estrechez severa fibrótica y acartonada de origen
autoinmune. Se da en alérgicos o atópicos. • Clínica o Asintomática. o Incapacidad de retraer el prepucio dejando libre el glande.

40
8
OSCE EUNACOM SP |||||||||||||||||||||||||||||||||||||||||||||||||||||||||||||||||||

o Micción: Si la estrechez es severa puede haber englobamiento prepucial distal con goteo al final de la micción, con chorro débil. No es frecuente que
cause obstrucción urinaria.
o Adherencias balanoprepuciales: lesiones fisiológicas.
• Diagnóstico: Clínico
• Complicaciones o Infecciones localizadas en el prepucio: Prepucitis, balanitis o balanopostitis.
 Compromiso progresivo del prepucio y el glande
 Inflamación, dolor localizado, disuria, signos de masaje como fisuración (probable causa de la infección)
 Gérmenes propios de la zona, no causa CEG y el manejo es local.
o Parafimosis
 Se produce por la retracción violenta de un prepucio con estrechez moderada con estrangulamiento a nivel del surco balanoprepucial.
 Produce edema prepucial y glandular progresivo con incapacidad de reducción
• Tratamiento o Fimosis verdadera
▪ Circuncisión
• > 3 años de edad
• Fimosis secundaria
o Balanitis y balanopostitis recurrentes
▪ Indicación relativa de circuncisión debido a recurrencia de infección.
• Complicaciones de la cirugía o Hemorragia
 Complicación más frecuente
 Extensión variable
 Ocurre en las primeras 24-48 horas desde la operación.
 Buen pronóstico. No compromete hemodinámicamente al paciente.
o Meatitis
 Infección localizada del glande que compromete el meato.
 Produce secreción sero-purulenta y disuria. Eventualmente retención urinaria.
 Ocurre en los primeros 7 días post operación.
 Manejo: Aseo local y ATB tópico de amplio espectro.
o Parafimosis
 Precoz: preoperatorio inmediato, por edema; o tardío por estenosis o recidiva de la fimosis.
 Manejo: Frío local y reducción o Estenosis meatal
 Estenosis secunadaria a meatitis.

40
9
OSCE EUNACOM SP |||||||||||||||||||||||||||||||||||||||||||||||||||||||||||||||||||

 Puede ocasionar obstrucción urinaria. Debe resolverse prontamente.


 Manejo: Queratolíticos por dos semanas. SI no cede, meatoplastía. o Recidiva de la fimosis
 Se produce por dejar prepucio largo o estenótico post cirugía. ▪ Puede darse en balanitis xerótica que queda con prepucio largo
▪ Manejo: Recircunsición. o Fístula uretral
 Iatrogénica intraquirúrgica por mal uso de electrobisturí. o Bandas o bridas prepucioglandulares
 Poco frecuente
 Tardía. Formación de bandas fibróticas como puentes desde el prepucio al glande
 Se produce por liberación traumática de las adherencias balanoprepuciales al divulsionar el prepucio en la primera etapa de la
cirugía.
o Necrosis peneana
 Complicación muy grave por mal uso del electrobisturí durante la cirugñia ▪ Pronóstico reservado. Podría implicar cambio de
sexo en RN.

Testículo no descendido
• Introducción
o 0.8% mayores de 1 año y 3.4% de los RN normales.
o 30.3% de RNPT o Importante por las secuelas a largo plazo de no mediar corrección Esterilidad y malignización o Etiología desconocida en la
mayoría de los casos
• Definiciones o Criptorquidea: Testículo en el trayecto normal de descenso pero fuera de la bolsa escrotal.
 Criptorquidea intraabdominal: Por dentro del anillo inguinal profundo, en la cavidad abdominal
 Criptorquidea canalicular: Entre el anillo inguinal profundo y la entrada del escroto. o Ectopia: Testículo fuera de lugar normal de descenso.
Generalmente superficial a la aponeurosis del oblicuo externo.
o Testículo en ascensor:
 Normalmente en el escroto pero por acción del cremaster asciende, pudiendo llegar al canal inguinal
 Ocurre por estímulos como frío o temor.
 El reflejo cremastérico está ausente en menores de 3 meses y va aumentando en intensidad entre los 3 meses y los 7 años.
 Testículos obligados a descender manualmente por el examinador pero al dejarlos libres ascienden nuevamente Testículo criptorquídico
canalicular.
• Etiología Multifactorial o Eje hipotálamo-hipófisis-testículo
▪ Hipotálamo produce LH y FSH que generan mayor producción de testosterona (el descenso es regulado en parte por la testosterona)
o Sustancia inhibitoria de los conductos de Müller Influencia en las etapas iniciales del descenso o Presión intraabdominal Impulsa al testículo a
través del canal o Andrógenos

41
0
OSCE EUNACOM SP |||||||||||||||||||||||||||||||||||||||||||||||||||||||||||||||||||

 Aumentan en los primeros meses por disminución de la exposición a estrógenos maternos que inhiben la producción de andrógenos
o Gubernaculum testis: Varias inserciones embriológicas (escroto, inguinal, perineal, femoral). SI alguna es más firme se desarrollará ectopia. o
Torsión intrauterina Puede generar ausencia del testículo por necrosis (puede confundirse con agenesia). Es más frecuente en el lado izquierdo.
• Frecuencia o 3.4% de RN normales o 30.3% RNPT (<2500 gramos) o 10% bilateral o Monorquia (ausencia de 1
testículo) 4%, Anorquia (ausencia de ambos) 0.6%
• Clínica o Asociaciones con otras patologías o malformaciones
 Válvulas uretrales: Genera síntomas urinarios: (relativamente frecuente)
 Síndromes genéticos
 Hernia inguinal
 Hipospadia: Sospechar hiperplasia suprarrenal congénita o Estudio
 Eco y/o TAC
 Prueba HCG (de no contar con eco o TAC) Alza en testosterona basal post inyección indica presencia de tejido testicular. Negativa: Buscar
estado intersexual.
• Secuelas o Cambios histológicos a partir de los dos años
 Disminución de las espermatogonias
 Disminución del grosor de los túbulos seminíferos
 Fibrosis intersticial, degeneración de mitocondrias. o Esterilidad o Mayor riesgo de cáncer (pexia no disminuye el riesgo pero ayuda
al diagnóstico precoz)
• Tratamiento o Pexia entre 1.5 y 2 años de edad o Buscar hernia inguinal
• Complicaciones quirúrgicas: Poco frecuentes o Inmediatas: hematoma, infección Hop, sección de vasos, sección del
conducto deferente o Tardías: atrofia testicular, recidiva.

41
1
OSCE EUNACOM SP |||||||||||||||||||||||||||||||||||||||||||||||||||||||||||||||||||

Sinequia vulvar
• Definición o Es la unión del borde del introito de los labios menores por su borde libre, formando una cicatriz más o menos gruesa, dependiendo de la
antigüedad del cuadro.
o Patología adquirida, no congénita, cuya etiología no está totalmente esclarecida, pero que se presenta con más frecuencia en niñas con antecedentes
de dermatitis del pañal o de irritación de los genitales externos, lo que asociado al déficit de estrógenos propio de la etapa prepuberal, hace que esta
zona erosionada se adhiera y cicatrice unida, especialmente si la higiene genital es deficiente.
• Epidemiología o Es bastante frecuente, constituyendo aproximadamente 19% de las consultas en los policlínicos de
Ginecología Infantil dependientes de Servicios de Pediatría; o Si consideramos sólo las niñas prepuberales, este porcentaje sube a 37%. De estas, la mitad son casos nuevos
cada año y el resto, controles de sinequias diagnosticadas en años anteriores.
o Un estudio efectuado en 1992 que incluyó 1 200 controles de niña sana entre 1 y 18 meses de edad, encontró una incidencia de sinequia vulvar de 5%.
• Clínica o Esta patología es generalmente asintomática, pero si no se trata puede llevar a infección e incluso a retención urinaria.
o Es corriente que la madre no note esta alteración generando consultas a edad avanzada.
o En los casos en que se encuentre sinequia hay que preguntar por antecedentes de dermatitis del pañal, vulvitis, "goteo" posmiccional, fiebre de causa
inexplicada o infección urinaria, producida por la retención de restos de orina entre la sinequia y el introito, que forma una especie de fondo de saco.
o La edad de la primera consulta es variable, siendo en 51% de los casos antes de los 12 meses de edad y en 4% sobre los 6 años.
o Cuando la sinequia es total, a la inspección se encuentra una superficie plana desde el periné hasta el clítoris, con una línea sagital blanquecina o
transparente, no identificándose las estructuras propias de esta zona como introito, meato uretral, himen, orificio vaginal externo, etc.
o A veces se logra ubicar un orificio milimétrico bajo el clítoris, a través del cual fluye orina con dificultad.
o En las sinequias parciales es variable su longitud y por lo tanto el impacto en la emisión de orina.
• Tratamiento o El tratamiento es sencillo, pero debe controlarse periódicamente hasta que la niña inicie su desarrollo puberal, o sea entre los 8 y 9 años
de edad en la mayoría de ellas.
o Durante el examen debe efectuarse un masaje digital suave con vaselina en el sitio de unión. Si se separa, tomar muestra de secreción vaginal y test de
Graham para pesquisar un posible agente causal y efectuar tratamiento específico.
o En las niñas que usan pañales hay que lubricar el introito en cada muda después del aseo y prevenir las dermatitis del pañal. Una vez que la niña controle
esfínteres y deje de usar pañales, la lubricación debe efectuarse una a dos veces al día hasta que que inicie desarrollo puberal.
o El control se realiza cada 3 a 6 meses, dependiendo de su evolución, para prevenir recidivas.
o Si en la primera consulta la sinequia no se separa con la colocación de vaselina con un pequeño masaje, hay que aplicar una crema de estrógenos
(estradiol o estriol, no broparoestrol) en el sitio de unión 3 veces al día hasta que se separe, con un máximo de 15 días y controlar.
o Una vez separada, hay que efectuar exámenes de secreción vaginal y test de Graham, manteniendo lubricado el introito en cada muda si usa pañales o
1 a 2 veces al día después del control de esfínteres, como veíamos en el párrafo anterior.
o Hay que explicar claramente a la madre los efectos y riesgos de la aplicación de estrógenos tópicos, para que no se entusiasme y los siga colocando, o
los repita sin supervisión especializada.
o El estrógeno tópico se absorbe y su uso indiscriminado puede producir pigmentación de genitales externos o aréolas, aumento de volumen mamario y
aun pseudopubertad precoz si se prolonga su aplicación en el tiempo.
o Hay que recalcar que el tratamiento de la sinequia es médico y no quirúrgico, ya que si bien con una intervención se separan los labios, al no tomar las
medidas de prevención se vuelven a unir, formando una cicatriz más fibrosa que la original, siendo más difícil su tratamiento. Además existe el riesgo
de la anestesia y el trauma psicológico que significa toda operación quirúrgica.

41
2
OSCE EUNACOM SP |||||||||||||||||||||||||||||||||||||||||||||||||||||||||||||||||||

Como conclusión podríamos decir que la sinequia vulvar es una patología frecuente, en un porcentaje importante no es detectado por el pediatra, transcurre un tiempo valioso entre
su detección y la referencia al especialista y que los tratamientos previos son generalmente insuficientes o deficientes y sin las indicaciones de prevención de recidivas. Las sinequias
recientes son de tratamiento fácil, con un tratamiento adecuado la respuesta es excelente y no es necesario el tratamiento quirúrgico.

41
3
OSCE EUNACOM SP |||||||||||||||||||||||||||||||||||||||||||||||||||||||||||||||||||

Hernia inguinal e hidrocele testicular


• Embriología o Proceso vaginal
 Conformado por peritoneo que protruye a través del anillo inguinal profundo formando un saco que sigue al
gobernaculum y al testículo hasta el escroto, formando la túnica vaginal.
 En la niña se llama conducto de Nuck y sigue al ligamento redondo hasta los labios mayores.
 Permanece abierto en el 80% de los fetos al nacer, 50% al año y 15% adultos. ▪ Falla en su cierre puede generar
• Hernia inguinal incompleta
• Hernia inguinoescrotal o completa
• Hidrocele comunicante
• Hidrocele testicular
• Quiste del cordon (o quiste de Nuck en mujeres).
• Epidemiología o < 15 años: 5 – 8% o 1 año: 80% antes del primer mes o Es más frecuente en RNPT, fibrosis quística o enfermedad
de Ehlers-Danlos o Es más frecuentes en niños que en niñas (4-6:1) o 60% son derechas, 25% izquierdas y 15% bilaterales
• Clínica o Aumento de volumen inguinal que aumenta con el llanto, tos o al obrar. o Siempre es indirecta en niños
• Diagnóstico Clínico
• Diagnóstico diferencial: Hidrocele testicular (transiluminación +), hernia femoral (se palpa bajo el ligamento inguinal), Ovario dentro
de saco herniario.
• Tratamiento Siembre quirúrgico y lo más pronto posible
• Conceptos o Hernias
 Hernia irreductible
• Definición o Aquella donde el contenido no puede volver al abdomen o Puede estar atascada (vísceras sin daño vascular) o
estrangulada (vísceras con daño vascular)
• Clínica o Aumento de volumen doloroso difícil de reducir o Nauseas y vómitos por el dolor, aparición tardía y no en todos los casos.
• Manejo
o Debe intentarse la reducción inmediata, si no es posible, cirugía urgente
(compromiso intestinal) o Esclarecer el tiempo de evolución
 < 6 horas Reducción inmediata
 > 6 horas Cirugía o Tratamiento: Hernioplastía. Si hay
estrangulamiento, laparotomía para explorar contenido intestinal. ▪ Hernia directa
• Muy rara en niños. El diagnóstico diferencial es intraoperatorio
• Tratamiento: Reparación de la pared posterior ▪ Hernia femoral
• Baja incidencia

41
4
OSCE EUNACOM SP |||||||||||||||||||||||||||||||||||||||||||||||||||||||||||||||||||

• Aumento de volumen bajo el ligamento inguinal


o Hidrocele
▪ Hidrocele comunicante
• Definición o Similar a hernia inguinoescrotal o Salida de líquido peritoneal a través de anillo inguinal profundo estrecho
• Clínica o Aumento de volumen escrotal que aparece durante el día y desaparece en la noche (durante el reposo)
o Aumento de volumen escrotal no doloroso, blando, transiluminación (+)
• Tratamiento Hernioplastía
▪ Hidrocele o quiste del cordón (quiste de Nuck en niñas)
• Definición: Acumulación de líquido peritoneal en el proceso vaginal.
• Clínica: Masa en región inguinal, no dolorosa ni reductible ▪ Hidrocele testicular
• Definición o Acumulación de líquido en la vaginal del testículo o Muy frecuente en el RN. Desaparece en el primer año de
vida o Si no desaparece asocia hernia inguinal indirecta o Pre escolar y escolar generalmente es secundario a trauma,
infección o tumor.
• Clínica o Masa testicular blanda e indolora, no reductible o Transiluminación (+)
• Tratamiento o Expectante el primer año o Quirúrgico si está a tensión o es muy sensible (compresión testicular) o Niños
mayores: Buscar etiología (trauma, infección o tumor) y tratarla. Cirugía solo después del tratamiento.

Cuerpo extraño
• Introducción o La ingesta de CE es la segunda causa de endoscopía en niños o La mayoría en el TGI pasan sin ocasionar problema, solo el 20% requiere extracción endoscópica y
>
1% cirugía o La aspiración puede ser una emergencia si se obstruye la VA, más frecuentemente se aloja en el bronquio fuente derecho, dando síntomas persistentes y daño
pulmonar con bronquiectasias si no se extrae

Cuerpo extraño en la vía aérea


• Epidemiología o Causa de obstrucción con resultado de muerte en los menores de 2 años o Grupo etario más frecuente: pre escolar
(por autonomía de movimiento, explora, coge objetos) o Principal factor de riesgo de muerte característica propia de la VA: menor
diámetro lo que facilita la obstrucción
• Tipo de cuerpo extraño
o Alimentos: Lo más frecuente (maní, semillas) o Juguetes o Monedas o Clips
• Ubicación más frecuente o Bronquio fuente derecho 60% o Bronquio fuente izquierdo 32% o Traqueal 113% o Laringe 3%
• Presentación clínica o Variable dependiendo del objeto
 Cavidades nasales: DR, secreción nasal de mal olor, rinolalia, estritdor
 Laringotraqueal: DR aguda, estridor, disfonía

41
5
OSCE EUNACOM SP |||||||||||||||||||||||||||||||||||||||||||||||||||||||||||||||||||

 Síntomas asfícticos o síndrome de penetración


• Síndrome de penetración o Apremio de inicio brusco o Tos brusca en crisis o Estridor
o Obstrucción total o parcial de la VA
o Sibilancias generalizadas o silencio de un campo pulmonar o Hiperinsuflación localizada
o Tos, sibilancias y disminución del murmullo localizado (triada clásica 50%) • Tratamiento o Evaluación inicial
▪ Determinar la gravedad: conciencia, nivel de obstrucción.
• Obstrucción total con víctima conciente o > 1 año Maniobra de Heimlich o < 1 año Compresiones torácicas y dorsales
• Inconciente RCP con despeje rápido de la VA
▪ Paciente sintomático pero respira
• Obstrucción parcial No realizar maniobras
• Pisición de confort
• Oxígeno
• Hospitalizar en UCI para diagnóstico y localización de cuerpo extraño para extracción con broncoscopía
• Radiografía de tórax portátil
▪ Asintomático con examen pulmonar normal
• Con síndrome de penetración o Rx tórax AP y lateral normal Hospitalizar para mayor estudio
• Sin síndrome de penetración claro o Rx tórax AP y lateral normal Alta
o Radiografía de tórax
 Muestra el CE
 Signos indirectos que permiten sospechar
• Hiperinsuflación dinámica
• Atelectasias
• Asimetría de los campos pulmonares
• Desviación mediastínica
• Alteraciones de la columna de aire de la vía aérea
• Presencia de edema de partes blandas
 Solicitar siempre AP y LAT desde nariz hasta burbuja gástrica
• Monedas en esófago son frontales y en vía aérea laterales.
Cuerpo extraño intranasal
• No empujarlo

41
6
OSCE EUNACOM SP |||||||||||||||||||||||||||||||||||||||||||||||||||||||||||||||||||

• Remover con clip o aspiración directa, presionando la narina CL

Ingestión de cuerpo extraño


• Epidemiología o Frecuente en pediatría, en menores de 5 años
o La mayoría pasan fácilmente las estrecheces fisiológicas y anatómicas del tubo y se eliminan de forma espontánea
o Una parte puede presentar rechazo alimentario, vómitos, disfagia, salivación y fiebre o Complicación Obstrucción y perforación
• Tratamiento o Asegurarse que se ha ingerido y no aspirado (síndrome de penetración) o Determinar el tipo de cuerpo extraño o Tiempo desde la
ingesta o Síntomas que hagan sospechar CE en esófago: dolor, disfagia, sialorrea o SI es radiopaco, solicitar Rx de cuello, tórax y simple de abdomen
para verificar presencia y localización.
o NO USAR CONTRASTES
• Localizaciones o CE esófago
 Deben retirarse cuanto antes, evitar esofagitis, perforación, fístulas
 Considerar remoción urgente
• Pilas
• Objetos cortantes o punzantes riesgo de lesión de mucosa y mediastinitis a las pocas horas
o CE estómago
 Mantener conducta expectante (lo más probable es que avance)
 Pilas, luego de 24 – 48 horas en estómago, deben removerse con endoscopía porque el jugo gástrico puede filtrar líquido caústico
o CE intestino
 Se dejan eliminar espontáneamente
 Revisar las deposiciones
 Reconsulta por falta de eliminación, no hacer radiografía hasta 8 días, lo más probable es que se haya eliminado, si no, seguir esperando y no
se retira a menos que hayan síntomas de obstrucción o abdomen agudo
 Imanes, si son dos, deben ser extraídos ya que pueden atraerse y producir isquemia y perforación.

41
7
OSCE EUNACOM SP |||||||||||||||||||||||||||||||||||||||||||||||||||||||||||||||||||

Hemorragia digestiva
• Introducción o Es una emergencia cuando es de gran
volumen o A la confirmación de sangre, se debe estimar la
cantidad y el tratamiento de la causa subyacente
• Etiología
o RN
 Sangre materna ingerida
 Colitis alérgica Moco sanguinolento en heces
 Enfermedad péptica
 Coaguloatía Parto a domicilio (sin vitamina K)7
 Vólvulo Hematemesis, hematoquecia. Abdomen doloroso y distendido o Lactante
 Invaginación Hemorragia rectal
 Divertículo de Meckel Rectal, masiva, roja birllante, sin dolor
 Enteritis bacteriana Diarrea sanguinolenta, fiebre
 Enfermedad péptica o > 2 años
 Lesión por AINE
 Enteritis bacteriana
 Divertículo de MEckel
 Sindrome de Mallory Weiss Hematemesis rojo brillante, sigue a arcadas ▪ SHU Rectal, trombocitopenia, anemia, uremia.
 Hemorroides
• Características distintivas o Sangre en el pañal
 Puede ser del tracto urinario, vagina, incluso exantema del pañal intenso o Determinar si hemorragia GI es alta o baja (Treitz)
 Hematemesis es siempre proximal
 Rectal puede ser alta o baja
 Melena Alta
 Roja brillante Baja (colon o recto)
• Estudios de laboratorio o A todos
▪ Hemograma, plaquetas, coagulación, función hepática, sangre oculta en heces, grupo y Rh.
o Diarrea sanguinolenta
 Clostridium difficile en coprocultivo
 Colonoscopía

41
8
OSCE EUNACOM SP |||||||||||||||||||||||||||||||||||||||||||||||||||||||||||||||||||

 TAC con contraste o Hemorragia rectal con heces formadas


 Tacto rectal
• Imágenes o A todos
 Radiografías de abdomen seriadas o Hematemesis
 Endoscopía o Hemorragia con dolor y vómitos (obstrucción intestinal) ▪
Radiografías de abdomen seriadas

41
9
OSCE EUNACOM SP |||||||||||||||||||||||||||||||||||||||||||||||||||||||||||||||||||

Neutropenia, leucemia y linfoma

Neutropenia
• Definición Disminución del RAN o RAN < 1500 o Agranulocitosis <
500
• Clasificación o Leve: 1000 – 1500 o Moderada: 500 – 1000 o Severa:
< 500
• Causas o Producción medular disminuida
 Trastorno de celula precursora Leucemia, anemia aplasica, mielodisplasia
 Deficiencia de Vit B12 o Ac. Fólico
 Quimio y radioterapia
 Enfermedades congénitas poco
frecuente, se da en RN o Aumento de destrucción
celular Neutropenia
 Infecciones graves febril

 Infección viral Influenza, sarampión,


varicela
 Infecciones bacterianas Fiebre tifoidea (raro)
 Mecánica Hiperesplenismo
 Inmune
o Pseudoneutropenia temporal, en infecciones virales o bacterianas

Cáncer en pediatría
• Epidemiología o Leucemias son 40%, seguidos de tumores del SNC y linfomas
• Leucemias o Epidemiología
 Neoplasia más frecuente de la niñez (40%)
 Incidencia de 4 – 5 x 100.000 ▪ Sobrevida
• L. linfoblástica aguda 75%
• L. Mieloide aguda 50%
• L. Mieloide crónica 60% (niños más grandes)
o Sospecha clínica
 Fiebre + citopenia
 Anemia en > 3 años sin sangrado

42
0
OSCE EUNACOM SP |||||||||||||||||||||||||||||||||||||||||||||||||||||||||||||||||||

 Síndrome hemorrágico
 Dolor óseo
 Bicitopenias o Clínica
 Leucemia aguda
• Síntomas y signos o Insuficiencia de hemopoyesis (triada)
 Anemia
 Neutropenia
 Trombocitopenia o Dolor óseo Muy frecuente o
Hepatoesplenomegalia o Adenopatías
• Cuadros
o L. linfoblástica aguda
▪ Epidemiología
• Neoplasia más común en niños
• Sobrevida importante 70%
•Más frecuente en hombres entre 1 – 6 años
▪ Diagnóstico: Hemograma, anemia, leucocitosis o leucopenia, trombocitopenia, blastos en sangre periférica
• Definitivo Mielograma
 Tratamiento PINDA • Linfomas o Epidemiología
 13% de los Ca infantiles.
 Principalmente niños mayores (10 años) o Sospecha
 Adenopatía indolora > 15 días o crecimiento progresivo + pérdida de peso
 Dolor abdominal difuso
 Masa abdominal
 Tumor maxilar de crecimiento rápido o Cuadros clínicos
 Linfoma de Hodgkin
• Epidemiología
o 6%
o Más frecuente en hombres y > 10 años o Presencia de
células de Reed Sternberg
• Clinica o Adenopatía indolora supraclavicular o cervical, firmes, gomosas.

42
1
OSCE EUNACOM SP |||||||||||||||||||||||||||||||||||||||||||||||||||||||||||||||||||

o Sintomas opresivos del mediastino o Fiebre, sudores


nocturnos, pérdida de peso ▪ Linfoma no Hodking • Epidemiología o
Menos frecuente
o Inmunodeficiencias aumentan el riesgo o VEB se relaciona
o Sobrevida mejor 60 – 75% • Adenopatías o Definición
 Ganglio es adenopatía cuando es > 1 cm
 La mayoría secundarias a infecciones o Conducta sospechar cancer en toda adenopatía que
 Unilateral, > 2 – 3 cms, indolora, adherida
 Dura, firme, abollonada
 Supraclavicular, mediastínica o posterior al ECM
 Sin regresión con ATB
• Empírico Amoxicilina por 10 días
• Si tiene puerta de entrada Cloxacilina

42
2
OSCE EUNACOM SP |||||||||||||||||||||||||||||||||||||||||||||||||||||||||||||||||||

PATOLOGÍA INMUNOLÓGICA

Reacciones alérgicas y anafilaxia


• Introducción o Reacción de hipersensibilidad sistémica, de mecanismo inmunológico, no inmunológico o idiopñatico o Las causas más frecuentes en
niños y adolescentes son: alimentos, drogas e insectos o Diagnóstico es clínico o La manifestación clínica más frecuente es el compromiso cutáneo
(urticaria y angioedema) y síntomas respiratorios
o El manejo es ABC y adrenalina IM
• Definición o Reacción de hipersensibilidad sistémica inmediata, por la liberación de sustancias vasoactivas. Lo más frecuente es la mediada por IgE
o Ingreso del antígeno puede ser VO, inyectable o tópico o Inicio de los síntomas puede ser minutos a horas, entre más precoz puede ser de mayor
gravedad
• Fisiopatología o Histamina es el mediador más importante: actúa en receptores H1 y H2, produce vasodilatación y aumento de la permeabilidad
vascular.
o Desplazamiento de líquido del intra al extravascular (hasta 50% en 10 min). Como compensación se liberan catecolaminas y angiotensina I pasa a II, lo
que aumenta la RVP, para revertir vasodilatación
o Pacientes que toman Inhibidores de la angiotensina II o B bloqueadores presentan intensa hipotensión
• Mecanismos o Inmunológico
 Dependiente de IgE: Unión a receptores IgE de mastocitos y basófilos
 Independiente IgE: Activación de mastocitos por opiáceos, vancomicina, medios de contraste
o No inmunológico o Idiopático: alteración clonal de mastocitos
• Etiología o Causas más frecuentes: Alimentos, drogas, insectos
 Alimentos: leche, huevo, soya, trigo, maní y pescado (90%)
 Drogas: PNC, aspirina, AINES, quimioterapia
 Insectos: Avispas, abejas, hormigas
 Látex
 Vacunas: Componentes que asocian látex, huevo, gelatina, neomicina
 Perioperatorio: relajantes musculares, látex, ATB, opiodes, coloides, productores sanguíneos ▪ Ejercicio (anafilaxis inducida)
 Inmunoterapia
 Ideopática
• Diagnóstico Clínico o Criterios clínicos (Un grupo de criterios por si solo)
▪ Enfermedad de inicio agudo (minutos a horas) después de exposición con compromiso de piel, mucosa o ambos; asociado a uno de los siguientes
• Compromiso respiratorio
• Hipotensión arterial o síntomas asociados a disfunción orgánica ▪ Dos o más de los siguientes

42
3
OSCE EUNACOM SP |||||||||||||||||||||||||||||||||||||||||||||||||||||||||||||||||||

• Compromiso súbito de piel, mucosas o ambos


• Compromiso respiratorio súbito
• Hipotensión arterial súbita o síntomas asociados a disfunción orgánica (síncope, incontinencia, hipotonía)
• Síntomas gastrointestinales súbitos
▪ Presión arterial reducida o Laboratorio confirmaría pero no se usa de rutina
• Clínica o Más frecuente
 Compromiso cutáneo (80-90%) : Urticaria y angioedema
 Síntomas respiratorios (60 – 70%)
 Síntomas CV (20 – 30%) o Graves
 Estridor, disnea, sibilancias por edema de la VA, broncoespasmo
 Hipotensión, vértigo, síncope y shock por colapso cardiocirculatorio o Otras
 Cólicos abdominales, nauseas, vómitos y diarrea
 Rinitis
 Prurito sin exantema o Anafilaxia bifásica Fase temprana y tardía con resolución temporal entre ambas.
• Diagnóstico diferencial o Reacciones vasopresoras Sin urticaria
• Factores de riesgo para anafilaxia severa o fatal o Edad: adolescentes o Enfermedades
concomitantes: Asma, enfermedades CV, mastocitosis, alteraciones clonales de
mastocitos, rinitis alérgica y eccema
o Drogas: AINES, iECA, Antagonistas B adrenérgicos (bradicardia paradójica, hipotensión profunda y broncoespasmo severo)
o Cofactores que amplifican la respuesta: ejercicio, infecciones agudas intercurrentes, fiebre, estrés emocional, síndrome pre menstrual
• Grados de anafilaxia o Leve
 Piel Prurito súbito en ojos y nariz, eritema facial, urticaria, angioedema
 GI Prurito oral, hormigueo oral, hinchazón de labios, nauseas o vómitos, dolor abdominal
 Respiratorio Congestión nasal o estornudo, rinorrea, prurito en garganta, leves sibilancias
 CV Taquicardia
 Neurológico ansiedad o Moderada
 GI Agrega dolor abdominal cólico, diarrea, vómitos
 Respiratorio: Agrega ronquera, tos irritativa, disfagia, sibilancias moderadas
 Neurológico: Agrega mareo, sensación de muerte inminente o Severo (shock)
 GI Agrega pérdida de esfínter
 Respiratorio Agrega cianosis, saO2 < 92%

42
4
OSCE EUNACOM SP |||||||||||||||||||||||||||||||||||||||||||||||||||||||||||||||||||

 CV Agrega hipotensión, paro, arritmia, bradicardia severa


 Neurológico Agrega confusión o pérdida de conciencia
• Tratamiento agudo
o Inmediato
 Remover la exposición
 ABC
 Adrenalina IM tan pronto se pueda (IM tiene rápido peak plasmático)
• Dosis: 0.01 mg/kg o 0.01 ml/kg de ampolla sin diluir (1mg/ml) Repetir cada 5 – 15 minutos
o Medidas generales
 Decúbito supino con pies levantados (aumenta retorno venoso)
 Estabilizar y mantener VA (ventilar con bolsa mascarilla de ser necesario)
 O2 al 100%
 Via venosa permeable
 Volumen: Solución fisiológica
• 1 hora 30 ml/kg o 20 ml/kg cada 10-20 minutos
o Medidas si no responde a adrenalina
 Adrenalina en infusión monitorizar
• Dosis: 0.1 mcg/kg/min
 Antagonista H1 segunda línea en shock. Sirve para controlar el prurito y la urticaria
• EV
o Clorfenamina IM o IV lento
 > 12 años: 10 mg
 6 – 12: 5 mg
 6meses – 6 años: 2.5 mg
 < 6 meses: 250 mcg/kg o Ranitidina
 Dosis 1 mg/kg/dosis cada 8 horas (máximo 50 mg) a pasar en 15 minutos
• Oral o Cetirizina
 2 – 6 años: 5 mg día VO
 6 – 18 años 10 mg día VO ▪ Broncoespasmo resistente a
epinefrina: SBT
• 0.5 ml + 3.5 ml SF 0.9% NBZ

42
5
OSCE EUNACOM SP |||||||||||||||||||||||||||||||||||||||||||||||||||||||||||||||||||

• 10 puff cada 20 minutos hasta mejorar


 Hipotensión refractara a volumen y epinefrina: Dopamina
 Usuarios con B bloqueo con mala respuesta a epinefrina: Glucagón Inótropo u cronótropo positivo
• 20 – 30 mcg/kg
• 0.02 – 0.03 mg + infusión 5 – 15 mcg/min hasta respuesta
 Glucocorticoides sin efecto inmediato, para evitar reacciones bifásicas y en asma y otras patologías tratadas recientemente con corticoides
• EV
o Metilprednisolona 1 – 2 mg/kg/dosis
• Oral o Prednisona 0.5 -1 mg/kg/dosis cada 12 horas
o Observar en urgencias
▪ Ante posibilidad de reacción bifásica debe observarse hasta el cese de los síntomas
• 2 horas en leves • 4 – 10 horas en moderados
o Hospitalización
 Todo paciente con síntomas graves al inicio (hipotensión, DR severa) ▪ Reacción bifásica
 Alto riesgo: reacción a maní, asmáticos, usuarios de B bloqueadores.
 Hipotensión persistente que requirieron adrenalina EV o glucagón UPC o Indicaciones al alta
 Régimen: Evitar comida que gatilló anafilaxis ▪ Medicamentos
• Adrenalina IM
 15 – 30 kg 0.15 mg
 > 30 kg 0.3 mg
 < 10 kg 1 ml (jeringa cellada) o Indicaciones
 Moderadas-graves
 Con factores de riesgo de cuadro severo
 Inducida por ejercicio
 Ideopática
 Ruralidad
• AntiH2 y corticoides VO x 3 días o Cetirizina o Ranitidina o Prednisona ▪ Reconsultar SOS

42
6
OSCE EUNACOM SP |||||||||||||||||||||||||||||||||||||||||||||||||||||||||||||||||||

42
7

S-ar putea să vă placă și